You are on page 1of 184

,

HU
,U IN
NGUYN

, ,
P IRICHL
PHUONG PHA
,

`
VA UNG DUNG
.

,
BAN KHOA HOC VA
` XUT
` KY
THUT
NHA
.
.
` NI
HA
1999
.

,
`
` I NOI
U
LO

,
,
,,
,
d
Nguyn ly nhung c
ai l`ng v`
a c
ac ch
u tho d
a
uo. c bit d
n t`u rt
,
,
,,
, ,
l`
lu. Ngay trong chuong tr`nh ph thng co so ch
ung ta c
ung d
am
a
,
,
,
,
,
i phuong ph
quen vo
ap giai to
an n`
ay. Thu. c ra nguyn ly n`
ay mang
,`, ,
tn nh`
a b
ac hoc
Legien Dirichlet (1805. nguoi ,uc Pte Gutxtap
,
,
,
1859). Nguyn ly ph
at biu rt d
ung ta nht tho v`
ao
on gian: Nu ch
,
,
,
c
ac l`ng m`
a s l`ng t hon s tho, th` th n`
ao c
ung c
o mt
. l`ng nht t
,
nht hai con tho.
,
,
,
,
`ng nguyn ly d

Ch ba
ang loat
ac b`
ai to
an d
on gian nhu vy
a
. h`
. c
,
,,
d
uo. c giai.
,
,,
,
Cun s
ach d
lai
theo t`ung chu d
o lin quan d
uo. c bin soan
` c
n
.
.
,
,
,
,
,
c
p dung
nguyn ly, mi
ach giai trong v du. cua t`ung chuong l`
aa
.
,
,
,,

d
ai tp
o lin quan dn b`
ai
in h`nh nguyn ly irichl. B`
. giai truoc c
,
,
,
`

giai sau nn cn luu y khia d


ach. Voi mong mun c`
ung ban
oc
oc
. s
. d
.
,
,,
,

thao lun
ap chung minh to
an hoc
a hy vong
cung
. mt
. phu,ong ph
. v`
.

`
cp mt
ai liu
ac thy c gi
ao v`
a c
ac em hoc
. t`
. b ch cho c
. sinh ham
,
m t`m t`
oi trong to
an hoc,
ac gia manh
dan
ach
. t
.
. bin soan
. cun s
,
,
n

`
c cha
n`
ay. Do kha n
ang v`
a thoi gian c`
on han
ach cha
. ch, cun s
,
,
,
,
ng g
khng tr
anh khoi thiu s
ot. Ch
ung ti mong d
op y
uo. c su. d
o
,
,
,
,,
,

kin cua d
op y xin gui v Nh`
a xut ban Khoa hoc
a Ky
oc
. gia. Thu g
. v`
,
`
thut
a Ni.
. - 70 Trn Hung ao,
. H`
.
3

,
L`oi n
oi d`u

,
,
,
H`ng Tn d
d
T
ac gia xin chn th`
anh cam on PGS-TSKH
a
oc
,
,.
`

ng g
v`
ad
op nhiu y kin qu b
au trong qu
a tr`nh ho`
an chnh ban
o
,
thao.
,
T
ac gia

, ,
CHUONG

IRICHL VA
` VI DU.
NGUYN LY

1.1. Nguyn l
y irichl
,,
,
Nguyn ly irichl nhi`u khi ngu`oi ta goi
a Nguyn ly nhung
. ,l`
,
. c bit
ng
an keo. y l`
a mt
o nhi`u
on gian, d
a
. nguyn ly rt d
. c
,
,
,
ng dung
u
trong c
ac lnh vu. c kh
ac nhau cua to
an hoc.
ung nguyn
.
. D`
,
,
,`,
,

ng minh t`n tai


,i tnh
ang chu
ly n`
ay nguoi ta d d`
i tuo. ng vo
. mt
. d
,
,
,
,
,
cht x
ac d
Dang
o th ph
at biu nhu sau:
inh.
on gian nht c
.
. d
Nu c
o m vt
. t v`
ao n c
ai ng
an keo v`
a m > n th` c
o t nht mt
. da
.
,

ng
an keo chua t nht hai vt.
.
,
,,
,, ,
,
`ng vo
,ng minh d
i nguyn ly n`
Tuy ra
ay ngu`oi ta ch chu
uo. c su. t`n
,
,
,,
,,
,,
,
tai
a khng dua ra d
ap t`m d
cu. th, nhung
uo. c phuong ph
uo. c vt
. m`
.
,
,
,
,
,
trong thu. c t nhi`u b`
ai to
an ta ch c`n ch ra su. t`n tai
ad
u r`i.
. l`
,
,
,
Nguyn ly irichl l`
a mt
at biu
inh
. ly v` tp
. d
. ho. p huu han.
. Ph
,
chnh x
ac nguyn ly n`
ay nhu sau:
,, ,
,

Cho A v`
a B l`
a hai tp
o s ph`n tu huu
. , h,o. p khng rng c
, ,
,,
,
,n ho,n s lu,o.,ng ph`n tu, cua
han,
a s luo. ng ph`n tu cua A lo
. m`
, ,
ph`n tu, cua A cho tu,o,ng

,i mt
c n`
B. Nu vo
ao d
y, mi
. qui ,ta
,
,,
,
,ng vo
,i mt
u
kh
ac nhau cua A
. ph`n tu B, th` t`n tai
. hai ph`,n tu
, , ,
, ,
,i c`
m`
a ch
ung tuong u
ng vo
ung mt
. ph`n tu cua B.
,
,
,, ,
,
`ng c
, cho ra
d hiu ta cu
ac ph`n tu cua tp
a Nhung ng
an
. B l`

,,
Chuong 1. Nguyn ly irichl v`
a v du.

,
,, ,
,,
. t v`
keo v`
a c
ac ph`n tu cua A d
ao c
ac ng
an keo cua n
o. Trong
uo. c da
, ,
,, ,
,
,
`ng
`
ph
at biu cua nguyn ly trn c
ac phn tu huu han
uo. c tnh ba
. d
,
,
i tp
s tu. nhin, v` vy
c
o lin quan mt
. Nguyn ly irichl
. thit to
.
,
, ,
,

ho. p s tu. nhin v`


a c
ac tnh cht cua tp
ay.
. ho. p s n`

1.2. V du.

, ,
,
. 1.1. ky nim
am ng`
ay giai ph
ong Mi`n Nam, tai
anh
., 20 n
. ,mt
. th`
,
,
,
,
,
,
,

ng ngu`oi 20 tui. Ng`


ay 30
p ma
. t nhu
ph ngu`oi ta t chuc bui l ga
, .
,
trong bui ga
th
ang 4 n
am do
. p ma
. t c
o 400 thanh nin. Chung minh
,
,, , ,
,
`
ra
ng c
o t nht hai ngu`oi trong s ngu`oi t
oi du. c`
ung chung mt
ay
. ng`
sinh.
,
,
ng`
`,i giai. N
Lo
am 1995 c
o 365 ng`
ay. Ch
ung ta coi mi
ay nhu mt
.
,

ng
an keo v`
ad
a
nh
s
t
u
1
d

n
365
(ng
a
n
k
e
o
cu
i
c`
u
ng
l`
a
ng`
a
y
31

,
. t nhung thanh nin c
th
ang 12 n
am 1995). Ch
ung ta d
o ng`
ay sinh
a
, , ,
,
,
,n
. Nhung s thanh nin d
tuong ung v`
ao c
ac ng
an keo d
o
n du. l lo
,, ,,
,
hon s ng
an keo, theo nguyn ly irichl c
o t nht hai ngu`oi d
uo. c
. t v`
c
d
ao c`
ung mt
an keo. i`u d
o ngha l`
a ho. sinh c`
ung mt
a
o
. ng
.
ng`
ay.
,
,,
ngu,`o,i
. 1.2. Trong sinh hoc
`ng s t
oc trn d`u cua mi
. ngu`oi ta bit ra
,
,
,,
khng qu
a 200.000 c
ai. Chung minh ra
`ng trong s ngu`oi cua th`
anh
,
,
,
,
ph H`
a ni,
oi s dn hon 2.000.000, c
o t nht 11 ngu`oi c
o c`
ung s
. v

t
oc.
,
,,
,
`,i giai. Ch
nh s t`u 0 d
Lo
ung ta xet 200.000 ng
an keo d
uo. c d
a
n
,
,
ngu`oi dn H`
. t mi
199.999. Ch
ung ta d
a ni
v`
ao mt
an keo
a
. ng
,
, ,.,
,
,
,,
`ng s thu
tu. cua ng
m`
a s t
oc ba
an keo. Gia su khng c
o 11 ngu`oi
,
,,
ng
c
o c`
ung s t
oc, nhu vy
an c
o nhi`u nht l`
a 10 ngu`oi c
o c`
ung
. mi
s dn H`
s t
oc, do d
a ni
a 200.00010=2.000.000,
o
. nhi`u nht l`

1.2. V du.

,
,i gia thit l`
,n ho,n 2 triu.
ng vo
d
ay khng d
a s dn H`
a ni
i`u n`
u
. lo
.

,
,
. 1.3. Ba muo,i hoc
am b`
ai vit chnh ta. Mt
. sinh l`
. trong s hoc
. sinh
,
,

bi. 14 li, c`
do
on c
ac hoc
ac ma
c li t hon. Chung minh ra
`ng
. sinh kh
,
,
ba
c s li
`ng nhau.
c
o t nht ba ngu`oi ma
,
,,
,
`,i giai. Ch
nh s t`u 0 d
Lo
ung ta xet 15 ng
an keo d
ung
uo. c d
a
n 14. Ch

t mi hoc
ng s li
ng ba
ta d
ao mt
an keo mang s d
a
u
. sinh v`
. ng
, .
ba
`ng nhau,
cua hoc
ay. Nu khng c
o ba hoc
ao c
o s li
. sinh n`
. sinh n`
,
ng
th` trong mi
an mang s t`u 0,1,2,. . . ,13 se c
o nhi`u nht hai hoc
.
,
,
,
,

s luo. ng cua nhung hoc


sinh. Khi d
ay nhiu nht l`
a 28. Nu
o
. sinh n`

c 14 li (trong ng
hoc
thm v`
ao d
an keo s 14) ch
ung ta se
o
. sinh ma
,
,,
d
`

`
nhn
ay dn
uo. c nhiu nht 29 hoc
iu n`
n
. d
. sinh vit chnh ta, d
,
,
i d
cho.
su. v ly vo
i`u kin
a
. d

,
,,
nh
o t nht hai
. 1.4. Chung minh ra`ng trong mi
om ban
. 5 ngu`oi c
,
,
,
,
,
,`,
,
,
,
a nhu
ng ngu`oi trong nh
o c`
ung s luo. ng ngu`oi quen giu
om dos.
nguoi c
,
,,
,
,
`

Chung minh ra
ng c`
ung kt lun
oi nh
om ban
o s luo. ng
. nhu vy
. v
. c
th`
anh vin bt k`y.
,
,
ngu,`o,i
`,i giai. Ch
nh s t`u 0 dn 4. Mi
Lo
ung ta xet n
am ng
an keo, d
a
, ,,
,i s ngu,`o,i trong
. t v`
tham du. d
ao ng
an keo mang s tr`
ung vo
uo. c d
a
,,
quen.
nh
om m`
a ngu`oi d
o
,
,,
,
,,
a) Nu c
o mt
. ngu`oi khng quen ai ca trong, s nhung ngu`oi
,,
c`
on lai,
an s 4 l`
a trng (v` nguo. c lai
an 0 v`
a4
. th` ng
. th` ca hai ng
,
,
,
d
ngu`oi trong s 5
d
`u khng trng, dn
n v ly). Nhu vy,
. mi
,`, , ,
,i s lu,o.,ng 4 ng
. t v`
nguoi d
ao c
ac ng
an mang s 0,1,2,3 vo
an.
uo. c d
a
,,
,
,
,
T`u nguyn ly irichl suy ra t nht c
o hai ngu`oi o trong mt
an,
. ng
,
,
,
,
hay l`
a, ho. c
o chung s luo. ng ngu`oi quen.

,,
Chuong 1. Nguyn ly irichl v`
a v du.

,,
,,
,,
ngu,`o,i se d
b) Nu moi
o t nht mt
uo. c
. ngu`oi c
. ngu`oi quen, mi
,i s lu,o.,ng 4 ng
. t v`
d
ao c
ac ng
an mang s 1,2,3,4, vo
an. Ph`n c`
on lai
a
.
p dung
a
nguyn ly irichl.
.

,
,
,
ca
tham du. 16 di.
. 1.5. Trong mt
ong da
. p hai di
phai
. giai b
. ,Mi
.
,
,
,
,
th`o,i d
du v
oi nhau. Chung minh ra
`ng tai
o t nht
im cua giai c
. mi
,
du nhu nhau.
2d
o s trn
i
. c
. da
,
,
`ng
`,i giai. Ch
nh s t`u 0 d
Lo
ung ta xet 16 ng
an keo d
u y ra
a
n 15. Ch
,
,
,
,
,
d
n nht c
15 l`
a s luo. ng lo
ac trn
b
ong m`
a mi
c
o th d
th`oi
i
u tai
.
.
.
,
d
`ng
. t mi
d
ay d
ong v`
ao ng
an keo mang s ba
im d
ang xet. H
a
i
. b
,
,
`ng
d
. Ch
s c
ac trn
ad
ung ta nhn
i
a
u d
n th`oi d
im d
o
. m`
. d
. ra ra
,
,
,
,
,
c
ac ng
an 0 v`
a 15 khng th d
a nhu vy
`ng th`oi khng trng d
uo. c v`
.
,
p dung
c
o th a
nguyn ly irichl.
.

,
,
. 1.6. Trn trai dt sng ho,n 5 ty ngu`o,i, bit ra`ng khng qua 1% sng
,
,
,,
ung
trn mt
am tui. Chung minh ra
`ng t nht c
o hai ngu`oi sinh c`
. tr
mt
. giy d`ng h`.
,
,
`,i giai. Theo du,o,ng lich
Lo
anh 100 n
am c
o t hon 37000 ng`
ay.
. hin
. h`
,
,
,

100 n
Mi ng`
ay c
o 24 gi`o, mi gi`o c
o 3600 giy. Khi d
am c
o t hon
o
,
, `
,,
,
,,
3,33 ty giy. T`u d
ch
ung ta t`m d
ai
iu kin
uo. c nhung ngu`oi trn tr
.
,
,
,
,
,

d
a 100 tui t nht l`
a 99% t`u 5 ty ngu`oi ngha l`
a t nht
t khng qu
,
,
,,
. t 4,9 ty ngu`oi
p dung
c
o 4,9 ty. Vic
c`
on lai
a
nguyn ly irichl: d
a
.
.
.
,
an keo.
v`
ao 3,33 ty ng

,
. 1.7. Trong th`o,i gian keo d`ai mt
n
am hoc
mt
hoc
sinh giai t nht
.
.
.
.
,
,
,
ng`
mt
ai tp
ay. tr
anh c
ang tha
ng hoc
ang tu`n
. b`
. mi
. sinh giai h`
,
,
khng qu
a 12 b`
ai tp.
ra
`ng trong th`oi gian keo d`
ai lin
. Chung minh
,
,
ng`
ng 20 b`
tuc
ay hoc
ay phai giai du
ai tp
ay.
. mt
. s ng`
. sinh n`
. mi

1.2. V du.

,
,
,,
`,i giai. Ch
giai
ung ta ky hiu
a s luo. ng b`
Lo
ai tp
a
. a1 l`
. , hoc
. sinh d
,,
giai trong hai ng`
trong ng`
ay d
a s luo. ng b`
ai tp
ay
`u tin, a2 l`
a
. d
,
,,
`

`
giai trong ba ng`
d
a s luo. ng b`
ai tp
d
ay d
a v.v. a77
u, a3 l`
a
u, v`
,.
,
,,

`
`
giai trong 77 ng`
l`
a s luo. ng b`
ai tp
ay d
a
u (11 tun). Theo gia
. d
,
,
thit a77 11.12 = 132. Ch
ung ta xet tp
ac s tu. nhin
. ho. p c
M = { a1 , a2 , a3 , . . . , a77 , a1 + 20, a2 + 20, a3 + 20, . . . , a77 + 20}. N
o
,,
,
,

chua 154 phn tu v`


a s lon nht trong ch
ung l`
a a77 + 20 152.
`ng nhau. Nhu,ng

Theo nguyn ly irichl trong M c


o t nht hai s ba
c
ac s a1 , a2 , a3 , . . . , a77 l`
a ho`
an to`
an kh
ac nhau. suy ra t`n tai
a
. ak v`
,
`
al m`
a ak = al + 20, l < k 77. Nhu vy
ay
iu n`
. ak al = 20, d
,
,
,
,
,

c
o ngha l`
a tu ng`
ay thu l + 1 d
ay thu k hoc
ay phai giai
n ng`
. sinh n`
ng 20 b`
d
ai.
u
,
, ,
,
,,
. 1.8. Trong mt
. khu tp
. th s,ng 123 ngu`oi. Tng s tui cua ho. ,
,,
,
,,
l`
a 3813. Chung minh ra
`ng c
o th chon
100 ngu`oi sng o khu tp
. th
.
,
, ,
, ,

n`
ay, m`
a tng s tui cua ho. khng nho hon 3100.

,
,
, ,,
,,
`,i giai. Ch
Lo
ung ta h
ay chon
100 ngu`oi nhi`u tui nht v`
a gia su
.
,
, ,
, ,
,, ,
ngu`oi tre nht trong s
tng s tui cua ho. nho hon 3100. Khi d
o
,
,, ,,
,,
. t kh
ngu`oi d
l`
a 3100:100=31 tui. Ma
ac ngu`oi n`
ay khng
uo. c chon
.
, ,
,
, ,
,`,
tng s tui cua 23
tre hon 23 nguoi c`
on lai
ach chon.
o
. theo c
. Khi d
,
, ,
,
,,
,n ho,n 23.31=713. Suy ra tng s tui cua tt ca
ngu`oi n`
ay khng lo
, , ,
,,
d
moi
n
. ngu`oi sng trong tp
. th nho hon 3100+713=3813 dn
v ly.
, ,
,
. 1.9. Nam ca. p vo., ch`ng t chuc mt
. p ma
. t. Khi ga
. p nhau ho.
. bui ga
,
,
,
,
ba
t tay nhau, nhung khng ai tu. ba
t tay ngu`oi trong gia d`nh m`nh
,`,
,
ba
v`
a nguoi m`
a ch`ng m`nh (hoa
. c vo. m`nh) da
t tay r`i. C
ung khng
,
,
,
` ng ban d`u,
ai ba
t tay c`
ung mt
. p ch
uc mu
. ngu`oi hai l`n., Sau cu,c
. ga
, ,
,`,
,
,
,
ng ngu`oi c
`n ng tn l`
mt
a H`
ung hoi tt ca nhu
o ma
. t, k ca
. nguoi da

10

,,
Chuong 1. Nguyn ly irichl v`
a v du.

,,
,
ba
a ho. da
t tay duo. c bao nhiu l`n. Ho. nhn
vo. m`nh, l`
`ng chn
. thy ra
, ,
,
,
,`, , , , `
,
nguoi duo. c hoi d
ac con s kh
ac nhau. Nhu vy
ung
u tra l`oi c
. vo. cua H`

`
ba
da
t tay bao nhiu ln?
,
, ,
,,
mt
t tay khng qu
`,i giai. Mi
Lo
ngu`oi kh
ach ba
a 8 l`n. V` cu tra l`oi
.
,
,
,,
phai l`
cua 9 ngu`oi l`
a c
ac s kh
ac nhau nn c
ac s d
a 0,1,2,3,4,5,6,7
o
,
,
,`,
,,
,
t tay 8 l`n phai l`
. c ch`ng) cua ngu`oi khng
v`
a 8. Nguoi ba
a vo. (hoa
,,
,,
t tay 8 l`n m`
t tay l`n n`
khng ba
a
ba
ao (nu nguo. c lai
o
. th` ngu`oi d
,
,
,
,
,
,
,
t tay 7 l`n
nhi`u nht ch l`
a 7 l`n thi). Tuong tu. nhu vy
. ngu`oi ba
,`, ,
,
,
t tay mt
t tay 6 l`n c
. c ch`ng) ba
c
o nguoi vo. (hoa
o
. l`n, ngu`oi ba
,`, ,
,
,
,
,
,
t tay 2 l`n, ngu`oi ba
t tay 5 l`n c
. c ch`ng) ba
nguoi vo. (hoa
o ngu`oi vo.
,
,,
t tay 3 l`n. Ch c`
t tay
. c ch`ng) ba
(hoa
on lai
. mt
. ngu`oi duy nht ba
,
,
,
,
chnh l`
4 l`n, do
a ngu`oi vo. cua H`
ung.
,
,
,,
. 1.10. Mt
`ng: Mt
ung vuong
. cu chuyn
. c tch k lai
. ra
. l`n vua H`
,
18 c
o m`oi c
ac quan trong tri`u hop
mt
ai b`
an tr`
on. Theo
. ng`i quanh
. c
,
,
,,

`n d
vit tn cua mi quan trn b`
lnh
cua vua, mt
an truoc
a
.
. cn
. th
,
,,
ac quan trong tri`u khng duo. c b
ao
chic gh m`
a ng ta phai ng`i. C
,,

ng`i khng theo sa


dinh
truoc nn ho. da
p xp da
a chim ch mt
. ,m`
.
,
`

`
c
ach bt k`y. Chung minh ra
ng ng cn
o th quay chic b`
an sao
. thn c
,
ng vi. tr tn cua m`nh ?
cho t nht c
o hai ng quan ng`i du

,
,,
`,i giai. a
. t s luo. ng c
. t b`
ma
Lo
ac quan l`
a n. Khi d
an c
o n trang
ai,
o
. th
,
,
,
i c
i c
.
vo
ac trang
th
ai n`
ay d
ac quan l`
a bin d
ao d
i din
` tn n`
o
.
. vo
,
,
mt
i mi
Ngo`
ai ra vo
o mt
trang
th
ai, m`
a khi ng`i
. ng quan ch c
.
,.
,
,
n.
i chnh tn cua m`nh trn bin d
ng th` ng y d
d
u
i din
` sa
. vo
,
,
,
,
trang
Ngha l`
a, nu mi
th
ai cua b`
an (v` b`
an c
o th xoay d
uo. c) ta
.
, , ,
,
,
,
`ng s luo. ng c
i mt
ng vi. tr
cho tuong ung vo
s ba
ac quan ng`i d
u
,
,.
,
,
,
,
tn m`nh, th` tng cua tt ca nhung s nhn
ai
uo. c (moi
. d
. trang
. , th
, ,
,
p
b`
an) se khng nho hon n. Nhung mt
th
ai d`u tin cua sa
. trang
.

1.3. B`
ai tp
.

11

, , ,
). Nu gia, su,,
,i 0 (khng ai ng`i d
ng ch
ng vo
xp b`
an cho tuong u
u
,
, , ,
,i s nho ho,n 2
. t b`
trong n 1 trang
th
ai ma
an c`
on lai
tuong u
ng vo
.
.
,
,
,
, ,
,,
,c l`
. c 0), th` tng cua n s nhn
(tu
a ch c
o s 1 hoa
d
uo. c se nho hon
.
, ,,
,
. t b`
khng th d
n, d
th
ai ma
an c`
on
i`u d
o
uo. c. Suy ra t`u n 1 trang
.
,`,
,

i chnh tn
lai
o t nht mt
th
ai m`
a hai nguoi se d
i din
. vo
., c
. trang
.
cua m`nh.

` tp
1.3. Bai
.
. 1.11. Trong sn cung din
a vua hi
. nh`
. hop
. 2n(n 2) ng quan,
,

,ng
. t tai
quen bit khng t hon n ng c
. Chu
mi ng quan d
o ma
a
o
. d
,
,,
,,
b`
`ng ngu,`o,i xp b`
minh ra
an tr`
on c
o th xp d
an 4 ngu`oi sao
uo. c mi
,
ngu,`o,i d
,ng giu,a hai ngu,`o,i quen cua m`nh.
cho mi
u
,
chi`u 1km.
. 1.12. Mt
o dang
h`nh vung mi
. khu r`ung thng c
.
,
,
,
Trong r`ung c
o 4500 cy thng, cy to nht c
o d
u`ong knh 0,5m.
,
,
,
`

`
ng trong khu rung c
Chung minh ra
o t nht 60 manh d
t, din
. tch
,
2

mi manh 200m , khng c


o mt
ao.
. cy thng n`
,a
. 1.13. Trong mt
a s
ach c
o 25 ng
an. Ta thy c
o mt
an chu
. gi
. ng
,
`ng
,a s s
,ng minh ra
10 cun, c`
on c
ac ng
an kh
ac chu
ach t hon. Chu
, ,
,
,
,
a c`
c
o t nht ba ng
an s
ach chu
ung s s
ach nhu nhau (k ca nhung
ng
an khng c
o s
ach).
,
,
,,
`ng t ho.,p
nh s ba
. 1.14. Tai
anh ph bin xe t d
uo. c d
a
. mt
. th`
,
,,
`ng trn mt
,ng minh ra
,
chu c
ai r`i d
ay s. Chu
n d
oan
u`ong cu
. d
. d
,
,
c
o 11 chic t d
ung c
o hai chic t c
o c`
ung chu
i qua th` bao gi`o c
s tn
ung.
. c`
,
,,
,,
,n d
. 1.15. Mt
ong
uo. c boc
. chuyn tip s
. chic h` lo
. boi 4 tram
,
,
,
,
thng tin. Giua hai tram
dung c
ac trung tm ph
at s
ong
. ngu`oi ta xy
, , .
,,
,`,
,
,
n nht l`
v`
a nhn
ong, d
ong bao phu lo
ad
on c
o tm o
uong s
u`ong tr`
. s

12

,,
Chuong 1. Nguyn ly irichl v`
a v du.

`ng vo
,ng minh ra
,i bn trung tm
trung tm v`
a di qua hai tram.
Chu
.
,
,,
, ,
,
,,
. p tram
. t h` se d
o c
ac d
an b. ma
oan
uo. c phu
. giua cua t`ung ca
. th` to`
s
ong thng tin.

, ,
CHUONG

2
HOC
S
.

,
chia s tu. nhin
2.1. Phep
,
Trong c
ac phep tnh trn s nguyn: cng,
tr`u, nhn, chia, th`
.
,
. c bit.
phep chia l`
a rt d
o h`
ang loat
a tt ca
a
. Phep chia c
. tnh cht m`
,
c
ac phep tnh c`
on lai
o. V du. c
ac phep to
an d
`u thu. c hin
. khng c
.
,
,
,
,
,
,

i s 0 d
vo
uo. c, nhung ring phep chia cho s 0 th` khng d
uo. c. Phep
,
,
,
i phep chia cho 0. Vo
i c
. c bit
chia khng ch d
ac phep tnh cng,
a
. vo
.
,
,
,

i phep chia th`


tr`u, nhn trn s nguyn cho ta s nguyn, nhung vo
,

khng c`
ng v` khng phai l
tnh cht d
on d
uc n`
ao ta c
ung nhn
o
u
.
,
,,
,
,
d
a
uo. c s nguyn sau phep chia. Nh`o nhung di. bit
. cua phep chia m`
,
,
,

`
n mt
trong to
an hoc
. xy du. ng ha
. ly thuyt v phep chia nhung s
,
,,
nguyn. Nhung v du. v`
a b`
ai tp
ay c
o lin quan mt
. chuong n`
. thit
,
c lai
giua phep chia v`
a nguyn ly irichl, nn ch
ung ta nha
inh
.
. d
ngha phep chia:
,ng s nguyn, vo,i b > 0. Ch
`ng a
Cho a v`
a b l`
a nhu
ung ta n
oi ra
chia ht cho b, ky hiu
a b| a, khi t`n tai
. l`
. mt
. s nguyn q sao cho
,
,
c sau d
ng thu
ng a = bq.
d
a
u
,
,,
,, ,
. c b l`
Ch
ung ta thu`ong goi
s a l`
a bi
cua b, hoa
a uoc cua a. S q
.
.
,
,
,,
goi
a thuong s cua phep chia a cho b. Trong ph
at biu d
inh
. ngha
. l`
,
`ng a
trn, nu khng t`n tai
ao ca, th` ch
ung ta n
oi ra
. mt
. s q n`

14

,,
Chuong 2. S hoc
.

khng chia ht cho b v`


a ky hiu
a b 6 | a.
. l`
,
,,
,ng minh d
T`u dinh
ung ta d d`
ang chu
ac tnh cht
uo. c c
. ngha ch
sau
,i moi
1) Vo
ung ta c
o a| a, Phep chia ht c
o
. s nguyn a > 0 ch
,
tnh phan xa.
.
c c`u.

2) Nu b| a v`
a a|c th` b|c- phep chia ht c
o t nh ba
3) Nu b| a v`
a b|c, th` b|( ac).
,
4) Nu a, b, m, n l`
a nhung s nguyn v`
a nu c| a v`
a c|b, th`
c|(ma + nb).
,
inh
ly sau d
o quan trong
cho phep chia mt
y giu vai tr`
.
.
. s
nguyn cho mt
. s nguyn.
,
i hai s nguyn bt k`
Vo
y a v`
a b sao cho b > 0, t`n tai
. duy
,
,

ng s nguyn q v`
nht nhu
a r thoa m
an a = bq + r v`
a 0 r < b.
,
,
,
C`
on rt nhi`u tnh cht kh
ac cua s nguyn c
ung nhu s thu. c
,
,
,,
nhung ch
ung ta khng d
ng n`
ay, m`
a ch d`
ung c
ac tnh cht
i theo huo
,
, ,
cua s hoc
a Nguyn ly irichl d
ac b`
ai to
an.
giai c
. v`

2.2. Vdu.
,
,
,
. 2.1. Cho k l`a mt
a tp
. s tu. nhin, A l`
. ho. p g`m k + 1 s tu. nhin.
,,
,
Chung minh ra
`ng c
o t nht mt
. hiu
. hai ph`n tu trong A chia ht
cho k.
,
,, ,
`,i giai. Goi
Lo
a c
ac ph`n tu cua A, c`
on b1 , b2 , . . . , bk+1
. a1 , a2 , ,. . . , ak+1 l`
,
,
a1 =
l`
a nhung s du cua phep chia c
ac s trn cho k. Khi d
o
,
i c
kc1 + b1 , a2 = kc2 + b2 , . . . , ak+1 = kck+1 + bk+1 ,vo
ac s nguyn
c1 , c2 , . . . , ck+1 sao cho 0 b1 k 1, 0 b2 k 1, . . . , 0
,,
, , ,
,i s
bk+1 k 1. Mt
ng vo
. ph`n tu bt k`y as thuc
. A cho tuong u

2.2. Vdu.

15

,
,
,
,
,
,
ph`n tu,, cua
a B. Nhu vy,
mi
du bs cua n
o.Goi
ac s du l`
A
.
. tp
. ho. p c
,
,, ,
,,
, , ,
,
,
`
`

i mt
. t tuong ung vo
d
m tt ca c
ac
uo. c d
a
. phn tu cua tp
. ho. p B, g
,
,, ,
,
,
,
,

`
s nguyn t`u 0 d
n k 1. Nhung s luo. ng phn tu cua A theo gia
,,
thit l`
a k + 1, c`
on B c
o s luo. ng k. Theo nguyn ly irichl suy ra
,
,,
,
ngha l`
t`n tai
ac nhau cua A c
o c`
ung s du. i`u d
a,
o
. hai ph,`n tu kh
,
`

tn tai
ac nhau s v`
a t voi as = kcs + bs v`
a at = kct + bs
. hai ch s kh
,
,,
`
sau khi tru d
i cho nhau ta d
uo. c at as = k (ct cs ).

,
,
s khng
. 2.2. Cho A mt
. tp
. ho. p bt k`y g`m 101 s tu. nhin, mi
,
,
,
l
on hon 200. Chung minh ra
`ng trong A c
o t nht hai s m`
a mt
. s
n`
ay chia ht cho s kia.
,
,
, ,
,,
s a cua
`,i giai. Mi
,i k l`
i dang
a = 2 k b vo
a
Lo
A c
o th biu din duo
.
,
,

i mi s a thuc
s nguyn khng m, c`
on b l`
a mt
s le. Vo
. A cho
, . ,,
, , ,
,
,

`
i s b trong su. biu din o trn. Ba
ng c
tuong ung vo
ach n`
ay, mi
,
,
,,
,
,,
, , ,
,
,
,i mt
. t tuong u
ph`n tu a cua A d
ng vo
uo. c d
a
. ph`,n tu cua tp
. ho. p B
,
,,
,
,
,
g`m c
ac s le giua 1 v`
a 200. Nhung tp
ho. p B ch c
o 100 ph`n tu v`
.
,
,, ,
, ,
,n ho,n s ph`n tu, cua B. Ta c
p dung
vy
o th a
. s ph`n tu cua A lo
.
,,
`
u
kh
a
c
nhau
a
v`
a
a2
nguyn ly irichl, suy ra t`n tai
hai
ph
n
t
1
.
,
, , ,
,
,
i c`
thuc
a ch
ung tuong ung vo
ung mt
. A m`
. s cua tp
. ho. p B. Ngha
k
k
2
1
l`
a, a1 = 2 b, a2 = 2 b v`
a nu k1 < k2 , th` s a2 chia ht cho a1 .

,
,
s khng
. 2.3. Cho M l`a tp
a mi
. ho. p bt k`y g`m 75 s tu. nhin
. m`
,
,
,
,
,
,
s tu. nhin l nho ho,n hoa
l
on hon 100. Chung minh ra
`ng v
oi mi
. c
,, ,
ba
`ng 49 t`n tai
o hiu
a l.
. hai ph`n tu cua M c
. l`
,
,, ,
`,i giai. Goi
Lo
ac ph`n tu cua M l`
a x1 , x2 , . . . , x75 . Ky hiu
a tp
. c
. A l`
.
,
,
,,
,
,

ho. p c
ac s tu. nhin t`u 1 d

n
150.
V
o
i
m
i
s
1,
2,
3
.
.
.
,
75
cho
tu
o
ng

,,
,ng vo
,i c
u
ac s x1 , x2 , . . . , x75 , c`
on c
ac s 76, 77, 78, . . . 150 l`n luo. t
,ng vo
,i x1 + l, x2 + l, . . . , x75 + l. V` xm 100(m = 1, 2, . . . , 75) v`
u
a

16

,,
Chuong 2. S hoc
.

,
, , ,
ph`n tu,, cua
,i
ng vo
l 49 th` xm + l < 150. Suy ra mi
A tuong u
,, ,
,
,
,
mt
n 149. V` s ph`n
. ,ph`n tu cua B g`m nhung ,s tu. nhin t`u 1 d
,,
,,
,
,
n hon s ph`n tu cua B, theo nguyn ly irichl t`n tai
tu cua A lo
.
,
,,
, , ,
,
`
i c`
hai phn tu kh
ac nhau cua A, m`
a ch
ung tuong ung vo
ung mt
.
,
,, ,
,
,
,
`

`
i c
phn tu cua B. Nhung vo
ac gi
a tri. kh
ac nhau cua m tu 1 d
n 75
,
,,
, , ,
,

d
ac gi
a tri. kh
ac nhau cua x1 d
uo. c cho tuong ung voi c
n x75 trong
,
,
,,
,,
,
, , ,

,i
B. Tuong tu. c
ac gi
a tri. cua m o khoang 76 d
ng vo
n 150 tuong u
,
,
suy ra t`n tai
c
ac gi
a tri. kh
ac nhau trong khoang c`
on lai.
o
. T`u d
. xm
v`
a xn m`
a xm = xn + l, ngha l`
a xm xn = l.

,
,ng s tu., nhin v`
. 2.4. Cho k 1 v`a n 1 l`a nhu
a A l`
a tp
. ho. p g`m
,
,
,
s n`
(k 1)n + 1 s nguyn duo,ng, mi
ay d`u nho hon hoa
. c ba
`ng kn.
,
,
,
,
,
,
nhu,
Chung minh ra
`ng t nht c
o mt
o th biu din
. ph`n tu cua A c
,
,
,,
tng cua k ph`n tu trong A.
,
,
,
`,i giai. Vo
,i k = 1 b`
ng, ch
Lo
ai to
an hin nhin l`
a du
ung ta gia thit
,
`ng m n v`
k 2. Ky hiu
a s nho nht thuc
a
. m l`
. A. D thy ra
,
,
,
n hon m nhung khng
ng n m s thuc
t`n tai
a ch
ung lo
u
. d
. A m`
,,
vuo. t qu
a kn.
, ,
ng minh b`
chu
ai to
an ch
ung ta t`m hai s x v`
a y thuc
. A sao
,

thuc
cho x = y + (k 1)m; ngha l`
a biu din mt
ao d
o
. s n`
. A,
,
`

ng m. Ch
c
th`
anh tng k s hang
o k 1 s hang
o
. thuc
. A trong d
. ba
`

cn t`m s x thuc
a x > (k 1)m v`
a x (k 1)m thuc
. A m`
. A.
,
Tht
o kn (k 1)m =
. vy,
. trong khoang = ((k 1)m, kn] c
k (n m) + m s nguyn. V` k 2, nn (k 1)m m, theo nhn
.
xet ban d`u suy ra c
o nhi`u nht n m s trong khng thuc
A.
.
,a t nht s = k(n m) + m (n m) =
i`u n`
ay ngha l`
a A chu
,
(k 1)(n m) + m s. Nhung s n, v` (k 2)(n m) 0. Goi
.
,

a1 , a2 , . . . , as thuc
A,
v
o
i
(
k

1
)
m
<
a

kn,
i
=
1,
2,
.
.
.
,
s.
Khi
d
o
i
.

2.2. Vdu.

17

,
,
a nhung
nhung hiu
. a1 (k 1)m, a2 (,k 1)m, . . . , as (k 1)m l`
trong
s nguyn kh
ac nhau trong khoang [1, kn]. Nu mt
ao d
o
. s n`
ch
ung khng thuc
ung ta nhn
. A, th` theo nguyn ly irichl ch
.
,
,,
,

ng n 1 s trong khoang n`
d
ai A c
od
ay. Nhu
uo. c s n 1, v` ngo`
u
,
,c d
,ng minh s n. Suy ra t`n tai
,i bt d
ng thu
chu
vy
ai vo
a
a
. tr
. mt
.
hiu
. ai (k 1)m thuc
. A.

, ,
,
`, n + 1 s du,o,ng kh
. 2.5. Chung minh ra`ng tu
ac nhau nho hon 2n, c
o
,
,
,
,,
th chon
ung ba
`ng s thu ba.
. duo. c ba s sao cho tng hai s trong ch
,
,
`,i giai. Ky hiu
cho.
Lo
a nhung s d
a
. 0 < a1 < a2 < . . . < an+1 l`
Ch
ung ta xet c
ac hiu
a c
ac s
. , s a2 a1 , a3 a1 ,,. . . , an+1 a1 v`
,
a2 , a3 . . . , an+1 . V` tt ca c
ac s n`
ay d
ac s trn
`u nho hon 2n nn c
,
,
,
,,
`m trong khoang 1, 2, . . . , 2n 1. Nhu vy
ung ta se t`m d
ch na
uo. c
. ch
,,
,
,
`ng mt
, nht ba
, hai: ak a1 = al ,
mt
om thu
om thu
. s o nh
. s o nh
suy ra ak = a1 + al .

,
,
oi mt
o dang
. 2.6. Chung minh ra`ng v
. s bt k`y n t`n tai
. mt
. s c
.

111
. . 000} m`
a chia ht cho n.
| .{z
,
s
n chu
,
,
,
`,i giai. Ch
a nhung
Lo
ung ta xet nhung s 1, 11, 111, . . . , 111
. . 111} v`
| .{z
, s
n chu
,
cho g`m n ph`n
s du khi chia d
ay s trn cho n. V` d
ay s d
a
,,
,
,
,
,
ac nhau khi chia ch
ung cho n c
o s
tu, nn nhung s du duong kh
, ,
,,
luo. ng n 1. C
o th gia thit khng c
o mt
ao trong d
ay trn
. s n`
,
,
,,
,

chia ht cho n v` nu nguo. c lai


ai to
an d
a
uo. c giai. Khi d
o
. th` b`

se c
o hai s trong ch
ung, v du. 111
. . 111} v`
a 111
. . 111}, l > k,
| .{z
| .{z
,
s
, s
k chu
l chu
,
lk =
m`
a khi chia ch
ung cho n se cho c`
ung mt
o
. s du. Do d

18

,,
Chuong 2. S hoc
.

111
. . 000}
se chia ht cho n.
| .{z
, s 1, k chu
, s 0)
(l-k chu
,
,
,
. 2.7. Cho p l`a s nguyn t l
on hon 5. Chung minh ra
`ng t`n tai
. mt
.

s c
o dang
a chia ht cho p.
. 111 . . . 111 m`
,
`,i giai. Ta xet d
Lo
ay s 1, 11, 111, . . . , 111
. . . 1} . Nu trong d
ay trn
| {z
, s )
(p chu
, , ,
s vo
,i s
khng c
o s n`
ao chia ht cho p, th` ta cho tuong u
ng mi
, ,
, ,
,
du cua phep chia. Tp
ac s du ch c
o 1, 2, . . . , p 1 g`m p 1
. h,o. p c
,,
,
ph`n tu (v` 0 khng th c
o trong tp
ay). Nhung v` ch
ung ta c
op
. n`
,,

s o dang
o c`
ung
. , trn, nn theo nguyn ly irichl tn tai
. hai s c
,,
,
,

i m > n. Khi
l`
s du. Gia su c
ac s d
a 111
. . . 1} v`
a 111
. . . 1} vo
o
| {z
| {z
,
,
s )
s )
(m chu
( n chu

d
o
1

n
<
m

p.
V
y

.
. . . 1} =
111
. . 000}
111
. . . 1} 111
| {z
| {z
| .{z
,
,
,
s ) (n chu
s )
s 1, n chu
, s 0)
(m chu
(m-n chu
= 111
. . . 1} .10n
| {z
, s )
(m-n chu
Tch n`
ay chia ht cho p v` ( p, 10) = 1, suy ra

111
. . . 1}
| {z
, s 1)
(m-n chu
,
,
`m trong d
chia ht cho p v`
a n
o c
ung na
ay o trn. M`
a1 mn p
,
,

mu thun voi gia thit khng c


o s n`
ao trong d
ay chia ht cho p.

,
,
,
. 2.8. (` thi Olympic toan th gi
oi l`n thu 14) Cho M l`
a tp
. ho. p bt
,
,
,
,
s khng l
k`y g`m 10 s tu. nhin, mi
on hon 100. Chung minh ra
`ng
,
,
,
,,
,
t`n tai
a tng cua c
ac ph`n tu trong ch
ung
. hai tp
. ho. p con cua M m`
ba
`ng nhau.

2.2. Vdu.

19

,
,
,
`,i giai. C
,ng minh nu t`n tai
o th chu
hai tp
thoa m
Lo
an kt lun
.
.
.
,
,
,,
cua b`
ai to
an, th` ta c
o th chon
o c`
ung tnh cht
. duo. c hai tp
. con c
,
,
y nhung khng giao nhau. Tht
vy,
Cho X, Y l`
a hai tp
con cua M
.
.
.
,
,,
`ng nhau. Ch
c
o tng c
ac ph`n tu ba
ung ta ky hiu
ac ph`n
. X1 g`m c
,, ,
,,
,
,
tu cua X m`
a khng thuc
ac ph`n
. Y. Tuong tu. nhu vy
. Y1 g,`m c
,, ,
tu cua Y m`
a khng thuc
o r`
ang X1 v`
a Y1 c
o tng c
ac ph`n
. X. R
,,
,
,
`ng nhau m`
tu ba
a khng giao nhau. Goi
A l`
a tp
ho. p moi
tp
ho. p
.
.
.
.
,
,, ,
,,
cua
con khng rng
M. S luo. ng ph`n tu cua A l`
a 210 1 = 1023.
,
,, ,
,
,
Ch
ung ta xet tng S c
ac ph`n tu cua mt
o
. tp
. ho. p con nhu vy,
. r
,
`
r`
ang S 91 + 92 + + 100 < 10.100 = 1000. Nhu vy
. tn tai
.
,
, ,
khng qu
a 1000 tng kh
ac nhau. Ky hiu
a tp
ac
. B l`
. ho. p tt ca c
,
,
,, ,
,, ,
,
,,

`
s luo. ng phn tu cua B nho hon 1000 v`
tng nhu vy.
a nho
o
. Do d
,
,, ,
,,
, , ,
,
,
ph`n tu,, cua
. t tuong u
hon s luo. ng ph`n tu cua A. a
ng mi
tp
. ho. p
,
,
,
,,
`ng c
,i tng c
p dung
A vo
ac ph`n tu cua n
o. Ta thy ra
o th a
nguyn
.
,,
,
`

ly irichl o d
ac nhau c
o
y. Suy ra tn tai
. t nht hai tp
. ho. p con kh
,
,,
`
c`
ung mt
t
ng
c
a
c
ph
n
t
u
.
.
,
,
. 2.9. (` thi hoc
an Cp II to`
an quc 1983) Chung minh
. sinh gioi to
,
ra
`ng trong c
ac s tu. nhin th n`
ao c
ung c
o s k sao cho 1983k 1 chia
ht cho 105 .
,
,
,
`,i giai. Cho k ly gi
,c
Lo
a tri. t`u 1 d
ao biu thu
n 105 + 1 r`i thay v`
,,
1983k 1 se nhn
a tri. kh
ac nhau. Chia 105 + 1 s
uo. c 105 + 1 gi
. d
,
,
,,
,
,
5

v`ua nhn
ed
a 105 s du. Do d
uo. c nhi`u nht l`
o
. o trn cho 10 , s
,
,
theo nguyn ly irichl phai c
o t nht hai s cho c`
ung mt
. s du.
, ,,
l`
Gia su d
a s 1983m 1 v`
a 1983n 1(m > n). Th th` (1983m
o
1) (1983n 1) chia ht cho 105 m`
a (1983m 1) (1983n 1) =
,
(1983m 1983n ) = 1983n (1983mn 1). Nhung 1983 v`a 105 nguyn
,
t c`
ung nhau, do vy
o (1983mn 1) chia ht cho 105 . S
. phai c
,
k = m n thoa m
an d
ai.
i`u kin
. d`u b`

20

,,
Chuong 2. S hoc
.

,
,ng s nguyn a, b v`
. 2.10. Chung minh ra`ng t`n tai
a c, khng
. nhu
,
, `

dng th`oi ba
ng 0 v`
a gi
a tri. tuyt
a 1000000,
. di cua mi s khng qu

11
thoa m
an | a + b 2 + c 3| < 10 .

,
,
, ,
18
`,i giai. a
. t S l`
a tp
Lo
+ t 3 vo,i moi
. ho. p cua 10 s thu. c r + s 2
.

6
6
. t d = (1 + 2 + 3d)10 .
r, s, t thuc
a da
. {0, 1, 2, . . . , 10 1} v`
,

`
`
m trong khoang 0 x < d. Chia
mi x trong S d
Khi d
o
u na
,
d
`ng nhau, mi
d
ay th`
anh 1018 1 ph`n ba
od
ai
oan
oan
. d`
. n`
. nho c
d
18
e =
. Theo nguyn ly irichl t`n tai
. hai s trong 10 s
18 1
10
,
,
,
`m trong c`
cua S na
ung mt
ay ky hiu
a
oan
. d
. nho. Hiu
. cua hai s n`
. l`

107
chnh l`
a+b 2+c 3 d
a c
ac s a, b, c v` e < 18 = 1011 .
o
10

` tp
2.3. Bai
.
,
,
s khng
. 2.11. Cho A l`a tp
. ho. p bt k`y g`m 201 s tu. nhin, mi
,
,
,,
,
,
`ng A chu
ng minh ra
a t nht hai s, m`
vuo. t qu
a 300. Chu
a ty s cua
,
ch
ung l`
a l
uy th`ua bc
. ba.
,
,
. 2.12. Cho k l`a s tu. nhin bt k`y, c`on a v`a b l`a nhung s nguyn
,
`ng nu M l`
,ng minh ra
sao cho a b v`
a b a < 2k 2. Chu
a tp
ho. p
.
,
,
,
,
`m trong khoang [ a, b], v`
k s tu. nhin na
a l l`
a s tu. nhin thoa m
an
,, ,
,

`
1 l 2k + a b 2, th` c
o t nht mt
. hiu
. nhung phn tu cua
,
i l.
M tr`
ung vo
,,
s a1 , a2 , a3 , . . . , a41 , m`
ph`n tu,, ch, d
. 2.13. Cho dy
a mi
uo. c tao
.
, ,,
,,
,,

c
boi s 1 v`
a, s 2, trong d
o t nht 21 s ch d
b
o
i
c
a
c
s
1.
o
uo. c tao
.,
,
,,
,
`
ng minh ra
ng t`n tai
Chu
ay c
o tng
. mt
. s ph`n tu lin tip cua d
`
ng d
ng 20.
ba
u
,
`ng t`n tai
,ng minh ra
. 2.14. Chu
. mt
. s tu. nhin n, sao cho s

2.3. B`
ai tp
.

21

ng nu ta thay 139
111
. . . 1} chia ht cho 139. (B`
ai to
an c`
on d
u
| {z
, s )
(n chu
`ng mt
,i 10).
ba
ung nhau vo
. s nguyn t c`
,,
,
`ng trong moi
,ng minh ra
. 2.15. Chu
. s tao
. boi 100 chu s N t`n tai
.

mt
s
chia
h
t
cho
1967.
.
,,
`ng bao gi`o, c
,ng minh ra
. 2.16. Chu
ung t`m d
uo. c s
19971997. . . 19970. . . 0 chia ht cho 1998.
,
`ng c
,ng minh ra
. 2.17. Chu
o mt
s tu. nhin chia ht cho 1997, m`
a
.
,
,

bn chu s cui c`
ung cua n
o l`
a 1998.
`ng nu c
,ng minh ra
. 2.18. Chu
ac s nguyn m v`
a n nguyn t c`
ung
,, ,
k

nhau th` t`m d


uo. c s tu. nhin k sao cho m 1 chia ht cho n.

22

,,
Chuong 2. S hoc
.

, ,
CHUONG

S
DAY

y s v han
3.1. Nguyn l
y irichl cho da
.
,,
Trong ph`n n`
ay ch
ung ta xet nguyn ly irichl duo
i dang:
.
,
,
. t v han
ung ng
an k
eo m`
a ch
ung ta d
Nu c
o huu han
a
. nh
.
,
,

, th` t nht c
ao d
o mt
an k
eo chua v han
nhung vt
o
. v`
. ng
.
,

c
o.
nhung vt
a
. d
,
,
, ,,
`ng nguyn ly n`
Ch
ung ta d c
o cam tuong ra
ay l`
a hin nhin nn
,
, ,
`ng phan chu
,ng c
ng minh nguyn ly n`
t ch
o. Ba
o th chu
ay
u y d
n n
,,
,
ng. Trong s hoc,
l`
ad
o lin quan d
a
u
n v han
. tp
. ho. p c
. ph`n tu l`
,

dy s. Ch
ung ta bit rt nhi`u d
ay s d
ay cp s cng,
d
ay
ep
. nhu d
.
,
,
. c d
cp s nhn, d
ay c
ac s nguyn t, hoa
ay Fibonaxi,. . . Chuong
,
,
, ,
p dung
n`
ay ch
ung ta ch quan tm d
d
at biu trn d
n a
i`u ph
giai
.
s. Nhu,ng tp
c
ac b`
ai to
an lin quan d
ac b`
ai
n dy
. v han
. trong c
,,
,
to
an duoi d
ac d
ay s.
y ta xet nhu c

3.2. V du.
, s cui c`
. 3.1. Xet day s 6, 62 , 63 , 64 , 65 , . . . , 6n , . . . v`a vit 4 chu
ung
,
,
`

cua c
ac s n`
ay 0006, 0036, 0216, 1296, 7776, . . .. Chung minh ra
ng ba
t
,
,
vu
tu`n ho`
` mt
` a lp
dy
d`u tu
ao do
a dy
an.
. s n0 n`
. l`
,
,
,,
4
`,i giai. V` t`n tai
Lo
ach chon
ac nhau
. huu han
. s luo. ng (10 ) c
. kh

24

,,
Chuong 3. D
ay s

,
,,
c cha
n t`m d
cho cha
ay d
c
ac s c
o 4 chu s, nn trong d
a
uo. c hai
,
,,
c
ach chon
o c`
ung 4 chu s cui. C
o ngha l`
a t`m d
uo. c hai s n0
. c
,
,i ch
v`
a n0 + t m`
a vo
ung th` 6n0 v`
a 6n0 +t+1 c
o c`
ung 4 chu s cui
,
(6n0 +t+1 6n0 = 104 .6k). Noi chung, chu s 6n v`a 6n+t vo,i bt k`y
,
n > n0 se c
o c`
ung 4 chu s cui (6n+t 6n = 104 .6nn0 ).

. 3.2. (` thi Toan Olympic quc t l`n 17 nam 1975) Cho


,
,
a1 , a2 , . . . , an , . . . l`
a d
ay t
ang nga
. t c
ac s tu. nhin. Chung minh ra
`ng
,
, ,
,,
,
,
trn c
duoi dang
v han
ac ph`n tu an cua dy
o th biu din
.
. c
,,
,
,
,
ng s nguyn duong v`
a y l`
a nhu
a p 6= q.
an = xa p + yaq ,o dy x v`
,
,
,
`,i giai. Nu a1 = 1 kt lun
Lo
ai to
an l`
a hin nhin. Tht
. cua ,b`
. vy,
.
,

i moi
vo
n

3
s
h
ang
a
c
o
bi
u
di
n
d
ang
a
=
a
+
(
a
+
n
n
n
n 1
.
.
.
an1 ) = 1.an1 + ( an an1 ).a1 c
o tnh cht mong mun. Ch
ung ta
,
,
,
,
ng minh t`n tai
n hon 1 sao cho v han
se chu
ac s hang
. ,ch s p lo
. c
.
,
,
,
,

i dang

cho c
cua d
ay d
o th vit duo
xa
+
ya
v
o
i
c
a
c
s
nguyn
a
p
1
.
,
,,
, , ,
,
s hang
,i
. t tuong u
duong thch ho. p x v`
a y. Mi
cua d
ay ta d
ng vo
a
.
,
,
,
,
s du cua n
o khi chia chnh n
o cho a1 . Tp
ac s hang
. ,ho. p tt ca c
.
,
,
,
,

cua d
ay l`
a v han,
c`
o
n
t
t
c
a
c
a
c
kh
a
n
a
ng
c
ua
s
du
khi
chia
c
a
c
s
.
,
,,
,
,
`
ng to ra
ng v han
chu
hang
a huu han.
o
. cho a1 l`
. i`u d
. ph`n tu
,i n1 < n2 < . . . < nk < . . .
an1 , an2 , . . . , ank , . . . , vo
,
,
cho c`
ung mt
s du r khi chia cho a1 . Khng mt tnh tng qu
at
.
,
,
,
,
,
,
ta gia thit n1 > 1, v` trong tru`ong ho. p nguo. c lai
ac s
. ta xet c
,
an2 , an3 , . . . , ank , . . . c
ung l`
a d
ay v han
a cho c`
ung s du r khi chia
. v`
,
,
,
i moi
cho a1 . Vo
. k = 1, 2, . . . t`n tai
. s nguyn duong xk sao cho ank =
a n k a n1 = ( x k a 1 + r ) ( x 1 a 1 + r ) = ( x k x 1 ) a 1
xk a1 + r. Khi d
o
,
,
,c ank = an1 + ( xk x1 ) a1 =
i moi
ng thu
suy ra vo
od
a
. k 2 ta c
,
,
1.an1 + ( xk x1 ) a1 . Ngha l`
a, nhung s an2 , an3 , . . . , ank , . . . ., c
o biu
,
,
,
`i hoi. Tht
din v`
a c
ac tnh cht nhu b`
ai to
an d
a n1
o
. vy,
. ch s 1 v`

3.2. V du.

25

,
,
, , ,
,
kh
ac nhau v` theo c
ach chon
n hon 1. Ch c`
on phai
. trn n1 thu. c su. lo
,
,,
`ng s xk x1 l`
,i k 2, d
ng dinh

kha
a s nguyn duong vo
i`u d
o
. ra
,
ng v` t`u n1 < nk suy ra x1 < xk .
d
u
,
,
. 3.3. Cho s tu. nhin bt k`y k. Chung minh ra`ng t`n tai
t
. s nguyn
,
,
p v`
a mt
ay s tu. nhin t
ang nga
. t a1 , a2 , . . . , an , . . . sao cho tt ca c
ac
. d
,, ,
,
`

ng s nguyn
phn tu cua d
ay p + ka1 , p + ka2 , . . . , p + kan , . . . l`
a nhu

t.
,
,
,
`,i giai. Ky hiu
,i moi
Lo
a tp
ac s nguyn t. Vo
. P l`
. ho. p tt ca c
.
,

i = 0, 1, . . . , k 1 ky hiu
P
l`
a
t
p
h
o
p
c
a
c
s
nguyn
t
m`
a
khi
i
.
.
.
,
`ng moi
`m trong mt
chia cho k c
o s du i. D thy ra
. s nguyn t na
.
,,
,

trong c
ac tp
h
o
p
P
,
P
,
P
,
.
.
.
,
P
.
B
o
i
v`

s
nguyn
t
l`
a
v
h
an,
0 1 2
k 1
. ,.
.
,

,a

vy
t
nh
t
ph
ai
c
o
m
t
trong
s
c
a
c
t
p
h
o
p
P
,
P
,
P
,
.
.
.
,
P
ch
u
k 1
.
. ,
.
. 0 1 2
,,
,a v han
v han
t. Gia su Pi chu
a ky hiu
a ph`n
. s nguyn
. s v`
. p l`
,
,, ,
,i
moi
tu nho nht cua n
o. Khi d
o dang
o
. s x thuc
. Pi c
. ,x = p + ka vo
,
,
,
mt
a c
ac ph`n tu cua Pi xp theo
. s tu. nhin a. Ly x1 , x2 , x3 , . . . l`
xn p
,
`ng
, tu., lo
,n d`n. Vo
,i moi
. t an =
thu
. D thy ra
a
. s tu. nhin n d
k
s nguyn t p v`
a d
ay a1 , a2 , . . . , an , . . . c
o tnh cht mong mun.
,
,
. 3.4. Cho f l`a da thuc k di s v
oi h. s nguyn v`
a a1 , a2 , . . . , an , . . . l`
a
,
,
,

ng s nguyn thoa m
d
ay nhu
an h. thuc an+1 = f ( a, a2 , . . . , ank+1 ),
,,
,

v
oi moi
a n k. Ch
ung ta xet s duong bt k`y m
. s nguyn n, k m`
,
,
,
,
v`
a v
oi moi
a s du khng m nho nht cua
. n = 1, 2, . . .. Ky hiu
. an l`
,
an theo m dun m. Chung minh ra
`ng d
ay a1 , a2 , a3 , . . . , an , . . . . l`
a d
ay
`
tun ho`
an.
,
,
,,
`,i giai. Ch
,c
ng d
Lo
ung ta se su dung
kha
sau: Nu g l`
a d
inh
a thu
.
.
,i h. s nguyn v`
k d
a x1 , x2 , . . . , xk , y1 , y2 , . . . , yk l`
a c
ac s
i s vo
nguyn sao cho x1 y1 (mod m), x2 y2 (mod m), . . . , xk yk
(mod m), th` g( x1 , x2 , . . . , xk ) g(y1 , y2 , . . . , yk ) (mod m).

,,
Chuong 3. D
ay s

26

,
`ng mt
Moi
cua d
ay ba
ac s a1 , a1 , a3 , . . . , an , . . .
. s hang
.
. trong c
,
,
p thu
tu., g`m k ph`n tu,
0, 1, . . . , m 1. Ch
ung ta xet c
ac b. sa

( a1 , a2 , . . . , ak ), ( a2 , a3 , . . . , ak+1 ), . . . , ( an , an+1 . . . , an+k1 )..


,
,
,,
p nhu, vy,
C
o tt ca v han
ac b. k s
. b. sa
. nhung s luo. ng c
,
,
(1 , 2 , 3 , . . . , k ), voi 0 i m 1, i = 1, 2 . . . , k l`a huu han
.
,
`ng mk theo ly thuyt t ho.,p). Theo nguyn ly irichl t`n tai
(ba
hai
.
,
ch s i v`
a j, i < j sao cho
a i = a j , a i +1 = a j +1 , . . . , a i + k 1 = a j + k 1
. c l`
hoa
a
x1 y1 (mod m), x2 y2 (mod m) , . . . , xk yk (mod m).
,
,
T`u d
ay a1 , a2 , a3 , . . . , an . . . l`
a tu`n ho`
an (chu k`y cua n
o
y suy ra d
,, ,
,
,
c vo
i h. s nguyn nn
l`
a uoc s cua j i). Tht
ad
a thu
. vy,
. v` f l`
,
ng minh trn ch
theo c
ach chu
ung ta c
o
f ( ai+k1 , ai+k2 , . . . , ai ) f ( a j+k1 , a j+k2 , . . . , a j ) (mod m)
,
= ai+k a j+k (mod m) hoa. c l`a ai+k a j+k . Bin di mt
ut d
. ch
,
,
,
`ng vo
c sau an+(i j) an .
i moi
ng thu
thy ra
od
a
. n i ta c

,
,
. 3.5. Cho day x1 , x2 , . . . , xn , . . . . duo.,c xac dinh
. theo cng thuc sau
x1 = 1, x2 = 0, x3 = 2, xn+1 = 2xn1 + xn2 , n 3.
,,
,
,
,
Chung minh ra
`ng v
oi moi
. s tu. nhin m t`n tai
. hai ph`n tu lin tip
,
cua d
ay m`
a ch
ung d`u chia ht cho m.
,
,
`,i giai. Cng thu
,c h`i quy trn c
Lo
o th vit lai
.
xn2 = xn+1 2xn1
(3.1)
,
,
,
,
`ng d
,c
ch ra ra
ay c
o kha n
ang ph
at trin v` pha tr
ai, tu
T`u do
,i n 0. V du. vo
,i n = 2, 1, 0 ch
l`
a x
ac d
xn vo
ung ta nhn
inh
.
.
,
,,
,
`

d
u
o
c
x
=
0,
x
=
0,
x
=
1.
Nhu
m
uc
3.4
ch

ra
r
a
ng
d
a
y
.
0
2
1
.

3.2. V du.

27

,
, , , ,
ng x1 , x2 , . . . , xn , . . . theo
x1 , x2 , . . . , xn , . . . g`m nhung s du tuong u
,

`
,c (3.1) suy ra mi
mdd un m, l`
a d
ay tun ho`
an. T`u cng thu
,
,, ,
,
ph`n tu cua d
ay { xn } v`
a suy ra ca { xn } x
ac d
duy nht t`u 3
inh
.
,
,, , ,
c n
nu (r1 , r2 , . . . , rk ) l`
ph`n tu truo
o. Khi d
a ph`n chu k`y cua d
ay
o
,
`
`
`
x1 , x2 , . . . , xn , . . . th` phn n`
ay se chuyn d
an v pha tr
ai
ng
. tun ho`
,
cua d
ay . . . , x3 , x2 , x1 , x0 , x1 , x2 , . . . v`
a se c
o dang
.
. . . , r1 , r2 , . . . , r k , r1 , r2 , . . . , r k , r1 , r2 , . . . , r k . . .

(3.2)

,
`ng x1 = x0 = 0, suy ra x1 = x0 = 0. T`u, (3.2)
By gi`o ta ch
u y ra
,,
,
`ng d
,a v s ca
. p ph`n tu
suy ra ra
ay c
ac s du theo mdd un m chu
`ng khng. N
. p s lin tip
lin tip ba
oi c
ach kh
ac t`n tai
ac ca
. v s c
,
,,

. p d
cua d
ay x1 , x2 , . . . , xn , . . . m`
a mi ph`n tu trong ca
`u chia ht cho
m.
,
,
,
ngha ba
`ng c
ac da
ng thuc F1 =
. 3.6. Day s Fibonaxi duo.,c dinh
.
,
F2 = 1, Fn+2 = Fn+1 + Fn , n 1. Chung minh ra
`ng t nht mt
. trong
,
,,
1.000.000.000 ph`n tu d`u tin cua d
ay chia ht cho 10.000.

,
, ,
`,i giai. Tu,o,ng tu., nhu, 3.5 ch
Lo
ung ta xet c
ac s du cua c
ac s
,
,
,
ng o, vi.
cho khi chia cho 10.000. Ky hiu
trong d
ay d
a
u
. s du d
, k khi chia cho 10 000 l`
th` r1 = 1, r2 =
tr thu
a rk . Khi d
o
1, r3 = 2, r4 = 3, . . . . . . .rk = rk1 + rk2 . R
o r`
ang c
o 10.000 s
,
2
. p s
c
du kh
ac nhau do d
o 10000 = 100000000 (tr
am triu)
ca
o
.
,
,
. p s du (r1 , r2 ), (r2 , r3 ), (r3 , r4 ) . . .
du kh
ac nhau. Xet 100000001 ca
. . . (r100000001 , r100000002 ). Theo nguyn ly irichl trong s n`
ay c
o t
,
,
,
,
c l`
i n, p
p s tr`
nht 2 ca
ung nhau, tu
a t`m d
a p vo
uo. c hai s n v`
, . ,
, ,
d
`u nho hon 100000002,n nho hon p sao cho rn = r p , rn+1 = r p+1 .
,
,
, ,
, ,
Nhung nu bit s du cua tng hai s v`
a s du cua mt
. s th` s
,
,,
du kia c
ung tnh d
o r n 1 = r p 1 , r n 2 = r p 2 , . . . .
uo. c. V` vy
. ta c
,c
cho d
cng thu
n khi r2 = 1 = r pn+2 , r1 = 1 = r pn+1 , p dung
.

,,
Chuong 3. D
ay s

28

,,
,
,i p n 100000001 1 =
s du h`i qui o trn ta c
o r pn = 0 vo
,
,
,ng o, vi. tr p n se thoa m
100000000. Ngha l`
a s d
an d
ai
u
i`u kin
. b`

ra, chia ht cho 10 000.

. 3.7. (inh
ly Fecma) Nu mt
.
. s nguyn t p khng chia ht s
,
,
ng a p1 1( mod p).
nguyn a, th` da
ng thuc sau du
,
,
,
`,i giai. Ch
,ng minh mnh
Lo
ung ta chu
d
at hon. Cho m > 1
` tng qu
.
,
,i m. Ch
l`
a s tu. nhin bt k`y v`
a a l`
a s nguyn t c`
ung nhau vo
ung
,
,
,
ta xet d
ay nhung l
uy th`ua lin tip cua a
a1 , a2 , a3 , . . .

(3.3)

v`
a ky hiu
.
r ,r ,r ,...
(3.4)
,1 2 3
,
, , , ,
l`
a nhung s du tuong u
ng cua (3.3) khi chia cho m, ngha l`
a
ak rk

(mod m), 1 rk m 1.

,,
,,
,
s luo. ng c
Khi d
ac s trong (3.3) l`
a v han,
c`
on nhung s o (3.4)
o
.
,
,
,
,,
ch c
o th nhn
a tri. trong 1, 2, 3, . . . , m 1 nn s luo. ng l`
a
. nhung gi
,
,
,
,

huu han.
du rk se c
o
. Suy ra theo nguyn ly irichl, giua nhung s
,

,i
t nht hai s tr`
ung nhau; n
oi c
ach kh
ac tn tai
a j vo
. hai ch s i v`
,
ch
i 6= j sao cho ri = r j . Khi d
ung ta c
o ai a j ( mod m). Theo gia
o
,,
,i i 6= j ch
thit ( a, m) = 1, vo
ung ta nhn
uo. c ai j 1( mod m).
. d
,
,
,c sau
ng thu
Ch
ung ta c
o kt lun
a
. t`n tai
. s tu. nhin l sao cho d
ng:
d
y d
u
al 1 (mod m)
(3.5)
- S l trong (3.5) khng x
ac d
on t`n tai
inh
. duy nht, thm
. ch c`
.
,
,

v s s tu. nhin l thoa m


an (3.5).
,
,`,
,
- Trong truong ho. p m = p l`
a s nguyn t, Fecma t`m ra l c
o th
chon
a s p 1.
. l`

3.2. V du.

29

,
,
,,
,
`ng l c
,ng minh ra
o th chon
-Tru`ong ho. p m bt k`y th` Ole chu
a
. l`
,
,
,,

`
h`
am ch s cua m (ch
ung ta khng xem xet vn d
ay o d
n`
y, d
c
.
,
,
,

cun s
gia c
o th t`m trong bt cu
ach s hoc
ao).
. n`

v han
. 3.8. Cho x1 , x2 , x3 , . . . l`a dy
ac s nguyn v`
a k l`
a mt
. c
. s
,
,
,
,
s g`m nhu
ng ph`n tu,
tu. nhin bt k`y. Chung minh ra
`ng t`n tai
dy
.
,
,
,
m`
lin tip cua dy,
a tng cua ch
ung chia ht cho k.
,
, ,
,,
,
`,i giai. Ch
i han
Lo
ung ta c
o th gio
lai,
giua moi
b. k ph`n tu lin
.
.
.
,
,
,,
,,
c
tip cua dy
o th chon
d
s ph`n tu c
o tnh cht mong
uo. c mt
.
.
, ,
,
,,
mun. d
on gian ta xem xet k ph`n tu d
`u tin x1 , x2 , x3 , . . . , xk .
,
Ch
ung ta xet tng
S1 = x 1 , S2 = x 1 + x 2 , S3 = x 1 + x 2 + x 3 , . . . , S k = x 1 + x 2 + + x k
,
,,
trong s trn chia ht cho k, th` b`
Nu mt
tng n`
ao d
ai to
an d
o
uo. c
.
,
,,
,,
giai. Nguo. c lai,
ac s S1 , S2 , . . . , Sk (c
o s luo. ng k) khi chia cho k
. c
,,
,
,
d
ac s du 1, 2, 3, . . . , k 1. T`u nguyn ly irichl suy ra c
o mt
uo. c c
.
,
,
. p ch s i v`
a j, 1 i < j k, m`
a c
ac tng Si v`
a S j cho c`
ung mt
s
ca
.
,
,,
,
tng c
du khi chia cho k. Khi d
ac ph`n tu lin tip xi+1 , xn+2 , . . . , x j
o
,
cho chia ht cho k, v` xi+1 + xn+2 + + x j = S j Si .
cua d
ay d
a

,
,
, s. Chu,ng minh ra
. 3.9. Cho day v han
ac chu
`ng v
oi moi
. c
. s tu.
,
nhin n, nguyn t c`
ung nhau v
oi 10, trong d
ay v han
. trn t`n tai
.
,,
,
,

mt
nh
o
m
ch
u
s
lin
ti
p,
m`
a
s
t
ao
b
o
i
c
a
c
ch
u
s
trong
nh
o
m
(vi
.
.
,
,
,
,
,
,
,

theo thu tu. ch s l


on dung truoc) chia ht cho n.
,
,
`,i giai. Cho d
ung ta xet c
ac s
Lo
ay c
ac chu s a1 , a2 , . . . , an , . . .. Ch
A 1 = a 1 , A 2 = a 2 a 1 , . . . , A n = a n a n 1 . . . a 1 , . . . , A n +1 = a n +1 . . . a 1 .
,
,
,,
,
,,
,
V` s luo. ng nhung s n`
ay l`
a n + 1, c`
on s luo. ng kha n
ang cua s du
khi chia ch
ung cho n l`
a n, nn theo nguyn ly irichl t`n tai
. t nht

,,
Chuong 3. D
ay s

30

,
ung l`
a Ai v`
a A j , (i < j). Khi
hai s cho c`
ung mt
. ch
. s du ta ky hiu

hiu
d
oi c
ach kh
ac
o
. A j Ai chia ht cho n. Hay n
A j Ai = a j . . . a1 ai . . . a1 = a j . . . ai1 .10 ji+1
v` (n, 10) = 1, nn a j . . . ai1 chia ht cho n.

. 3.10. Cho k l`a s nguyn du,o,ng bt k`y v`a


x1 , x2 , . . . , x n , . . .
y1 , y2 , . . . , y n , . . .
,

ng chui s nguyn bt k`y. Chu,ng minh ra


l`
a nhu
`ng t`n tai
. p
. v s ca
,
,
,

ch s (i, j), v
oi i < j sao cho mi tng
x i +1 + x n +2 + + x j ; y i +1 + y n +2 + + y j
d`u chia ht cho k.
,
,
,
`ng trong b. s k2 ph`n tu, lin tip
`,i giai. Ch c`n chu
,ng minh ra
Lo
,
,
,
,, ,
trn c
,i tnh cht d
ch ra. V` vy
cua 2 dy
o th chon
d
uo. c tng vo
a
.
.
,
,
,,
d
cho.
ac chui
ch
ung ta ch quan tm d
`u tin cua c
a
n k2 ph`n tu d
,
,
,
`ng c
Ba
ach tng qu
at h
oa c
ach giai b`
ai to
an 3.8, ly tng
S1 = x 1 , S2 = x 1 + x 2 , S3 = x 1 + x 2 + x 3 , . . . , S k 2 = x 1 + x 2 + + x k 2
T1 = y1 , T2 = y1 + y2 , T3 = y1 + y2 + y3 , . . . , Tk2 = y1 + y2 + + yk2
,,
m = 1, 2, 3, . . . , k2 d
,ng ca
,i mi
,i ca
. t tuong u
. p (Sm , Tm ) vo
. p
Vo
a
,
,
,
( RSm , RTm ) cua nhung s du, khi chia Sm v`a Tm cho k. V` RSm
, ,
,
v`
a RTm l`
a mt
ac s 0, 1, 2, . . . , k 1, nn t ho. p tt ca
. trong c
c
ac dang
ac nhau ( RSm , RTm ) l`
a khng qu
a k2 . Nu t`n tai
. , kh
,.
,

mt
ch

s
m,
sao
cho
(
RS
,
RT
)
tr`
u
ng
v
o
i
(
0,
0
)
,
th`

m
oi
t
ng
m
m
.
.
Sm = x1 + x2 + + xm v`
a Tm = y1 + y2 + + ym d
`u chia ht cho
,
. p s ( RSm , RTm ), m = 1, 2, . . . .., k2
k. V` nu khng nhu vy,
ac ca
. th` c
,
,
,,
c
o nhi`u nht l`
a k2 1 kha n
ang kh
ac nhau. Nhung s luo. ng

3.3. B`
ai tp
.

31

,
`ng
. p s n`
ay l`
a k2 suy ra c
o t nht hai trong ch
ung ba
nhung ca
,
nhau. N
oi c
ach kh
ac, t`n tai
a j, sao cho 1 i <
. hai ch s i v`
,,
,
2
s
j k v`
a ( RSi , RTi ) = ( RS j , RTj ). Trong tru`ong ho. p n`
ay mi
xi+1 + xn+2 + + x j = S j Si ; yi+1 + yn+2 + + y j = Tj Ti
d
`u chia ht cho k.
,
,,
: y l`
Ch
uy
a b`
ai to
an tng qu
at h
oa b`
ai to
an 3.8. Mo rng
kt
.
,
qua n`
ay c
ac ban
ay xem v`
a l`
am b`
ai tp 3.15.
. h

` tp
3.3. Bai
.

,
, ,
,
. 3.11. Co t`n tai
a c
ac chu s cui c`
ung cua n
o
. luy th`ua cua s 3 m`
l`
a 0001 khng ?
,
,
,,
. 3.12. Cho F l`a tp
a x1 , x2 , . . .
. huu han
. nhung s nguyn duong v`
,,
,

, xn , . . . v`
a y1 , y2 , . . . , yn , . . . l`
a hai dy v han
nhung ph`n tu thuc
.
.
,
,
`ng t`n tai

,ng minh ra

F. Chu
nh
u
ng
ch

s
i
v`
a
j,
i
<
j
sao
cho
t
ch
.
,
,
cua xi+1 , xi+2 , . . . , x j v`
a yi+1 , yi+2 , . . . , y j l`
a mt
o l
uy th`ua bc
. s c
. k.
,
`ng
. 3.13. Cho u1 , u2 , . . . , un , . . . l`a day nhung s nguyn xac dinh
. ba
,
,
2
c u1 = 39, u2 = 45, un+2 = un+1 un (n 1). Chu
ng minh
cng thu
,,
,
`ng 1986 chia ht cho v s nhung ph`n tu trong d
ra
ay n`
ay.
,
,
. 3.14. Cho k l`a mt
ay x1 , x2 , . . . , xn , . . . thoa m
an c
ac
. s tu. nhin. D
,
1
,c x0 = 0, x1 = 1 v`
,i moi
ng thu
d
a xn = ( xn+1 xn1 ) vo
a
. n 1.
k
`ng giu,a nhu,ng s x1 , x2 , . . . , x1986 t`n tai
,ng minh ra
Chu
a
. hai s m`
,
tch cua ch
ung chia ht cho tich 19.86.
,
. 3.15. Cho k l`a s nguyn duo,ng v`a
x11 , x21 , . . . , xn1 , . . .
x12 , x22 , . . . , xn2 , . . .

,,
Chuong 3. D
ay s

32

...............
x1s , x2s , . . . , xns , . . .
,
,i
. p ch s (i, j), vo
t`n tai
l`
a s d
ay s nguyn. Khi d
v han
c
ac ca
o
.
.
,
i < j sao cho c
ac tng sau d
y
xi1+1 + xi1+2 + + x1j

d
`u chia ht cho k.

xi2+1 + xi2+2 + + x2j


.....................
xis+1 + xis+2 + + x sj

, ,
CHUONG

4
H`INH HOC
.

4.1. V du.

, ,
,p
Trong s c
ac b`
ai to
an h`nh hoc
trong to
an hoc
t ho. p c
o mt
.
.
. lo
,
`ng phu,o,ng ph
b`
ai to
an giai ba
ap irichl rt thun
a r
o r`
ang.
. tin
. v`
,
,
,
,
Ban
o th t`m thy nhung c
ach giai kh
ac, nhung v` muc
oc
ch
. d
. c
., d
,
,
,
chuyn d
ap ch
ung ta d
ung ta ch khao
` phuong ph
ang xet nn ch
s
at c
ac v du. sau.
,
,,
,
,
,
. 4.1. Trong h`nh vung v
oi canh
1 don vi. d
101 dim. Chung
uo. c chon
.
.
,
,
, , ,, , ,
chon
ac dim da
o th phu boi du`ong
minh ra
`ng c
o n
am dim trong c
. c
1
tr`
on b
an knh .
7
,
`,i giai. Chia h`nh vung ra ra 25 h`nh vung con c
Lo
o canh
0,2.
,
, .
,
,
,
chon
Nhung h`nh vung n`
ay c
o s luo. ng 25 v`
a v` tt ca s d
im d
a
.
,
,
,

l`
a 101, th` t nht c
o mt
h`
nh
vung
nh
o
ch
u
a
t
nh
t
5
d
i
m.
M`
a

.
,
1
1
,,
`ng < .
b
an knh d
on ngoai
u`ong tr`
. tip h`nh vung nho ba
7
5 5
,
. 4.2. Chung minh ra`ng trong moi
. khi da din
. l`i t`n tai
. t nht hai
ma
. t c
o c`
ung s canh.
.

,
,
`,i giai. Ky hiu
,n nht cua khi d
. t c
Lo
F l`
a ma
o s canh
lo
a din.
.
.
. Nu
,
. t (v` c
. t
s canh
cua F l`
a k, th` khi d
o t nht k + 1 ma
o k ma
a din
.
. c

34

,,
Chuong 4. H`nh hoc
.

,
,,
ma
,i F), c`
. t l`
c
o canh
chung vo
on s luo. ng c
ac canh
cua mi
a mt
.
.
.

. t
trong c
ac s 3, 4, . . . , k. Theo nguyn ly irichl c
o t nht hai ma

c
o c`
ung s canh.
.
,
, ,,
,
. 4.3. Trong ph`n trong cua mt
h`nh tr`
on v
oi du`ong knh 5 don vi,
.
,
,.
,`,
,
`

nguoi ta chon
ng t nht c
o hai dim
. , bt k`y 10 dim. ,Chung minh, ra
,
chon
trong c
ac dim da
o khoang c
ach nho hon 2.
. c
,
,,
`,i giai. Chia d
Lo
on
u`ong tr`
,
`
ng nhau
th`
anh 8 re quat
. ba
C9
C
,
,,
,
C2

voi g
oc o tm mi re quat
a
. l`
A1
,
,,
B
F
on d
450 v`
a du. ng d
`ng
u`ong tr`
C8
C3
A
,i b
tm C1 vo
an knh 1. Ky
E
D
,
hiu
a nhung
. C2 , C3 , . . . ,, C9 l`
O
B1
,
,
C7
C1
h`nh t`u t
am re quat
i
. tr`u d
,
,

ph`n m`
a du`ong tr`
on C1 d
a
C4
,
,

chim. C
o th chung minh
C6
,
C5
,,
, hai d
d
ao
uo. c bt cu
im n`
thuc
chn h`nh
. mt
. trong
,
,
H`nh 4.1:
`
trn d
o khoang c
ach nho
u c
,
,,
,
,
hon 2. Tht
ao d
on d
im roi v`
u`ong tr`
`ng tm th`
. vy,
. nu hai d
,
,
,
,
,,
,
,
,
khoang c
ach giua ch
ung nho hon 2. Gia su hai d
a B roi
im A v`
,
v`
ao mt
s t
am re quat.
an knh OC v`
a OD ly
. CDEF trong
. Trn b
,
, , ,
tuong ung hai d
a B1 sao cho OA1 = OA; OB1 = OB, ngha
im A1 v`
,,
[ A\
l`
a AB A1 B1 (theo d
ly h`
am cosin, boi v` AOB
inh
1 OB1 ).
.
,
,
`ng A1 B1 max{ A1 D, A1 E}. Tht
`m trong
y ra
im B1 na
. vy
. d
,
,
,,
ng tao
d
boi h`nh chiu H cua A1 trn OD v`
a t nht mt
oan
. tha
.
,
,.
,
,,

trong hai d
i
m
D,
E,
ch
a
ng
h
an
d
i
m
D.
B
o
i
v
y
h`
nh
chi
u
HD

.
.
,
,
,`,
,

cua d
u
o
ng
xin
A
D
khng
b
e
ho
n
h`
nh
chi
u
HB

1
1 cua A1 B1 trn
,ng minh nhu, trn ta c
OD. Ngha l`
a A1 B1 A1 D. C
ung chu
o

4.1. V du.

35

, ,
nh gi
a
DA1 max{ DF, DC }, EA1 max{ EF, EC }. T`u su. d
a
EF2 < CD2 = OC2 + OD2 2.OC.OD. cos 450

25 25 2
=2
< 3, 75 < 4
4
4
v`
a
EC2 = FD2 = OF2 + OD2 2OF.OD. cos 450

25 5 2
5.1, 4
= 1+

< 7, 25
= 3, 75 < 4,
4
2
2
,,
ta d
uo. c AB A1 B1 max{ DF, DC, EF, EC } < 2.
,
, ,,
,
,, , `
d
. 4.4. Gia su mi
ma
. t pha
ng duo. c son ba
ng mt
im trong mt
.
. trong
,
,
,
`
nht

hai m`u do v`
a xanh. Chung minh ra
ng c
o mt
ao d
o
. h`nh chu
. n`
,
,
,
trong ma
. t pha
ng m`
a bn dnh cua n
o c`
ung m`u.
,
`,i giai. D thy theo nguyn ly
Lo
,
P1
P4
P3
P2
irichl, mt
im
. tp
. bt k`y 7 d
,
m`
a son mt
hai m`u th` t
. trong
Q2
Q1
,
Q4
Q3
nht c
o4d
ung m`u. Trn
im c`
,
,
,,
ng c
mt
o7d
R2
u`ong tha
im th`
R1
R3
R4
. d
,
,
,

ch
ung ta phai c
o4d
i
m
th
a
ng

, ,,
l`
h`
ang c`
ung m`u, gia su d
a
o
,
H`nh 4.2:
P1 , P2 , P3 , P4 c
o c`
ung m`u d
o. Ta
,
,
,
,,
,,
,i d
ng song song vo
chiu nhung d
ay xung hai d
im n`
u`ong tha
u`ong
, , ,
,a ch
,i
chu
ung tao
a ( R1 , R2 , R3 , R4 ) tuong u
ng vo
. ra ( Q1 , Q2, , Q3 , Q4 ) v`
,
,
( P1 , P2 , P3 , P4 ). Nhung dim n`ay tao
mt
ung
. ra
. s h`nh chu nht,
. ch
,
,
,
ta ch
u y d
ac h`nh chu nht
od
a Pi , i = 1, 2, 3, 4. Nhu vy
n c
nh l`
. , c
.
,
,
,

nu 2 d
i
m
b
t
k`
y
c
ua
Q
l`
a
d
o
th`

ta
c
o
k
t
qu
a
m
t
h`
nh
vung

,
, .
,,
,
`
Pi Pj Q j Qi c
od
nh
c`
u
ng
m
u.
Tu
o
ng
t
u
cho
c
a
c
d
i

m R. Nu d
`ng
.
,
,
,
,
,
,
. c
th`oi khng c
od
a R thoa m
an tru`ong ho. p trn th` c
o 3 (hoa
im Q v`

36

,,
Chuong 4. H`nh hoc
.

,
,
,
,
v`
c
hon) d
ao d
a3d
ao d
o c`
ung m`u xanh. Nhung
im Q n`
o
im R n`
o
,
,
,
,i c
. p d
trong b. ba nhu vy
phai c
o ca
ra h`nh chu nht
vo
ac
i tao
.
.
.
,
,
`

d
ac d
a R.
nh mu xanh trong s c
im Q v`
, ,,
,, ,
,
, nht
. 4.5. Gia su mt
an c`o h`nh chu
o 4x7 vung duo. c son den
. b`
. c
,
,
,
,
hoa
. c tra
ng. Chung minh ra
`ng v
oi c
ach son m`u bt k`y, trong b`
an c`o
,,
, nht
lun t`n tai
ac vung, m`
a bn o g
oc l`
a c
ac
. h`nh chu
. g`m c
`
c`
ung mu.

,
`,i giai. Ch
,ng
Lo
ung ta chu
,
minh cho b`
ai to
an b`
an c`o
so,n m`u c
3 7. Mu
o
, ,
,
,

`
th xy ra voi b`
an co n`
ay
,
,

`
c
o dang
t
u
1
d

n
8.
Gi
a

.
,,
su mt
ac ct
. trong s c
.
thuc
d
ang
1.
B`
a
i
to
a
n
s
e
.
.
,,
,
ng minh nu tt
d
uo. c chu
,
,
ca nhung ct
on lai
. c`
. trong
6 ct
ac dang
1, 2,
. thuc
. c
, . ,,
,
,
3, 4. Nhu vy
. gia su tt ca
H`nh 4.3:
c
ac ct
on lai
. c`
. thuc
. dang
.
. c 8. Khi d
theo nguyn ly irichl hai trong s s
5, 6, 7, hoa
au ct
o
.
,
,,
,

c
o hai ct
c`
u
ng
m
t
d
ang
v`
a
nhu
v
y
b`
a
i
to
a
n
c
u
ng
d
u
o
c
ch
u
ng
.
.
.
.
.
,
,,
,
,ng minh ho`
minh. Chu
an to`
an tuong tu. nu mt
c
t
c
o
dang
8. Gia
. .
.
,,
,
. c 8. Nhu vy
su khng c
o ct
ao trong 7 ct
o dang
1 hoa
o7
. n`
. c
.
. ta c
,
i 6 dang.
ct
Theo nguyn ly irichl c
o hai ct
ung dang
a b`
ai
. vo
.
. c`
. v`
,
,,
,
ng minh d
to
an d
uo. c chu
`y d
u.
,
,
. 4.6. Nam dim A, B, C, D, E na`m trong mt
ma
. t pha
ng v`
a toa
.
. d.
,
,
,
ng tam
cua ch
ung l`
a c
ac s nguyn. Chung minh ra
`ng trong s nhu

4.1. V du.

37

,
,
,
,
trong c
gi
ac m`
a dnh cua n
o l`
a ba dim n`
ao do
ac dim n`
ay, c
o t nht
,
ba tam gi
ac v
oi din
a c
ac s nguyn.
. tch l`
,
,
,
`ng nu mt
`,i giai. Ta c
,ng minh ra
Lo
o th chu
trong c
ac toa
d
. cua
.
.
,
,
,
n, th` din
cho thay d
c
ac d
ac d
s cha
tch cua tam
nh tam gi
a
i mt
.
.
,
,
,
,
gi
ac c
ung thay d
s nguyn. Mt
c
ach tng qu
at hon ta c
o th
i mt
.
.
,
,
,
,
ng d
ac toa
d
tam gi
ac thay d
kha
inh
. d
nh cua mt
i mt
. nu c
.
.
. s
,
,

n, th` din
cha
tch cua n
o c
ung thay d
i mt
. s nguyn. V` vy,
. nu
,.
,
,
,
i nhn
a s nguyn, th` din
din
ac mo
uo. c l`
. tch cua tam gi
. d
. tch
,
ng minh).
tam gi
ac ban d
ung l`
a s nguyn (h
ay ve h`nh v`
a chu
`u c
,
,
,
,
cho A, B, C, D, E l`
V` nhung toa
ac d
a nhung s
. cua c
im d
a
. d
,
n thch ho.,p,
nguyn, sau khi thm v`
ao c
ac toa
d
ay nhung s cha
. n`
.
,
,
toa
moi
cho
th` mi
c
ac gi
a tri. 0 v`
a 1. Do d
5d
. se ch nhn
o
im d
a
. d
.
.
,
,,
tao
ac d
nguyn ly
im (0, 0), (0, 1), (1, 0), (1, 1). p dung
. nn boi c
.
,
,
,

irichl suy ra t nht hai d


ac d
im trong c
im A, B, C, D, E bin d
i
,
, ,,
l`
th`
anh c`
ung mt
a
im trong {(0, 0), (0, 1), (1, 0), (1, 1)}. Gia su d
o
. d
,
ng d
A, B. Ch
ung ta se kha
ac tam gi
ac ABC, ABD v`
a
inh
. din
. tch c
,

ABE l`
a nhung s nguyn. Tht
ac tam gi
ac n`
ay bi. bin th`
anh
. vy,
. c
,
,

ng (do A v`
d
tha
a B bin th`
anh c`
ung mt
tch
oan
im) nn din
., d
, . ,
, .
,,
`

ng 0. Vy
anh cua ch
ung ba
ung
. truoc khi bin di, din
. tch cua ch
,

phai l`
a s nguyn.
,
,
,
. 4.7. Trong mt
ma
. t pha
ng cho mt
tp
ho. p A c
o n dim (n 2),
.
.
.
,
,
,,
,
,
mt
. p dim duo. c ni v
oi nhau ba
`ng doan
tha
ng. Chung minh
. s ca
.
,
,
,,
,,
,
ra
`ng trong A c
o t nht hai d
oi c`
ung s luo. ng c
ac dim
im duo. c ni v
kh
ac thuc
. A.
,
,
`,i giai. Vo
,i moi
Lo
ung ta ky hiu
S( a) l`
a
im bt k`y a thuc
. d
. A ch
, .
,
,
,
,
,
ng. B`
s luo. ng nhung d
a a ni th`
anh d
ai to
an
im cua A m`
oan
. tha
,
,
,
`ng t`n tai
ng d
mun kha
ac nhau a1 v`
a a2 cua A
inh
im kh
. ra
. hai d

38

,,
Chuong 4. H`nh hoc
.

`ng 0 S( a) n 1 vo
,i
m`
a S( a1 ) = S( a2 ). Ch
ung ta thy ngay ra
,
,,
,
. t kh
moi
ac, c
ung khng t`n tai
im
. ph`n tu a thuc
. A. Ma
. nhung d
,,
x1 v`
a x2 thuc
a S( x1 ) = n 1 v`
a S( x2 ) = 0. Boi v` d
ay
i`u n`
. ,A, m`
,
,
,
,

i tt ca c
c
o ngha l`
ad
ac d
on lai
on x2
im x1 ni vo
im c`
. cua A, c`
,
,
,
,

i d
khng ni vo
ao cua A, d
ac s nguyn t`u 0
im n`
n d
n v ly. C
,,
,
d
o s luo. ng l`
a n. V` 0 v`
a n 1 khng d
a gi
a tri.
n n 1 c
`ng th`oi l`
,
,
,,
`

vua S nn S nhn
a n 1 gi
a tri.
ai to
an d
uo. c giai suy
. B`
. nhiu nht l`
,
`
ra tu nguyn ly irichl.

,,

. 4.8. Cho da giac d`u 100 canh


ni
tip trong du`ong tr`
on k. Mi
.
.
,
,,
,
dnh duo. c g
an mt
ac s 1, 2, . . . , 49. Chung minh ra
`ng trn k
. trong c
,
t`n tai
a CD v
oi c
ac tnh cht sau:
. hai cung AB v`
,
,
,
cho.
a) C
ac dim A, B, C v`
a D l`
a dnh cua da gi
ac d`u da
,
b) C
ac dy cung AB v`
a CD song song v
oi nhau
,,
,
c) Nu A, B, C v`
a D c
o nh
an tuong ung l`
a c
ac s a, b, c, d th` a + b =
c + d.
,
,,
`,i giai. Trong d
Lo
on k c
o
u`ong tr`
,`,

d
ac nhau m`
a
ung 50 d
uong knh kh
D
C
,
,
,`,

d
i
m
cu
i
c
ua
d
u
o
ng
k
nh
l`
a
m
t

.
,
,

d
ac d
d
nh cua d
a gi
`u 100 canh
a
.
,
,
cho. Nu PQ l`
a mt
u`ong
. trong s d
,,
v`
knh d
ad
an nh
an
o
ir nh P d
uo. c g
,
,
,
i nh
p, c`
on Q vo
an q th` d
u`ong
, , ,
,
i s nguyn
knh PQ tuong ung vo
B
A
| p q|. Ro r`ang, 0 | p q| 48.
,,
d
`ng c

Ba
ach n`
ay mi
u`ong knh d
a
H`nh 4.4:
,
,
,
,
xet (tng s c
o 50 tt ca) d
uo. c cho
, , ,
,i mt
tuong u
ng vo
ac s 0, 1, 2, . . . , 48. Suy ra c
o t nht hai
. trong c

4.1. V du.

39

,,
,,
, , ,
,i c`
. t tuong u
ng vo
d
ung mt
u`ong knh d
uo. c d
a
. s.
,
,,
, , ,
l`
Ta ky hiu
a AB v`
a CD, c`
on c
ac d
an tuong u
ng
o
nh d
uo. c g
. d
,
, ,
c
ac nh
an a, b, c, d. Khng mt tnh tng qu
at ch
ung ta c
o th gia
dy cung AC v`
thit c a v`
a b d. Khi do
a BD c
o c
ac tnh cht
,,
,
,
,
i d
. t tuong
d
ung ta d
mong mun. Tht
a
a
u`ong knh AC ch
. vy,
. vo
,
`ng nhau,
,ng vo
,ng vo
,i a c, c`
,i d b v`
u
on BD u
a nhung s n`
ay ba
,,
ch
ung ta nhn
a a + b = c + d. Ngo`
ai ra,
uo. c a c = d b hay l`
. d
,
,
gi
i CD (thm
tu
ac ABCD l`
a h`nh vung, nn AB song song vo
. ch
AB = CD).

,
,
,
,,
,,
. 4.9. Tt ca cac dim trong mt
. t pha
ng duo. c bi boi n m`u
. ma
,,
,,
kh
ac nhau. Cho truoc 2n 1 du`ong tr`
on d`ng tm kh
ac nhau
,
,
,
,
,
,
k1 , k2 , . . . , k2n 1 . Trong c
ac du`ong tr`
on du. ng b
an knh tuong ung
,
ca
OA1 , OA2 , . . . , OA2n 1 sao cho mi
. p b
an knh khng c
o dim
,
,,
on
chung n`
ao kh
ac ngo`
ai O. Chung minh ra
`ng t`n tai
. mt
. du`o,ng tr`
n
k i , i = 1, 2, . . . , 2 1 sao cho trn n
o v`
a trn b
an knh OAi cua n
o c
o
,
,
,
,
,
,
,
ng dim tuong ung Xi v`
nhu
a Yi , m`
a ch
ung duo. c bi c`
ung mt
a
. m`u v`
,
Yi khng tr`
ung v
oi O v`
a Ai .
,
`,i giai. Ky hiu
cho l`
Lo
ac m`u d
a c1 , c2 , . . . , cn . Ch
ung ta se tnh
a
., c
,
,
,
,
,
,
i ch
s luo. ng c
ac t ho. p kh
ac nhau cua c
ac m`u m`
a vo
ung c
o th
,
,
ng. Moi
bi m`u c
ac d
ac trng
im trn mt
., h`nh pha
. tp
. con kh
, , ,
,
{i1 , i2 , . . . , ik }, 1 k n cua tp
. ho. p N = {1, 2, . . . , n} tuong ung
, ,
,
,
i t ho. p m`u {ci1 , ci2 , . . . , cik }. R
vo
o r`
ang nhung tp
ac trng
. con kh
,
, ,
,
,
,
,
,
`ng c
ng vo
i nhung t ho. p m`u
kh
ac nhau cua N ba
ach n`
ay tuong u
, ,
,
kh
ac nhau. Ngo`
ai ra, moi
o th nhn
. t ho. p m`u {ci1 , ci2 , . . . , cik } c
, .,
,,
,
,

`
d
u
o
c
nh
o
xy
d
u
ng
t
p
con
kh
a
c
tr
ng
{
i
,
i
,
.
.
.
,
i
}
.
Suy
ra,
t
ho. p
.
1 2
k
.
.
,
,
,
,
i ch
nhung m`u c1 , c2 , . . . , cn m`
a vo
ung c
o th bi c
ac d
im mt
. h`nh
,
, , ,
,
,
,
i nhung tp
ng l`
pha
a tuong ung mt
ac trng
. - mt
. vo
. ho. p con kh

40

,,
Chuong 4. H`nh hoc
.

,
, ,
,
cua N = {1, 2, . . . , n}. Ngha l`
a s c
ac t ho. p kh
ac nhau cua m`u
,
,
,
,
`ng s c
ba
ac tp
ac trng cua N, c
o tt ca 2n 1 tp
. ho. p con kh
. ho. p
,
nhu vy.
.
,
,,
,,
`ng t`n tai
Truo
c tin ch
ung ta gia thit ra
on k i , i =
u`ong tr`
. d
,
,
n
1, 2, . . . , 2 1 m`
a mun bi kn n
o phai d`
ung tt ca c
ac m`u
,
,
ch
c1 , c2 , . . . , cn . Khi d
ung ta chon
a d
o
im bt k`y Yi l`
im trong
. d
,
, ,,
,,
,
`ng m`u c1 . Nhung trn k i c
cua b
an knh OAi . Gia su Yi d
o
uo. c bi ba
,
,
,
,
t nht mt
cho
ung m`u nhu vy,
im Xi , d
uo. c bi c`
a
. d
. v` moi
. m`u d
,
,
,
,
,
,
,
t ga
. p trn k i ; vy
d
ai to
an d
`u ba
uo. c giai trong tru`ong ho. p gia thit
. b`
, ,,
,`,
,,
,
n`
ay. Ta xet truong ho. p c`
on lai:
Gia su khng c
od
on n`
ao m`
a
u`ong tr`
.
,
,
,
,
,
,
,
,
i to`
nhung d
od
an b. t ho. p c1 , c2 , . . . , cn .
im cua n
uo. c bi m`u vo
, n
,
, ,
,
,,
,i n
c
Khi d
o tt ca 2 2 kha n
ang t ho. p m`u, vo
o c
ac d
o
im d
uo. c
,
,,
,
bi cho c
ac d
on k1 , k2 , . . . , k2n 1 . Boi v` 2n 1 > 2n 2 nn
u`ong tr`
, ,
,,
c
o hai d
on k i v`
a k j , i < j trn n
o se c
o c`
ung mt
t ho. p m`u.
u`ong tr`
.
, ,,
,
,
t cua OAi v`
,n ho,n k j . Ky hiu
Gia su k i c
o b
an knh lo
ad
a
im ca
. Yi l`
,
,
,`,
,
,
`m trn k j v`
Yi na
k j . Trong truong ho. p d
a v` nhung d
a
o
im cua k i v`
, ,
,
,,
,
,
,
i c`
kj d
ung mt
o dim Xi d
uo. c bi vo
uo. c bi
. t ho. p m`u, trn k i c
,
i Yi .
c`
ung m`u vo

,
,
,
,
,
. 4.10. Tng cua d. d`ai mt
. t pha
ng l`
a 4. Chung
. s vecto, trong ma
,
,
`, nhu
,ng vecto, n`
minh ra
`ng tu
ay c
o th chon
a tng d.
. mt
. s vecto m`
,
,
,
d`
ai cua ch
ung l
on hon 1.
,
,
,
,
`,i giai. Ch
Lo
ung ta d
ao h. toa
a xet vecto d
ua v`
. v`
ai
. d
. din
. nhung
,
,
,
,
cho tai
vecto d
ung ta chiu nhung vecto n`
ay xung
a
im gc. Ch
. d
,
,
,
,
,
vecto c
truc
a Oy. V` mi
o d. d`
ai nho hon tng cua c
ac
. Ox v`
. toa
. d
,
,
,
,
d
ai h`nh chiu cua n
o xung hai truc;nn
tng d
ai cua tt ca
. d`
. d`
.
,
, ,
,
trn t nht mt
h`nh chiu cua c
ac vecto lo
n hon 4. Khi d
o
. trong
,
,
,
,,
,
n ho,n 1,
4 nua truc
ai cua h`nh chiu se lo
. tng d
. d`
. cua h. toa
. d

4.2. B`
ai tp
.

41

,
,
,
,
, , , ,
c
ng se
d
o ngha l`
a tng cua d
ai cua nhung vecto tuong u
i`u d
o
. d`
,
,
,
,
,

n hon 1. (d
n hon th` tt nhin d
lo
lo
ai h`nh chiu d
ai vecto
. d`
a
. d`
,n ho,n.)
c
ung lo

` tp
4.2. Bai
.

,
,
,ng
. 4.11. Trong h`nh vung vo,i canh
1d
on vi. cho 112 d
im. Chu
.
,
, , 2
`ng t nht hai trong s d
c
minh ra
o khoang c
ach nho hon .
o
15
, 25 n
. 4.12. (` thi Toan Olympic quc t l`n thu
am 1984) Trong
,
,
,
,
d
i mi
. t pha
ng cho hai d
ma
ac nhau O v`
a A. Vo
im kh
im X thuc
.
,
`
. t pha
ng kh

ma
ac O ch
ung ta ky hiu
a
(
X
)
l`
a
d

d
o
b
a
ng
radian

.
.
,
,,
,
cua g
oc AOX, d
o theo chi`u nguo. c kim d
`ng h` t`u OA d
n OX
,`,
(0 a( X ) 2). C`on c( X ) l`a duong tr`on tm O v`a ban knh co d.
,
a( X )
,,
,
d
. t
d`
ai OX +
. Cho truo
c b. huu han
m`u v`
a mi
im trong ma
.
OX
,
,
,,
`ng 1 trong s d
`ng t`n tai
,ng minh ra
ng d
. Chu
pha
d
uo. c bi ba
o
im
,.
,
,,
,
,
i a( X1 ) > 0 v`
X1 vo
a trn d
on c( X1 ) c
o t nht 1 d
u`ong tr`
im d
uo. c
,i X .
bi c`
ung m`u vo
1

,
,
,
. 4.13. Trong ma. t phang cho n dim, n 7, sao cho khoang cach
,
,
,
,,
ca
mt
,i
. p d
mi
ung l`
a kh
ac nhau. Mi
d
im giua ch
im duo. c ni vo
.
,
,
,,
`ng khng c
,ng minh ra
d
o nht. Chu
od
ao d
im g`n n
im n`
uo. c ni
,
,i nhi`u ho,n 5 d
vo
ac.
im kh
`ng trong mt
,ng minh ra
. 4.14. Chu
nh tr`
on b
an knh 1, khng
. h`
,
,
,
,
,,
,
th chon
a n
am d
a khoang c
ach giua hai d
uo. c qu
im m`
im mt
. d
.
,
,
n hon 1.
lo
,
,
. 4.15. Ngu`o,i ta quang 120 h`nh vung co kch thuo,c 1 1 v`ao mt
.
,
,,
,
,
`

h`nh chu nht


k
ch
thu
o
c20

25.
Ch
u
ng
minh
r
a
ng
v
o
i
m
oi
c
a
ch
s
a
p
.
.

42

,,
Chuong 4. H`nh hoc
.

,
,,
,
c`
trng d
on ch
xp c
ac h`nh vung th` o trong h`nh chu nht

. vn
,,
. t mt
d
on d
a
u`ong knh 1.
. h`nh tr`

, ,
CHUONG

,,
IRICHL
MO RNG
NGUYN LY
.

,,
5.1. Nguyn l
y irichl mo rng
.

,,
,
,
,,
Cho A l`
a tp
a s luo. ng
. huu han
. nhung ph`n tu, ky hi, u
. s( A) l`
,,
,,
,
c
ac ph`n tu thuc
o th mo rng
. A. Nguyn ly irichl c
. nhu sau:
,,
,
,
,
Nu A v`
a B l`
a nh
ung tp
uu han
a s( A) > k.s( B), o
. ho. p h
. v`
, ,
,
ph`n tu, cua A cho
v`
d
a mt
ao d
a nu mi
y k l`
o
. s tu. nhin n`
,
,,
, , ,
,i mt
cua B, th` t`n tai
ng vo
ph`n tu n`
ao d
tuong u
o
.
. t nht,k +, 1
,, ,
,
,
,
,
,
ng vo
i c`
a ch
ung tuong u
ung mt
ph`n tu cua A m`
. ph`n tu cua
B
,,
,
xet
Tht
a nguyn ly irichl m`
a ta d
a
. vy,
. tru`ong ho. p k = 1 l`
, ,
,
,
,
ng minh mnh
i gi`o. chu
trong c
ac b`
ai tp t`u d
ung
`u to
` trn ch
. d
, ,, .
, ,,
,, ,
,
,
,
ng vo
i nhi`u nht k ph`n tu,
ta gia su mi ph`n tu cua B ch tuong u
,
,
,i gia thit s( A) > k.s( B).
s( A) k.s( B) tr
cua A. Khi d
ai vo
o
,
,
,,
, , ,,
d
. Ngay t`u nhung chuong d
Ch
u y
ay d
`u nguyn ly n`
a
uo. c su
,
,
,ng minh tu,o,ng tu., nhu, trn. tip tuc
,i c
dung
vo
ach chu
.
. hiu su
,
,,
,i ch
thm nguyn ly o dang
mo
ung ta xet mt
ai to
an d
in
.
. loat
. b`
h`nh.

44

,,
,,
Chuong 5. Mo rng
nguyn ly irichl
.

5.2. V du.

,
. 5.1. Trong mt
o canh
`ng 1 ta chon
. h`nh vung c
., ba
. bt k`y 51 dim.
,
na
Chung minh ra
`ng t nht c
o ba dim trong s do
`m trong mt
. h`nh
vung c
o canh
0,2.
.
,
`,i giai. Ch
Lo
ung ta chia h`nh vung th`
anh 25 h`nh vung con c
o
,
,
,`,
,
`
i c
ng c
ng song song vo
canh
0,2 ba
ac d
ac canh
cua h`nh
uong tha
.
.
, ,
,
,,
,

ng mi h`nh vung vua nhn


vung.Nu gia su ra
d
uo. c chua khng
.
,
,
,
,
,n se c
qu
a2d
ac d
o s d
im th` tt ca c
im trong h`nh vung lo
im
d
nhi`u nht l`
a 2 25 = 50 dn
n v ly.

,
,
,,
y da gi
. 5.2. Tt ca 9 canh
v`
a 27 du`ong cheo cua mt
op da
ac
.
. h`nh ch
,
,
,
,,
,
,
,
9 dnh duo. c bi son: mt
son do, c`
on lai
., s bi
. bi son xanh. Chu,ng
,
,ng dnh
minh ra
`ng t`n tai
op, m`
a ch
ung l`
a nhu
. ba dnh cua h`nh ch
,
,, ,
,
ung mt
cua mt
ac v
oi c
ac canh
duo. c son c`
. h`nh tam gi
.
. m`u.
,
,
,, , `
`,i giai. 9 canh
ng hai m`u. V` 9 >
Lo
bn cua h`nh ch
op d
uo. c son ba
.
,,
,
,,
4.2, t`u nguyn ly mo rng
trn suy ra t nht c
o 5 canh
bn d
uo. c bi
.
, ,,.
,,
l`
c`
ung mt
m`u. Gia su d
a SA1 , SA2 , SA3 , SA4 , SA5 d
ung
o
uo. c bi c`
, . ,,
,
,
, ,,
,

m`u d
o,
o
d
y
S
l`
a
d
nh
h`
nh
ch
o
p.
Khng
anh
hu
o
ng
t
o
i
k
t
lu
n

.
, ,,
,
,,
p

b`
ai to
an ch
ung ta gia su c
ac d
i
m
A
,
A
,
A
,
A
,
A
d
u
o
c
s
a
p
x

2
3
5 .
1
4
,
,
,
theo chi`u nguo. c kim d
trong c
ac canh
cua ng
u
`ng h`. t nht mt
.
,
, . ,,
,`,
y; gia su d
l`
a A1 A2 . Ch
ung ta
gi
ac l`
ad
ac d
o
uong cheo cua d
a gi
a
,
,
,
,
,
xet tam gi
ac A1 A2 A4 . Nhung canh
cua n
o l`
a c
ac d
u`ong cheo cua d
a
.
,
,
,
,
y (do c
gi
ac d
ach xp trn) v`
a suy ra ch
ung d
a
uo. c bi son.Nu tt ca
,
giai xong.
c
ac canh
A1 A2 , A2 A4 , A4 A1 l`
a m`u xanh th` b`
ai to
an d
a
.
,,
,,
,
Tru`ong ho. p nguo. c lai
ac canh
a m`u
. mt
. trong c
. ,A1 A2 , A2 A4 , A4 A1 l`
,
, ,,
l`
c
d
a A1 A2 . Khi d
ac canh
cua tam gi
ac SA1 A2 l`
a m`u
o, gia su d
o
o
.
,
d
o.

5.2. V du.

45

,
,
. 5.3. Chung minh ra`ng trong moi
ac l`i v
oi s canh
cha
n t`n
a gi
. d
.
,
,`,
,
tai
oi mt
n`
ao cua da gi
ac.
. duong cheo khng song song v
. canh
.
,
,
`ng qui nap
`,i giai. Ba
Lo
o th d
. ta c
,ng minh du,o.,c moi
,i n
chu
ac vo
a gi
. d
n ( n 3) , ,
d
canh
c
o
u`ong cheo.
.
2
,
By gi`o ch
ung ta xet d
ac l`i bt
a gi
,
i k ( 2) l`
k`y P c
o 2k canh
vo
a mt
.
. s

d1
d2
d k 1
a

nguyn.
H`nh 5.1:
, ,,
,
,`,
,
i mt

Gia su mi d
n`
ao d
uong cheo cua P song song vo
o
. canh
.
,
,
,`,
, , ,
,

i canh
mi d
cua P. Khi d
o th cho tuong ung vo
song
o
uong cheo d c
.
,
,
,
i d. Ky hiu
song vo
a s c
ac d
ung ta c
o
u`ong cheo ch
. s l`
2k (2k 3)
s=
= k(2k 3)
2
= 2k(k 2) + k > (k 2).2k.
,,
,
Nhu vy
mo rng
suy ra t`n tai
. theo nguyn ly irichl
.
. k1
,
,`,
, , ,
,i
d
ac P m`
a ch
ung tuong ung vo
uong cheo d1 , d2 , . . . , dk1 cua d
a gi
,
c`
ung mt
a cua d
ac, ngha l`
a a, d1 , d2 , . . . , dk1 song song
a gi
. canh
.
,
,
,
`m trong
i nhau. Suy ra c
vo
ac d
ung na
u`ong cheo d1 , d2 , . . . , dk1 c`
,
,,
,,
. t pha
ng x
mt
ac d
a v` P l`
ad
ac l`i (h`nh ve).
inh
a gi
. boi canh
. n ua m a
.
,
,,
,
ng kh
Ngo`
ai ra d1 , d2 , . . . , dk1 v`
a a l`
a nhung doan
tha
ac nhau v`
a boi
.
,
,
,
,
,
,,
d
v` s luo. ng cua ch
ung l`
a k, mi
ac P l`
ad
nh cua d
a gi
im d
`u cua
,
,
, ,,
,ng
trong ch
ao d
ung. Khng anh huong d
mt
oan
o
n kt qua chu
. d
. n`
,
,,
,i a trong c
minh ta gia thit d1 l`
ad
ac d
oan
i vo
u`ong cheo
. xa nht d
,
`ng d
`m
t`u ly lun
d1 , d2 , . . . , dk1 . Khi d
trn suy ra ra
ac P na
o
a gi
,.
,,
,,
,i
. t pha
ng x
to`
an b. trong mt
ac d
ay tr
ai vo
inh
. boi d1 . i`u n`
. nua ma
,
tnh l`i cua d
ac P.
a gi

,
,
,
doan
`,ng ca
. 5.4. Trong ma. t phang cho 6 dim. Mi
ng ni tu
. p
. tha

46

,,
,,
Chuong 5. Mo rng
nguyn ly irichl
.

,
,
,
,,
,
dim duo. c bi m`u d
. c xanh. Chung minh ra
`ng ba dim trong s
o hoa
,
,
,
,
,,
c
ac dim l`
a dnh cua mt
ac, m`
a c
ac canh
cua n
o duo. c bi c`
ung
. tam gi
.
`
mt
. mu.
,
,
`,i giai. Ky hiu
cho v`
Lo
A l`
a mt
trong c
ac dim d
a xet n
am d
a
oan
.
.
.
,
,
,
,
,
,
,
ng, c

tha
o d
a A. Nhung d
th
a
ng
n`
a
y
d
u
o
c
so
n
hai
nh chung l`
oan

.
.
,,
,
m`u v`
a v` 5=2.2+1, suy ra t`u nguyn ly irichl mo rng
ch
ung
.
, ,,
l`
ta c
o t nht ba doan
ung m`u. Gia su d
a AB1 , AB2 , AB3 m`u
o
. c`
,
ng B1 B2 , B2 B3 , B3 B1 l`
xanh. Nu mt
ai d
a xanh th` t`n tai
oan
. v`
. tha
, .
,
,

mt
tam
gi
a
c
v
o
i
ba
c
anh
xanh
v`
a
c
o
d
nh
l`
a
A.
N
u
ba
d
o
an
th
a
ng

.
.
,.
,
ra l`
B1 B2 , B2 B3 , B3 B1 l`
ad
tam gi
ac thoa m
an d
an d
a
o, th` mt
` to
a
,.
B1 B2 B3 (c
ac canh
d
`u m`u d
o).
.

,
,,
. 5.5. Cho day v han
ac s tu. nhin u1 , u2 , . . . , un , . . . duo. c x
ac dinh
. c
.
,
theo cng thuc sau u1 = 3, un+1 = (n + 1)un n + 1, n = 1, 2, . . ..
,
,
,
Cho n s tu. nhin bt k`y v`
a tp
ho. p M g`m un dim sao cho khng
.
,
,
,
,
doan
c
o ba d
ao tha
ng h`
ang. Mi
ng ni hai dim kh
ac nhau
im n`
. tha
,,
, `
cho. Khi do
t`n tai
trong M d
ng mt
uo. c bi son ba
. trong n m`u da
. 3
,
,
,
dim trong M l`
a dnh cua mt
ac d`ng m`u.
. tam gi
,
`ng qui nap
`,i giai. Chu
,ng minh ba
,i n = 1 ch
Lo
ung ta c
o
. theo n. Vo
,
,
,
,
,
,
c tnh d
i n = 2 t`u cng thu
u1 = 3 v`
a kt lun
a hin nhin. Vo
uoc
. l`
, ,, .
,,
,`,
,
,
ng minh o b`
chu
u2 = 6, d
a truong ho. p ta d
ai trn. Gia su b`
ai
y l`
a
,
,
,,
,
,
,
i un d
ng ni d
to
an vo
a c
ac d
o mt
im v`
oan
uo. c bi boi n m`u, c
. tha
.
,
,
,
`

i c
`
tam gi
ac vo
ac canh
c`
u
ng
m
u.
By
gi
o
ta
x
e
t
u
d
i
m
b
t
k`
y
,
m`
a
n +1
.
,`,
,
,
,
`ng n + 1 m`u: c1 , c2 , . . . , cn+1 . Ly
d
ung d
uong ni giua ch
uoc bi ba
, .
,
,
,
,i c
A l`
a mt
ac d
ay c
o th ni vo
ac d
on
im trn. im n`
im c`
. trong c
,
,,
,
c h`i
ng bi m`u. Ma
. t kh
lai
ac do cng thu
uo. c un+1 1 d
oan
. d
. tha
,
qui un+1 1 = (n + 1)(un 1) + 1, nhu vy
theo nguyn ly irichl
,
,.
,,
,,
ng c
mo rng
o chung d
ung m`u.
oan
nh A d
uo. c bi c`
. t nht un d
. tha

5.2. V du.

47

,
, ,,
,,
ng han
Gia su AB1 , AB2 , . . . , ABun d
ung m`u, cha
uo. c bi c`
. m`u c1 .
,
, ,
,
,
Nhung kha n
ang c
o th xy ra nhu sau:
,
,
,
ng ni giua c
a) Nu mt
ac d
ac d
oan
im B1 , B2 , . . .
. trong c
. tha
,
,,
`ng m`u c1 th` t`n tai
. p d
, Bun theo t`ung ca
ac d
uo. c bi ba
`ng
. tam gi
,
,
,
m`u c
od
ac canh
cua tam gi
ac n`
ay son m`u c1 .
nh A. C
.
,
,
ng ni giua c
b) Khng c
o mt
d
n`
ao trong c
ac d
tha
ac
oan
oan
.
.
.
,
,, ,
a
p dung
d
an
im B1 , B2 , . . . , Bun d
uo. c son m`u c1 . Khi d
o
. , qui nap
. to
,
,
,
,
,
hoc
d
ay c
o th d
im B1 , B2 , . . . , Bun . i`u n`
uo. c v` tt ca
. cho nhung
,
,,
,
ng d
ni d
c
ac d
oan
a
uo. c bi son n m`u c2 , c3 , . . . cn+1 . Trong
. tha
,,
,
d
tru`ong ho. p n`
ay ta c
ung dn
ac d
n kt lun
`ng
. t`n tai
. mt
. tam gi
m`u.

,
,,
,
,,
. 5.6. Cho s hang
cua d
ay un duo. c dinh
ai truoc. Tp
.
.
. ngha nhu b`
,,
,,
,
`

,ng s tu., nhin tu


`, 1 d
ho. p nhu

n
u

1
d
u
o
c
phn
chia
b
a

ng
phu
o
ng

.
n
,,
,
,
`
ph
ap bt k`y v`
ao n tp
h
o
p
con,
m`
a
t
u
ng
di mt
khng c
o ph`n tu
.
.
.
,
,
,
chung. Chung minh ra
`ng t`n tai
hai s cua c`
ung mt
a
.
. tp
. ho. p con, m`
,
,
,
ung c
ung na
`m trong tp
ay ; hoa
. c l`
a t`n tai
tng cua ch
. ho. p con n`
. mt
.
,

s trong mt
t
p
h
o
p
con
n`
a
o
d
o
,
m`
a
sau
khi
nhn
d
i
v
n
thu
c
t
p

. .
.
. .
,
ho. p con n`
ay.
,
,
,
`,i giai. Ky hiu
cho l`
Lo
a l`
a A1 , A2 , . . . , A n .
a
. nhung tp
. ho. p con d
,
,,
,
nh
Ch
ung ta xet tp
a ch
ung duo. c d
im, m`
a
. ho. p bt k`y M g`m un d
,
,
`ng phuong ph
trong c
s ba
ap n`
ao d
ac s 0, 1, 2, . . . , un 1. Ch
ung
o
,
,
,
`ng n m`u
ng ni c
ta bi m`u nhung d
ac d
oan
im trong M ba
. tha
,
,
, i v`
,i
ng ni c
c1 , c2 , . . . , cn theo c
ach sau: oan
ac d
a j, vo
im thu
. tha
,
,, , `
,
ng m`u ck khi v`
i>jd
a ch khi hiu
c
uo. c son ba
., i j thu
. tp
. ho. p
,
,
,
,,
cua M l`
Ak . Theo b`
ai to
an truo
c 5.5 ba d
ao d
ad
im n`
o
nh cua mt
.
, ,,
,
tam gi
ac c`
ung m`u. Gia su c
ac d
i
m
n`
a
y
l`
a
i,
j,
k
v`
a
i
>
j
>
k.
Khi

ung mt
d
o
c
a
c
s
i

j,
j

k
v`
a
i

k
=
(
i
j) + ( j k) na`m trong c`

48

,,
,,
Chuong 5. Mo rng
nguyn ly irichl
.

,
tp
a ch
ung c
o c
ac tnh cht ta c`n t`m.
. ho. p As v`

,
,
. 5.7. Trong ma. t phang c dinh
h. toa
d. Oxy. Ch
ung ta xet tp
ho. p
.
.
.
,
,,
,
,ng dim v
,ng s nguyn
R g`m nhu
oi toa
a nhu
. ( x, y), o dy x, y l`
. d
,
,, , `
d
v`
a 1 x 12, 1 y 10. Mi
ay duo. c son ba
ng mt
im n`
. m`u
,
,
,
nht
tra
ng, xanh hoa
. c do. Chung minh ra
`ng t`n tai
h`nh chu
o c
ac
. ,c
,
,.
,
,
,
ng dim cua
canh
oi c
ac truc
a dnh cua n
o l`
a nhu
. song song v
. toa
. d,
. m`
,, ,
R duo. c son c`
ung mt
. m`u.
,
,
`,i giai. Trong chu
,ng minh ta d`
ng d
Lo
ung hai kha
inh
. sau:
,
,
,
,i n ph`n tu,, th` s lu,o.,ng tp
a) Nu X l`
a tp
. huu han
. vo
. ho. p con
,
,,
n ( n 1)
,i x X, y X l`
. p ph`n tu { x, y}, vo
cua X g`m c
ac ca
a
.
2
,
,
,,
ng
b) Moi
an bt d
`u thoa m
a
. s duong bt k`y x1 , x2 , . . . , xn d
,c
thu
x12 + x22 + + xn2
x + x2 + + x n 2
( 1
)
2
2
, ,
`ng ch xy ra khi x1 = x2 = . . . = xn .
Du ba
,
,
,,
Tro lai
ai to
an d
ac d
a 120, nn
ang xet, v` tng s c
im ta xet l`
. b`
,
,
, ,,
,
,

,i mi
t nht c
o 40 d
ung m`u, gia su l`
a m`u d
im d
uo. c bi c`
o. Vo
,
,
,`,
ng d
i = 1, 2, . . . , 12 ch
ung ta ky hiu
ad
uong tha
i qua d
im (i, 0)
. li l`
,
,
,
i truc
v`
a song song vo
o ni d
im d
o, i = 1, 2, . . . , 12,
. Oy. Nu trn li c
,
,
,
. p
n1 + n2 + + n12 40. Ngo`
ai ra t`u ni d
o th tao
ac ca
im c
. ra c
,,
, , n i ( n i 1)
i = 1, 2, . . . , 12
,i mi
p c
di d
o s luo. ng
. Ngha l`
a vo
uo. c sa
2
,
n ( n 1)
. p s nguyn { j1 , j2 } thoa m
ng i i
an 1 j1
t`n tai
ca
u
. d
2
,
,
,,
. t kh
10, 1 j2 10 v`
a c
ac d
a {i, j2 } l`
ad
ac s luo. ng
im {i, j1 } v`
o. Ma
,
,
,
,i 1 j1 10, 1 j2 10
. p nhung s nguyn { j1 , j2 } vo
cua tt ca ca

5.2. V du.

49

,
,
10(10 1)
`ng tng c
,ng minh ra
= 45. Ch
ung ta phai chu
ac s
2
n1 ( n1 1) n2 ( n2 1)
n12 (n12 1)
,
,...,
2
2
2
,
,
,
,
,
n hon 45. T`u d
,i
suy ra t`n tai
lo
ac nhau i1 v`
a i2 vo
o
. nhung ch s kh
,i 1 j1 10, 1 j2
. p s nguyn { j1 , j2 } vo
1 i1 < i2 12 v`
a ca
,
,
,,
,
10 sao cho 4 d
ad
im {i1 , j1 }, {i1 , j2 }, {i2 , j1 }, {i2 , j2 } l`
o. Boi nhung
,
,
,
,
,i c
,i truc
d
ay l`
ad
ac canh
song song vo
im n`
nh cua h`nh chu nht
. vo
.
.
,
,,
,

toa
d
,
nhu
v
y
b`
a
i
to
a
n
d
u
o
c
ch
u
ng
minh.

.
.
.
.
,
,
,
,ng minh
Nhu vy,
on phai chu
. ta ch c`
`ng
ba

n1 ( n1 1) n2 ( n2 1)
n ( n 1)
+
+ + 12 12
> 45
2
2
2
,
,,
,c b)
ng thu
Su dung
bt d
a
.
S=

1
((n1
2
1
= ((n1
2

1 2 1
1
1
1
1
1
1
) ) + ((n2 )2 ) + + ((n12 )2 )
2
4
2
2
4
2
2
4
1 2
1 2
1 2
1
) + (n2 ) + + (n12 ) ) 12
2
2
2
8
1
1
1
(n1 ) + (n2 ) + + (n12 )
1
2
2
2 )2 3
.12(
2
12
2
1
1 2 3
= ((n1 + n2 + + n12 ) 12 )
24
2
2
2
3
34
3
1
> 45
(40 6)2 =
24
2
24
2

S=

v` n1 + n2 + + n12 40.

,
. 5.8. (B`ai thi Olympic toan quc t l`n thu 20, 1978) Mt
an
. hi
. to
,,
,
,
hoc
ac th`
anh vin o 6 nuoc. Danh s
ach c
ac hi
. bao g`m c
. vin g`m
,`, , ,
,
,
` 1 dn 1978. Chung minh ra
nh s b
1978 nguoi duo. c da
ao danh tu
`ng
,
t`n tai
o s b
ao danh gp di s b
ao danh cua
. t nht mt
. hi
. vin c

50

,,
,,
Chuong 5. Mo rng
nguyn ly irichl
.

,
,
,,
mt
ac c`
ung nuoc, hoa
. c ba
`ng tng hai s b
ao danh cua hai
. hi
. vin kh
,, ,
hi
ung mt
oi m`nh.
. vin c`
. nuoc v
,
,,
`,i giai. T`u, 329.6 < 1978 suy ra mt
Lo
ac nuo
c (ky hiu
a
. trong c
., l`
,
,
A) c
o khng t hon 330 dai
a ch
ung ta c
o th vit
. biu trong hi
. v`
,
s b
ao danh a1 < a2 < . . . < a330 < . . . Ch
ung ta xet nhung hiu
.
,

xi = a330 ai , i = 1, 2, . . . 329. Nu c
o mt
s
x
n`
a
o
d
o
tr`
u
ng
v
o
i
aj

i
.
,
, ,
(s b
ao danh cua mt
ung ta c
o a330 = ai + a j ,
aj i biu cua A) th` ch
. d
,
,
ng minh xong. Nu xi 6= a j vo
i moi
chu
b`
ai to
an d
a
a
. i, j, th` s xi l`
,
,
,
,
,
c c`
s b
ao danh cua mt
on lai.
a. i biu thuc
. s d
. 5 nuo
. By gi`o,
,,
v` 65.5 < 329 , th` t nht c
o mt
am nuo
c n`
ay (ky hiu
l`
a
. trong n
, .
,
B) se c
o khng t hon 66 th`
anh vin, m`
a s b
ao danh cua ho. l`
a
l`
mt
ac s x1 , x2 , . . . , x329 . Cho c
ac s d
a b1 < b2 < b3 <
o
. trong c
,
i bi = xni , i = 1, 2, . . . , 66. Ch
. . . < b66 < . . . vo
ung ta lai
. xet hiu
.
,

yi = b66 bi , i = 1, 2, . . . , 65 Nu mt
hi
u
n`
a
o
d
o
tr`
u
ng
v
o
i
s
b
a
o

.
.
,
, ,
,
i hai s i
danh b j cua mt
ai
. d
. biu cua B th` b66 = bi + b j . Nu vo
ch
ung ta c
o yi = ak , th` ak = b66 bi = xn66 xni =
v`
a k n`
ao d
o
. c l`
a330 an66 ( a330 ani ) = ani an66 hoa
a ani = an66 + ak . Nu hai
,
,`,
,
,
truong ho. p trn khng xy ra, th` nhung s n`
ay se l`
a s b
ao danh
,
,
,,
,,
cua d
on lai
a suy ra t nht mt
ac nuo
c n`
ay
ai
. biu 4 nuoc c`
. v`
. trong c
,
i s b
c
o s hi
a 17 vo
ao danh yi . Tip tuc
qu
a tr`nh
. vin t nht l`
, .
,
. p lai
nhu vy
a la
ung ta c
o kt lun
ai to
an.
. v`
. ly lun
. trn ch
. cua b`

,,
,,
. 5.9. (` thi Toan v dich
o
. nuo, c Anh, 1978) Mt
. h`nh lp
. phuong c
,
,
`
`
`
canh
ba
ng 15 chua 11.000 dim. Chung minh ra
ng c
o mt
.
. h`nh, cu
,
,
,
`

b
an knh ba
ng don vi. chua t nht 6 dim trong s 11.000 dim da
cho.
,
,
,,
canh
`ng
`,i giai. Chia mi
Lo
cua h`nh lp
anh 13 ph`n ba
.
. phuong th`
,
,
,
,
3
nhau, th th` h`nh lp
anh 13 = 2187
`u d
uo. c chia th`
. phuong ban d

5.2. V du.

51

,
,,
,,
,a 11.000
h`nh lp
phuong nho. V` h`nh lp
phuong ban d
`u chu
.
.
,
,
,
,,
,a t nht 6 d
d
im, nn t`n tai
im.
. mt
. h`nh lp
. phuong nho chu
,
,
15
,,
b
D thy canh
cua h`nh lp
a
, do d
an knh h`nh
o
.
. phuong nho l`
13r
r
,
1
15 2
1 675
,
,
c`u ngoai
ar=
3( ) =
<
. tip h`nh lp
. phuong nho l`
2
13
2 169
r
1 676
1
,a t
=
4 = 1. Vy:
an knh 1 chu
. T`n tai
. mt
. h`nh c`u b
2 169 , 2
,
cho.
nht 6 d
im trong 11.000 d
im d
a

,
,
,
. 5.10. Trong ma. t phang cho mt
im, mt
. , s huu han
. d
. s trong
,,
`ng d
`ng mt
ng. Ch
ch
ung d
tha
ung ta n
oi ra
ay
uo. c ni lai
oan
. ba
. d
,.
,
,
,
ng d
a mt
e1 , e2 , . . . , em t`u c
ac d
tha
oan
uo. c ni l`
.
. dy chuy`n, nu
,
. p d
ng li`n nhau ei v`
moi
a ei+1 c
o chung mt
ut,
oan
`u m
. ca
. tha
. d
,
,
,
,
ng trong dy chuy`n
i = 1, 2, . . . , m 1. S luo. ng m nhung doan
. tha
,
goi
ad
ai cua n
o. Ch
ung ta xet b`
ai to
an:
. d`
. l`
,
,
,
,
,,
`, c
,ng doan
Cho n dim, tu
ac dim n`
ay du. ng nhu
tha
ng, s luo. ng doan
.
.
,
,
,
,
,
, ,
,,

du. ng l`

tha
ng da
a q. Gia su nhu
ng doan
th
a

ng
d
u
o
c
d
a
nh
s
m
t
c
a
ch

.
.
.
,
`
trong c
bt k`y ba
ng c
ac s 1, 2, . . . , q. Khi do
ach tao
. h`nh nhu vy
. t`n
,
2q
,
tai
oi d
ai nho nht l`
a
, trn dy chuy`n n`
ay
. d`
. mt
. dy chuy`n v
n
,
,
,
,
,
ng doan
c
ac s ung v
oi nhu
ng th`
anh ph`n lp
ay s giam
. tha
. nn mt
. d
nga
. t.
,
,
,
,
,
`,i giai. Goi
ng,
Lo
a cu h`nh bt k`y cua nhung d
ad
im v`
oan
. G l`
. tha
,,
`ng s tu., nhin nhu, d
,i moi
nh s ba
ra. Vo
m`
a ch
ung d
ai d
uo. c d
a
` b`
a
.
,
`
v thuc
ung ta ky hiu
ad
ai cua dy chuyn d`
ai nht
d`
. G ch
. LG (v) l`
, .
,
,
,
t d
`ng phuong ph
trong G, m`
a n
o ba
a giam d`n. Ba
ap qui
`u t`u v v`
,
,
,
,
,,

ng thuc sau d
nap
ung ta se chung minh bt d
an
a
uo. c thoa m
. ch
LG (v1 ) + LG (v2 ) + + LG (vn ) 2q

(5.1)

52

,,
,,
Chuong 5. Mo rng
nguyn ly irichl
.

,
,
,
,,
,ng
od
a tt ca c
ac d
y v1 , v2 , . . . , vn l`
im thuc
. G. Kt lun
. phai chu
,,
,
minh suy ra t`u (5.1) theo nguyn ly irichl mo rng:
Mt
.
. trong
2q
,
,
,
`ng
n hon hoa
. c ba
c
ac s LG (v1 ), LG (v2 ), . . . , LG (vn ) lo
, nhu vy
.
n
,
,
,
,
,
ng
i tnh cht d
ch ra. Do d
ch c`
t`n tai
on phai chu
a
o
. dy chuy`n v,o
,
,
,
,,
,,
,
i n d
ng d
an vo
aqd
minh (5.1) d
im cua G v`
oan
uo. c
uo. c thoa m
. tha
,
,
,
,
, `
`ng qui nap
,i q = 1 bt d
nh s nhu m ta o d
d
ai. Ba
a
u b`
ang
. vo
,
,
,
,
,
,
c (5.1) l`
thu
a hin nhin. Gia su (5.1) thoa m
an cho moi
. cu, h`nh
,
,
,
i q 1 d
i n d
ng v`
vo
tha
a ch
ung ta xem xet cu h`nh vo
a
oan
im v`
, .
,
,
,
i hai d
ng. Nu d
ng ky hiu
qd
a q ni vo
a
oan
oan
im vi v`
. tha
. th, a
. l`
,
,
,,
0
ng n`
v j th` ch
ung ta bo d
ay v`
a nhn
oan
uo. c cu h`nh G cua n
. , tha
. d
,
,
i cu h`nh n`
ng. Vo
d
a q1 d
ay theo qui nap
ung ta
im v`
oan
. tha
. ch
,
c
o LG0 (v1 ) + LG0 (v2 ) + + LG0 (vn ) 2(q 1). By gi`o ch
ung ta
,
,
,
xet d
at t`u x j c
od
ai LG0 (v j ),
im v j . Dy chuy`n giam d`n xut ph
. d`
,
,,
,
,
,
,
ng trong dy chuy`n n`
v`
a nhung d
ay d
oan
uo. c ky hiu
. tha
. boi nhung
1, 2, . . . , q 1. Nu ni thm v`
trong dy
s n`
ao d
ao dy chuy`n
o
,
,,
ng c
mt
d
o ky hiu
ung ta se nhn
oan
uo. c dy chuy`n
. th,a
. q ch
. d
,.
,
,
i d
i d
giam vo
a vo
ai LG0 (v j ) + 1. Theo d
ngha
im d
`u vi v`
. d`
inh
.
,
`ng mt
,ng
cua LG (vi ) ch
ung ta c
o LG (vi ) LG0 (v j ) + 1. Ba
ach chu
. c
,,
,
,,
,ng minh d
minh ho`
an to`
an tuong tu. ch
ung ta c
ung chu
uo. c LG (v j )
,
,i moi
. t kh
L0G (vi ) + 1. Ma
ac, LG (vk ) = LG0 (vk ) vo
im vk thuc
. d
. G
m`
a n
o kh
ac vi v`
a v j . Suy ra, LG (v1 ) + LG (v2 ) + + LG (vn )
LG0 (v1 ) + LG0 (v2 ) + + LG0 (vn ) + 2 2(q 1) + 2 = 2q.

` tp
5.3. Bai
.
,

. 5.11. (` thi v dich


. o - Balan, 1978) Cho 1978 tp
. ho. p, mi
,,
,
,
`ng hai tp
ng mt
tp
o 40 ph`n tu. Bit ra
o du
. ho. p c
. ho. p bt k`y c
.,
,,
,,
,
`
`
`
`

ng minh ra
ng, tn tai
phn tu chung. Chu
. mt
. phn tu thuc
. tt ca
,
1978 tp
. ho. p trn.

5.3. B`
ai tp
.

53

,, ,
,
`ng moi
,ng minh ra
,a 55 s chon
d
. 5.12. Chu
uo. c t`u tp
. tp
. ho. p chu
.
.
,
,
,
`

a hai s m`
ng 9.
ho. p s {1, 2, 3, . . . , 100} chu
a hiu
ung ba
. cua ch
,
,
,
. 5.13. Cho n l`a s tu. nhin. Cho n tp
ho. p con A1 , A2 , . . . , An cua
.
,
,i tnh cht: vo
,i moi
. t pha
ng vo
ma
. i = 1, 2, . . . , n t`n tai
. n + 1 phep
,
,
,

tinh
ac anh cua Ai qua c
ac phep tinh
i mt
. tin sao ,cho c
. tin t`u, ng d
.
,
`
`
ng minh ra
ng tn tai
. t
khng c
od
mt
d
im chung. Chu
im trn ma
.
.
,
,
`m trong bt cu
, tp
ng, khng na
ao, i = 1, 2, . . . , n.
pha
. ho. p Ai n`
,
,
,i canh
. 5.14. Trong ph`n trong cua mt
ac d
ai 15 d
`u vo
on
. tam gi
. d`
,
`ng phu,o,ng ph
,ng minh
vi,
ung ta chon
ap bt k`y 111 d
im. Chu
. ch
. ba

,
,
,,
`ng t`n tai
,i d
ra
on vo
a n
o phu t nht 3 d
u`ong knh 3, m`
im
. h`nh tr`
,
,,
,
chon
trong s nhung d
im d
a
. o trn.

. 5.15. (` thi v dich


om
. Quc gia Bungari, 1977) Trong mt
. nh
,`,
,`,
,
nguoi, hai nguoi X v`
a Y goi
a quen nhau gi
an tip, nu ho. tru. c
. l`
,
,,
. c l`
tip quen nhau, hoa
a nu t`n tai
. mt
. dy truy`n nhung ngu`oi
Z1 , Z2 , . . . , Z p sao cho X v`
a Z1 quen nhau, Z1 v`
a Z2 quen nhau,...
,,
,
g`m 134 ngu`oi, c`
, Z p v`
a Y quen nhau. Cho bit nh
om d
on giua
o
,
,, ,
,,
nh
bit t nht c
mi
om nho 8 ngu`oi t`u nh
om d
o hai ngu`oi quen
a
,
,,
`ng c
,ng minh ra
gi
an tip. Chu
o mt
om nho 12 ngu`oi trong nh
om
. nh
,
,`,
,
,
. p hai ngu`oi trong nh
bit, m`
nguoi d
a moi
om nho n`
ay d
a
`u quen
. ca
nhau gi
an tip.

54

,,
,,
Chuong 5. Mo rng
nguyn ly irichl
.

, ,
CHUONG

` TP
BAI
. S HOC
. NNG CAO

,
, ,
6.1. .inh l
y co ban cua s hoc
.

, ,,
, ,
Trong ly thuyt s hoc
o mt
inh
. ly co ban:
. co so c
. d
,
,
,
,
`,a
Moi
on hon mt
o th phn tch ra tch c
ac thu
. s tu. nhin l
. d`u c
,
,
l`
s nguyn t v`
a phn tch do
a duy nht nu ta khng d y dn thu
, ,
`,a s.
tu. cua c
ac thu
,
Cho p1 , p2 , . . . , pn l`
a nhung s nguyn t kh
ac nhau. Trong
,
,,
,
chuong n`
ay ch
ung ta xet mt
., s kt lun
. v` v` tp
. ho. p M cua
,
,
`m trong
c
ac s tu. nhin m`
a n
o c
o th phn tch ra c
ac th`ua s na
,
, ,
,
,
i dang
{ p1 , p2 , . . . , pn }. Moi
o th biu din duo
. s x cua M c
.
x = p11 p22 . . . pnn

,,
od
a c
ac s nguyn khng m.
y 1 , 2 , . . . , n l`

6.2. V du.
,
,
`, tp
. 6.1. Tu
ach bt k`y 2n + 1 s. Chung minh
. ho. p M chon
. mt
. c
,
`,a chon
ra
`ng t`n tai
a tch cua ch
ung l`
a mt
. hai s trong tp
. vu
. m`
. s
,,
chnh phuong.
,
,
1 2
n
`ng mt
`,i giai. Nhn
Lo
a s
. xet ra
. s tu
. nhin x = p1 p2 . . . pn l`
,
,
,,
n. Ch
chnh phuong khi v`
a ch khi tt ca c
ac s m
ud
ung ta
`u cha

56

,,
Chuong 6. B`
ai tp
. s hoc
. nng cao

,
,,
, , ,
,i dang
,i b.
chon
biu din moi
ng vo
o trn v`
a cho tuong u
a
. s d
. vo
.
,,
, ,
a c
ac s du cua c
ac s m
u
n-s (1 , 2 , . . . , n ), o d
y 1 , 2 , . . . , n l`
, , ,
. c i = 1
tuong ung 1 , 2 , . . . , n khi chia cho 2. R
o r`
ang i = 0 hoa
,
,
,

tu. n s g`m
i moi
vo
a b. xp thu
. i = 1, 2, . . . , n. Vy
. (, 1 , 2 , . . . , ,n ) l`
,
,,
,
s 0 v`
a 1. Theo ly thuyt t ho. p, tt ca n-b. nhu vy
o s luo. ng
. c
,,
,
l`
a 2n , c`
on c
ac s ta d
o s luo. ng l`
a 2n + 1. Nhu vy
ang xet c
. t nht
p xp g`m s 0 v`
c
o 2 s trong ch
ung c
o c`
ung b. sa
a s 1 ging
, ,,

1
,i
l`
nhau. Gia su c
ac s d
a x = p1 1 p2 2 . . . pnn v`
a y = p1 p2 2 . . . pn n vo
o
,
,c sau c`
ng thu
ch
ung c
o ( 1 , 2 , . . . , n ) = ( 1 , 2 , . . . , n ) . a
ung c
o

c
ngha l`
a i = i , i = 1, 2, . . . , n . Do d
ac s m
u i v`
a i c
o c`
ung
o
,
,
,

n le nhu nhau voi bt k`y i = 1, 2, . . . , n. Khi d


1 + 1 ,
tnh cha
o

n v`
2 + 2 , . . . , n + n , l`
a c
ac s cha
a theo nhn
`u tch
. xet ban d
1 2
n
1 + 1 2 + 2
n + n
1 2
n
ng
xy = ( p1 p2 . . . pn )( p1 p2 . . . pn ) = p1
p2
. . . pn
d
u
,,

l`
a s chnh phuong.
,
,
,
,a n + 1 s trong tp
. 6.2. Chung minh ra`ng giu
hop M c
o th chon
.
.
,
,,
,,

duo. c mt
ai s m`
a tch cua ch
ung l`
a mt
. v`
. s chnh phuong.
,
,
,
tp
`,i giai. Cho x1 , x2 , . . . , xn l`
,i mi
Lo
a nhung s bt k`y cua M. Vo
.
,
,

con kh
ac trng {i1 , i2 , . . . , ik } cua tp
ung ta
. ho. p {1, 2, . . . n + 1},, ch
n c
xet c
ac xi1 , xi2 , . . . , xik (tt nhin s n`
ay c
ung thuc
M).
Bi
u
di
ac
.
,
,
,
,
tp
s n`
ay theo dang
a mi
. chun v`
. ho. p con {i1 , i2 , . . . , ik } cho tuong
,,
,
, ,
,
ng vo
i n-b. (1 , 2 , . . . , n ) , o d
a c
ac s du cua
u
y 1 , 2 , . . . , n l`
, , ,
,
,,
c
ac s m
u tuong u
ng 1 , 2 , . . . , n khi chia cho 2. Nhung s luo. ng
,
,
cua {1, 2, . . . , n + 1} l`
nhung tp
ac rng
a 2n+1 1, c`
on s
. con kh
,,
,
,
n
p g`m nhung s 0 v`
ac n-b. sa
a 1 l`
a 2 . Suy ra t`n tai
luo. ng c
. nhung
,
tp
ac trng kh
ac nhau {i1 , i2 , . . . , ik } v`
a { j1 , j2 , . . . , jl }
., ho. p con kh
,
, , ,
,
p cua
i c`
cua {1, 2, . . . , n + 1}, m`
a ch
ung tuong ung vo
ung mt
. n-b. sa
,
,
nhung s du. i`u n`
ay c
o ngha l`
a nu xi1 xi2 . . . xik = p11 p22 . . . pnn

v`
a x j1 x j2 . . . x jk = p1 1 p2 2 . . . pn n th` c
ac s m
u i v`
a i c
o c`
ung tnh

6.2. V du.

57

, ,
,
,
n le vo
i i = 1, 2, . . . , k. i`u n`
cha
ay c
o ngha l`
a tch cua nhung
,,
s xi1 , xi2 , . . . , xik , x j1 , x j2 , . . . , x jk l`
a chnh phuong. Nu {i1 , i2 , . . . , ik }
,
,,
,,
d
v`
a { j1 , j2 , . . . , jl } khng c
o ph`n tu chung, th` b`
ai to
an d
a
uo. c giai.
,,
,,
,
. p lai
ng
Tru`ong ho. p nguo. c lai,
u
. trong P = xi1 xi2 . . . xik x j1 x j2 . . . x jk la
. d
,

nhung s xs , m`
a s thuc
{i1 , i2 , . . . , ik } v`a { j1 , j2 , . . . , jl }. Ch
ung ta
, .
,
,
,
,
,,

loai
ac nhung xs nhu vy
a nhn
uo. c tch cua
. tru trong P tt ca c
. v`
. d
,,
ng l`
v`
ai s trong s x1 , x2 , . . . , xn+1 m`
a n
od
a chnh phuong.
u

,
,
,
,a moi
`, tp
. 6.3. Chung minh ra`ng giu
3.2n + 1 s tu
ho. p M c
o th
.
.
,
,
,,
`,a bc
chon
a tch cua ch
ung l`
a l
uy thu
. duo. c 4 s m`
. bn cua mt
. s.
,
`,i giai. V` 3.2n + 1 > 2n + 1 giu,a nhu,ng s d
chon
Lo
ai 6.1
a
. theo b`
,
,,

c
o hai s m`
a tch cua ch
ung l`
a mt
ach hai s
. s chnh phuong. T
,
n
p dung
trn ra th` c`
on lai
aa
tip 6.1 cho hai s nua m`
a
. 3.2 1 s v`
.
,
,
,,
. p lai
tch cua ch
ung l`
a s chnh phuong. Ch
ung ta c
o th la
qu
a
tr`
nh
.
,
,
(3.2n 1) (2n + 1)
,
n`
ay t nht
= 2n + 1 l`n. Gia su ch
ung ta nhn
.
2
,, n
,
,
,
i xi yi l`
. p ( x1 , y1 ), ( x2 , y2 ), . . . , vo
d
a c
ac s chnh phuong,
uo. c 2 + 1 ca
n
`ng theo c
. p s
i = 1, 2, . . . , 2 + 1. Ch
u y ra
ach chon
ac ca
. trn th` c
,
( x1 , y1 ), ( x2 , y2 ), . . . , ( x2n +1 , y2n +1 ) t`ung di mt
ac nhau. n d
y
. kh

,
suy ra c
ac s x1 y1 , x2 y2 , . . . , x2n +1 y2n +1 l`
a nhung s nguyn v`
a
,
,
,
`ng tch cua hai s
a
p dung
thuc
o
. M. Khi d
. mt
. l`n nua 6.1 ch ra ra
,,
,
4
trong ch
n`
ao d
ung l`
a chnh phuong. Nhu vy
a
o
. xi yi x j y j = t ngha l`
,
,
,
tch cua 4 s t`ung di mt
ac nhau xi , yi , x j , y j l`
a l
uy th`ua bc
. kh
. bn
,
cua mt
. s.
,
,ng s tu.,
. 6.4. Cho p l`a mt
a a1 , a2 , . . . , a p+1 l`
a nhu
. s nguyn t le v`
2
, ,
,
,
,
nhin kh
ac nhau nho hon p. Chung minh ra
`ng v
oi moi
. s tu. nhin
, ,
,
,
r nho hon p, t`n tai
o th ba
`ng nhau) ai v`
a a j , m`
a tch cua
. hai s (c
,
ng l`
ch
ung khi chia cho p c
o s du d
a r.
u

,,
Chuong 6. B`
ai tp
. s hoc
. nng cao

58

,
, ,,
`,i giai. p dung
mt
Lo
ab
inh
. ly trong ly thuyt s co so: Nu a v`
.
. d

l`
a hai s nguyn t c`
ung nhau, th` tn tai
. mt
. s nguyn x sao cho
ax 1 (mod b).
i = 1, 2, . . . , p + 1 , c
,i mi
ac s ai v`
a p l`
a nguyn t c`
ung
Vo
2
,
nhau, v` 1 ai p 1 v`
a p l`
a s nguyn t. p dung
d
ly v`ua
inh
.
.
,
,i moi
ph
at biu trn, t`n tai
ac s nguyn b1 , b2 , . . . , b p+1 sao cho vo
.
. c
2
,
,
,
p +1
c sau d
ng thu
i = 1, 2, . . . , 2 c
ac d
an
a
`u thoa m
a i bi 1

(mod p)

(6.1)

Ch
ung ta xet d
ay s a1 , a2 , . . . , a p+1 , rb1 , rb2 , . . . , rb p+1 . Ch
ung ta c
o
2
2
,
tt ca p + 1 s sao cho t nht hai s trong ch
ung khi chia cho p
,
,
c
o c`
ung mt
ac s a1 , a2 , . . . , a p+1 kh
ac nhau
. s du. Theo gia thit c
2
, ,
,
,
ho`
an to`
an v`
a nho hon p. Suy ra s du cua ch
ung theo mdd un p
,i p, nn d d`
l`
a kh
ac nhau. V` r l`
a s nguyn t c`
ung nhau vo
ang
,
,
,
,
`
ng minh d
ng c
chu
ac s rb1 , rb2 , . . . , rb p+1 cho s du ho`
an to`
an
uo. c ra
2
,
,
kh
ac nhau khi chia cho p. Ngha l`
a ch t`n tai
a j sao
. hai ch s i v`
,
,
,
cho ai rb j (mod p). T`u (6.1) ch
ung ta nhn
uo. c ai a j rb j a j
. d
r.1 r (mod p).
,
,
,
,
. 6.5. B d` Tue: Cho n l`a s tu. nhin l
on hon 1 v`
a a l`
a s nguyn
,
t`n tai
t c`
ung nhau v
oi n. Khi do
ac s nguyn x v`
a y, m`
a ch
ung
. c
,
,,
,
,
thoa m
an phuong tr`nh ax y (mod n) v`
a c
ac bt phuong tr`nh

1 x [ n ], 1 | y | [ n ].

,
,
,,
`,i giai. Ch
Lo
ung ta xet tt ca c
ac s c
o dang
au v, o d
a v chay
y u v`
.
.

,
,
,

d
c
l
p
nhau
trong
c
a
c
s
0,
1,
2,
.
.
.
,
[
n
]
.
S
lu
o
ng
t
t
c
a
c
a
c
s
d
o
. .

. 2
2
,
,
2
n hon n v` ([ n] + 1) > ( n) = n.
l`
a ([ n] + 1) . Suy ra s n`
ay lo
,,
,
. p
ng n s du kh
Boi c
od
ac nhau theo md
u
un n, nn t`n tai
. hai ca
,
s kh
ac nhau (u1 , v1 ), (u2 , v2 ) t`u c
ac s nguyn sao cho 1 ui

6.2. V du.

59

[ n], 1 vi [ n], i = 1, 2 v`a au1 v1 au2 v2 (mod n).


,,
,,
,ng minh d
D d`
ang chu
ac v2 . Tht
uo. c v1 kh
. vy,
. nu nguo. c lai
. th`

au1 au2 (mod n),suy ra u1 u2 (mod n), do a l`


a s nguyn t
, ,
,
,
,
i n. Ma
. t kh
ac, u1 , u2 l`
a c
ac s tu. nhin nho hon n. Nhu
c`
ung nhau vo
,
,
,
,c sau c`
ng thu
vy
ung ch c
o mt
ang duy nht u1 = u2 . Do
a
. d
. kha n
,
,i c
. p (u1 , v1 ), (u2 , v2 ) tr`
ca
d
ung nhau, tr
ai vo
ach chon
trn. Nhu vy
o
.
.
,
, ,

u1 v`
a u2 c
ung phai kh
ac nhau. C
o th gia thit u1 > u2 m`
a khng
,
,
, ,,
,
,

`
. t x = u1 u2 , y =
anh huong d
n kt qua chung minh. By gio ta d
a

,
`
suy ra 1 x [ n], 1 |y| [ n] v`
v 1 v 2 . Tu d
a ax y
o
(mod n).
, ,
,
. 6.6. p dung
d` Tue: Moi
s nguyn t dang
4k + 1 c
o th biu
.
.
. b
,
,
,,
du,o,i dang
din
. tng b`nh phuong cua hai s nguyn.

,
,
`,i giai. Ch
,ng minh kha
ng d
Lo
ung ta chu
a s
inh
. sau: Nu p = 4k + 1 l`
,
,
2
nguyn t, th` phuong tr`nh x 1 (mod p) c
o nghim.
. Tht
. vy,
.
V` p l`
a s nguyn t, theo d
l
y
Wilson,
(
p

1
)
!

1
(
mod
p
).
inh
.
p +1
Ngo`
ai ra p 1 1 (mod p), p 2 2 (mod p), . . . , p 2
p +1
2 (mod p ). Suy ra,

( p 1) ( p + 1)
2
2
p1 2
1
1
) )
(1 )(2 ) . . . ((
2
p 1
p1 2
p1 2
= (1) 2 (1.2 . . .
) = (1)2k ((
)!)
2
2
p1 2
= ((
)!) (mod p).
2
,
p1
,
,,
a nghim
Nhu vy
. cua phuong tr`nh trn. V` a2 + 1
. a = ( 2 )! l`
,i p. By gi`o, ta a
p
chia ht cho p nn a l`
a nguyn t c`
ung nhau vo
,
dung
b d
a p. T`n tai
ac s nguyn x v`
a y m`
a
` Tue cho hai s a v`
.
. c
1 ( p 1)! = 1.2 . . . .

60

,,
Chuong 6. B`
ai tp
. s hoc
. nng cao

a2 x 2 y2
1 x [ p], 1 |y| [ p] v`
a ax y (mod p). Khi do
,
,
2
2
2

(mod p), nhu vy


o
. x y (mod p), v` a 1 (mod p). T`u d
2
2
2
2

. t kh
suy ra x + y chia ht cho p. Ma
ac 1 x p, 1 y p. Ta
, ,
2
2
`

ng x = p v`
thy ra
a y = p khng th xy ra v` p l`
a s nguyn t, suy
ra 0 < x2 + y2 < 2p v`
a v` x2 + y2 chia ht cho p, nn x2 + y2 = p.

,,
,
,,
`ng moi
,ng minh ra
: Tuong tu. , chu
Ch
uy
. s, p,, sao cho phuong
,,
tr`nh x2 2 (mod p) c
o nghim,
d
o th biu din duo
i dang
`u c
.
.
,
2
2
i x, y l`
p = x + 2y vo
a c
ac s nguyn.

. 6.7. (` thi Toan Olympic quc t l`n 18 nam 1976) Cho h. p


,
,,
,
phuong tr`nh v
oi q = 2p n
a11 x1 + a12 x2 + + a1q xq = 0
.....................
a p1 x1 + a p2 x2 + + a pq xq = 0
,
,
,
Tt ca c
ac h. s aij thuc
`ng t`n tai
. ,tp
. hop {1, 0, ,1}. Chung minh ra
.
nghim
a n
o thoa m
an
. ( x1 , x2 , . . . , xq ) cua h,
. m`
,
,
a) tt ca x j ( j = 1, 2, . . . , q) l`
a nhung s nguyn;
b) t nht c
o mt
a x j 0;
. j( j = 1, 2, . . . , q) m`
,
c) v
oi moi
o x j q;
. j( j = 1, 2, . . . , q) ta lun c
,
`,i giai. Xet b. ( x1 , x2 , . . . , xq ) g`m nhu,ng s nguyn bt k`y, m`
Lo
a
,
,
,,
ch
ung thoa m
an | x1 | p, | x2 | p, . . . , | xq | p. Boi v` tt ca c
ac
,
,
,
,
,
,
i vic
. c 1, vo
h. s cua h. phuong tr`nh ch l`
a -1,0 hoa
thay c
ac n
.
,
,,
phu,o,ng tr`nh na
`m
x1 , x2 , . . . , xq ch
ung ta nhn
d
a tri. cua mi
uo. c gi
.
,
,
i = 1, 2, . . . , p ch
i mi
trong khoang [ pq, pq]. Tht
ung ta
. vy,
. vo
c
o | ai1 x1 + ai2 x2 + + aiq xq | | x1 | + | x2 | + + | xq | pq. Suy
,
,
,
,
,,
, , ,
ra, nu thay nhung n trong tt ca phuong tr`nh cua h. tuong u
ng

6.2. V du.

61

,,
,i x1 , x2 , . . . , xq se nhn
vo
d
uo. c b. p s nguyn (y1 , y2 , . . . , y p ). Tt
.
,
,
,
,
`m trong khoang [ pq, pq]. Trong khoang
ca nhung s n`
ay d
`u na
,
,
p xp
ng 2pq + 1 s nguyn. Suy ra giua nhung b. sa
n`
ay c
o d
u
,,
,,
,
. t kh
p ph`n tu nhu o trn c
o (2pq + 1) p b. xp kh
ac nhau. Ma
ac
,,
,
,
,

s luo. ng nhung b. xp q-phn tu ( x1 , x2 , . . . , xq ) m`


a |xj | p
,
,
q

ng tu q = 2p suy ra
voi j = 1, 2, . . . q, l`
a (2p + 1) . D thy ra
q
p
(2p + 1) > (2pq + 1) . Theo nguyn ly irichl t`n tai
. hai b. q0
0
0
00
00
00

s nguyn ( x1 , x2 , . . . , xq ) v`
a ( x1 , x2 , . . . , xq ) kh
ac nhau m`
a ch
ung
,
,
0
00

thoa m
an | x j | p, | x j | p voi j = 1, 2, . . . q v`
a sau khi th v`
ao
,,

. t
h. phuong tr`nh cho c`
ung mt
. b. p s nguyn (y1 , y2 , . . . , y p ). a
0
00
0
00
0
00
x1 = x1 x1 , x2 = x2 x2 , . . . , xq = xq xq . R
o r`
ang ( x1 , x2 , . . . , xq )
,
,,
,i moi
l`
a nghim
a x j l`
a c
ac s nguyn vo
. cua h. phuong tr`nh v`
.
0
0
0
00
00
00
j = 1, 2, . . . , q. V` ( x1 , x2 , . . . , xq ) v`
a ( x1 , x2 , . . . , xq ) l`
a hai b. q s
ho`
an to`
an kh
ac nhau, th` t nht mt
ac s x j kh
ac khng.
. trong c
,

Cui c`
ung voi moi
ung ta c
o | x j | = | x 0j x 00j |
. j = 1, 2, . . . , q ch
,
,
| x 0j | + | x 00j | p + p = 2p = q. Nhu vy
a nghim
. ( x1 , x2 , . . . , xq ) l`
. cua
,,
o tnh cht mong mun.
h. phuong tr`nh c

. 6.8. (` thi Toan Olympic quc t l`n 28 nam 1987) Cho


,
,ng s thu.,c v`
x1 , x2 , . . . , xn l`
a nhu
a x12 + x22 + + xn2 = 1. Chung minh
,
,ng s nguyn a1 , a2 , . . . , an
ra
`ng v
oi mi
s nguyn k, k 2, t`n tai
. nhu
, `
khng d`ng th`oi ba
ng khng sao cho | ai | k 1, i = 1, 2, . . . , n v`
a
,
,
,
(
k

1
)
n
thoa m
an bt phuong tr`nh sau | a1 x1 + a2 x2 + + an xn | kn 1
,
,
,,
`,i giai. Ch
,c Cosi-Buniakovski-Svars
ng thu
Lo
ung ta su dung
bt d
a
.
q
q
|1 1 + |2 2 + + |n n | 21 + + 2n 21 + + 2n
,
,
`ng xy ra
,i moi
,d
s thu. c 1 , 2 , . . . n , 1 , 2 , . . . , n . Du ba
us ng vo
.
,
khi v`
a ch khi t`n tai
. mt
. s sao cho 1 = 1 , 2 = 2 . . . , n =
n .

,,
Chuong 6. B`
ai tp
. s hoc
. nng cao

62

,
By gi`o ch
ung ta xet c
ac s y0 = k2 1 , y1 = k2 1 + 1, . . . ,
,
,,
,
ung l`
a k v`
a hiu
yk1 = k2 1 + (k 1) = k2 1 S luo. ng cua ch
. , t`ung
,
. p trong ch
ca
ung l`
a nhung s nguyn, m`
a gi
a tri. tuyt
o
i cua n
. d

`
p xp n-th`
khng qu
a k 1. Mi b. sa
anh phn = (b1 , b2 , . . . , bn ),
,,

,i moi
trong y1 , y2 , . . . , yn vo
od
a mt
ao d
y bi l`
o
. s n`
. i = 1, 2, . . . , n,
, , ,
,

. t tuong ung voi mt


ch
ung ta d
a
. s S = b1 x1 + b2 x2 + + bn xn .
, ,
,
`

ng thuc Cosi
Tu bt d
a
S = b1 x1 + b2 x2 + + bn xn
q
q
2
2
2
b1 + b2 + + bn x12 + x22 + + xn2
q
= b12 + b22 + + bn2
,
Nhung |bi |

,i moi
vo
. i = 1, 2, . . . , n sao cho
q
k 1
|S | b12 + b22 + + bn2
n
2

,,
. c l`
a k2 1 n S k2 1 n. Theo phuong ph
hoa
ap n`
ay n-b. =
,,

,i moi
(b1 , b2 , . . . , bn ), o dy bi l`a mt
ac s y0 , y1 , . . . , yk1 vo
. trong c
.
,,
, , ,
,

i = 1, 2, . . . , n, d
uo. c d
a. t tuong ung voi mt
. s S = b1 x1 + b2 x2 +
k 1
,,
,,
k 1
+ bn xn trong doan
. d
uo. c
. = [ 2 n, 2 , n]. S luo. ng n-b
,
p xp l`
,i d
sa
a kn . Chia ra kn 1 d
ai kkn11 n. T`u
oan
. d`
. nho vo
nguyn ly irichl suy ra t`n tai
a
. hai n-b. = (b1 , b2 , . . . , bn ) v`
,
= (c1 , c2 , . . . , cn ), m`
a nhung s S = b1 x1 + b2 x2 + + bn xn
, , ,
`m trong
,i ch
v`
a S = c1 x1 + c2 x2 + + cn xn tuong u
ng vo
ung na
,
. t a1 = b1 c1 ,a2 = b2 c2 , . . . , an = bn
c`
ung mt
oan
. ,d
. nho. a
,
,
,
cho. Tht
cn . D kim tra d
an d
uo. c a1 , a2 .., an thoa m
i`u kin
a
. ,d
.
,

vy,
v
o
i
m
oi
i
=
1,
2,
.
.
.
,
n
s
a
=
b

c
l`
a
hi
u
c
ua
hai
s
n`
a
o
i
i
i
.
.
.
,,
,
trong y0 , y1 , . . . , yk1 nhu d
n
d
oi o trn v`
a l`
a s nguyn khng
o
a
,,
vuo. t qu
a k 1. V` hai n-b. trn kh
ac nhau ho`
an to`
an th` t nht
mt
ac s ai = bi ci kh
ac khng. Ngo`
ai ra, |b1 x1 + b2 x2 +
. trong c
k 1
2

6.2. V du.

63

+ bn xn | = | x1 (b1 c1 ) + x2 (b2 c2 ) + + xn (bn cn )| = |S

S | kkn11 n.

,
b. g`m 11 s thu.,c kh
. 6.9. Chung minh ra`ng mi
ac nhau trong
,
,
,
,,
khoang [1,1000] c
o th chon
duo. c hai s x v`
a y, m`
a ch
ung thoa m
an
.
,
,
bt da
ng thuc sau

0 < x y < 3 3 xy
(6.2)
,
,
`,i giai. Ch
cho
Lo
ung ta xet c
an bc
ac s trong b. s d
a
. ba cua c

,
3
`
cho suy ra 1
xi 10, i =
x1 , x2 , . . . , x11 T`u d
iu kin
a
. d
,
,`,
`ng nhau.
`
1, 2, . . . , 11. Ch
ung ta chia khoang [1,10] ra muoi phn ba

`m trong
, c
Khi d
o t nht mt
trong hai s 3 x1 , 3 x2 , . . . , 3 x11 na
o
, .

l`
c`
ung mt
ac s d
a 3 xi v`
a 3 x j , i 6= j v`
a xi > x j ,
oan
o
. d
. nho. Nu c
ch
ung ta c
o

p
9
< 1.
(6.3)
0 < 3 xi 3 x j
10

,
,
,
3
3
i (6.3) ta c
o 0 < xi
Nhu vy,
. 0 < ( xi 3 x j ) < 1, kt ho. p vo

3
x j < 1 + 3 3 xi x j ( xi 3 x j ) < 1 + 3 3 xi x j .
,
,i b. 10 s thu.,c trong khoang
ng vo
: B`
Ch
uy
ai to
an khng c`
on d
u
,
3 3
3
[1,1000]. Phan v du:
a i > j ch
ung
. b. s 1 , 2 , . . . , 10 . nu i 6= j v`
,
3
2
2
ta c
o (i j) 1, nhu vy
a ij =
. i +, ij + j ,1 + 3ij. Ngha l`
,
2
2
c xy ra khi i = j + 1.
(i j)(i + ij + j ) 1 + 3ij. dang thu
,
,a ch
. 6.10. Cho 7 s thu.,c bt k`y. Chu,ng minh ra`ng giu
ung c
o th
,
,,
chon
ng han
a y, sao cho
. duo. c hai s, cha
. x v`

3
xy

.
0
1 + xy
3

,
,
`,i giai. C
cho ky hiu
Lo
ac s d
l`
a x1 , x2 , . . . , x7 . Muc
d
ung
ch cua ch
a
.
.
,
,,
,,

ta l`
a biu din moi
s
du
o
i
d
ang
x
=
tg

,
o
d
y

l`
a
m
t

i
i
i
.
.
. s
,

trong khoang ( , ), i = 1, 2, . . . , 7. Ch
ung ta chia d
ay ra
oan
. n`
2 2

64

,,
Chuong 6. B`
ai tp
. s hoc
. nng cao

`ng nhau, ngha l`


`ng . D d`
s
au d
od
ai ba
a ba
ang thy
oan
. d`
. con c
6
`ng t nht c
`m trong mt
ra
o hai s trong 1 , 2 , . . . , 7 c`
ung na
. doan
.
,

`
. Nu ch

coni do
ung ta ky hiu
c
a
c
s
d
o
l`
a

v`
a

,
th`

t
u
d
o

i
j
i
j
.
,

am s tg l`
a t
ang trong khoang ( 2 , 2 ),
suy ra 0 i j 6 . V` h`
suy ra

tg i tg j
xi x j

3
0 tg(i j ) =
=
tg =
.
1 + tg i tg j
1 + xi x j
6
3

, ,
,
`ng c
: Ba
Ch
uy
ach giai cua hai b`
ai tp
ay ch
ung ta c
o th s
ang
. n`
,
,,
,
,
,
,
i c
tao
ai to
an tuong tu. , m`
a vo
ach giai b`nh thu`ong kh
o
. ra,mt
. loat
. b`
,,

m`
a giai quyt d
uo. c.

` tp
6.3. Bai
.
. 6.11. (` thi Toan Olympic quc t l`n 26 nam 1985). Cho tp
.
,
,, ,
,
,
ho. p M g`m 1985 s tu. nhin, khng c
o s n`
ao c
o uo
c s lon hon
,
,
,
,,
`ng t`u, nhu,ng ph`n tu, cua M c
,ng minh ra
26. Chu
o th chon
uo. c 4
. d
,
,
,
,
s t`ung d
ac nhau m`
a tch cua ch
ung l`
a l
uy th`ua bc
i mt
. kh
. 4 cua
,
9
,i tp
ng vo
mt
u
. s nguyn. (k,t lun
. d
. ho. p g`m 3.2 + 1 = 1537 s
,
,,
m`
a nhung uo
c s cua ch
ung khng qu
a 23).
,
`ng c
,ng minh ra
. 6.12. Cho bn s duo,ng bt k`y. Chu
o hai s trong
,
,
,
,
ng han
, cha
bn s d
a y, thoa m
an bt phuong tr`nh sau
o
. x v`

xy
0
2 3.
1 + x + y + 2xy
, ,
,
`ng mt
,ng minh ra
. 6.13. Chu
s tu. nhin c
o th biu din th`
anh
.
,
,
,
,,
tng b`nh phuong cua hai s nguyn khi v`
a ch khi, trong phn tch
,
,
c cua n
ra dang
o, c
ac th`ua s nguyn t dang
o
`u c
. chnh ta
. 4k + 3 d
n.
s m
u cha

6.3. B`
ai tp
.

65

, ,
`ng t`u, k + 1 s, m`
,ng minh ra
a ch
ung nho hon 2k, lun
. 6.14. Chu
,
,
,
,,
,
lun c
o th chon
d
a ty s cua ch
ung l`
a mt
l
uy th`ua
uo. c hai s m`
.
.
,
cua 2.
,
`ng t`u, (n 1)2 + 1 s
,ng minh ra
. 6.15. Cho n l`a mt
., s le. Chu
,
,
,,
nguyn bt k`y c
o th chon
ung chia
. duo. c n s sao cho tng cua ch

ht cho n.

66

,,
Chuong 6. B`
ai tp
. s hoc
. nng cao

, ,
CHUONG

` TP
BAI
. DY S NNG CAO

7.1. V du.
,
. 7.1. Cho day Fibonaxi u1 = u2 = 1, un = un1 + un2 . V
oi moi
. s
,
,,
,
,
,

a nhu
ng s hang
` d`u dy dn s hang
nguyn duong m, th` giu
. thu
. tu
m2 1 t`n tai
. mt
. s hang
. chia ht cho m.
,
, ,
`,i giai. Ch
Lo
ung ta ky hiu
k l`
a ph`n du cua s k cho m. Ch
ung ta
.
,
,
. p ph`n du cua d
xet c
ac ca
ay Fibonaxi khi chia cho m:

( u 1 , u 2 ). ( u 2 , u 3 ), ( u 3 , u 4 ), . . . , ( u n , u n +1 ), . . .

(7.1)

`ng nhau khi


. p ( a1 , b1 ) v`
Nu ch
ung ta qui d
a ( a2 , b2 ) l`
a ba
inh
. hai ca
,
,
,
,
. p ph`n du
a1 = b1 v`
a a2 = b2 , th` s tt ca kha n
ang cua c
ac ca
khi chia cho m l`
a m2 . V` th ch
ung ta ly m2 + 1 s hang
d
`u tin
.
,
,
. p tr`
cua d
ay (7.1) th` trong ch
ung phai c
o hai ca
ung nhau,( theo
, ,,
,
. p sau la
. p lai.
. p (uk , uk+1 )
c
nguyn ly irichl ) v`
a t`u d
ac ca
o
. Gia su ca
,
`ng
ng minh ra
. p d
. p lai
l`
a ca
ung ta se chu
`u tin la
. trong (7.1). Ch
, ,,
,
,
`ng ca
. p n`
. p (1, 1). Tht
. p
ca
ay ba
a ca
. vy,
. gia su nguo. c lai,
. ngha l`
,
i k > 1. Khi d
. p lai
ch
d
a ( u k , u k + 1 ) , vo
ung ta se t`m
o
`u tin la
. l`
,,
`ng (uk , uk+1 )
. p (ul , ul +1 ), (l > k ), m`
d
a n
o ba
uo. c trong (7.1) mt
. ca
,
. Nhu vy
a uk1 = uk+1 uk , do ul +1 = uk+1 v`
a
. ul 1 = ul +1 ,ul v`
,
`ng
ul = uk , nn ph`n du cua ul 1 v`
a uk1 khi chia cho m c
ung ba
,
suy ra (uk1 , uk ) = (ul 1 , ul ), vy
nhau, ngha l`
a ul 1 = uk1 . T`u d
o
.

,,
s nng cao
Chuong 7. B`
ai tp
. dy
,, ,
`m trong d
. p (uk1 , uk )na
n
ca
ay (7.1) truo
c ca (uk , uk+1 ). i`u d
oi
o
,
,
,
`
`
i gia thit
ng (uk , uk+1 ) khng phai ca
p la
p lai d
ln ra
ai vo
u tin, tr
,. , . .
. t ra. Ngha l`
d
a k > 1 khng th xay ra, vy
a
. k = 1.
, ,,
,
. p (1, 1) l`
. p d
. p lai
Nhu vy
a ca
o
`u tin la
. ca
. trong (7.1). Gia su n
2
. p lai
. c l`
la
a (ut , ut+1 ) = (1, 1). Ngha l`
a ut
. t l`n (1 < t < m + 1). hoa
,
,
v`
a ut+1 khi chia cho m cho ph`n du c`
ung l`
a 1, vy
ung
. hiu
. cua ch
,
,
,
t1
chia ht cho m. Nhung ut+1 ut = ut1 . Nhu vy
. s hang
. thu
,
cua d
ay Fibonaxi se chia ht cho m.
68

, ,,
,
, , ,
,
. 7.2. Gia su a v`a x l`a cac s tu. nhin thu. c su. l
on hon 1 v`
a ( x, a 1) =
,
,
,
1. D
ay s v han
ac dinh
uo. c x
. {un } d
. nhu sau
un = ax n a + 1, n = 1, 2, . . .
,
,
Chung minh ra
`ng trong d
ay s n
oi trn chua v han
. s di mt
. nguyn

t c`
ung nhau.
,
,
,
,
,
`,i giai. Gia thit phan chu
,ng trong d
Lo
ay s ch c
o huu han
. s
u i1 , u i2 , . . . , u i k d
ung nhau.
i mt
. nguyn t c`
. t q = ui1 ui2 . . . uik . Xet (q + 1) s sau a, ax, ax2 , . . . , ax q . Theo
a
nguyn ly irichl t`n tai
. hai s nguyn r, s sao cho 0 r < s q
v`
a
axr ax s

(mod q) = axr ax s 0 (mod q) hay

axr (1 x sr ) 0 (mod q)
,
Theo gia thit ta c
o ( x, a 1) = 1, nn suy ra

(7.2)

( axr , ui j ) = 1, j = 1, 2, . . . , k.

(7.3)

( axr , q) = 1.

(7.4)

,
T`u (7.3) suy ra

7.1. V du.

69

,
,i l N.
a (7.4) c
o x sr 1 (mod q) = x sr = lq + 1 vo
T`u (7.2) v`
Xet s uik+ j = ax sr a + 1. Vy
.
uik+ j = a(lq + 1) a + 1 = qal + 1.

(7.5)

,
T`u (7.5) ta c
o

(uik+ j , ui j ) = 1, j = 1, 2, . . . , k.
(7.6)
,
, ,
`ng lun c
,c (7.6) chu
,ng to ra
H. thu
o th b sung thm v`
ao b. s
,
,
thoa m
i, m`
q = ui1 ui2 . . . uik c
ac s mo
a b. s n`
ay vn
an d
i`u kin:
.

`
bt k`y hai s n`
ao c
ung nguyn t c`
ung nhau. iu n`
ay c
o ngha l`
a

cho c
trong d
ay {un } d
o v han
ung nhau.
a
i mt
. s d
. nguyn t c`

,
. 7.3. Cho {un } l`a day cac s tu. nhin tang d`n: u1 < u2 < u3 < . . .
,
,
,
v`
a thoa m
an di`u kin
oi moi
a s tu. nhin.
. u1 = 1, un+1 2n, v
. n l`
,
,
,
Chung minh ra
`ng v
oi moi
s tu. nhin n t`n tai
ac s hang
a uq
.
. c
. u p v`
,
cua d
ay sao cho u p uq = n.
,
, ,
,
,,
, ,
`,i giai. Gia su, n N l`
Lo
a s tu. nhin cho truo
c. T`u gia thit suy ra
,,
,
s hang
mi
u1 , u2 , . . . , un+1 khng vuo. t qu
a 2n. Xet tp
.
. ho. p 2n s
,
,
. p
tu. nhin sau {1, 2, . . . , 2n}. Ch
ung ta chia tp
ay ra l`
am n ca
. ho. p n`
,
,
a khng t ho,n
(1, n + 1), (2, n + 2), . . . , (n, 2n). Do tp
ho. p trn chu
.
,, ,
(n + 1) ph`n tu cua day {un } da cho (v` noi ring u1 , u2 , . . . , un+1 da
,
thuc
. tp
. ho. p y), vy
. , theo nguyn ly irichl t`n t,ai
. hai s hang
.
,,
. p ( gia su u p > uq ).
kh
ac nhau u p v`
a uq cua d
ay thuc
ao mt
. v`
. ca
,
,

`
. p d
ng n, nn ch
Nhung hiu
ung ta c
o u p uq = n.
u ba
. s cua mi ca

,
. 7.4. Cho {uk }, k = 1, 2, . . . , n l`a day s tu. nhin sao cho
1 u1 u2 . . . un v`
a u1 + u2 + + un = 2n

,,
s nng cao
Chuong 7. B`
ai tp
. dy

70

,
`, d
Chung minh ra
`ng nu n cha
n v`
a un 6= n + 1, th` tu
ay trn lun chon
.
,
,
,,
`

ba
ra duo. c mt
ay con m`
a tng c
ac s hang
ay con do
ng n.
. d
. cua d
,
`,i giai. a
. t Sk = u1 + u2 + + uk , k = 1, 2, 3, . . . , n. Xet n + 2 s
Lo
{0, u1 un , S1 , S2 , . . . , Sn } Theo nguyn ly irichl th` t nht hai s
,
,
,
khi chia cho n c
o c`
ung ph`n du. Vy,
o 4 kha n
ang sau d
y:
. ch c
1) (u1 un ) chia ht cho n.
Do u1 + u2 + + un nu1 = 2n nu1 = u1 2
,
a) Nu u1 = 2 th` t`u 1 u1 u2 . . . un v`
a u1 + u2 + +

n nn n = 2m vy
un = 2n suy ra u1 = u2 = . . . = un = 2. Do n cha
.
u1 + u2 + + um = 2m = n
,
. c
b) Nu u1 < 2 th` t`u u1 un chia ht cho n, suy ra un = 1 hoa
l`
a un = 1 + n ( do u1 nguyn nn u1 = 1 v`
a 1 un 2n suy ra
,,
,
. t kh
ac
d
uo. c kt lun
. trn). Nhung un 6= n + 1 suy ra un = 1. Ma
1 u1 u2 . . . un vy
. th` u2 = u3 = . . . = un1 = 1. Suy ra
u1 + u2 + + un = n, v ly.
,
,
,,
,
Nhu vy
ay, ta ch ra t`n tai
ay con u1 , u2
. trong tru`ong ho. p n`
. d
n
,
i m = .
, . . . , um sao cho u1 + u2 + + um = n vo
2
2) S j Si , ( j > i ) chia ht cho n.
,
, ,
ng
Ta c
o S j Si = ui+1 + ui+2 + + u j . R
o r`
ang v phai cua d
a
,c trn c
thu
o t nht mt
m`
a uk 1, k = 1, 2, . . . , n, suy ra
. s hang
.
,
,, ,
. t kh
S j Si 1. Ma
ac c
ung hiu
ac ph`n tu cua
u c
. trn nu khng d
,
d
ay th` bao gi`o ta c
ung c
o S j Si < u1 + u2 + + un 2n 1.

cui c`
Do d
ung ta c
o 1 S j Si < u1 + u2 + + un 2n 1
o
, ,

. c
m`
a S j Si chia ht cho n. d
ay ch xy ra khi S j Si = n hoa
i`u n`
l`
a ui+1 + ui+2 + + u j = n.
3) Si chia ht cho n.

7.1. V du.

71

Ta c
o 1 Si Sn1 = 2n un < 2n m`
a Si chia ht cho n, suy
. c l`
ra Si = n hoa
a u1 + u2 + + ui chia ht cho n.
,
,i k n`
4) Sk v`
a u1 un cho c`
ung ph`n du khi chia cho n, vo
ao
, 1 k n 1. Suy ra Sk (u1 un )|n = (u2 + u3 + +
d
o
uk + un )|n. M`
a u2 + u3 + + uk + un 2n u1 < 2n. Suy ra
u2 + u3 + + uk + un = n.
,
,
,,
`ng
ay con m`
a tng cua ch
ung ba
T
om lai
uo. c d
. lun lun chon
. d
n.

,
,
. 7.5. Cho day s nguyn u1 , u2 , . . . , un v
oi n 2. Chung minh ra
`ng
1 k1 < k2 < . . . < k m n
t`n tai
ay con uk1 , uk2 , . . . , ukm trong do
. d
2
2
2
sao cho uk1 + uk2 + + ukm chia ht cho n.
,
`,i giai. Xet (n + 1) s
Lo
0, u21 , u21 + u22 , u21 + u22 + u23 , . . . , u21 + u22 + + u2n
,
Chia c
ac s n`
ay cho n, th` ch
ung cho nhi`u nht n s du. Theo
,
,
,
,
l`
nguyn ly irichl t`n tai
ung s du, gia su d
a u21 +
o
. hai s cho c`
u22 + + u2j v`
a u21 + u22 + + u2k (0 j < k n). C
o ngha
2
2
2
2
2
2
l`
a s (u1 + u2 + + u j ) (u1 + u2 + + uk ) chia ht cho n.
,
. c l`
a u j+1 + u j+2 + + uk chia ht cho n. D
ay con phai t`m l`
a
Hoa
u j +1 , u j +2 , . . . , u k .

. 7.6. Cho day Fibonaxi 1, 2, 3, 5, 8,. . . .a. t f (n) = 19852 +


,
,
1956n2 + 1960 Chung minh ra
`ng t`n tai
uk cua d
ay
. v han
. s hang
.
Fibonaxi, sao cho f (uk )|1989
,
`,i giai. a
. t h(n) = 4n2 + 33n + 29 = h(n) = 1989(n2 + n + 1)
Lo
,
suy ra f (n)|1989 h(n)|1989.
f (n). T`u d
o

72

,,
s nng cao
Chuong 7. B`
ai tp
. dy

Xet d
ay s {vn } sau d
y, trong d
o

v0
= 1, v1 = 1
vn+1 = vn + vn+1 , n = 1, 2, . . .
,,
N
oi c
ach kh
ac d
ay {vn } l`
a d
ay sinh ra boi d
ay Fibonaxi {un } :
,,
`
ng c
1, 1, 2, 3, 5, . . . ba
ach thm v`
ao truoc d
ay n`
ay ba s hang
. -1, 1, 0.
,
Goi
a ph`n du trong phep chia vi cho 1989(i = 0, 1, 2, . . .).
. ri l`
,
. p s sau d
Nhu vy
o 0 r 1988. Xet d
ay c
ac ca
y
. ta c

(r0 , r1 ), (r1 , r2 ), (r2 , r3 ), . . . .


,
ca
. p ri ch nhn
. p kh
V` mi
a tri,
ac ca
ac
. vy
. mt
. trong 1989 gi
. s c
,
2
2
theo nguyn ly irichl th` trong 1989 +
nhau ti d
a 1989 . T`u d
a l`
o
, ,,
. p d`u tin t nht c
. p tr`
. p y l`
1 ca
o hai ca
ung nhau. Gia su hai ca
a
(r p , r p+1 ), (r p+q , r p+q+1 ), p, q N
i`u y c
o ngha l`
a r p = r p+1 , r p+q = r p+q+1 . Theo c
ach x
ac d
inh
.
d
ay, ta c
o
v p1 = v p+1 v p = r p1 = r p+1 r p
,,
,
Tuongtu. , ta c
o
v p+q1 = v p+q+1 v p+q = r p+q1 = r p+q+1 r p+q
,
,,
,
suy ra r p1 = r p+q1 . Tuongtu. , ta c
T`u d
o
o
r p 2 = r p + q 2
.......
r 2 = r q +2
r 1 = r q +1
r0 = r q
,
T`u r0 = rq , r1 = rq+1 v`
a vn+1 = vn + vn+1 suy ra ri = ri+q , i =
0, 1, 2, . . .. Do vy
. r0 = rq = r2q = r3q = . . . = rkq , k 1, suy

7.1. V du.

73

,
ra h(vkq ) = 1989.A + h(1) = 1989.A. Nhung k = 1, 2, 3 . . . vkq
,
d
a s Fibonaxi suy ra c
o v s s hang
vkq cua d
ay Fibonaxi m`
a
`u l`
.
f (vkq )|1989.

,
. 7.7. Cho day s {un } xac dinh
. nhu sau
u1 = 20, u2 = 100, v`
a un+1 = 4un + 5un1 1976, n = 1, 2, . . .
,
,
Chung minh ra
`ng t`n tai
ay s chia ht cho 1996.
. t nht mt
. s cua d
,
`,i giai. a
. t un = 1996pn + qn , n = 1, 2, . . . trong d
pn , qn l`
Lo
a c
ac
o
s nguyn v`
a 0 qn 1995. Xet d
ay s

( q 1 , q 2 ), ( q 2 , q 3 ), . . . , ( q n , q n +1 ), . . .
,

V` d
ay n`
ay v han
a s c
ac s qi l`
a huu han
. , m`
. nn t`n tai
. hai s
,
,
,
,
tu. nhin l, m ( gia su m > l) sao cho (ql , ql +1 ) = (qm , qm+1 ) hiu
`ng
,ng minh ra
theo ngha ql = qm v`
a ql +1 = qm+1 . Ch
ung ta se chu
ql 1 = qm1 . Tht
. vy,
.
5(um1 ul 1 ) = (um+1 4um + 1976) (ul +1 4ul + 1976)

= ( u m +1 u l +1 ) 4 ( u m u l )
,
Do uk = 1996pk + qk , nn t`u (7.7) c
o

(7.7)

5(um1 ul 1 ) = 1996( pm+1 pl +1 ) (qm+1 ql +1 )

4[1996( pm pl ) (qm ql )]
,
`ng nhau v`
Thay nhung gi
a tri. ba
ao ((7.8) ta d
i d
n
5(um1 ul 1 ) = 1996[( pm+1 pl +1 ) 4(qm ql )]

(7.8)
(7.9)

,
T`u (7.9) suy ra 5(um1 ul 1 )|1996, m`
a (5, 1996) = 1 = (um1
ul 1 )|1996 = 1996( pm1 pl 1 ) + (qm1 ql 1 )|1996

= (qm1 ql 1 )|1996.

(7.10)

74

,,
s nng cao
Chuong 7. B`
ai tp
. dy

Do 0 qm1 1995, 0 ql 1 1995

= 1995 qm1 ql 1 1995

(7.11)

,
T`u (7.10) v`
a (7.11) suy ra (qm1 ql 1 ) = 0 hay qm1 = ql 1 .
,
,,
,
, tip tuc
Tuong tu. ch
ung ta c
ung c
o th d
a cu
i d
n qm2 = ql 2 v`
.
,
nhu th d
i dn
q2 = q2 + (m 1) v`
a q1 = q1 + ( m l )
(7.12)
,
`ng uml chnh l`
,ng minh ra
Ta chu
a s hang
ay m`
a chia ht cho
. cua d
1996. Tht
ach x
ac d
ay, ta c
o
inh
. d
. vy
. theo c
5uml = uml +2 4uml +1 + 1976

= 1996pml +2 + qml +2 4(1996pml +1 + qml +1 ) + 1996


= 1996( pml +2 + 4pm+l +1 ) + (qml +2 + 4qml +1 ) + 1996
,
,c trn ta c
ng thu
Thay (7.12) v`
ao d
o
a
5uml = 1996( pml +2 4pml +1 ) + (q2 4q1 ) + 1996.

(7.13)

Do
u1 = 20 = u1 = 0.1996 + 20 = q1 = 20
u2 = 100 = u2 = 0.1996 + 100 = q2 = 100
,
Vy
. t`u (7.13) suy ra
5uml = 1996( pml +2 4pml +1 ) + 1996.
(7.14)
,
Do pml +2 v`
a pml +1 l`
a s nguyn, vy
. t`u (7.14) suy ra 5uml |1996
m`
a (5, 1996) = 1 suy ra uml |1996.

,
. 7.8. Cho day s tu. nhin u1 , u2 , . . . , un+1 sao cho 1 u1 < u2 <
,
,
. . . < un+1 2n . Chung minh ra
`ng t`n tai
a j sao
. hai s tu. nhin i v`
cho u j chia ht cho ui , ( j > i ).
,
,
,, , ,
, , ,
`,i giai. Ky hiu
Lo
a uo
c s le lon nht cua ui tuong u
ng, ngha l`
a
. vi ,l`
,
,
p
i vi le v`
(i = 1, 2, . . . , n + 1).
ui = 2 i .vi , vo
a s tu. nhin pi n`
ao do

7.1. V du.

75

,i moi
Do 1 u1 < u2 < . . . < un+1 2n suy ra vo
. i = 1, 2,,. . . , n + 1
,

ta c
o vi < 2n . Xet (n + 1) s le v1 , v2 , . . . , vn+1 . C
ac s le n`
ay d
`u
, ,
,
,,
,
,
`ng n. Vy
duong v`
a nho hon 2n. nn s luo. ng cua ch
ung ba
. theo
`

nguyn ly irichl tn tai


a
. hai s i, j sao cho 1 i < j n + 1 m`
,
pj
p
i
. t trn th` ui = 2 vi v`
vi = v j . Theo c
ach d
a u j = 2 v j . Nhung
a
pj
p
i
ui < u j suy ra 2 vi < 2 v j , m`
a vi = v j . Suy ra 2 pi < 2 p j . Do pi , p j l`
a
,
,
pj
p
i

c
ac s nguyn duong nn 2 chia ht cho 2 . Ngha l`
a u j chia ht
cho ui .

,
. 7.9. (` thi Toan Olympic Quc t l`n thu 13) Day s {un }, n =
,
,
n
2, 3, 4, . . . x
ac dinh
`ng d
ay s n`
ay
. nhu sau un = 2 3. Chung minh ra
,
,,
,
`

chua mt
ac phn tu, sao cho bt k`y hai s n`
ao cua tp
. tp
. v han
. c
.
,

ho. p n`
ay c
ung nguyn t c`
ung nhau.
,
,
,
,,
`ng quy nap.
`,i giai. Ta chu
,ng minh ba
xy du. ng d
Lo
Gia thit d
a
uo. c
.
,, ,
k ph`n tu cua d
ay
u1 = 2n1 3, u2 = 2n2 3, . . . , uk = 2nk 3
,
,i 1 i, j k, o, d
,i moi
m`
a vo
y 2 = n1 <
. i 6= j, th` (u j , ui ) = 1, vo
n2 < . . . < n k .
,
Ch
ung ta se xy du. ng s uk+1 = 2nk+1 3 nguyn t c`
ung nhau
,
`ng c
i c
. t l = u1 .u2 . . . uk . Xet
vo
ac s u1 , u2 , . . . , uk ba
ach sau d
y: a
,
,
0
1
l
(l + 1) s 2 , 2 , . . . 2 Khi chia cac s n`ay cho l ta duo.,c mt
. tp
. ho. p
,
g`m l s du. Vy
o t nht hai s cho ta
. , theo nguyn ly irichl c
,,
,
,
r
s
l`
c`
ung ph`n du. Gia su hai s d
a 2 v`
a 2 , (s > r ). By gi`o chon
o
. s p
,
r
s
s
r

s
s
sao cho pl = 2 2 = 2 (2 1). Do l l`
a s le nn 2 khng chia
r
r

s
. t kh
ht cho l . Ma
ac (2 , 2 ) = 1 suy ra (2rs 1) chia ht cho l.
,
r s
i`u n`
ay ngha l`
a t`n tai
. s tu. nhin q sao cho 2 1 = ql, suy ra
2rs+2 3 = 4.2rs 3 = 4(ql + 1) 3 = 4ql + 1.

76

,,
s nng cao
Chuong 7. B`
ai tp
. dy

. t uk+1 = 2rs+2 3 = ql. Do uk+1 = 4ql + 1 v`


a
a l =
u1 .u2 . . . uk , suy ra uk+1 > uk .
,i
(uk+1 , ui ) = 1 vo
Do uk+1 = 4ql + 1, suy ra (uk+1 , 1) = 1. Do d
o
moi
. i = 1, 2, . . . , k.
,
,
,
,,
xy du. ng d
Nhu vy
ay con u1 , u2 , . . . , uk thoa m
an
a
uo. c d
. khi d
,
,
,
,
,
i u1 , u2 , . . . , u k ,
ay con mo
d
ai, th` se xy duo. ng d
uo. c d
i`u kin
`u b`
. d
,
uk+1 c
ung c
o tnh cht y. Theo nguyn ly qui nap
ung ta xy du. ng
. ch
,
,,
cho c
d
ay con v han
ay d
o tnh cht:
uo. c d
a
. u1 , u,2 , . . . , un , . . . . cua d
,,
,
ca
. p ph`n tu n`
Bt cu
ao cua d
ay con y c
ung nguyn t c`
ung nhau.

,
. 7.10. Cho u1 v`a u2 l`a hai s nguyn duo,ng nguyn t c`
ung nhau.
,
D
ay s {un } x
ac dinh
oi u1 , u2 l`
a hai s hang
on khi n =
. v
. d`u tin, c`
2, 3, . . . ta x
ac d
inh
.
u n +1 = u n u n 1 + 1
,
,
Chung minh ra
`ng v
oi moi
. i > 1 t`n tai
. j > i sao cho u j chia ht cho
ui .
,
,
,,
,
`,i giai. Ly i > 1 t`
Lo
uy y, v`
a p l`
a uo
c s nguyn t cua ui . Xy du. ng
,
d
ay s {vn } nhu sau

0
vn p 1
vn un (mod p)
. p
Ta c
ung c
o vn+1 = vn vn1 + 1 (mod p), n = 2, 3 . . . V` c
ac ca
,
,
(vn , vn1 ) l`a v han,
a vn ch nhn
a tri. suy ra {vn }
. m`
. huu han
. gi
,
,
`ng
,ng minh ra
. Ta c`n chu
phai l`
a d
ay tu`n ho`
an t`u mt
uc n`
ao d
o
. l
,
,
t`n tai
. s nguyn duong k p > 0 sao cho
vi+k p = 0.
,
Xet hai kha n
ang sau

(7.15)

7.1. V du.

77

, ,,
,
,i chu k`y T, trong d
s > i.
ay {vn } tu`n ho`
an t`u vs vo
1) Gia su d
o
Ta c
o
v s +1 = v s v s 1 + 1 = v s + T +1 = v s + T v s + T 1 + 1
v s v s 1 = v s + T v s + T 1

(mod p).

(mod p)
(7.16)

a) Nu vs = vs+T 0 (mod p), th` vs = vs+T = 0, do 0 vn


,
ch vic
p 1, n. Khi d
o
. chon
. k p = s i, Suy ra vi+k p = vs = 0,
ng.
vy
u
. (7.15) d
,
b) Nu vs = vs+T 6= 0, th` t`u (7.16) c
o vs1 = vs+T 1 . Suy ra
,
,
vo

`
`
,i
{vn } tun ho`an khng phai bat du t`u vs di`u n`ay mu thun
,
,
c
ach chon
ang n`
ay khng xy ra.
. s.Kha n
,,
,
,c (7.15) d
ng trong tru`ong ho. p 1).
T
om lai
u
. h. thu
,
,c (7.15) hin nhin d
ng. Ta c
2) Nu s i th` h. thu
o v i +1
u
,
ui+1 (mod p) ui ui1 + 1 (mod p) vy
. vi ui (mod p) nhung
,
,
,,
c nguyn t cua ui suy ra vi 0 (mod p), nhu vy
do p l`
a uo
. v i +1
1 (mod p) hay l`
a vi+1 = 1.
,,
,
,
Tuongtu. , do vi+k p = 0 suy ra vi+k p +1 = 1. Nhu vy
. suy ra {vn }
,
,,
,
,
c l`
i chu k`y k p tu
i moi
c
ung tu`n ho`
an vo
a vo
o
. l nguyn duong, ta c
vi+l.k p = 0.

(7.17)

Ngha l`
a ui+l.k p chia ht cho p.
,
,
,,
,
,,
,i moi
Nhu vy
c nguyn t p cua ui ta xy du. ng d
uo. c s k p
. vo
. uo
ng.
sao cho (7.17) du
,
,
,
Goi
a bi
ac s k p , theo (7.3)
. m l`
. s chung nho nht cua tt ca c
,
,
,
i moi
suy ra vi+lm = 0 vo
duong. Suy ra ui+lm chia ht cho
. l nguyn
,
,
,
,
,c l`
c s nguyn t cua ui . Tu
,i
tt ca c
ac uo
a ui+lm chia ht cho ui vo
,
,,
,ng minh.
moi
od
i`u phai chu
. s nguyn duong l. Ly j = i + lm, ta c

78

,,
s nng cao
Chuong 7. B`
ai tp
. dy

` tp
7.2. Bai
.
, ,
. 7.11. Nu ba s nguyn t thu. c su. lo,n ho,n 3 lp
anh mt
. th`
. cp
,
s cng
ung chia ht cho 6.
. th` cng sai cua ch
,,
,
. 7.12. cho f (n) l`a h`am xac dinh
ac s nguyn duong nhu
. trn tp
. c

,i mi
sau: nu n = a1 a2 . . . ak , th` f (n) = ( a1 + a2 + + ak )1998 . Vo
,
,
,
s nguyn duong n, lp
ay v han
. d
. ui (n), i = 1, 2, . . . nhu sau
ui (n) = f ( f (. . . f (n)i l`n f
|
{z
}
,,
`ng vo
,ng minh ra
,i moi
Chu
ay
. n nguyn duong, t`n tai
. p sao cho d
ui (n), i = p, p + 1, . . . l`
a d
ay tu`n ho`
an.
,ng
. 7.13. Cho u1 , u2 , . . . , un l`a day s t`
uy y g`m n s hang.
Chu
.
,,
`ng lun lun trch d
minh ra
ay con sao cho nu goi
a
uo. c mt
. d
. S l`
,
,, ,
,
i s nguyn g`n nht
tng c
ac ph`n tu cua d
ay con y, th` S kh
ac vo
1
,,
,,
.
mt
a
. luo. ng khng vuo. t qu
n+1
`ng nu nhu,ng s nguyn a v`
,ng minh ra
. 7.14. Chu
a m nguyn t
,
c`
ung nhau, th` t`n tai
a tch na chia cho m cho
. mt
. s tu. nhin n m`
,
s du 1.

, ,
CHUONG

,
THU,C VO

` MT
S
I TP
. TRU
.
.

8.1. Tp
u mt
. tr`
.

,
,,
,
,,
,i kh
ng thu. c ch
Trn d
ung ta thu`ong quan tm to
ai nim
u`ong tha
.
,
,
,
, ,
ng), ta c
mt
(mt
o th hiu mt
oan
. khoang
. d
. tha
. khoang trn
,
,
,`,
,
, `
,
, ,

ng thu. c l`
m giua hai d
d
a tp
ac s thu. c na
uong tha
im
. , ho. p, tt ca c
,
,
,
`m trong
cho. Nhu vy
d
ac d
mt
khoang c
o th na
a
im d
x
inh
.
. hai d
.
,
,
`m ngo`
. c na
;d
ng,
hoa
ai khoang d
a xut ph
at cua kh
ai nim
o
y l`
o
. d
,
,
,, ,,
,

mo,.. o ng`
anh giai tch, giai tch h`
am trong to
an hoc
cao cp. Nhung
.
,
,,
,i c
, m`
o d
a ch quan tm to
ac kh
ai
y ta khng quan tm d
i`u d
o
,
, ,
,
,
,

nim
a s thu. c, ky
y voi quan d
im kin thuc so cp. Voi a, b l`
. sau d
,
,
,
,
,,
`
hiu
a khoang d
a goi
l`
a khoang
uo. c tnh ca hai d
im d
u a, b v`
. [a,b] l`
.
,
,
,, , ,
ng. Khoang mo d
d
l`
a ( a, b), khng ly hai d
o
uo. c ky hiu
im d
`u
.
,
,
,,
,,
,,
ng (nua mo ) do vic
a, b. Ngo`
ai ra ta c`
on xet khoang nua d
o
. ta ch
,
,
. c b,v du. nhu [ a, b) hoa
c ( a, b]. Mt
ly mt
trong hai d
im d
`u a hoa
, .
,.
,.
,

khoang trong tp
a suy tho
ai nu n
o ch l`
a mt
im
. , s thu. c goi
. l`
. d
,
`

(tuc l`
a khi hai d
ung nhau).
im d
u tr`
,
,
,
,
Mt
a tr`
u mt,
nu moi
khoang
. tp
. ho. p A cua s thu. c goi
. l`
.
.
,, ,
,a mt
khng suy tho
ai d
`u chu
. s ph`n tu cua A.
, ,
, ,
,
Mt
a tp
ho. p s huu ty g`m c
ac s c
o th biu
. v du. d thy l`
.
,
,
,
,,
,
din nhu mt
a tp
. thuong cua hai s nguyn v`
. ho. p s v ty g`m

80

,,
, ,
Chuong 8. S thu. c voi tp
u mt
. tr`
.

,
, , `
,
,ng s khng phai l`
u mt
nhu
a s huu ty d
a nhung tp
u l`
. tr`
. trong
,

tp
. s thu. c.
,
,,
,,
Ngu`oi ta c`
on mo rng
kh
ai nim
tr`
u mt
cho mt
khoang trn
.
.
.
.
,
,
,
,,
,
,,
ng thu. c. Cho l`
ng
d
a mt
u`ong tha
u`ong tha
. khoang bt k`y trn d
,
,
,
,
thu. c. Ch
ung ta goi
u. c l`
a tr`
u mt
. ,tp
. ho. p B g`m nhung s th
. trong
,
,
,
a nhu,ng
i moi
, nu vo
ai cua d
`u chu
. khoang con khng suy tho
,
,, ,
,
,
ph`n tu cua B. R
o r`
ang tp
ac s huu ty tr`
u mt
. h,o. p c
. trong moi
.
,
,

khoang; v`
a tp
h
o
p
c
a
c
s
v
t
y
c
u
ng
c
o
t
nh
ch
t

y.
.
.
,
,,
,
,i moi
Nhung ky hiu
ung: vo
a
uo. c d`
. hay d
. s thu. c x, ky hiu
. [ x ] l`
,
,
,
n nht khng vuo. t qu
s nguyn lo
a x, { x } l`
a s x [ x ]; [ x ] v`
a {x}
,
,
goi
a ph`n nguyn v`
a ph`n thp
cua s x. T`u d
o
inh
. ngha c
. , l`
. phn
, ,
,
th thy ngay 0 x [ x ] < 1. hiu r
o hon kh
ai nim
u mt
. tr`
.
,,
trong ph`n n`
ay ta xet mt
lo
at
b`
a
i
to
a
n
c
o
lin
quan
v`
a
c
a
ch
s
u
d
ung
.
, ., .
, ,
ng minh d
nguyn ly irichl d
chu
inh ly co ban Kronecker.

8.2. V du.

,
,
,
,
. 8.1. Nu mt
s thu. c l`
a tr`
u mt
a r l`
a s thu. c kh
ac
. tp
. ho. p A cua
. v`
,
, `
,
khng, th` tp
oi a chay
ung l`
a
. ho. p gm tt ca tch s {ra} v
. trong A c
tp
u mt.
. tr`
.
,
,
,,
,
`,i giai. C
,ng
Lo
o hai tru`ong ho. p xy ra: r > 0 v`
a r < 0. V` c
ach chu
,
,,
,
,,
,
minh ho`
an to`
an tuong tu. nn ta ch xet tru`ong ho. p d
a
`u. Cho x v`
,
x
y
,
< v`
y l`
a hai s thu. c v`
a x < y. Khi do
a theo gia thit A l`
a tp
.
r
r
,,
x
y
tr`
u mt,
. th` t`n tai
. ph`n tu a thuc
. A sao cho r < a < r . Suy ra
,
,,
,
,a s dang
,i a
x < ar < y. Nhu vy
ra vo
. moi
. khoang mo ( x, y) chu
.
thuc
. A.

,
,
,
,
. 8.2. (inh
oi moi
ac s c
o
. ly Kronecker) V
. s v ty , tp
. ho. p tt ca c

8.2. V du.

81

,,
,ng s nguyn bt k`y.
a n l`
a nhu
dang
a tp
u mt,
. m + n l`
. tr`
. o dy m v`
,
,
,
`ng moi
`,i giai. C`n phai chu
,ng minh ra
Lo
khoang khng suy tho
ai
.
,
,
,
,
a nhung d
chu
ung ta chia khoang [0,1] ra
im dang
. ,m + n. Ch
,
,

th`
anh mt
s huu han
khoang con 1 , 2 , . . . , k m`
ad
ai cua mi
. d`
.
.
,
, ,
,
,
`ng vo
i moi
khoang con nho hon d
ai cua . Ch
u y ra
s a t`n tai
. d`
.
, .
`

mt
s
nguyn
n,
sao
cho
s
a
+
n
n
a
m
trong
kho
ang
[0,1].
Suy
ra
.
,
, , ,
,i moi
,i mt
vo
o th cho tuong u
ng vo
s dang
. s nguyn m c
. m + n,
,.
,
,
,,
`m trong khoang [0,1]. Nhung d
ng [0,1] d
m`
a n
o na
oan
uo. c chia
. tha
,
,
,
,
ra huu han
. khoang nho. Suy ra t`n tai
. nhung s nguyn m, m1 , n, n1 ,
,
`m trong c`
,i m 6= m1 v`
vo
a nhung s m + n v`
a m 1 + n 1 na
ung mt
,
,
,
,
, , .
,
khoang c
khoang nho i . Do d
ach giua hai d
ay nho hon d.
o
im n`
,
, ,
d`
ai cua . Ch
ung ta c
ung thy khng th xy ra m + n = m1 + n1 ,
,
n1 n
,
,
, ,
c n`
ng thu
v` t`u da
ay suy ra =
m`
a lai
l`
a mt
s huu ty, tr
ai
.
.
m

m
1
,
,i gia thit.
vo
,
,
`ng s (m m1 ) + (n n1 ) kh
ng d
Nhu vy
ung ta kha
ac
inh
. , ra
. ch
,
,
,
,
c s
suy ra mt
khng v`
a c
o gi
a tri. tuyt
o
i nho hon . T`u d
. s uo
. d
,
,
`m trong khoang .
cua n
o, c
ung c
o dang
. m + n, se na
, ,,
,
,
tp
. 8.3. Cho l`a mt
o dang
o
. s v ty duong. Khi d
. ho. p tt ca s c
.
,
,
,

m n l`
a tp
tr`
u
m
t,v
o
i
m
v`
a
n
l`
a
nh
u
ng
s
t
u
nhin
b
t
k`
y
.
.
.
.
,
,
,
,
`,i giai. Kt lun
ng
Lo
ai n`
ay manh
hon d
ly trn v` n
o kha
inh
.
. cua b`
.
, ,
,
,
c
d
ung tr`
u
inh
ij nh ly d
o
. mt
. tp
. s thu. c nho hon tp
. m ta trong d
mt.
.
,
,
`m o,, bn phai s khng v`
Cho khoang khng suy tho
ai na
a c
o
,
,
, ,
, ,,
,
ng minh c
d
ai e > 0. Khng anh huong d
o th gia
. d`
n kt qua chu
,
e
,
,
thit e < 1. Ngo`
ai ra ta c
o th chon
. s tu. nhin k sao cho k . T`u
,i p v`
8.1 v`
a 8.2 suy ra tp
ac s kp + kq vo
a q l`
a c
ac s nguyn bt
. c

82

,,
, ,
Chuong 8. S thu. c voi tp
u mt
. tr`
.

,
k`y, l`
a tp
u mt.
a t`n tai
ac s nguyn p v`
a q thoa m
an
. tr`
. Ngha l`
. c
0 < kp + kq < e.
(8.1)
,
,
,
`ng giua nhung ca
. p s nguyn ( p, q) trong (8.1)
Ch
ung ta se ch ra ra
, ,
,
,,
,
. p m`
c
o nhung ca
a p > 0. Tht
. vy,
. tru`ong ho. p p = 0 khng th xy
,
vo
,i d
bit l`
ra v` t`u (8.1) suy ra 0 < kq < e, mu thun
a kq
i`u ta d
a
,
,
,
,

ng minh cho tru`o,ng


l`
a s nguyn v`
a e < 1. Do vy
on phai chu
. ch c`
,,
,
,
,i s, t l`
ho. p p < 0. Trong tru`ong ho. p n`
ay s c
o dang
kps + kqt vo
a
.
,
,

`
nhung s nguyn bt k`y, tao
anh tp
u mt.
. th`
. tr`
. T`u (8.1) suy ra tn

tai
a t sao cho
. s nguyn s v`
0 < kps + kqt < kp + kq.
(8.2)
,
,ng minh lai
ng. V` th ta ch xet
Nu s < 0, th` kps > 0 v`
a chu
u
. d
,
s 0. T`u (8.1) v`
a (8.2) suy ra
0 < kp(1 s) + kq kpt < e.
(8.3)
,
,
,
,c (8.2) khng th xy ra, nn s > 0. Nhu,ng
,i s = 0 bt d
ng thu
V` vo
a
,,
,
boi v` s l`
a s nguyn, th` t`u s > 0 suy ra s 1. Tai
. v` ((8.3) khng
,
, ,
,
,
,
c cui c`
i s = 1, t`u bt d
ng thu
th xy ra vo
ung lai
a
. suy ra s > 1.
,
c
Nhu vy
o ngha l`
a t`n tai
ac s nguyn p
o
. , kp(1 s) > 0. i`u d
. c
,
v`
a q thoa m
an (8.1) v`
a p > 0. T`u (8.1) suy ra
e
kq < e kp < e p = e(1 p) 0

,,
,
Vy
ach n`
ay (8.1) d
n < e,
uo. c d
ua v` dang
. kq < 0. Theo c
. 0 < m
,
,,
,,
,
,
,

od
a n l`
a nhung s tu. nhin. Boi v` e l`
ad
ai cua, t`u bt
y m v`
. d`
,
,
,,
,c cui c`
ng thu
cua m n, c
d
ung suy ra mt
c s n`
ao d
ung c
o
a
o
. uo
,,
,
,
,
,
`
ng minh d
m trong . Tuong tu. c
dang
n`
ay, se na
ung chu
uo. c khi
.
,,
`

m o bn tr
na
ai s khng.
,
,
,ng s dang
. 8.4. Cho l`a s v ty bt k`y. Khi do tp
. ho. p nhu
. {n},
,
,
,
v
oi n l`
a s tu. nhin bt k`y, l`
a tp
u mt
. tr`
. trong khoang (0,1).

8.2. V du.

83

,
,
,
`ng moi
`,i giai. C`n phai chu
,ng minh ra
khoang con khng suy
Lo
.
,
,
,a s c
bin ( a, b) cua (0,1) chu
o dang
{n} vo,i mt
.
. s tu. nhin
,ng minh cho tru,`o,ng ho.,p khi l`
. Ch
n n`
ao d
ung ta chu
a mt
o
. s
,,
,
,
,
i n, m l`
duong. Theo b`
ai 8.3 nhung s c
o dang
n m vo
a c
ac s tu.
.
,
,
nhin, tao
anh tp
u mt.
a t`n tai
. , th`
. tr`
. Ngha l`
. nhung s tu. nhin
,
,
m v`
a n thoa m
an 0 a < n m < b < 1. Nhung t`u d
inh
. ngha
,,
[n] suy ra 0 n [n] < 1. Ch
ung ta nhn
uo. c hiu
. d
. hai s,
,
nguyn m v`
a [n] thoa m
an 1 < m [nm] < 1, d
ay ch
i`u n`
,
,
,
`
`
ng s
m = [n]. Tu (8.1) ch ra ra
xy ra khi m [nm] = 0, do d
o
,
,

{n} = n [n] na`m trong khoang ( a, b). Voi > 0, ch


ung ta d
a
,

chung minh xong.


,,
,
,
> 0 v`
Tru`ong ho. p < 0. Khi d
a t`u b`
ai 8.3 suy ra t`n tai
o
.
,
,
,
nhung s tu. nhin m v`
a n, thoa m
an 1 b < n() m <
,,
a 0. Nhn cac v vo,i -1 ch
ung ta nhn
uo. c 0 a < n m <
. d
,
`m trong
b 1. Nhu vy
ang tnh ra m = [n] v`
a {n} na
. d d`
,
khoang ( a, b).
,
,
,
,
. 8.5. Tp
ac s dang
oi n l`
a s tu. nhin bt k`y
. ho. p tt ca c
. {log n} v
l`
a tp
u mt
. tr`
. trong (0,1).

,
,
,
,
`,i giai. Ch
,ng minh mo, rng
Lo
ung ta c
o th chu
hon mt
ut: tp
.
. ch
.
,
,
,

ho. p c
ac s dang
{
n
log
2
}
v
o
i
n
l`
a
s
t
u
nhin
b
t
k`
y
,
l`
a
t
p
tr`
u
m
t
.
.
.
.
trong (0,1).
,
,ng minh log 2 l`
d
muc
dch n`
ay ch
ung ta chu
a mt
at
.
.
. s v
,
,`,
,,
,
,
ty. Tht
. vy,
. trong truong ho. p nguo. c lai
. th` t`n tai
. hai s tu. nhin
p
p
, ,
,c cui
ng thu
p v`
a q, m`
a log 2 = , ngha l`
a 2 = 10 q . L
uy th`ua d
a
q
,,
,i q ch
c`
ung vo
ung ta nhn
ay tr
ai d
ly
uo. c 2q = 2 p .5 p . i`u n`
inh
.
. d
,
,
,
,
,
co ban cua s hoc
. v` vic
. phn tch ra th`ua s nguyn t. Nhu vy
.

84

,,
, ,
Chuong 8. S thu. c voi tp
u mt
. tr`
.

,
, ,
a mt
log 2 thu. c su. l`
8.4 suy ra tp
ac s c
o dang
. s v ty. p dung
.
. c
.
{n log 2} l`a tr`
u mt
. trong (0,1).
,
,i moi
. t kh
Ma
ac, theo tnh cht cua logarit, log 2n = n log 2 vo
.
n = 1, 2, . . .
,
,
. 8.6. Cho m l`a s nguyn, n l`a s nguyn khng m. Tp
. ho. p tt ca
m
c
ac s c
o dang
l`
a tp
u mt.
.
. tr`
.
2n
,
,
`ng
`,i giai. Do c
,ng minh ra
Lo
ac b`
ai to
an trn, ch
ung ta ch c`n chu
m
3
,
,,
,
,i m, n l`
vo
a nhung s nguyn duong, l`
a tr`
u mt
tp
. ho. p s dang
.
.
n
2
,
,
,
,
i moi
. p s duong
c
trong khoang (0, +). i`u d
o ngha l`
a vo
o
. ca
,
3m
,
,

< b.
a, b( a < b) t`n tai
nh
u
ng
s
t
u
nhin
m
v`
a
n
th
oa
m
a
n
a
<
.
.
2n
,
,
,
c n`
p bt d
ng thu
Sau khi logarit h
oa ca
ay theo co s 2, ch
ung ta
a
,.
,,
,
,
,
,
,

c tuong d
. p bt d
ng thu
nhn
uo. c ca
a
uong log2 a < m log2 3 n <
. d
,
,
,
,
,
ng minh tuong tu. nhu, 8.5 cho thy log2 3 l`
log2 b. C
ach chu
a v ty.
,
`ng trong d
,a
d
Khi d
o
inh
oan
. ly Kronecker ch ra ra
. (log2 a, log2 b) chu
s c
o dang
. {m log2 3 n}.
,
,
,
,
. 8.7. Tp
ac s hang
cua d
ay x
ac dinh
`ng cng thuc
. ho. p tt ca c
.
. ba
,
n
1
xn = [log n]+1 , n = 1, 2, 3 . . ., l`
a tr`
u mt
. trong khoang ( 10 , 1).
10

,
,
1
`,i giai. Ly hai s thu.,c a v`
Lo
a b sao cho
a < b 1. C`n phai
10
,
n
,
`ng t`n tai

,ng minh ra
chu
s
t
u
nhin
n,
th
oa m
an a < [log n]+1 <
.
.
10
,
,
, ,

b. Sau khi logarit h


oa bt d
a
ng
th
u
c
trn
v
o
i
co
s
10
ch
ung ta

,, ,
,
,
,
,
,
c tuong d
ng thu
nhn
uo. c bt d
a
uong 1 + log a < log n [log n] =
. d
,
,
,
{log n} < 1 + log b. Nhung khoang mo, vo,i cac d`u m
ut 1 + log a
,
,
`m trong khoang (0, 1). i`u n`
v`
a 1 + log b r
o r`
ang na
ay giai thch tai
.
,
,
,,
,
,

sao t`n tai


m
t
s
t
u
nhin
n
th
oa
m
a
n
b
t
d
a
ng
th
u
c
sau
c`
u
ng,
b
o
i

.
.
.

8.2. V du.

85

,
v` theo 8.5 tp
ac s c
o dang
{log n} l`a tr`
u mt
. ho. p c
.
. trong (0, 1).
,
,
`
`
ng minh.
Nhu vy
iu ta cn chu
. th` a < xn < b, d

,
,
,ng s l
`,a cua 2, m`
. 8.8. Chung minh ra`ng t`n tai
uy thu
a
. v han
. nhu
,

`
khi vit theo co s 10 ch
ung lun lun ba
t d`u ba
ng 8975.
,
,
m
`ng nh
`,i giai. Mt
,i
t d
Lo
om s 8975 khi v`
a ch khi vo
`u ba
. s 2 ba
,
,
,
,
,,
,c sau d
ng thu
, nhung da
mt
ao d
an
o
uo. c thoa m
. s tu. nhin n n`
,
,
,,
n
m
n
8975.10 2 < 8976.10 . Nhu c
ach giai c
ac b`
ai truoc ch
ung ta
,
,
,
,
,
,
c trn v`
c tu,o,ng
ng thu
ng thu
logarit h
oa bt d
a nhn
ac bt d
a
uo. c c
a
. d
,,
d
a mt
uong log 8975 m log 2 n < log 8976. Tai
. v` log 2 l`
. s v
,
,
,
,
,
i n, m l`
ty, nn nhung s c
o dang
m log 2 n vo
a nhung s tu. nhin
.
. p s
tao
anh mt
u mt.
ay c
o ngha l`
a t`n tai
. th`
. tp
. tr`
. i`u n`
. v s ca
,
,
,
,
c sau c`
ng thu
tu. nhin m v`
a n thoa m
an c
ac bt d
ung, suy ra c
ung
a
,
,
,
,
,
c truo
c d
ng thu
. Ch
ng khi
u y b`
ai to
an c`
on d
thoa m
an c
ac bt d
o
u
a
,
,
,
,
`
`ng
ng co s 10 cua logarit v`
thay co s hai ba
a s 8975 c
o th thay ba
, ,
,ng minh d
mt
ay d`
anh cho ban
i`u n`
oc.
. t ho. p s bt k`y, chu
. d
.

,
,
,
, s d`u tin cua
. 8.9. V
oi moi
a chu
. s tu. nhin n, ky hiu
. xn l`
,
, ,
s 2n (trong c
ach vit v
oi co s 10). Chung minh ra
`ng d
ay s
,
tu`n ho`
x1 , x2 , . . . , xn , . . . khng phai l`
a dy
an.
,
, ,
,
,,
`,i giai. Gia su, ngu,o.,c lai
Lo
a t`n tai
a
. l`
. nhung s nguyn duong k v`
d sao cho xk = xk+d = xk+2d = . . . = xk+nd = . . . N
oi c
ach kh
ac
,
k
k
+
d
k
+
2d
k
+
nd
2 ,2 ,2
,...,2
, . . . c
o c`
ung chu s daah u tin trong c
ach
,
,
,
,
,
`ng moi
i co s 10. B`y gi`o ch
ng d
vit vo
ung ta kha
inh
. ra
. s tu. nhin
, ,
,
,
log
N
[
log
i dang
N c
o th biu din duo
N = 10
= 10 N ]+{log N } =
.
10[log N ] .10{log N } . V` 0 {log N } < 1, nn 1 10{log N } < 10, suy
,
, ,
,
,i ph`n nguyn cua cua s 10{log N } .
ra chu s d
ung vo
`u tin cua N tr`
,
,
,
ng
n d
an bt d
y suy ra t`n tai
a
. mt
. chu s s, 1 s 9, thoa m

86

,,
, ,
Chuong 8. S thu. c voi tp
u mt
. tr`
.

k+nd
,c sau: s 10{log 2 } < s + 1 vo
,i n = 0, 1, 2, . . .. Sau khi logarit
thu
,
,c trn ch
ng thu
h
oa d
ung ta c
o log s {(k + nd) log 2} < log(s +
a
,
`

1). iu n`
ay ngha l`
a tt ca c
ac s c
o dang
{(k + nd) log 2}, n =
.
,
`
,ng minh
m trong khoang [log s, log(s + 1)). D d`
ang chu
0, 1, 2, . . . na
,
,
`ng tp
tr
ay tr`
u mt
ai
ra
oan
o
. ho. p nhung s n`
. trong d
. (0,1). i`u d
,
,, ,
,

`
i kt lun
vo
o d
oan
im
. : tn tai
. tp
. con mo cua d
. , (0,1) khng c
,,
,
,
,
,
`
i [log s, log(s + 1)). Nhu vy
chung vo
a sai.
iu gia su nguo. c lai
. d
. l`
,
,
,
,
. 8.10. Hay t`m trong doan
ng [0, ], tt ca s thu. c t thoa m
an
. tha
,
,,
,

bt phuong tr`nh cos nt cos t v


oi moi
. s tu. nhin n.

,
`ng nu t c
`,i giai. - Ch
,ng minh ra
nu
Lo
ung ta se chu
o tnh cht d
a
,
,
, ,, t
t
,
trong b`
ai to
an th` ty s l`
a mt
a
. s huu ty. Tht
. vy
. gia su l`

,
,
,
t 6= ; Khng mt tnh tng qu
s v ty. Khi do
at ch
ung ta c
o th
,
, ,,
`ng 0 t < . V` t l`
gia thit ra
a s v ty duong, theo d
ly
inh
.
2
,
,
,
,,
Kronecker t`n tai
a n, thoa m
an bt phuong
. nhung s tu. nhin m v`
t
t
t
,
<m
n < 1
, hay l`
a t < mt 2n < 2 t. T`u
tr`nh
2 , 2
2
tnh cht cua h`
am cos trong d
ung ta suy ra cos mt =
oan
. [0, 2 ] ch
,
,
`

cos(mt 2n ) < cos t. iu n`


ay tr
ai voi c
ach chon
. t, do vy
. ty s
t
, ,
l`
a s huu ty.

, ,
,,
p
,
,,
Nhu vy
o th biu din duo
i dang
t = 2, o d
a q l`
a
y p v`
. t c
.
q
,
nhung s nguyn t c`
ung nhau v`
a 0 p q (tai
. v` 0 t 2 ).
,
q
,

n, th` n = l`
Nu q l`
a s cha
a s tu. nhin v`
a ngo`
ai ra p l`
a s le, v` p
2
v`
a q l`
a nguyn t c`
ung nhau. V` vy
. cos t cos nt, = cos p =, 1,
,
`
suy ra cos t = 1 v`
tu d
a t = . R
o r`
ang thoa m
an tt ca c
ac
o
`
cho.
d
iu kin
a
. d
,
,
,
,
,,
,
,ng
Ch
ung ta ch c`
on phai xet tru`ong ho. p q l`
a s le. C
o th chu

8.3. B`
ai tp
.

87

`ng vo
`ng mt
,i moi
trong dy
minh ra
ao d
o
. n = 1, 2, . . . cos nt ba
. s n`
sau d
y
2
2
2
2
, cos 1. , cos 2. , . . . , cos((q 1) ).
(8.4)
q
q
q
q
,,
Tht
y 0 r q 1, th` cos nt = cos(sqt +
. vy,
. nu n = sq + r, o d
,
rt) = cos(2ps + rt) = cos rt. Nu q = 2k + 1 l`
a mt
. s le, th` s
,
k
k+1
nho nht trong (8.4) l`
a c
ac s cos
2 v`
a cos
2. Ly
2k + 1
2k + 1
p
t =
2, 0 t , c
o tnh cht mong mun, ch
ung ta se
2k + 1
,
,
,
`ng
,ng minh khi d
p = k. d
chu
ay ch c`n ch ra ra
o
at
ch n`
. muc
. d
k
,,
t`n tai
a cos nt = cos
2. Tht
. mt
. s nguyn duong n m`
. vy:
.
2k + 1
,
,

`
v` 2k + 1 v`
a p l`
a nhung s nguyn t c`
ung nhau tn tai
. nhung s
,
,
nguyn m v`
a n thoa m
an 0 n 2k + 1 v`
a (2k + 1)m + pn = k. T`u
,
k
,c sau c`
ng thu
d
ung suy ra 2m + nt =
2. Suy ra cos nt =
a
2k + 1
k
k
`ng nhu,ng s c
2. D d`
ang thy ra
o dang
2 v`
a
cos
.
2k + 1
2k + 1
,
k+1
2, k = 0, 1, 2, . . . l`
a nghim
cua b`
ai to
an. Kt lun
ung
.
. cui c`
2k + 1
,
,
k
k+1
,
,i
l`
a nhung gi
a tri. cua t phai t`m l`
a t = v`
a
2,
2, vo
2k + 1
2k + 1
k = 0, 1, 2 . . . . . .
1 = cos 0.

` tp
8.3. Bai
.
,
,
. 8.11. Cho a, b, c l`a nhung s thu. c sao cho [ an] + [bn] = [cn] vo,i
,
`ng t nht mt
,ng minh ra
moi
ac s a, b l`
a
. trong c
. s tu. nhin n. Chu
nguyn.
, ,
,
`ng tp
,ng minh ra
. 8.12. Cho l`a mt
ho. p c
ac s c
o
. s huu ty. Chu
, .
dang
u mt
. {n}, n = 1, 2, . . . khng tr`
. trong khoang (0, 1).

88

,,
, ,
Chuong 8. S thu. c voi tp
u mt
. tr`
.

,
, ,
`ng vo
,ng minh ra
,i mt
. 8.13. Chu
a mt
. s v ty bt k`y v`
. s huu ty
,
bt k`y , tp
ho. p s c
o dang
{n + }, n = 1, 2, . . . l`a tr`
u mt
.
.
. trong
,
khoang (0, 1).

,
,
`ng tp
,ng minh ra
. 8.14. Chu
ho. p s dang
{ n} vo,i n l`a s tu. nhin
.
.
,
l`
a tp
u mt
. tr`
. trong khoang (0,1).

, ,
CHUONG

,
,
,

NHUNG UNG DUNG


KHAC CUA
.
IRICHLE
NGUYN LY

,
,
9.1. Xp x mt
. s thu. c
,
,
t`n tai
. 9.1. Cho x l`a mt
on n l`
a mt
. s thu. ,c, c`
. s tu. nhin. Khi do
.
,

ng s nguyn p v`
nhu
a q thoa m
an 1 q n v`
a




x p 1 .
(9.1)

q
nq
,
,
`,i giai. Ch
ung
Lo
ung ta xet nhung s kx [kx ](k = 0, 1, 2, . . . , n). Ch
,
,
`
`

m trong khoang [0, 1]. Ch


gm n + 1 s v`
a na
ung ta chia khoang [0, 1]
,
,
,
khoang
`
ng nhau 1 , 2 , . . . , n v`
ra n khoang con ba
ad
ai cua mi
. d`
`ng 1 . Theo nguyn ly irichl t`n tai
ac nhau k v`
al
n`
ay ba
. hai s kh
n
,
`m trong 0, 1, 2, . . . , n, sao cho nhung s kx [kx ] v`
`m
na
a lx [lx ] na
,
,
,
,
m. Do d
khoang c
trong c`
ung mt
ach giua ch
ung
o
. khoang con thu
1 ,
1
c l`
khng qu
a , tu
a |kx [kx ] (lx [lx ])| , hay l`
a
n
n
1
(9.2)
|(k l ) x ([kx ] [lx ])| .
n
,
,
,
,,
, ,,
,ng minh ta c
Boi v` k 6= l, khng anh huong d
o th
n kt qua chu
,
`ng k > l. Bo,,i v` ngo`
gia thit ra
ai ra c`
on c
o 0 k n, 0 l n,
. t q = k l v`
p v`
nn 1 k l n. Ta d
a p = [kx ] [lx ]. Khi d
aq
a
o

,
,,
,ng u
,ng dung
Chuong 9. Nhu
kh
ac cua nguyn ly irichle
.
,
,
,i c
. t n`
a thoa m
l`
a nhung s nguyn v`
an 1 q n. Vo
ach da
ay (9.2)
1 ,
, `
d
o (9.1).
ua v dang
y chia hai v cho q ta c
. |qx p| n , t`u d
90

,
,
,
,
. 9.2. V
oi moi
han
q, v
oi mi
q t`n
. s thu. c x t`n tai
. v
. s tu. nhin
,
,
,
tai
ung thoa m
an bt da
ng thuc.
. s nguyn p, sao cho ch




x p 1 .
(9.3)

q
q2
,
r ,
, , ,
`,i giai. Nu x l`
i r l`
a s nguyn v`
a s l`
a
Lo
a s huu ty, tu
c l`
a x = vo
s
,
,
,
. t p = mr
ng, v` c
s tu. nhin, th` kt lun
ai to
an l`
ad
o th d
u
a
. cua b`
,
,
,
,
,
i s tu. nhin bt k`y m. Vo
i tt ca c
v`
a q = ms vo
ach chon
a q nhu
. p v`
,
,,
`ng khng. Nhu, vy
vy
an, v` v tr
ai lun lun ba
uo. c thoa m
. , (9.3) d
.
,
,
,
,, ,
,,
,
,

ch c`
on phai xet tru`ong ho. p x l`
a s v ty. Gia su ch c
o huu han
s
.
,
,
,
i ch
tu. nhin q, m`
a vo
ung t`n tai
an (9.3), ky
. s nguyn p thoa m
,
i bt k`y = 1, 2, . . . , l ky hiu
hiu
ung l`
a q1 , q2 , . . . , ql . Vo
a s
. ch
. p l`
,
nguyn thoa m
an






x p x p



q
q
,
,,
,
i moi
vo
s nguyn p. V` s x l`
a v ty, moi
a tri. tuyt
ai
i o pha tr
. gi
. d
,.
,
,
,
,
,

c trn l`
ng thu
bt da
a duong, s nhung gi
a tri. tuy t
a huu
i n`
. d
ay l`


1
p
,

han.
. V` vy
. t`n tai
. s tu. nhin n, sao cho n < x q , ( =

1, 2, . . . , l ).
,,
,
Theo b`
ai truo
c se t`n tai
a s nguyn
p, sao cho
. s tu. nhin q v`



,
p
1
1
,
x
an v`
a 1 q n. Nhung khi d
(9.1) thoa m

=
o
q
qn
qq
,
1
,
,
,c
,i mt
ng thu
, suy ra q = q vo
s = 1, 2, . . . , l T`u nhung bt d
a
.
2
q




1
p
1
1
1
p
,

. Nhu vy
<
v` th
trn suy ra < x
x
.

n
q
q
nq
n
nq

,
,
9.1. Xp x mt
. s thu. c

91

, ,,
,
,,
q < 1, d
ay khng th d
a s tu. nhin. Ta nhn
uo. c
i`u n`
uo. c v` q l`
. d
d
i`u v ly.
,
,
,
. 9.3. V
oi moi
ng thuc
. c > 2 bt da




2 p 1 .
(9.4)

q
qc
, , ,
,
u han
ng ch v
du
oi hu
. p s nguyn p v`
a s tu. nhin q.
. ca
,
,
`,i giai. Ch
,ng minh bt da
,c sau l`
ng thu
ng
Lo
ung ta chu
ad
u



2 p 1 .
(9.5)

q
3 2q2
,
1
,
,i moi
,i moi
ng
vo
a vo
a
. s nguyn p v`
. s tu. nhin q. V` q2 1, bt d



,
,
p
1
,
,

c (9.5) tt nhin thoa m


thu
an, khi 2 > . Nhu vy
ch
.
q
3 2



p
1
1
,
,
,
,
2
c`n xet tru`ong ho. p 2 . Nhung khi d
o
q
3 2
3 2

1
p
2 + suy ra
q
3 2

p
(9.6)
0 < < 2 2.
q
. t kh
Ma
ac




p
p

2



2 q . 2+ q
2q p2


p
2 =


.
=
(9.7)




p
p
q
2 2
2+
q


q
q

,
,
Nhung s 2q2 p2 l`
a s kh
ac khng, v` 2 l`
a mt
. s v
ty. Ngo`
ai



p
,
ra n
o l`
a s nguyn nn |2q2 p2 | 1 . T`u (9.7) suy ra 2
q
1
,

v`
i (9.6) cho ta (9.5).
a c`
ung vo

p
2

q 2+
q

,
,,
,ng u
,ng dung
Chuong 9. Nhu
kh
ac cua nguyn ly irichle
.
,
, ,,
,
,c (9.4) d
,i mt
ng thu
. p s ( p, q) n`
ng vo
. T`u
ao d
Gia su bt d
u
o
a
. ca
,
,
1
1
,c c2 . V` c > 2 bt d
,c
ng thu
ng thu
(9.4) v`
a (9.5) suy ra d
a
a
q
3 2
,
,
,
,
s q nhu, vy

,i mi
ng cho huu han
n`
ay ch d
s
t
u
nhin
q. Vo
ch
u
.
.
.
,
,
,c
,i ch
ng thu
c
o nhi`u nht hai s nguyn p, vo
ung thoa m
an bt d
a
, ,
,
. p s p v`
(9.4). Ngha l`
a (9.4) ch thoa m
an cho huu han
a q.
. ca
92

,
,
. 9.4. Nu D l`a s tu. nhin bt k`y, khng l`a s chnh phuo,ng, th` t`n
,
,
,,
tai
ac ca
. p s tu. nhin ( x, y), l`
a nghim
. v han
. c
. cua bt phuong tr`nh
sau

| x2 Dy2 | 1 + 2 D.
(9.8)
,
,
`ng t`n tai
`,i giai. Theo b`
. p s ( x, y) tu.
Lo
ai 9.2 ta bit ra
. v han
. ca
nhin, sao cho


x
D 1 .
(9.9)
y

y2
,
,c sau l`
. t kh
ng thu
ng
Ma
ac bt d
ad
a
u





x x
+ D = ( D ) + 2 D x D + 2 D.
y
y
y

ca
,i mi
. p ( x, y) ta c
Suy ra vo
o

x
+ D 1 + 2 D.

y
2
y
,
,
,i c
. p ( x, y), sao cho thoa m
Nhu vy,
ach chon
an (9.9)
. vo
. bt k`y ca
,,
ch
ung ta nhn
. duo. c

| x2 Dy2 | = | x y D |.| x + y D |

1 1
1
( + 2 Dy) 2 + 2 D 1 + 2 D.
y y
y
`ng bt phu,o,ng tr`nh (9.8) c
,ng minh ra
chu
i`u n`
ay d
o v han
a
.
, ,
nghim
trong
t
p
h
o
p
s
t
u
nhin.
.
.
.
.

,
,
9.1. Xp x mt
. s thu. c

93

,
,
. 9.5. Nu D l`a s tu. nhin bt k`y, khng l`a s chnh phuo,ng, th`
,,
,
phuong tr`nh x2 Dy2 = 1 c
o t nht mt
oi
. nghim
. nguyn (u, v) v
v 6= 0.
,
,
,
,
`,i giai. Ch
. p ( x, y) s tu. nhin, m`
Lo
ung ta xet tt ca c
ac ca
a n
o thoa

. p
m
an 1 2 D x2 Dy2 1 + 2 D. Theo b`
ai 9.4 c
o v s ca
,
,
,
,
,
,
,
2
c trn, Nhung biu thu
c x Dy2
ng thu
s tu. nhin thoa m
an bt d
a
,
,
ch c
o huu han
gi
a tri. v` ch
ung l`
a c
ac s nguyn trong d
oan
. (1
.

,,
2 D, 1 + 2 D ). Ngha l`
a t`n tai
. mt
. s nguyn k, sao cho phuong
,
tr`nh x2 Dy2 = k c
o v han
y). R
o r`
ang k 6= 0,
. nghim
. tu. nhin ( x,

, ,
,
,

v` nu nguo. c lai
d
n mu thun do s D khng th biu
. th` dn
,
x
, ,
,,
,
,
. p n`
i dang
din duo
ay c
o th chon
. huu ty D = y . Giua nhung ca
. t
,
. p kh
nht hai ca
ac nhau ( x1 , y1 ) v`
a ( x2 , y2 ) m`
a ch
ung thoa m
an
x1 x2 (mod |k |), y1 y2 (mod |k |).
(9.10)
,
,
,
,
,
,
. p s du theo
Tht
c`n ch ra tt ca c
ac kha n
ang cua c
ac ca
. vy,ch
.
,,
,
,
. p kh
md
o s luo. ng huu han.
o ngha l`
a t`n tai
ac
un |k | c
. C
. nhung ca
,
,
2
2
2
nhau nhung s tu. nhin ( x1 , y1 ) v`
a ( x2 , y2 ), m`
a x1 Dy1 = x2
,
,
,
2
c (9.10).
ng thu
Dy2 = k v`
a thoa m
an da
,
,
c
ng thu
Ch
ung ta xet d
a

( x1 y1 D )( x2 + y2 D ) = ( x1 x2 y1 y2 D ) + ( x1 y2 x2 y1 ).
(9.11)
,
T`u (9.10) ch
ung ta c
o
x1 x2 y1 y2 D x12 Dy21 k 0

(mod |k|),

x1 y2 x2 y1 x1 y1 x1 y1 0 (mod |k |).
t`n tai
Khi d
a v, sao cho x1 x2 y1 y2 D = ku, x1 y2
o
. s nguyn u v`
,
x2 y1 = kv. V` vy
o th vit th`
anh
. (9.11) c

( x1 y1 D )( x2 + y2 D ) = k(u + v D ).

,
,,
,ng u
,ng dung
Chuong 9. Nhu
kh
ac cua nguyn ly irichle
.
,
,
,c sau c`
,i nhau trong c
ng thu
Nhn t`ung s hang
ung ch
ung ta
ac da
. vo
,,
nhn
uo. c
. d
94

k2 = ( x12 Dy21 )( x22 Dy22 ) = k2 (u2 Dv2 ),

,
,
,,
. p (u, v) l`
suy ra u2 Dv2 = 1, ngha l`
t`u d
a ca
a nghim
o
. cua phuong
tr`nh.
,
,
,
,ng minh v 6= 0. Nu gia thit ngu,o.,c lai,
Ch c`
on phai chu
ung
. ch

ta se c
o x1 x2 = y1 y2 D = |k | v`
a x1 y2 = x2 y1 . Khi d
o

|k|y2 = |( x1 y2 ) x2 y1 y22 D | = |y1 x22 y1 y22 D | = y1 | x22 Dy22 | = |k|y1


,
,
,
,
t`u d
o kha n
ang khi y1 = y2 . i`u n`
ay khng th
y, v` k 6= 0 ch c
,
,
xy ra v` ( x1 , y1 ) kh
ac ( x2 , y2 ) , c`
on t`u y1 = y2 suy ra x1 = x2 .
,
,ng s thu.,c v`

. 9.6. Cho x1 , x2 , . . . , xn l`a nhu


a N l`
a s tu. nhin. Khi do
,
n
ng s nguyn p1 , p2 , . . . , pn , q sao cho 1 q N v`
t`n tai
a
. nhu




xi pi 1 ,
(9.12)

q
Nq
,
v
oi moi
. i = 1, 2, . . . , n.
,
,,
,
`,i giai. Ch
,ng minh b`
,i
Lo
ung ta chu
ai to
an cho tru`ong ho. p n = 2. Vo
,,
,,
,
,
,ng minh ho`
,n ho,n 2 chu
tru`ong ho. p n lo
an to`
an tuong tu. v`
a d`
anh
,
,
,
,

cho ban
a x2 l`
a nhung s thu. c, c`
on N l`
a s tu.
oc.
. d
. Nhu vy
. cho x1 v`
,
`ng t`n tai
,ng minh ra
nhin. Ch
ung ta se chu
nhung s nguyn p1 , p2
.
,
,i 1 q N 2 v`
v`
a q vo
a thoa m
an








x1 p1 < 1 , x2 p2 < 1 .
(9.13)

q
Nq
q
Nq
,
ng v`
. t pha
Ch
ung ta c d
a xet h`nh vung Q
h. toa
d
inh
. trong ma
.
.
,
,
,
i c
vo
ac d
a (0, 1) nhu h`nh ve. Chia Q ra N 2
nh (0, 0), (1, 0), (1, 1) v`
,
,
1 `
,,
`ng nhau vo
,i canh
ng c
ng
l`
a
ba
ac d
h`nh vung nho ba
u`ong tha
.
N
,i truc
,i N=7).
song song vo
. (trong h`nh ve ta chia vo
. toa
. d

,
,
9.1. Xp x mt
. s thu. c

95

,
. p s c
By gi`o ch
ung ta ch
u y dn ca
o dang
(qx1 [qx1 ], qx2
.
,,
,
[qx2 ]), o dy q nhn
a tri. nguyn 0, 1, 2, . . . , N 2 . V` 0
. nhung gi
,
,
,
ca
. p nhu vy c
qxi [qxi ] < 1, i = 1, 2, . . . , N 2 mi
o th coi nhu
,
,
s
`ng c
. p toa
mi
mt
ach d
. cua d
im trong h`nh vung Q. Ba
o
. ca
. d
,
,
,
,
ng trong h`nh vung Q , s
0, 1, 2, . . . , N 2 tao
im tuong u
. ra mt
. d
,,
,
,,
l`
luo. ng c
ac s d
a N 2 + 1. Nhung Q d
o
uo. c chia ra N 2 h`nh vung
,
nho, suy ra t`n tai
ac nhau q1 , q2 trong d
oan
. hai s nguyn kh
. [0, N ],
,
, , ,
,

m`
ad
ung c
o c
ac toa
im tuong ung voi ch
. (q1 x1 [q1 x1 ], q1 x2
. d
`m trong mt
[q1 x2 ]), (q2 x1 [q2 x1 ], q2 x2 [q2 x2 ]) v`a c`
ung na
. h`nh
, ,
1
i canh
c
vung nho vo
. i`u d
o ngha l`
a
o
.
N
1
|(q1 x1 [q1 x1 ]) (q1 x2 [q1 x2 ])| ,
N
1
|(q2 x1 [q2 x1 ]) (q2 x2 [q2 x2 ])| .
(9.14)
N
,
, ,
t q =
Khng mt tnh tng qu
at c
o th gia thit q1 > q2 . Nu d
a
, .
,c
ng thu
q1 q2 , p1 = [q1 x1 ] [q2 x1 ], p2 = [q1 x2 ] [q2 x2 ] th` bt d
a
(9.14) c
o dang
.

|qx1 p1 |

1
1
, |qx2 p2 | .
N
N

(9.15)

`ng 1 q N 2 , v` 1 q2 < p1 N 2 . Chia hai v


Ch
ung ta thy ra
,
,,
cua (9.15) cho q ch
ung ta nhn
uo. c (9.13).
. d
,,
,ng
,i mt
chu
Hai b`
ai to
an duo
id
tnh cht m`
a ta d
y lin quan to
a
.
, ,,
,,
,,
,
minh o chuong 2. mo rng
tnh cht n`
ay ch
ung ta d
ao d
ua v`
inh
.
.
ngha.
,
`ng
Nu a1 , a2 , . . . , a2n+1 l`
a 2n + 1 s thu. c (n 1). Ch
ung ta n
oi ra
,
d
ay n`
ay c
o tnh cht P, nu bt k`y 2n s trong ch
ung c
o th chia
,
,
nh
l`
am hai nh
om, mi
om n s, sao cho tng cua c
ac s trong hai
`ng nhau.
nh
om ba

96

,
,,
,ng u
,ng dung
Chuong 9. Nhu
kh
ac cua nguyn ly irichle
.

,
,ng s nguyn
. 9.7. Chung minh ra`ng moi
., b. 2n + 1 s g`m nhu
,,
duong c
o tnh cht P, th` tt ca c
ac s d`u ba
`ng nhau.
,
,
s.
`ng qui nap
`,i giai. Chu
,ng minh ba
,n nht cua dy
Lo
theo s lo
.
,
,
`ng 1 th` tt ca c
n nht ba
Nu s lo
ac s c`
on lai
ung d
a 1, nn
`u l`
. c
,
, ,,
,
,

b`
ai to
an d
a giai. By gio gia su kt lun
ung voi moi
. d
. b. 2n+1 s
,,
,
,
s khng vuo. t qu
nguyn duong m`
a mi
a k, k 2 v`
a c
o tnh cht P.
,
,
s
Ly p1 , p2 , . . . , p2n+1 l`
a b. s nguyn duong c
o tnh cht P v`
a mi
,,
,
khng vuo. t qu
a k + 1. T`u d
ai to
an suy ra p1 , p2 , . . . , p2n+1
i`u kin
. b`
,
n le.
c
o c`
ung tnh cha
n, ch
a) Nu p1 , p2 , . . . , p2n+1 l`
a c
ac s cha
ung ta xet c
ac s
p1 p2
p2n+1
, ,...,
. Ch
ung ta thy ngay ch
ung c
ung c
o tnh cht
2 2
2
,
,
,i moi
P. Ngo`
ai ra moi
a k v` pi k + 1 vo
. s khng vuo. t qu
. i =
p2
p2n+1
p1
`
ng qui nap
1, 2, . . . , 2n + 1. Ba
. suy ra 2 = 2 = . . . = 2 .Ngha
l`
a p1 = p2 = . . . = p2n+1 .
,
`ng qui
,ng ba
b) Nu p1 , p2 , . . . , p2n+1 l`
a c
ac s le, ch
ung ta chu
nap
ay p1 1, p2 1, . . . , p2n+1 1. Ch
ung c
ung c
o tnh cht
. cho d
,,
,,
P v`
a khng vuo. t qu
a k, ch
ung ta nhn
uo. c p1 1 = p2 1 = . . . =
. d
p2n+1 1. Ngha l`
a p1 = p2 = . . . = p2n+1 .

,
,ng s thu.,c du,o,ng
. 9.8. Chung minh ra`ng moi
b. 2n+1 s g`m nhu
.
,
c
o tnh cht P, th` tt ca c
ac s d`u ba
`ng nhau.
,
,
`ng nu c
`,i giai. - Hin nhin ra
Lo
ac s a1 , a2 , . . . , a2n+1 c
o tnh cht
,
P v`
a q l`
a s thu. c bt k`y, th` c
ac s qa1 , qa2 , . . . , qa2n+1 c
ung c
o tnh

cht P.
,
,
,,
,
,i tnh
By gi`o cho x1 , x2 , . . . , x2n+1 l`
a nhung s thu. c duong vo
,
,
,
,
,
ng d
cht P. Nu tt ca c
ac s l`
a huu ty th` d
b`
ai to
an
i`u kha
inh
. cua
,
,
suy ra khng kh
o. Tht
a bi
. vy,
. ky hiu
. q l`
. s chung nho nht cua

,
,
9.1. Xp x mt
. s thu. c

97

,
s c
qx1 , qx2 , . . . , qx2n+1 l`
mu
ac s trn. Khi d
a nhung s nguyn
o
,,
,,
,ng minh b`
duong c
o tnh cht P, theo c
ach chu
ai truo
c ch
ung ta c
o
,
qx1 = qx2 = . . . = qx2n+1 . V` vy
on
. x1 = x2 = . . . = x2n+1 . Ch c`
,
,
,
,
,

ng minh tru`ong ho. p c


o trong dy x1 , x2 , . . . , x2n+1 mt
phai chu
. s
,
, ,
,
,
v ty. Nhung theo kt qua cua b`
ai 9.6 t`n tai
a c
ac
. s tu. nhin q, v`
,
,
,

s nguyn duong p1 , p2 , . . . , p2n+1 thoa m


an

| xi

pi
|<
q

1
1
1+ 2n+1

, i = 1, 2, . . . , n.

(9.16)

q
,`,
,
,i q > (2n)2n+1 v`
Truong ho. p ring, t`n tai
a
. p1 , p2 , . . . , p2n+1 , q vo
,
1
, i = 1, 2, . . . , n.
ch
ung thoa m
an |qxi pi |
1
q 2n+1
,
,
. t i = qxi pi , i = 1, 2, . . . , 2n + 1. Nhu ph`n d
ng d
a
`u kha
inh
.
,
`

ng qx1 , qx2 , . . . , qx2n+1 c


ra
o tnh cht P. Ch
ung ta se chung minh
`

ng p1 , p2 , . . . , p2n+1 c
, v
ra
ung c
o tnh cht P. Chon
o
. 2n s trong d
,

du. nhu p1 , p2 , . . . , p2n . V` qx1 , qx2 , . . . , qx2n c


o tnh cht P, khng
,
,
,

ng thuc qx1 + qx2 + + qxn =


at ch
ung ta ly d
mt tnh tng qu
a
( p1 + 1 ) + ( p2 + 2 ) + +
qxn+1 + qxn+2 + + qx2n , khi d
o
( pn + n ) = ( pn+1 + n+1 ) + ( pn+2 + n+2 ) + + ( p2n + 2n ).
Ch
ung ta vit lai
.
p1 + p2 + + pn pn+1 pn+2 p2n =

= n +1 +

+ + 2n 1 2 n .
(9.17)
, n +2 ,

ng thuc trn l`
V bn tr
ai d
a s nguyn nn s = n+1 +
a
,
n+2 + + 2n 1 2 n c
ung l`
a s nguyn. Nhung
1
| | | n +1 | + | n +2 | + +
| i |
, i = 1, 2, . . . , n. Do d
o
1
2n
+
1
q
2n
|2n | + |1 | + |2 | + + |n | <
< 1, v` q > (2n)2n+1 . Khi
1
q 2n+1

,
,,
,ng u
,ng dung
Chuong 9. Nhu
kh
ac cua nguyn ly irichle
.
, ,
,c tru,o
,c d
ng thu
ta c
= 0 v`
o p1 + p2 + + p n =
d
a t`u d
o
a
o
,
,
,,
,
ng dinh
pn+1 + pn+2 + + p2n Nhu vy
ung ta kha
uo. c nhung
. d
. ch
,
s nguyn p1 , p2 , . . . p2n+1 c
o tnh cht P. Ch
ung ta c
o th kt lun
.
,,
d
uo. c p1 = p2 = . . . = p2n+1
,
,,
,ng minh d
chu
Thu. c cht d
ung ta d
n d
y ch
a
uo. c :
,
,
Nu p1 , p2 , . . . , p2n+1 , q l`
a nhung s nguyn thoa m
an (9.16) v`
a
2n
+
1
q > (2n)
th` p = p1 = p2 = . . . = p2n+1 . Suy ra (9.16) c
o dang
.

98

p
| xi | <
q

1
q

1
1+ 2n+1

, i = 1, 2, . . . , 2n + 1

,
,
`ng t`n tai
ng d
Trong 9.6 ch
ung ta kha
ra
inh
.
. v han
. s, tu. nhin q
,
,
`ng
thoa m
an (9.16). Khng mt tnh tng qu
at ch
ung ta gia thit ra
,,
,
2n
+
1
l`
nhung s d
a q1 < q2 < . . . < qk < . . ., o d
. Trong
o
y q1 > (2n)
,`,
,
,
i moi
truong ho. p n`
ay vo
. k t`n tai
. s nguyn pk sao cho

| xi

V` lim

1
1
1+
qk 2n+1

pk
|<
qk

1
1
1+ 2n+1

, i = 1, 2, . . . , 2n + 1.

(9.18)

qk

p
,
= 0 khi k tin to,i v c`
ung, t`u (9.18) suy ra lim k =
qk

,
d
,i v c`
,i moi
xi khi k tin to
ung, vo
ay
. i = 1, 2, . . . , 2n + 1. Nhung mi
,
,
,
i han,
s thu. c khng c
o nhi`u hon mt
. gio
. suy ra x1 = x2 = . . . =
x2n+1 .

,
s c
, s, bao gi`o,
. 9.9. Chung minh ra`ng trong tam s, mi
o ba chu
,
,,
,,
, s v`
c
ung chon
o ba chu
a d khi ghep lai
. duo. c hai s c
. ta duo. c mt
. s
chia ht cho 7.
,
,
`,i giai. Khi mang t
Lo
am s chia cho 7 th` th n`
ao c
ung c
o hai s c
o
, ,,
,
,
l`
c`
ung s du (nguyn ly irichl ). Gia su hai s d
a abc v`
a . Hin
o

9.2. B`
ai tp
.

99

,
ach chon
o abc = 7m + r
nhin abc = 1000abc + . T`u c
. ta c
,
,
i 0 r < 7 v`
v`
a = 7n + r vo
a m, n, r l`
a c
ac s tu. nhin. V`
vy
. abc = 1000(7m + r ) + (7n + r ) = 7(1000m + n) + 1001.r =
7(1000m + n + 143.r ).
,
,
. 9.10. Cho a, b, c, d l`a cac s nguyn. Chung minh ra`ng tch cua cac
hiu
a c b chia ht cho 12.
. b a, c a, d a, d c, b d v`
,
`,i giai. C`n chu
,ng minh tch: P = (b a)(c a)(d a)(d c)(d
Lo
`ng mt
b)(c b) chia ht cho 12=4.3. Ch
ung ta bit ra
. s nguyn
,
,
bt k`y khi chia cho 4 th` ch c
o c
ac s du 0, 1, 2, 3.
,,
c nu c
o hai s khi chia cho 4 m`
a
Trong bn s a, b, c, d cho truo
,
,
c
o c`
ung s du th` hiu
ung se chia ht cho 4. Nu khng c
o
. cua ch
,
,
hai s n`
ao khi chia cho 4 cho c`
ung s du th` trong bn s phai c
o
,
,
,
,
n v`
n c
hai s cha
a hai s le. V` hiu
ung nhu hiu
. cua hai s cha
. cua
,
n nn P chia ht cho 4.
hai s le d
a s cha
`u l`
,,
. t kh
Ma
ac trong bn s a, b, c, d lun t`m d
uo. c hai s khi chia cho
,
,
hiu
3 th` c
o c`
ung s du (nguyn ly irichl ). Do d
ung chia
o
. cua ch
ht cho 3, suy ra P chia ht cho 3. T
om lai
. P chia ht cho 12=4.3.

` tp
9.2. Bai
.
,
,
t`n tai
. 9.11. Cho x l`a mt
on n l`
a mt
o
. s thu. c, c`
. s tu. nhin. Khi d
.


,
1
p
,
nhung s nguyn p v`
a q thoa m
an 1 q n v`
a x
.
q
( n + 1) q
,
,
,
. 9.12. Cho x1 , x2 , . . . , xm l`a nhung s thu. c v`a n l`a s tu. nhin. Khi
,
, `
ng
t`n tai
d
o
. nhung s nguyn p1 , p2 , . . . , pn , q khng d`ng th`oi ba
0, sao cho ta c
o q n( = 1, 2, . . . , m) v`
a
1
| q1 x1 + q2 x2 + + q m x m p |
.
( n + 1) m

100

,
,,
,ng u
,ng dung
Chuong 9. Nhu
kh
ac cua nguyn ly irichle
.

`ng vo
,ng minh ra
,i moi
p s nguyn p v`
. 9.13. Chu
ca
a q > 0 ta d
`u


. .

,
3 2
p
,c sau 2
ng thu
c
o bt d
.
a
q
q2
`ng vo
,ng minh ra
,i moi
. p s nguyn p v`
a q > 0 ta d
. 9.14. Chu
`u
. ca



,
1
p
,c sau 3
ng thu
c
o bt d
.
a
q
3 3q2
,
,
,,
. 9.15. Cho m, n v`a s l`a nhung snguyn v`a l`a nghim
. cua phuong
2
`ng nu l`
,ng
tr`nh bc
a
. hai mx + nx + s = 0, (m 6= 0). Chu
minh ra


,
,
p
c ,
,,
i mi
s v ty, th` t`n tai
an 2 vo
. mt
. s duong c thoa m
.
q
q
. p s nguyn p v`
ca
a q > 0.

, ,
CHUONG

10

IRICHL
NGUYN LY
CHO DIN
. TICH

,
biu nguyn l
10.1. Phat
y irichl cho din
. tch

,
,,
,
,
. t pha
ng,
Trong chuong n`
ay ch
ung ta xet nhung tp
. ho. p trn ma
,
,
,
nhung phep to
an trn c
ac tp
ac ban
o
. ho. p nu c
. chua quen bit c
,
,,
,
i c
th xem o Phu. d
ach. Ch
ung ta quan tm to
ac kh
ai nim
nh cui s
.
sau d
y:
,
,
. n, khi t`n tai
. t pha
ng goi
Mt
l`
a bi. cha
. tp
. ho. p trong ma
. mt
.
,
, .
,
,
`
a to`

h`nh tr`
on chu
an b. c
ac d
h
o
p
d
o
.
Khi
khng
t
n
t
ai
im cua tp

.
.
.
,
,
,

mt
h`
nh
tr`
o
n
n`
a
o
nhu
trn
th`

t
p
h
o
p
d
o
g
oi
l`
a
t
p
h
o
p
khng
b
i.

.
.
.
.
.
.
,
,,
,
`
. t pha
. n. V du. nhu mt
. n c`
ng l`
ac li l`
a tp
on nua ma
a
cha
a gi
. d
. bi. cha
,
,
,
,

ng minh d
. n. D d`
tp
ang chu
ac tnh cht sau
uo. c c
., ho. p khng bi. cha
,
,
. n
cua nhung tp
. ho. p bi. cha
,
,
,
,
. n l`
1. Ho. p v`
a giao cua huu han
nhung tp
a mt
.
. bi. cha
. tp
. bi.
,
. n. Hiu
. n l`
. n.
cha
a mt
. cua hai tp
. bi. cha
. tp
. bi. cha
,
,
. n l`
. n. Mt
2. Mt
a mt
. tp
. ho. p con cua mt
. tp
. bi. cha
. tp
. bi. cha
.
,
,
,

tp
h
o
p
ch
u
a
m
t
t
p
h
o
p
con
khng
b
i
ch
a
n
th`

n
o
c
u
ng
khng
b
i.
. .
.
.
. .
.
. n.
cha
,
,
,
,
,
. t pha
ng,
Mt
a dim bin cua tp
im P goi
. d
. l`
. ho. p A trong ma

102

,,
Chuong 10. Nguyn ly irichl cho din
. tch

,
,
,a nhu,ng d
nu moi
nh tr`
on tm tai
P c
o chu
A v`
a ca
im thuc
. h`
.
.
,
,
,
,
,
,
nhung d
A.Tp
ho. p tt ca c
ac d
im khng thuc
im bin cua A goi
.
.
.
,
,
,`,
l`
a bin cua A v`
a ky hiu
a K ( A). V du. bin cua h`nh tr`
on l`
ad
uong
. l`
,
,

i c`
. c bit
tr`
on vo
ung tm v`
a b
an knh. V du. d
a tnh cht bin cua
a
. v`
,
,
tp
. ho. p nhu sau:
,
,
, ,
,
3. Cho tp
an b. nhung d
o toa
im c
. huu ty
. ho. p A g`m to`
. d
,i toa
trong mt
ac d
. c
ir nh (0, 0), (0, 1), (1, 0), (1, 1).
. h`nh vung vo
. d
,
`ng bin cua A l`
D d`
ang thy ra
a to`
an b. h`nh vung n
oi trn.
,
,
,
,i moi
. t pha
ng d
4. Vo
ac d
o
im trong ma
`u c
. tp
. ho. p A g`m c
,
c sau K (K ( A)) K ( A).
cng thu
,
,
,
. p tp
. t pha
ng d
5. Moi
an c
ac
`u thoa m
. ca
. ho. p bt k`y trong ma
,
c sau
cng thu
K ( A B) K ( A) K ( B)
K ( A B) K ( A) K ( B)
K ( A\ B) K ( A) K ( B)
,
,
,
,
,
,
Mt
ad
im P goi
im trong
im trong cua tp
. ,d
. l`
. ho. p A nhung d
`m tron
. t pha
ng, khi t`n tai
ma
on tm P m`
a n
o na
. h`nh tr`
. trong A.
,
,
,
,
T`u d
ay thy ngay l`
a moi
inh
im trong cua tp
`u
. ngha n`
. d
. ho. p A d
,
,
,
,
,
ng: trong v du. 3 o trn tp
thuc
oc lai khng du
. A. i`u ngu
. ho. p A
, . .
khng c
o mt
ao. Ta thy ngay
im trong n`
. d
,
,
,
6. Mt
d
A l`
ad
a chi khi n
o khng
im thuc
im trong cua A khi v`
.
.
,
,
,
,
,
l`
ad
ay giai thch tai
im bin cua A. i`u n`
im trong
. sao nhung d
,
,`,
`
m trn d
cua h`nh tr`
on khng na
on.
uong tr`
,
,
,
,
. c bit,
loai
ac tp
ung ta d
ao mt
ai nim
a
ua v`
. tr`u c
. ho. p d
. ch
. kh
.
,
,
,
,
p tp
. c chung cho lo
. c bit
. t pha
ng : Mt
d
trong ma
a
a
.
. ,ho. p khng d
, .
,
`
. t, khi bin
. n c
t pha
ng goi
tp
ac d
a b ma
im trong ma
., ho. p bi. cha
. l`
,
,.
,

cua n
o khng chua d
im trong (cua bin).
,
. c d
. t trong ma
. t pha
ng.
V du. c
ac h`nh tr`
on hoa
ac d
a b` ma
a gi
`u l`

,
10.1. Ph
at biu nguyn ly irichl cho din
. tch

103

,
,
`ng c
. t. Ba
Nhung trong v du. 3 tp
a b` ma
ach d`
ung
. ho. p A khng l`
,
`
`
c
ac d
a c
ac d
ung
inh
inh
. ngha v tp
. ngha trong phn trn ch
. ho. p v`
,
,
,
ng minh d
ta chu
uo. c
,
,
. t, nhung tp
7. Nu A v`
a B l`
a hai b` ma
a
. ho. p A B, A B v`
,
,
. t trong ma
. t pha
ng.
A\ B c
ung l`
a nhung b` ma
,
. t v`
a A khng c
o chung d
8. Nu A, B v`
a C l`
a c
ac b` ma
im trong
,
,i B v`
,i C, th` A khng c
,i B C.
vo
a vo
od
im trong chung vo
,
,
, ,
ng, nhi`u khi
Mt
inh
. ly co ban trong h`nh hoc
., trong nhung d
. pha
,,
,
o ph thng ch
ung ta cng nhn
`:
. nhu mt
. tin d
,
,
,
,
,,
. t A nhung d
. t pha
ng c
9. Moi
o th cho tuong
im trong ma
. b` ma
,
,ng vo
,i mt
u
. s thu. c khng m S( A) sao cho
,i l`
a) S() = 1, vo
a mt
o canh
l`
a 1;
. h`nh vung c
.
,
. t khng c
b) Nu A v`
a B l`
a hai b` ma
od
im trong chung, th`
S ( A B ) = S ( A ) + S ( B ).
, , ,
,,
,i c
Phep cho tuong u
ng S vo
ac tnh cht trn d
ac d
uo. c x
inh
. mt
.

c
ach duy nht.
,
,
. t A bt k`y, s S( A) goi
. t
Cho b` ma
l`
a din
tch cua A. Nhung ma
.
.
,
,,
,`,
,
d
d
ac truong ph thng l`
a h`nh chu nht,
ac,
a
uo. c xet trong c
. tam gi
,
i kh
h`nh tr`
on,. . . v`
a s S( A) theo d
ung vo
ai nim
inh
. ngha trn tr`
.
,
,
,
,
,
`

din
t
ch
c
ua
c
a
c
h`
nh
n`
a
y.
V
o
i
c
a
ch
tr
u
u
tu
o
ng
h
o
a
kh
a
i
ni
m
di
n
.
.
.
.
,
,
tch ch
ung ta d d`
ang khao s
at tnh cht v` din
t
ch
c
ua
c
a
c
h`
nh.
.
,
,
,,
,ng minh d
T`u a) v`
a b) ch
ung ta c
o th d d`
ang chu
uo. c:
,
. t v`
10. Nu A v`
a B l`
a nhung b` ma
a A B, th` S( A\ B) =
S ( A ) S ( B ).
,
. t t`ung d
11. Nu A1 , A2 , . . . , An l`
a c
ac b` ma
o
i mt
. khng c
,
d
im trong chung, th` S( A1 A2 . . . An ) = S( A1 ) + S( A2 ) +

104

,,
Chuong 10. Nguyn ly irichl cho din
. tch

+ S ( A n ).
Nguyn l
y irichl cho din
a mt
. t, c`
on
. b` ma
. tch: Nu A l`
A1 , A2 , . . . , An l`
a c
ac b` ma
. t sao cho Ai A(i = 1, 2, . . . , n) v`
a
S( A) < S( A1 ) + S( A2 ) + + S( An ), th` t nht c
o hai b` ma
. t
,
trong s c
ac b` ma
. t trn c
o mt
. dim trong chung.
,
,
C
ung nhu nguyn ly d
ung ta c
ung c
o th thy d
`u tin, ch
i`u
,
, ,,
,
,
,
ng minh d
n`
ay l`
a hin nhin v`
a chu
o
uo. c. Tht
. vy,
. Gia su khng c
,
,
. p n`
. t d
cho c
theo
ca
ao trong nhung ma
od
a
im trong chung. Khi d
o
,
ng d
kha
o S ( A1 A2 . . . A n ) = S ( A1 ) + S ( A2 ) + +
inh
. 11. ta c
. t kh
S ( A n ) . Ma
ac Ai A(i = 1, 2, . . . , n) suy ra A1 A2 . . . An
,
,
ng
c
o S( A1 A2 . . . An ) S( A). Thng nht c
ac bt d
A, t`u d
o
a
to
,c lai
,i v ly.
thu
ung ta c
o S( A) < S( A), dn
. ch
,
`ng nguyn ly trn bao tr`
C
o th thy ra
um nguyn ly irichl
,
,
,
cho nhung tp
huu han.
Ngo`
ai ra nguyn ly irichl trn c
o th cu.
.
.
,
,
,
,,
,p b` ma
. t thng dung
th h
oa cho nhung lo
trong chuong tr`nh ph
.
,
,
thng v`
a c
ac kh
ai nim
ai, th tch c
ung c
o c`
ung tnh cht nhu
. d`
. d
,
,
,,
ch
kh
ai nim
ung ta c
o th ph
at biu nguyn
o
. din
. tch o trn. Do d
,,
n kh
ly irichl theo c
ac phuong a
ac nhau.
,
,
ng doan
12.Cho nhu
ng 1 , 2 , . . . , n na
`m trong doan
a
. tha
. v`
,
,
,
,
,
t nht c
tng d
ai cua 1 , 2 , . . . , n l
on hon d. d`
ai cua . Khi do
o
. d`
,
,
,
ng doan
hai trong s nhu
ng 1 , 2 , . . . , n c
o dim chung.
. tha
,
,ng da din
13. Cho nhu
P1 , P2 , . . . , Pn na
`m trong da din
P v`
a tng
.
.
,
,
,
,
,
,
t nht c
th tch cua P1 , P2 , . . . , Pn l
on hon th tch cua P. Khi d
o hai
o
,
,
ng da din
trong s nhu
o dim chung.
. P1 , P2 , . . . Pn c
,
,
,,
ng cung c1 , c2 , . . . , cn na
14. Cho nhu
`m trn du`ong tr`
on c v`
a tng
,
,,
,
,
t nht c
on c. Khi do
o
d. d`
ai cua c1 , c2 , . . . , cn l
on hon d. d`
ai du`ong tr`
,
,

ng cung c1 , c2 , . . . cn c
hai trong s nhu
od
im chung.

10.2. V du.

105

,
,
Tt ca c
ac ph
at biu 12, 13, 14 ch
ung ta d
a nguyn ly
`u goi
. l`
, ,
,
,
ng minh d
irichl v`
a ban
o th chu
ac nguyn ly n`
ay.
oc
uo. c c
. d
. c

10.2. V du.
,
,
. 10.1. Cho M l`a mt
ac l`i v
oi din
a chu vi P. Chung
. da gi
. tch S v`
minh ra
`ng
, , ,,
,
,
, S
a) M c
on v
oi b
an knh l
on hon ;
o th phu duo. c mt
. h`nh tr`
P
,
S
b) B
an knh cua c
ac h`nh tr`
on na
`m trong M khng qu
a .
P
,
,
,
canh
`,i giai. a) Ch
Lo
ung ta du. ng trn mi
d
ac M mt
a gi
.
. h`nh chu
S
,
nht
. chi`u cao h = P , nhu h`nh 10.1.

M
O

h=
H`nh 10.1:

C
S
P
H`nh 10.2:

,
,
,
,
,
Nhung h`nh chu nht
ay c
o nhung d
ung; n
oi
im chung giua ch
. n`
,
`

m tron
chung mt
nh chu nht
na
ung ta c
o
. s h`
. khng
. trong M. Ch
,
,
,
,
,

th tnh to
an tng din
ac h`nh chu nht
ay l`
a S. Nhu vy
. tch cua c
. n`
,
,
,
, , .
,
`

phn cua M bi. c


ac h`nh chu nht
o din
. phu phai c
. tch nho hon S.
,
,
,
`ng t`n tai
i`u n`
ay ch ra ra
im O cua M khng thuc
. mt
. d
.

106

,,
Chuong 10. Nguyn ly irichl cho din
. tch

,
,
,
,
,
ao ca. Nhu vy
o ngha l`
a khoang c
h`nh chu nht
ach t`u O d
n
. n`
. c
,
, ,
S
n ho,n h = . Ta ly O l`
c
ac canh
cua M phai lo
a tm h`nh tr`
on b
an
.
P
S
`m tron
knh R > , h`nh tr`
on n`
ay se na
. trong M.
P
`m trong M. Ch
ung ta xet
b) Cho h`nh tr`
on C tm O b
an knh R na
,
,
,
,
,
gi
c
ac tam gi
ac m`
a hai d
o l`
a hai d
ac, c`
on
nh cua n
nh lin tip cua tu
,
,
,
,
,
ba l`
d
a tm h`nh tr`
on O. u`ong cao ha. t`u O xung c
ac canh
nh thu
.,
,
,
,
,
,
n hon R. T`u d
cua tam gi
ac n`
ay lo
ac din
t
ch
cua
y suy ra tng c
.
, ,
S
`ng S v`
ch
ung ba
a khng nho hon P.R. Suy ra R
(H`nh 10.2).
P

,
. 10.2. Trong khng gian cho 30 vecto, khac khng. Chung minh ra`ng
,
,
,
,a ch
c
trong s do
o hai vecto m`
a g
oc giu
ung nho hon 450 .
,
, ,
,
`ng tt ca
`,i giai. C
Lo
o th gia thit ra
,
,
vecto c
o chung d
im d`u O. Ly OA
A
`ng 1 trn vecto, thu
, nht.
,i d
vo
d`
. ai ba
,
,
,
i
Ch
ung ta du. ng h`nh n
on d
nh O vo
0
,, ,
0
J 45
truc
a g
oc o d
a 45 . B`
ai
nh l`
. OA, m`
,,
O
,ng minh nu ch
ung
to
an se d
uo. c chu
,
`ng t nht hai trong s 30
ta ch ra ra
,,
,,
h`nh n
on (duo. c xy duong theo c
ach
,
,
,
,
ng vo
i 30 vecto d
cho) c
trn u
od
a
im
trong chung (h`nh 10.3).
H`nh 10.3:
,i tm
Ch
ung ta xet h`nh c`u S vo
l`n du.,ng h`nh n
t ma
. t c`u S mt
O v`
a b
an knh 1. Mi
on ca
. h`nh
,
,
,
,
`ng
i din
vo
a c
o th tnh to
an d
ung thy ra
uo. c. Ta c
. tch 1 m`
,
,
hai h`nh n
on c
o d
a chi khi nhung ph`n
im trong chung khi v`
,
,
, `
. t c`u c
trn ma
ung phai c
o d
ay v`
a
im trong chung. T`u d
iu n`

10.2. V du.

107

,
,
,
nguyn ly irichl ch
ung ta ch c`n thit kim tra tng din
. tch
,
,
,
`
`
`
n hon din
. t cu lo
. t cu (ba
ng 4).
cua 30 h`nh trn ma
. tch ma
p
!


2+ 2

Ch
ung ta c
o 1 = 2 1 cos
= 2 1
. Vy
.
8
2
!
p
p

14
2+ 2
2+ 2
,,
,,
,
<
30.2 1
> 4 tuong duong voi
2
2
15

2
1
167
. c l`
a <
hoa
.
2
225
,
,
. 10.3. Nu mt
. t A trong ma
. t pha
ng thoa m
an di`u kin
. b` ma
.
,
,
S( A) > 1. th` n
o lun lun chua t nht hai dim trong ( x1 , y1 ),
,ng s nguyn.
( x2 , y2 ) m`a hiu
a y2 y1 l`
a nhu
. x2 x1 v`

,
,
d
`,i giai. Qua mi
,i
Lo
im (m, n) vo
y
,
toa
d

nguyn
ch
u
ng
ta
k
e
c
a
c

.
. ,
,
,,
,,
,ng v`
ng du
ng
d
ad
u`ong tha
u`ong tha
ngang (h`nh 10.4).
,,
,,
Ch
ung ta se tao
d
i
uo. c luo
.
3 2 1 0 1 2 3 x
,
,
,
i toa
nhung d
im vo
. d., nguyn.
,,
. t pha
ng ra c
Luoi nguyn chia ma
ac

`
ng nhau, mi h`nh
h`nh vung ba
`ng
vung c
o din
a 1. Ch
u y ra
. tch l`
,
H`nh 10.4:
h`nh vung c
mi
o th x dich
n
. d
,
,
,i mt
tr`
ung vo
h`nh vung kh
ac, ch c
o kh
ac l`
a toa
d
. cua h`nh vung
.
.
,
,i chuyn d
mo
ung l`
a c
ac s nguyn.
n c
,,
Ch
ung ta chon
i nguyn l`
am gc c
. ,mt
. h`nh vung trong luo
,
d
inh
. r`i dich
. ,chuyn moi
. h`nh vung v` h`nh vung gc. Nhu vy
.,
,
`m trong c
ac h`nh vung kh
ac nhau d`u chuyn
nhung ph`n cua A na
,
,
,

v` h`nh vung gc (h`nh ve). Tng cua din


ac ph`n d
o
. tch cua c

108

,,
Chuong 10. Nguyn ly irichl cho din
. tch

,
`ng din
,n ho,n 1. Suy ra t nht hai trong
ba
a suy ra lo
. tch cua A v`
,
`m trong h`nh vung dich
s c
ac ph`n na
od
n se c
im trong chung
., d
, , ,
,i hai
( x0 , y0 ). Trong tp
A ban d
ng vo
`u th` d
im ( x0 , y0 ) tuong u
.
,
d
ac nhau ( x1 , y1 ) v`
a ( x2 , y2 ) m`
a x1 x0 , y1 y0 , x2 x0 , y2
im kh
,

y0 l`
a c
ac s nguyn. Nhu vy
a y2 y1 c
ung l`
a snguyn.
. th` x2 x1 v`

,
,
,ng dim trong ma
. 10.4. Cho A l`a tp
ho. p l`i v`
a bi. cha
. n nhu
. t
.
,
,
,
ng dim thuc
pha
ng, c`
on P1 , P2 , P3 , P4 , P5 l`
a nhu
A. Goi
a tp
. Ai l`
.
, .
,
,
,

`
ho. p nhn
t
u
A
sau
m
t
ph
e
p
t
inh
ti
n
c
a
c
d
i
m
theo
vecto
P
P
(
i
=

1
i
.
.
.
,
,
`
1, 2, 3, 4, 5). Chung minh ra
ng t nht c
o hai tp
ac
. ho. p trong s c
,
Ai (i = 1, 2, 3, 4, 5) c
o dim chung.
,
`,i giai. Ch
Lo
ung ta chia ra hai
,
Q
Qi
,`,
,
. c
truong ho. p A c
o th c
o hoa
,
,
c
o th khng c
od
im trong.
Q00
,
,`,
,
1. Truong ho. p A c
od
im
,
0
trong. Ky hiu
a tp
. A l`
. ho. p,
,, ,
Pi
P1
m`
a n
o nhn
uo. c t`u A, sau
. d
,
khi t
ac d
phep vi. tu. tm
ng
.
H`nh 10.5:
,

P1 v`
a h. s vi. tu. 2. Khi d
o
,c sau d
ng Ai A0 (h`nh 10.5).
cng thu
u
,
,
Tht
a Qi l`
a anh cua Q qua phep tinh
. tin theo
. vy,
. nu Q A v`
,
vecto P1 Pi (xem h`nh ve), th` P1 Pi Qi Q l`
a h`nh b`nh h`
anh. Ky hiu
.
,
,
,,
00
00
`
Q l`
a trung d
i
m
c
ua
P
Q
.
R
o
r`
a
ng
Q

A,
v`

Q
v`
a
P
l`
a
ph
n
tu

1 i
i
,
,
,
. t kh
cua A v`
a A l`i. Ma
ac P1 Qi = 2P1 Q00 v`
a v` A0 l`
a anh cua A qua
,
,
,
0
`m trong tp
phep vi. tu. tm P1 v`
a h. s 2, nn d
im Qi na
. ho. p A .
,
tp trong c
tp
`m trong A0 . `ng th`o,i mi
Nhu vy
ac tp
. mi
. Ai na
.

10.2. V du.

109

,
,i A v`

cua Ai d
a suy ra S( Ai ) = S( A). Do d
`ng dang
o
. vo
S( A1 ) + S( A2 ) + S( A3 ) + S( A4 ) + S( A5 ) = 5S( A).
,
,
0 `ng dang vo
,i A vo
,i h. s 2. Suy ra
Nhung tp

. ho. p A d
.

(10.1)

S( A0 ) = 4S( A).

(10.2)
,
,
. t kh
ac A c
od
a t`u (10.1) v`
a
Ma
im trong, v` th S( A) > 0. Ngha l`
0
(10.2) suy ra S( A1 ) + S( A2 ) + S( A3 ) + S( A4 ) + S( A5 ) > S( A ).
,
,,
,,
,
,
Nhu vy
ai to
an d
uo. c suy ra t`u nguyn ly
. trong tru`ong ho. p 1 b`
irichl cho din
. tch.
,
,`,
,,
,
`m trn mt
2. Truong ho. p A khng c
od
im trong, A se na
u`ong
. ,d
,
,
,,
`m trn mt
ng, do
ng. Tht
tha
im na
u`ong tha
. vy,
. nu t nht ba d
. d
,
,
,
,
a to`
i dnh l`
tnh cht l`i A chu
an b. h`nh tam gi
ac vo
a c
ac d
im
,
,
,,
. n trn mt
n`
ay, suy ra A c
od
im trong. Nhung tp
u`ong
. l`i,, bi. cha
. d
,
,
,,
ng chnh l`
ng trn du`ong tha
ng n`
tha
ad
ay. Ph`n c`
on lai
oan
. tha
. ly
,,
,
,
,
,
,
,
,
,
lun
tu. nhu ph`n trn. Nhung trong tru`ong ho. p trn d
u`ong
., tuong
,
,
ng ch c`n 3 d
tha
a nguyn ly irichl v` d
ai.
im v`
. d`
,
,
,
. 10.5. (inh
a tp
. t pha
ng
. ly Minkovski) Cho A l`
. ho. p dim trong ma
,
c
o tnh cht l`i, bi. cha
. n v`
a di xung qua gc toa
a S( A) > 4. Khi
. d. v`
,
,
,
,
,
ng dim trong kh
A chua nhu
do
ac dim gc v
oi toa
. d. nguyn.

,
1
,
`,i giai. T
Lo
ac dung
ln A phep vi. tu. tm l`
a gc toa
a h. s .
. v`
.
. d
,
,,
,2,
,i A v`
Nhu vy
ung ta nhn
a c
o kch thuo
c
uo. c A0 , d
`ng dang
. ch
. d
. vo
,
,,
1
,
,
`ng nua kch thuo
c cua A. Suy ra S( A0 ) = S( A) > 1. Theo b`
ai
ba
4
,
,
0
a t nht hai d
10.13 suy ra A chu
ac nhau ( x1 , y1 ) v`
a ( x2 , y2 ),
im kh
,
,ng qua
a y2 y1 l`
a nguyn. V` tp
m`
a c
ac hiu
i xu
. x2 x1 v`
. h,o. p A d
,
,
0 ung vy. i`u do
`ng d
gc toa
im
. nn tp
giai thch ra
. d
. ho. p A c
.
,
0
0 ung l`i. Do do
( x1 , y1 ) thuc
trung d
im
. A . V` A l`i nn A c

110

,,
Chuong 10. Nguyn ly irichl cho din
. tch

,
,
0 a c
o toa
cua c
ac d
ut ( x1 , y1 ) v`
a ( x2 , y2 ) c
ung thuc
.
im m
. d
. A v`
,
x2 x1 y2 y1
,
,
,
i 2 th` d
(
,
). Nhn toa
ay vo
. n`
uo. c toa
. cua mt
. d
. d
.
2
,2
,
,
,,
d
im thuc
uo. c
. A. Nhu vy
. ( x2 x1 , y2 y1 ) thuc
., A. im, ta t`m d
kh
ac gc toa
a c
o toa
a l`
ad
. v`
. nguyn v`
im trong cua tp
. d
. d
. A.

. 10.6. Nu A l`a mt
. t,
. b` ma
(10.3)
,
,
,ng b` ma
l`
a nhu
. t v`
a thoa m
an Ai A(i = 1, 2, . . . , n), c`
on k l`
a s tu.
nhin m`
a
A1 , A2 , . . . , A n .

k.S( A) < S( A1 ) + S( A2 ) + + S( An ).
(10.4)
,
,ng b` ma
th` t nht k + 1 trong s nhu
. t trn c
o mt
. dim trong chung.
,
,`,
,
`ng qui nap,Tru
`,i giai. Ch
,ng minh ba

Lo
ung ta chu
ong ho. p k = 1 d
o
.
,
,,
,
,
,
,
ng minh o trn. By gi`o gia su b`
chu
l`
a nguyn Ly irichl d
ai to
an
a
,
,
ng minh n
d
ng cho k, ch
ng cho k + 1.
d
ung ta phai chu
o c
ung d
a
u
u
,
,
c
ng thu
a c
o bt d
Ta c d
inh
a
. mt
. k v`

( k + 1) S ( A ) < S ( A1 ) + S ( A2 ) + + S ( A n ).
(10.5)
,
,
,
,
`ng vo
,i (10.5) t`n tai
mt
d
ad
Ch
ung ta se ch ra ra
im l`
im trong cua
.
.
, ,
k + 2 tp
. ho. p cua (10.3). Do A A( = 1, 2, . . . , n), nn S( A )
,
,
suy ra S( A1 ) + S( A2 ) + + S( An ) nS( A). T`u bt
S( A), t`u d
o
,
,c sau c`
ng thu
d
ung v`
a (10.5) ch
ung ta c
o (k + 1)S( A) < nS( A) suy
a
ra k + 1 < n. V` vy
.
n k + 2.
(10.6)
,
,,
,

,ng

T`n tai
d
i
m
chung
cho
t
nh
t
k
+
2
t
p
h
o
p
(10.3)
d
u
o
c
u

.
.
.
. ch
,
,
t
`ng qui nap
minh ba
. theo n. T`u (10.4) suy ra (10.5), vy
. ta phai ba
,
,
d
`u t`u k + 2. Nhu vy
. vit lai
.
( k + 1 ) S ( A ) < S ( A 1 ) + S ( A 2 ) + + S ( A k +2 ).

(10.7)

10.2. V du.

111

,
,
,
,
,ng minh t`n tai
C`n phai chu
a n
o l`
ad
im trong cua A1 , A2 ,
. dim m`
. . . , Ak+2 . Ly
A0 = A \ Ak+2 ( = 1, 2, . . . , k + 1)

(10.8)

A00

(10.9)

= A Ak+2 ( = 1, 2, . . . , k + 1)

v`
a
A 0 = A \ A k +2

(10.10)

A00 = Ak+2 .

(10.11)
,
R
o r`
ang A0 A0 v`
a A0 A00 ( = 1, 2, . . . , k + 1) . V` c
o tt ca k + 1
,
,
0
,c trn suy ra
tp
am thu
. ho. p A , t`u bao h`

(k + 1)S( A0 ) S( A10 ) + S( A20 ) + + S( A0k+1 ).


,
,,
Nu ly (10.7) tr`u d
ung ta nhn
i (10.12) ch
uo. c
. d

(10.12)

(k + 1)S( A00 ) < S( Ak+2 ) + S( A100 ) + S( A200 ) + + S( A00k+1 ).


(10.13)
0
00
0
00
V` S( A) S( A ) = S( A ) v`
a S( A ) S( A ) = S( A ) do (10.8),
(10.11) nn t`u (10.13),(10.9) v`
a (10.11) suy ra
kS( Ak+2 ) < S( A1 Ak+2 ) + S( A2 Ak+2 ) + + S( Ak+1 Ak+2 ).
(10.14)
,
,

ng cho k theo qui nap


T`u (10.14) v`
a gia thit d
u
. suy ra A1
,
Ak+2 , A2 Ak+2 , . . . , Ak+1 Ak+2 c
o d
ay
im trong chung, d
i`u n`
,
,
,
c
o ngha l`
a tp
od
im trong chung. Nhu vy
. ho. p A1 , A2 , . . . , Ak+2 c
.
,
,
,
,

i n = k + 2 t`u (10.5) suy ra t nht k + 2 tp


vo
o
. ho. p t`u (10.3) c
,
d
im trong chung.
,
,
,
,i mt
By gi`o ch
ung ta gia thit vo
ra
., n k + 2 t`u (10.5) suy
,
,
,
t nht k + 2 tp
od
a se phai kt
im trong chung v`
. ho. p t`u (10.3) c
,
`ng t`u
lun
. ra

(k + 1)S( A) < S( A1 ) + S( A2 ) + + S( An ) + S( An+1 ). (10.15)

,,
Chuong 10. Nguyn ly irichl cho din
. tch

112

,
,
A1 , A2 , . . . , An+1 c
suy ra c
o t nht k + 2 tp
od
im
. ho. p trong dy
. t
trong chung. Tht
ung ta d
a
. vy,
. ch
A0 = A \ An+1 ,
A00

= A A n +1 ,

( = 1, 2, . . . , n).
( = 1, 2, . . . , n)

(10.16)
(10.17)

v`
a
A 0 = A \ A n +1

(10.18)

A00 = An+1 .

(10.19)

V` A0 A0 = A0 , A0 A00 = ( = 1, 2, . . . , n) v`
a A0 A00 =
A, A0 A00 = nn
S( A0 ) + S( A00 ) = S( A ), ( = 1, 2, . . . , n)

(10.20)

v`
a
S( A0 ) + S( A00 ) = S( A),.
(10.21)
,
,

ng minh mt

ng
Ch
ung ta se chu
trong
c
a
c
b
t
d
a
ng
th
u
c
sau
l`
a
d

u
.

(k + 1)S( A0 ) < S( A10 ) + S( A20 ) + + S( A0n )

(10.22)

. c l`
a
hoa
kS( A00 ) < S( A100 ) + S( A200 ) + + S( A00n ).
(10.23)
,,
,,
,
Tht
ung ta se c
o ( k + 1) S ( A 0 )
. vy,
. trong tru`ong ho. p nguo. c lai
. ch
S( A10 ) + S( A20 ) + + S( A0n ) v`
a kS( A00 ) S( A100 ) + S( A200 ) + +
,,
S( A00n ) Cng
a do (10.20), (10.21) ch
ung ta nhn
. hai v lai
. v`
. duo. c
S( A0 ) + kS( A) S( A1 ) + S( A2 ) + + S( An ).
(10.24)
,
,i S( A00 ) v`
Cng
a t`u (10.19), (10.21) ch
ung ta nhn
. hai v (10.24) vo
.
,,
d
u
o
c
.
( k + 1) S ( A ) S ( A1 ) + S ( A2 ) + + S ( A n )
,
,
,c
,i (10.15). Do d
ng thu
trong hai bt d
Nhung di`u n`
ay tr
ai vo
o
a
,
ng.
(10.22) v`
a (10.23) phai c
o t nht mt
ai d
u
. c

10.2. V du.

113

, ,,
,
,i n t`u, (10.22)
ng. Theo gia thit qui nap
Gia su (10.22) d
u
i vo
. d
,
,
A0 , A0 , . . . , A0 c
suy ra t nht k + 2 tp
im trong
. ho. p trong dy
n od
2
1
,

`ng kt lun
ng cho dy
chung. T`u (10.16) suy ra ra
ung d
u
. trn c
(10.3).
, ,,
, ,
,i k suy ra k + 1
ng. T`u gia thit qui nap
Gia su (10.23) d
u
i vo
. d
,
,
,
00
00
00 o dim chung v`
,i (10.17) ch
tp
a c`
ung vo

. ho. p trong A1 , A2 , . ,. . , An c
,
,
, ,
`ng t`n tai
ra ra
mt
d
a n
o l`
ad
im m`
im trong cua k + 1 tp
.
.
. ho. p t`u
, ,
(10.3) v`
a ca cua An+1 .
,
,
,
Nhu vy
ay A1 , A2 ,
. t`u, (10.15) suy ra k + 2 tp
. ho. p trong d
,
i n + 1. T`u,
ng vo
. . . , An+1 c
od
im trong chung. Suy ra kt lun
u
. d
,,
,
,i moi
,i (10.5) t`n
phuong ph
ap qui nap
. by ,gi`o lai
. suy, ra, vo
. n, vo
,
,
tai
a n
o l`
ad
im m`
im trong cua t nht k + 2 tp
. mt
. d
. ho. p trong
(10.3).
,,
,ng minh d
. Trong chu
Ch
u y
ly d`
ung phuong ph
ap qui nap
inh
.
.
,
,
,
,
,
,
c k sang k + 1 ta lai
theo k. Nhung d
ung phuong ph
ap qui
i t`u buo
. d`
,
,
,
,
,
ng minh qui nap
c n sang n + 1 ta
i chu
nap
. theo n. Vo
. theo n t`u buo
,,
,,
v`
lai
ung buoc k truoc d
a n.
o
. d`

. 10.7. Trong mt
o canh
l`
a
. h`nh vung c
.
,
,
,
,,
1 chua mt
on, Tng d. d`
ai cua
. s du`ong tr`
,
ch
ung l`
a 10. Chung minh ra
`ng t`n tai
. mt
.
,
,,
,ng
du`ong tha
ng, m`
a n
o ca
t t nht 4 trong nhu
,,
du`ong tr`
on n`
ay.
,
`,i giai. (H`nh 10.6) Ch
Lo
ung ta chon
. mt
.
canh
h`nh vung r`i chiu vung g
oc c
ac
.
,,
.
d
on xung canh
d
u`ong tr`
o
.

H`nh 10.6:

114

,,
Chuong 10. Nguyn ly irichl cho din
. tch

,
,,
`ng h`nh chiu cua mt
D thy ra
on b
an knh R l`
a mt
. du`ong tr`
.
,

d
od
ai 2R. V` vy
h`nh vung d
oan
. d`
a chon
. thang c
. trn canh
.
. se
,
,
10
,
,
i tng d
ng chiu xung vo
c
o nhung d
a .
oan
d`
. tha
. ai l`

10
,
> 3 = 3.1, theo nguyn ly irichl suy ra co mt
Nhung
.

,
,
,
,
ng d
thuc

d
ao do
ad
im M n`
im trong chung cua 4 d
oan
a
. AB l`
. tha
,
,
,
,
,
i AB
ng d
d
chiu xung. Khi d
oc vo
o
u`ong tha
i qua d
im M vung g
,`,

t 4 d
se ca
on.
uong tr`

,, ,
,,
,
. 10.8. Mt
on v
oi b
an knh 1 duo. c son
s cung na
`m trn d
u`ong tr`
, .
,
,
,
,
xanh. Tng d. d`
ai cua c
ac cung m`u xanh l
on hon . Chung minh
,
,,
, ,`,
on t`n tai
ra
`ng v
oi mi
duong knh d cua du`ong tr`
. dy cung song song
,
,
,
,
,
v
oi d, m`
a hai d`u cua dy cung duo. c son xanh.
,
`,i giai. (H`nh 10.7) Khng mt
Lo
C
,
,
A
tnh tng qu
at ch
ung ta gia thit
,
,
c
ac cung bi son khng c
o d
im
d
,`,
chung. Cho d l`
ad
uong knh bt k`y
,
,
cua du`ong tr`
on k v`
a c l`
a mt
. trong
,,
,,
d0
`
c
ac cung d
uo. c bi mu. Trn d
u`ong
0 `
ng c
tr`
on k ta tao
ach
. ra cung c ba
,
,
,

ng c qua d
ly d
i xu
u`ong knh d0
k
A0
,
i d.
vung g
oc vo
C0
H`nh 10.7:
D thy cung c0 cho c`
ung d
ai
. d`
,
,,
,i cung c. Ta l`
,i moi
vo
am nhu vy
on lai
uo. c
. vo
. cung c`
. th` trn k ,nhn
. d
,
,n ho,n 2. Nhu,ng d
mt
o tng d
ai lo
ai cua k l`
a 2,
. d`
. d`
. h. cung c
,
nhu vy
theo nguyn ly irichl c
o t nht hai cung trong h. n`
ay c
o
, .
dim chung.

10.2. V du.

115

,
,,
,
,
,
gia thit o trn nhung cung dang
ung ta d
c t`ung d
Nhung ch
a
i
.
,
0
`
c
ng cho c

khng c
od
ung d
ac cung c . Khi d
im chung, d
iu d
o
u
o
,
,
,
,
0
`

ng
i cung c2 , d
tn tai
od
im chung A vo
i x u
. mt
. cung bi son c1 c
,
,
0
i cung c2 qua d . R
i d c
vo
o r`
ang dy cung d
a song song vo
o
i qua A v`
,,
,,

`
`
tnh cht mong mun v` mt
on d
u o c1 , c`
u kia o c2 .
. d
,
. 10.9. Trong h`nh tr`on ban knh 1 ngu`o,i ta t so,n mt
. s dy cung.
,
,
,

`
t nhi`u nht k dy cung, th`
Chung minh ra
ng nu moi
du`ong knh ca
,
,
, .
, ,
,
t son nho hon k.
tng d. d`
ai cua tt ca c
ac dy cung da
,
`,i giai. Ly c l`
cho v`
Lo
a mt
on C d
a
a
. dy cung bt k`y trong h`nh tr`
,
0
,ng
l`
a cung nho trong hai cung do c tao
a cung d
i xu
. ra. Ky hiu
. l`
,
,
,,
t dy
,i qua tm cua h`nh tr`
vo
on. R
o r`
ang mt
u`ong knh cua C ca
. d
,
`m trong v`
cung c khi v`
a ch khi c
o hai d
ut na
a 0 .
`u m
, ,,
,
, ,
,
`ng tng d
n ho,n hoa
. c
ke lo
Gia su ra
ai nhung dy cung d
. d`
a
,
,
,
,
`

n hon k.
ng k. Khi d
tng d
ba
ai tt ca c
ac cung dang
lo
o
. d`
.
,
,
,
0
ng nhu vy
C
ung d
cho tng d
ai c
ac cung dang
. Suy ra tng
u
. d`
.
.
,
,
,n ho,n 2k. V` d
. c l`
d
ai cua c
ac cung dang
0 hoa
a lo
ai cua
. d`
. d`
.
,,
,,
,
d
on C l`
a 2, t`u nguyn ly irichl mo rng
suy ra t`n tai
u`ong tr`
.
. t
,
`

m trn t nht k + 1 cung c


nht mt
a n
o na
o dang
im thuc
. d
. C, m`
.
, ,,
,
, ,
0
t

`
`
. c . Nu tu d
hoa
ay ta ke d
o ca
im chung n`
uong knh cua C,th` n
,
i di`u kin
t nht k + 1 dy cung d
cho, d
ay tr
ai vo
a
i`u n`
`u
. trong d
,
,
ng d
ng.
b`
ai. Vy
ai to
an l`
ad
i`u kha
inh
u
. cua b`
. d
,
,
,
,
. 10.10. Mt
tp
ho. p M l`
a ho. p cua mt
s doan
tha
ng na
`m trong
.
.
.
.
,
,
,
,
,
a hai dim bt k`y cua M
khoang [0, 1]. Bit ra
`ng khoang c
ach giu
,
,
,
,ng doan
khng ba
`ng 0,1. Chung minh ra
`ng tng d. d`
ai cua nhu
. tao
.
,,
nn M khng vuo. t qu
a

a) 0, 55;
b) 0, 5.

116

,,
Chuong 10. Nguyn ly irichl cho din
. tch

,
,
, , ,
,
`ng phan chu
`,i giai. a) Chu
,ng minh ba
,ng. Gia su, tng S cua d
Lo
.
,
,
,
,
n hon 0,55. Ch
ng tao
d`
ai nhung d
ung ta xet tp
oan
. tha
. nn M lo
. N
,
,
, , `,
,

`
i 0,1. T`u,
ng c
ng cua M vo
nhn
ach cng
mi d
tha
uo. c tu M ba
oan
. d
.
.
,
,
d
cua b`
ai to
an suy ra M v`
a N khng c
od
ai
i`u kin
im chung. Ngo`
.
,
,
,
,
,

n hon 0,55. D thy hai tp


ng S nn lo
ra tng d
ai cua N ba
. d`
. ho. p
,
`m trong d
M v`
a N na
V` 2.S > 2.0, 55 = 1, 1, t`u nguyn
oan
. [0;1,1].
,
to
,i v ly.
ly irichl suy ra M v`
a N c
od
ay dn
im chung, d
i`u n`
, ,,
,
,
,
,
b) Gia su d
ai cua tt ca c
acd
. d`
oan
. trong
 M lon hon 0,5. Ch
ung
1
1 2
9
ta chia d
anh 10 ph`n 0,
,
,
,...,
, 1 . Ky
oan
. [0,1] th`
10  10 10 
10
,
i i+1
`m trong doan
hiu
a ph`n cua M na
,
, i = 0, 1, . . . , 9
. Mi l`
.
10 10
,
,
`ng c
ng tao ra Mi . Ba
v`
a Si l`
a tng d
ai c
ac d
a
ch tinh
. d`
oan
. 
. tha

,
2
9
1
,
tin thch ho. p ch
ung ta chuyn moi
,
,...,
,1
oan
. d
.
10 10
10


,
,
1
,i 0,
,i i = 2, 3, 4 . . . , 9. V`
to
. Ky hiu
Mi0 l`
a anh cua Mi vo
.
10
,
S = S1 + S2 + + S10 > 0, 5 = 5.0, 1, t`u nguyn ly irichl
,
,,
,
0
0 o dim
mo rng
suy ra c
o t nht 6 tp

.
. ho. p trong M0 , M1 , . . . , M9 c
1
trong 0,
chung. i`u n`
ay c
o ngha l`
a mt
ao d
l`
a kt
o
. s n`
10
, ,
,
,
,
, , ,
qua cua 6 d
ac nhau x1 , x2 , . . . , x6 cua M tr`u d
ng
im kh
i tuong u
,
k
k
k
,
2
6
,
1
trong
nhung s dang
, , . . . , , o dy k i l`
a mt
ao d
o
.
. s n`
10 10
10
`ng t nht c
0, 1, 2, . . . , 9, i = 1, 2, . . . 6. D d`
ang thy ra
o hai trong c
ac
,

s k1 , k2 , . . . , k6 l`
a lin tip (
ap dung
nguyn ly irichl). V du. nhu
.
k1
k2
1
= x2 , nn x2 x1 = (k2 k1 ) =
,
k2 = k1 + 1. V` x1
10
10
10
d
d
ay d
i`u n`
n
n v ly.

10.3. B`
ai tp
.

117

` tp
10.3. Bai
.
,
. t
. 10.11. Cho n l`a mt
on A l`
a mt
. s tu. nhin bt k`y, c`
. , b` ma
,
. t pha
ng, m`
t`n tai
trong ma
a s( A) > n. Khi d
o
im trong
. n+1 d
,
kh
ac nhau ( xi , yi ) (i = 1, 2, . . . , n + 1) cua A, m`
a hiu
a
. xi x j v`
,
yi y j (i, j = 1, 2, . . . , n + 1) l`
a nhung s nguyn.
,
. t A trong ma
. t pha
ng v`
. 10.12. Nu vo,i mt
b` ma
a mt
.
. s nguyn
,
,,
duong k thoa m
an S( A) < k, th` t`n tai
cho
. tin sao
. mt
. phep tinh
,
,
,
,

,i
anh cua A qua phep tinh
ay chua nhiu nht k 1 d
im vo
. tin n`
toa
. d. nguyn.
,
,ng qua gc toa
. t
. 10.13. Cho A l`a tp
ho. p l`i, d
i xu
. trong ma
.
. d
,
,
,

a t nht 2n + 1
ng, n l`
A chu
pha
a s tu. nhin v`
a S( A) > 4n. Khi d
o
,
,
i toa
d
ac nhau vo
im trong kh
. nguyn.
. d
,
,ng qua gc toa
. n, d
. 10.14. Nu A l`a tp
d
a
i xu
. v`
. ho. p l`i, bi. cha
,
,.
,
`
a t nht 307 d
ng 615, th` A chu
c
o din
a khoang c
ach
im m`
. tch ba
,
,
,
`
n hon hoa
. p trong ch
. c ba
ng 1.
giua moi
ung lo
. ca
,
,
,
,
,
. n nhung dim cua ma
. t pha
ng.
. 10.15. Cho F l`a tp
. ho. p l`,i, bi. cha
,
,
,
,
Ch
ung ta goi
cua tp
ho. p F l`
a khoang c
ach nho nht
. huu hiu
. d
.
.
,
,
,,
,
,,
,
,ng
ng song song o hai pha cua tp
giua hai d
u`ong tha
. ho. p F. Chu
,
,
`ng nu trong mt
,i d
. n vo
minh ra
o
. huu hiu
. tp
. ho,. p l`i, bi. cha
. d c
,
,
,
,
,
,
a mt
n hon
chu
m`
a tng d. huu hiu
ung lo
. s tp
. ho. p con
. cua ch
,
,
,
,

ng, m`
t t nht k + 1 tp
kd, th` t`n tai
a n
o ca
oi
u`ong tha
. d
. ho. p con n
trn.

118

,,
Chuong 10. Nguyn ly irichl cho din
. tch

, ,
CHUONG

11

, ,

TOAN HOC
. T HO
.P

11.1. V du.
,
. 11.1. (` thi Toan Olympic Quc t,1972) Cho tp
. ho. p g`m 10 s
,
,
, s. Chu,ng minh ra
c
c
o hai chu
`ng tp
o t nht hai tp
. ho. p do
. ho. p con
,
,,
,
ng ph`n tu trong ch
khng giao nhau, m`
a tng nhu
ung ba
`ng nhau.
,
,
,
`,i giai. Nu c
Lo
o hai tp
giao nhau m`
a tng trong ch
ung
. ho. p con
, ,
,,
,
`
ng nhau th` ch

ba
ung ta c
o th bo nhung ph`n tu chung d
i. Khi d
o
,
,, ,

c`
on lai
a tng c
ac ph`n tu cua ch
ung vn
. hai tp
. khng giao nhau v`
`
ng nhau.
ba
,
,
cua
Ch
ung ta tnh c
o bao nhiu tp
mt
. ho. p con khng rng
. tp
.
,
,,
,`,
,,
,
,
,
,
`

ho. p c
o muoi phn tu. S luo. ng nhung tp
. ho. p con ch chua 1 ph`n
,,
,
,
,
,
,
1 . S luong nh
,a 2 ph`n tu,
. c l`
tu c
o 10 hoa
a C10
ung tp
.
. ho. p con chu
,
,
,
2 . S lu,o,ng nh
3 ,. . . Suy ra
,a 3 ph`n tu, l`
l`
a C10
a C10
ung tp
ho. p con chu
.
.
,
,
,
,
tng s luo. ng c
ac tp
a
. ho. p con l`
1
2
3
10
C10
+ C10
+ C10
+ + C10
= (1 + 1)10 = 1023
,
,
,n ho,n 99. Vy
cho khng lo
i`u kin
ai ra l`
a 10 s d
a
. tng cua
. b`
,,
,
,
tp
c
ac s trong mi
a 99.10 = 990, nhu vy
. ho. p con khng vuo. t qu
.
,
,,
,
s luo. ng nhung tng kh
ac nhau nhi`u nht l`
a 990. Theo nguyn ly
,
,
,
`m 10 s se c
irichl trong s 1023 tp
ho. p con cua tp
o t
.
. ho. p g
,
,
,,
`ng nhau.
nht hai tp
a tng c
ac ph`n tu trong ch
ung phai ba
. m`

, ,
,,
Chuong 11. To
an hoc
. t ho. p

120

b`
`, 1 dn 15, mi
nh s tu
. 11.2. Cho 15 b`ai toan trac nghim,
da
ai
.
,
,
, ,
ng hoa
ch c
o hai kha n
ang tra l`oi : du
. c sai. C
o 1600 th sinh tham gia
, ,
,
,
`
ng 2 b`
thi, nhung khng c
o ai tra l`oi du
ai lin nhau. Chung minh ra
`ng
, ,
,
c
o t nht hai th sinh tra l`oi to`
an b. 15 b`
ai ht
. nhu nhau.
,
,
,,
th sinh tu,o,ng u
`,i giai. B`
,ng vo
,i mt
Lo
ai l`
am cua mi
ay 15 ph`n tu,
. d
,
ph`n tu,, hoa
. c l`
. c l`
ng) hoa
mi
a (d
a S (sai). Theo gia thit khng
u
,,
,,
c
o d
ay n`
ao c
o 2 ph`n tu k` nhau l`
a , , nn s ti d
ac ph`n tu
a c
SS trong d
ay l`
a 8.
0 k 8 th` s c
,i mi
ng k ph`n
Ta nhn
ac d
ay c
od
u
. thy vo
,,
,,
,
,
,,
,
15k +1
16k
mt
tu l`
a Ck
= Ck
(boi v` mi
ay nhu th tuong d
uong
. d
,
,
,
,i c
. t k tm b`a giua 15 k quyn s
vo
ach d
ach: c
o tt ca 16 k vi. tr
a
,
, ,
,
. t b`a, k ca hai d
).
d
ach do
d
a
`u cua ch`ng s
Vy
a s c
ac d
ay c
o tnh cht nu trn l`
a
. l`
8

Ck16k = 1 + 15 + 91 + 286 + 495 + 462 + 210 + 36 + 1

k =0

= 1597 < 1600


,
,
, ,
`ng phai c
,ng to ra
i`u n`
ay chu
o t nht 2 th sinh c
o c
ac cu tra l`oi
,
ht
. nhu nhau.
,
,,
. 11.3. Trong mt
h`nh 9 canh
d`u c
o mt
dnh duo. c t m`u tra
ng
.
.
.
,
,,
,
c`
on c
ac dnh kh
ac duo. c t den. Chung minh ra
`ng t`n tai
hai tam gi
ac
,
, . ,

`
phn bit
an da
ng (din
ng nhau) m`
a c
ac dnh cua mi tam
. to`
. tch ba
,,
`
gi
ac duo. c t c`
ung mt
. mu.
,
,
,
,,
,,
`,i giai. T`u, 9 d
Lo
ung c
o5d
nh d
uo. c t m`u th` t nht c
nh d
uo. c t
, ,,
ng. N
ng tao
l`
c`
ung mt
a m`u tra
am m`u tra
anh
o
. m`u. Gia su d
. th`
,
5.4.3
`ng
ng. y ra
C35 =
= 10 tam giac phn bit
ung mt
. c`
. m`u tra
2.3

11.1. V du.

121

,
2k
(k = 0, 1, . . . , 8) xung quanh tm O cua
9 ,
,
,
, ,,
h`nh 9 canh
d
ac d
`u khng anh huong d
n tp
nh cua h`nh 9
.
. M c
tam gi
canh
n`
ay. V` th sau khi quay mi
ac trong 10 tam gi
ac trn
.
2k
,,
mt
oc
(0 k 8) xung quanh O ta duo. c 10.9 = 90 tam giac
. g
9
,
,
,
,
c
od
ung khng th phn bit
nh trong tp
. ho. p M. Tt ca ch
. v` s
,
2.8.7
,
`ng C5 =
tam gi
ac kh
ac nhau c
od
=
nh trong tp
. ho. p M ba
3
2.3
78 < 90.
,
,,
ng 1 v`
Nhu vy
ac tra
a 2 sau mt
ai
uo. c hai tam gi
. ta t`m d
. v`
,
,
i c`
phep quay tr`
ung vo
ung mt
ac . Luu y l`
a hai tam gi
ac 1
. tam gi
,
,
v`
a 2 l`
a phn bit.
ac nhu nhau khng th c
o
. Tht
. th, hai tam gi
,,
,
d
ac.
uo. c sau khi quay t`u mt
. tam gi
tam gi
,i tam gi
V` mi
ac 1 v`
a 2 se tr`
ung vo
ac sau khi quay,
,
,
,,
`
`
ng mt
nn tu tam gi
ac 1 ba
o th nhn
uo. c tam
. phep quay ta c
. d
,
ng han
gi
ac 2 (cha
ac phep quay 1 , 2 ). Vy
a 2
. , c
. 1 v`
l`
a hai tam gi
ac phai t`m.
phep quay mt
oc
. g

,
, ,
. 11.4. Trn ma. t phang co n h`nh. Gia su, Si1 ...ik , l`a din
. tch ,ph`n
,
,
giao cua c
ac h`nh thu i1 , . . . , ik , c`
on S l`
a din
. t pha
ng bi.
. t,ch ph`n ma
, ,,
,
,
,

. Ky hiu
phu boi c
ac h`nh do
Mk l`
a tng tt ca c
ac Si1 ...ik , tuc l`
a tng
.
,
,
,
,
,
cho. Chung
din
ac h`nh c
o th c
o l`
a giao cua k h`nh da
. tch cua tt ca c
minh ra
`ng
a) S = M1 M2 + M3 . . . (1)n+1 Mn
,
b) S M1 M2 + M3 . . . (1)n+1 Mn v
oi m cha
n v`
a
,
,
S M1 M2 + M3 . . . (1)n+1 Mn v
oi m le.
,
,, ,
,,
n a s c
`,i giai. a) Ky hiu
Lo
ach chon
. Ck l`
. k ph`n tu t`u n ph`n tu, ta
,c Niuto,n ( x + y)n = nk=0 Ckn x k ynk . Ky hiu
c
o nhi. thu
a din
. Dm l`
.

, ,
,,
Chuong 11. To
an hoc
. t ho. p
,
,
, ,,
ng bi. phu boi d
. t pha
ng d
. t pha
ng m h`nh. Ph`n ma

tch ph`n ma
u
o
,
,
, ,,
,

`
.
gm nhung manh, mi manh bi. phu boi m h`nh x
ac d
ao d
inh
o
. n`
,
,,
,
,,
m

`
Khi tnh Mk din
. , tch mi manh nhu ,vy,
. se duo. c tnh Ck ln, boi v`
,
,
,
m

t`u m h`nh c
o th tao
o
. duo. c Ck giao cua k h`nh. Do d
122

Mk = Ckk Dk + Ckk+1 Dk+1 + + Ckn Dn


Suy ra M1 M2 + M3 + (1)n+1 = C11 D1 + (C12 C22 ) D2 +
+ (C1n C2n + C3n ) Dn = D1 + D2 + + Dn
,,
m = (1 +
,c Niuto,n C1m C2m + 3m (1)m Cm
boi v` do nhi. thu
m ) + 1 = (1 1) m + 1 = 1
C1m C2m + C3m (1)m Cm
,
`ng S = D1 + D2 + + Dn .
Cui c`
ung ch
ung ta luu y ra
,
b) T`u ph`n a) ta c
o
S ( M1 M2 + + (1)m+1 Mm ) =

= (1)m+2 Mm+1 + (1)m+3 Mm+2 + + (1)n+1 Mn


n

((1)m+2 Cmi +1 + + (1)n+1 Cni ) Di

i =1
,
,
,,
,ng minh
ta ch c`n phai chu
(Ta qui uo
c nu k > i th` Cki = 0. Do d
o
`ng:
ra
m + n + 1 C i 0 vo
i
i
i
,i i n.
Cm
n
+1 Cm+2 + Cm+3 . . . (1)
,
, ,
,c ( x + y)i = ( x + y)i1 ( x + y) suy ra d
,c
ng thu
ng thu
T`u d
a
a
1
i 1
Cji = Cji
1 + Cj .

Do d
o
i
i
i
m + n +1 i
Cm
Cn =
+1 Cm+2 + Cm+3 . . . (1)
i 1
i 1
i 1
i 1
= Cm
+ Cm
+1 Cm+2 + Cm+3 . . .
i 1
+(1)m+n+1 Cni11 + (1)m+n+1 Cni1 = Cm
Cni1
,
,
,
i

1
`ng Cn = 0 vo
i i n.
Cui c`
ung ch c`n luu y ra

11.1. V du.

123

,
o din
. 11.5. Mt
ai a
`ng 1 c
o 5 ming v
a, m`
a din
. c
. , tch ba
. tch cua
,,
,
,
ming v
mi
a khng nho hon 0,5. Chung minh ra
`ng lun t`m duo. c hai
,
, ,
ming v
a c
o din
ung khng nho hon 0,2.
. tch ph`n chung cua ch
,
, ,
,
`ng M, din
`,i giai. Gia su, din
o ba
Lo
ai a
ac
. tch c
. tch ph`n giao cua c
,
,
`
i1 , . . . , ik ba
ng Si1 ...ik , c`
ming v
a thu
on Mk = Si1 ...ik . T`u b`
ai trn suy
,
,,
ng
ra M M1 + M2 M3 + M4 M5 0, boi v` M S. C
ac bt d
a
, ,,
,
,
,c tu,o,ng tu., c
o, m`
thu
o th d
ai a
a c`
on cho
uo. c vit khng ch cho ca c
,
,
,
i c
o vo
t`ung ming v
a. Nu ta coi ming v
a S1 nhu mt
ai a
ac ming
. c
,,
v
a S12 , S13 , S14 , S15 th` ta d
uo. c S1 S1i + S1ij S1ijk + S12345
,
,
,,
,c nhu, vy
ng thu
0. Cng
c
ac bt d
a, ta d
a
uo. c
.
. cho tt ca 5 ming v
,,
M1 2M2 + 3M3 4M4 + 5M5 0 (hang
tu Si1 ...ik c
o trong c
ac
.
,
,
,
,

c cho c
ng thu
bt d
ac ming v
a i1 . . . ik , nn trong tng tt ca c
ac
a
,
,
,
,

c n
c 3( M M1 +
ng thu
ng thu
bt d
o c
o h. s k). Cng
c
ac bt d
a
a
.
M2 M3 + M4 M5 ) 0 v`
a M1 2M2 + 3M3 4M4 + 5M5 0,
,
,,
ng
bt d
ta d
v`
ao do
uo. c 3M 2M1 + M2 M4 + 2M5 0. Cng
a
.
,
,,
,
,c M4 2M5 0, (d
ac Si1 i2 i3 i4
thu
o trong tt ca c
uo. c suy ra t`u S12345 c
,
,
,
c l`
,c l`
tu
a M4 5M5 2M5 ), ta d
a
uo. c 3M 2M1 + M2 0, tu
,
,,
,
,
,

M2 2M1 3M 5 3 = 2. Boi v` t`u 5 ming v


a c
o th tao
uo. c
. d
,
,

. p, nn din
. p d
khng nho
10 ca
ac ca
o
. tch giao cua mt
. trong s c
, M2
hon
0, 2.
10
, ,,
,
,
,
dim cua
. 11.6. (` thi Toan v dich
CHLB uc, 1979) Gia su mi
.
,
,,
,
,,
ma
. t pha
ng duo. c t ba
`ng 1 trong n m`u (n l`
a s tu. nhin cho truoc).
,
,,
,
,
, nht
Chung minh ra
`ng t`n tai
oi c
ac dnh duo. c t c`
ung
. mt
. h`nh chu
. v
mt
. m`u.

,
,
`,i giai. Ta thit lp
. t pha
ng mt
Lo
ac vung
. ` c
. trong ,ma
. h. toa. d
,
,
i d
i 1 i n + 1 v`
g
oc tu`y y v`
a ch
u y to
a
im nguyn (i, j) vo

, ,
,,
Chuong 11. To
an hoc
. t ho. p
,
,

, tu., n + 1 dim trn mi


1 j nn+1 + 1. Vic
. t m`u k theo ,thu
d
c
d`
ong c
o nn+1 c
ach kh
ac nhau (mi
o n c
ach t). Do d
o
im c
o
,,
n
+
1

t nht l`
a 2 d`
ong trong s n
+ 1 d`ong duo. c t mu ging nhau
,
,
c t o,, d

, nht, d
(ngha l`
a hai d`
ong ging nhau v` m`u sa
im thu
im
,
,
,
, ,,
,
, hai,. . . ,d
, n + 1 cha
ng han
thu
tnh t`u tr
ai qua phai). Gia su
im thu
.
,
,

,i mi
hai d`
ong y l`
a d`
ong nhung d
o toa. d
a m; ngha l`
a vo
im c
. k v`
,
,,
`u. Boi v`
i {1, 2, . . . , n} c
ac d
a (i, m) c
o c`
ung mt
im (i, k ) v`
. m
,
,
,
ch c
o n m`u nn trong s n + 1 d
ong c
ac d
o tung
im trn d`
im c
,
,
,

`
t c
ng han,
l`
d
o t nht hai d
o c`
ung mu, cha
d
a2d
. k a
im c
o
im
.
,
,
,i c
( a, k) v`a (b, k). Khi do h`nh chu nht
ac d
nh ( a, k ), (b, k ), (b, m)
. vo
,
v`
a ( a, m) c
o4d
ung m`u.
nh c`
,
,
,
. 11.7. Cho 1000 dim trn ma. t phang M1 , M2 , . . . , M1000 . Chung
,,
, ,,
on b
an knh 1 n`
ao ta d`u t`m duo. c mt
minh ra
`ng trn bt cu du`ong tr`
.
,
,
,
,
`, S dn c
dim S sao cho tng khoang c
ach tu
ac dim M1 , M2 , . . . , M1000
, ,
khng nho hon 1000.
124

,
,
,
, ,
,,
`,i giai. Gia su, S1 v`
on
a S2 l`
a hai d
Lo
im tu`y y d
i tm cua d
u`ong tr`
T b
an knh 1. Ta c
o S1 S2 = 2, vy
. S1 M1 + S2 M2 S1 S2 = 2;
S1 M2 + S2 M2 2;. . . ; S1 M1000 + S2 M1000 2. V` th (S1 M1 +
S2 M1 ) + (S1 M2 + S2 M1 ) + + (S1 M1000 + S2 M1000 ) = (S1 M1 +
S1 M2 + + S1 M1000 ) + (S2 M1 + S2 M2 + + S2 M1000 ) 2000.
,
,
c cha
n thoa m
t nht mt
Do do
trong hai d
a S2 cha
an
im S1 v`
.
,
,
,
,

`
b`
ai ra l`
a tng khoang c
ach tu n
od
ac d
n c
im M1 , M2 , . . . , M1000
, ,
khng nho hon 1000.
,
,ng
. 11.8. Cho 37 dim phn bit
khng gian c
o toa. do. l`
a nhu
. trong
,
,
,
s nguyn sao cho khng c
o ba dim n`
ao tha
ng h`
ang. Chung minh
,
,
,
,
ra
`ng ta c
o th chon
a toa. d. giao dim c
ac trung tuyn cua
im m`
. 3d
tam gi
ac tao
anh d`u l`
a s nguyn.
. th`

11.1. V du.

125

,
,
,
,
d
`,i giai. Vo
,i mi
Lo
o toa. d
a nhung s nguyn (d
im
im x; y; z c
. l`
, , ,
,
, ,
y;
z l`
nguyn) ta cho tuong u
ng c
ac s x;
a nhung s du cua phep
chia c
ac s x; y; z cho 3. V` x nhn
a 3 gi
a tri. nn t nht
. khng qu
,
,
13 trong s 37 d
o c`
ung gi
a tri. (nu khng, th th` s c
ac d
im c
im
,
,
,n ho,n 12.3=36). Tu,o,ng tu.,, c
khng lo
o khng t hon 5 trong 13 dim
,
c
d
o gi
a tri. y nhu nhau.
o
,
,
,
Th th` giao dim c
ac trung tuyn cua tam gi
ac c
od
nh ( x1 , y1 , z1 ),
( x2 , y2 , z2 ) v`a ( x3 , y3 , z3 ) co toa. d.
x1 + x2 + x3
y1 + y2 + y3
z1 + z2 + z3
; y0 =
; z0 =
3
3
3
Nu x1 = x2 = x3 v`
a y1 = y2 = y3 th` c
ac s x0 v`
a y0 l`
a nguyn c`
on
,
,

s z0 ch nguyn khi v`
a ch khi z1 + z2 + z3 0( mod 3). Trong
,
,
,`,
,
`ng nhau v`
truong ho. p b`
ai ra ta chon
ra 5 d
a tt ca s x ba
a tt
im m`
.
,
,
,
,
,
`ng nhau. Nu trong c
ca s y ba
ac d
ay ta t`m d
a
im n`
uo. c ba d
im m`
,
,

c
ac s z nhn
ac gi
a tri. 0; 1; 2 th` voi c
ac d
o
im d
o ta c
. c
x0 =

z1 + z2 + z3 z1 + z2 + z3 0 + 1 + 2 0( mod 3)
,
,
,
,i 5 d
chon
Nu khng c
o nhung d
ay th` s z vo
im n`
im d
a
. se nhn
.
,
,,

khng qu
a 2 gi
a tri,
do
d
o
ta
t`
m
d
u
o
c
3
d
i
m
m`
a
gi
a
tr
i
z
nh
n
c`
u
ng

.
.
.
,
,

mt
gi
a
tr
i
m`
a
s
z
u
ng
v
o
i
n
o
l`
a
s
nguyn.
0
.
.
,
,
,
. 11.9. Trn ma. t phang cho 7 du`o,ng thang trong do khng co hai
,
,,
,
,,
du`ong tha
ng n`
ao song song. Chung minh ra
`ng ta t`m duo. c hai trong
,
, ,
,,
,a ch
7 du`ong tha
ng n
oi trn m`
a g
oc giu
ung nho hon 260 .

,
,
,
,,
`,i giai. Ta h
ng d
cho sao cho ch
Lo
ay tinh
ung
u`ong tha
a
. , tin tt ca 7 d
,
,
,`,
,

c`
ung d
oc
i qua mt
im c d
inh
uo. c 7 d
uong thang chia g
. O. Ta se d
. d
,
,
`
`

y
d
nh
O
th`
a
nh
14
ph
n.
V`

th
m
t
trong
c
a
c
g
o
c
t
ao
th`
a
nh
nh
o

.
.
0
,
5
, 360

hon
= 250 < 260 (nu tt ca cac goc d`u ba`ng nhau th` mi
14
7

, ,
,,
Chuong 11. To
an hoc
. t ho. p
,
,
,,
t nhau
,n ho,n 260 ). Nhu,ng g
ng ca
g
oc khng lo
oc giua hai d
u`ong tha
,
,
,
,
`ng g
ng ban d
ba
oc giua hai d
u`ong tha
`u.
126

nguyn bt
. 11.10. Cho A l`a mt
ac s nguyn v`
a x l`
a mt
. tp
. c
. s
,
,
k`y. Tp
a tinh
ac d
inh
. tin cua A, x
.
. ho. p x + A = { x + a| a A} goi
. l`
,,
,,
,
`
`
bo i x. Hay n
oi c
ach kh
ac tp
ac phn tu cua A cng
. ho. p x + A gm c
.
,
,
,
`

ng mt
thm voi c`
ung mt
s x. Ch
ung ta n
oi ra
. tp
. , ho. p nhung s
, .
nguyn l`
a k-nho nht, nu t`n tai
a ch
ung
. tin cua A, m`
. k tp
. tinh
,
,
,
,

tung d
o dim chung. V du:
a k-nho
i khng c
. moi
. tp
. s huu han
. l`
,
,,
,i moi
nht vo
ung c
o tp
a k-nho nht
. s nguyn duong k. C
. v han
. l`
,
,
,ng minh
nhu d
ay k + 1, (k + 1)2 , . . . , (k + 1)n , . . . Nhung ch
ung ta chu
kt lun
. sau:
, ,
,
,
,ng s nguyn khng th biu
tp
Cho n l`
a s tu. nhin. Khi do
ho. p nhu
.
,
,
,
nhu, ho.,p cua n tp
din
. ho. p (n + 1)-nho nht.
,
, ,,
,,
,
,
`,i giai. Ch
Lo
ung ta gia su nguo. c lai:
t`n tai
nhung tp
ho. p (n + 1).
.
.
,
s nguyn thuc
nho nht A1 , A2 , . . . , An sao cho mi
t trong
. m
,
, .
,
,
,i
nhung tp
ay. Theo d
inh
. ngha cua tp
. ho. p n`
. (n + 1)-nho nht vo
moi
. i = 1, 2, . . . , n t`n tai
. n + 1 s nguyn ai,1 , ai,2 , . . . , ai,n+1 sao cho
,
,
i 1 r n + 1, 1 s n + 1 v`
vo
a r 6= s nhung phep tinh
. tin
,
,
air + Ai v`
a ais + Ai khng c
o d
N l`
a tp
ho. p
im chung. Ky hiu
.
.
,
,
p thu
, tu., ( j1 , j2 , . . . , jn ) o, d
tt ca n-b. sa
y j1 , j2 , . . . , jn d
c
. lp
. nhn
.
,
,

i mi n-b. a = ( j1 , j2 , . . . , jn ) t`u N
c
ac gi
a tri. 1, 2, . . . , n, n + 1. Vo
,,
, , ,
,

i mt
cho tuong ung vo
s nguyn x a = a1j1 + a2j2 + + anjn . Boi
.
,
,
,
, ,
gia thit tp
v` ch
ung ta da
a ho. p cua c
ac
. ho. p nhung s nguyn l`
,

trong d
tp
c
ao do
ay
. ho. p A1 , A2 , . . . , An , mi s x a thu
. mt
. tp
. n`
,
,, ,
,
n

`
tp
ai ra s phn tu cua N l`
a (n + 1) (d
a tnh t
y l`
. ho. p trn. Ngo`
,,
,
,
ho. p).T`u nguyn ly irichle mo rng
ch
ung ta c
o t nht mt
.
. trong
( n + 1) n
,
,
,
a nhung s dang
i t nht
c
ac tp
x a vo
. A1 , A2 , . . . , An chu
.
n

11.2. B`
ai tp
.

127

,
b. a t`u N.

,
,
Khng mt tnh tng qu
at ch
ung ta gia thit A1 c
o tnh cht
,
,
,,
trn. Ch
ung ta xet tt ca c
ac ph`n tu cua N m`
a s x thuc
. A1 .
n
,,
( n + 1)
Ch
ung g`m t nht
ph`n tu. Nu = ( j1 , j2 , . . . , jn ) l`
a mt
.
n
,,
,
,
,
,
ng vo
i (n 1)-b.
ph`n tu trong ch
ung, th` ch
ung ta cho tuong u
,
0
p = ( j2 , j3 , . . . , jn ). V` 1 j n + 1 vo
i moi
sa
. = 2, 3,, . . . , n,
, ,
,
0
i c
theo tnh cht t ho. p vo
o nhi`u nht (n + 1)n1 kha n
ang.
n
,
( n + 1)
,
,c
ng thu
T`u bt d
> (n + 1)n1 suy ra t nht co hai n-b.
a
n
= ( j1 , j2 , . . . , jn ), = (k1 , k2 , . . . , k n ) ho`
an to`
an kh
ac nhau, m`
a
, , ,
,
i c`
x A1 , x A1 , tuong ung vo
ung mt
. (n 1)-b.
.
,
,
t nga
n gon
Ch
ung ta c
o th t
om ta
ac
. nhu sau: t`n tai
. hai b. kh
nhau = ( j1 , j2 , . . . , jn ) v`
a = (k1 , k2 , . . . , k n ) thuc
a x
. N, m`
,
A1 , x A1 , j1 6= k1 v`
a j2 = k2 , j3 = k3 , . . . , jn = k n . Nhung, x =
,
a1j1 + a2j2 + + anjn x = a1k1 + a2k2 + + ankn nhung v` j = k
= 2, 3, . . . n ch
,i mi
. c l`
vo
ung ta c
o x x = a1j1 a1k1 hoa
a
s x = a1k1 + x = a1j1 + x thuc
a1k1 + x = a1j1 + x . Khi d
ao
o
. v`
,,
,
tp
a a1j1 + A1 . Suy ra tp
tinh
tin A1 x
ac d
boi
inh
.
.
. ho. p a1k1 + A1 v`
.
,
,
t
,i gia thit khi ba
tr
c
ac s a1k1 v`
a a1j1 c
od
ai vo
im chung, d
i`u d
o
,
ng minh.
d
`u chu

` tp
11.2. Bai
.
. t ba d
. 11.11. a) Trong h`nh vung din
ac din
a
a gi
. tch 6 d
. tch
,
,
,
`

ng minh ra
ng trong s d
lun t`m d
3. Chu
ac m`
a din
o
uoc hai d
a gi
.
,
, , .
`
tch phn chung cua ch
ung khng nho hon 1.
,ng
. t 9 d
b) Trong h`nh vung din
ac din
a
a gi
. tch 5 d
. tch 1. Chu
,,
`ng trong s d
lun t`m d
minh ra
ac m`
a din
o
uo. c hai da gi
. tch ph`n

128

, ,
,,
Chuong 11. To
an hoc
. t ho. p

,
, , 1
chung cua ch
ung khng nho hon .
9
,
,
,
,,
. 11.12. Sau dim duo. c sap xp trn ma. t phang sao cho ba dim
,
,
bt k`y l`
ad
ac m`
a c
ac canh
c
od
ai kh
ac nhau.
nh cua mt
. d`
. tam, gi
.
,
,
`

ng minh ra
ng canh
Chu
ac tam gi
ac d
`ng
. , nho nht cua mt
. trong c
,
,
n nht cua mt
th`oi l`
a canh
lo
ac kh
ac.
.
. tam gi
,
,
. 11.13. Cho P1 , P2 , . . . , P7 l`a by dim trong khng gian, trong do
,
,
d
ng. T m`u mi
khng c
o bn d
ao d
im n`
`ng pha
oan
. Pi Pj (i < j)
,
,
,
`ng c
ng minh ra
i mt
. c d
vo
o hai tam
o hoa
en. Chu
. trong hai m`u d
,
c khng c
gi
ac d
o chung canh.
o n s a
.
,
`ng nhau,
. 11.14. Co hai da d`u duo.,c chia th`anh 1998 h`nh quat
. ba
d
`ng mt
v`
a trn mi
ach bt k`y (ba
a t mt
. c
. m`u) 200 h`nh quat.
.
,,
,

. t ch`ng ln nhau v`

C
ac d
a quay mt
d
a
theo
nh
u
ng
g
oc
a d
uo. c d
a

.
0
, 360
`ng t`n tai
,ng minh ra
l`
a bi
o khng
. cua 1998 . Chu
. t nht 94 vi. tr c
,, ,
qu
a 20 h`nh quat
ung nhau.
uo. c son tr`
. d
,
,
,
,
. 11.15. Trn ma. t phang cho n du`o,ng thang t`ung di khng song
,
,
,,
`ng g
,ng minh ra
,i nhau. Chu
ng n`

song vo
oc giua hai d
ao do
u`ong tha
1800
,
,
n hon
khng lo
.
trong s d
o
n

, ,
CHUONG

12

` TP H`INH HOC KHAC

MT
.
. S BAI
.

12.1. V du.

,
,
, nht
. 12.1. Trong h`nh chu
. t 6 dim. Chung minh ra
`ng trong
. , 3x4 da
,
,
,
,
,
a ch
lun t`m d
s do
o khoang c
ach giu
ung khng l
on
uo. c hai dim c

,
hon 5.
,
`,i giai. Chia h`nh chu, nht
Lo
am
. ra l`
,
5 h`nh nhu h`nh 12.1. Trong mt
.
se c
trong s c
ac h`nh d
o t nht
o
,
,
,
2 d
a khoang c
ach giua hai
im, v`

,
,n ho,n 5.
se khng lo
d
im d
o
H`nh 12.1:
,
,
,
. 12.2. Trn ma. t phang co 25 dim, bit ra`ng trong 3 dim bt k`y
,
, ,
,
lun c
trong s d
o 2 dim c
ach nhau nho hon 1. Chung minh ra
`ng
o
,
,
,
cho.
t`n tai
on b
an knh 1 chua khng t hon 13 d
im da
. h`nh tr`

,
, ,
,
,
,
`,i giai. Gia su, A l`
cho. Nu tt ca c
a mt
ac d
on lai
Lo
im d
a
im c`
. d
.,
`m trong h`nh tr`
na
on S1 tm A b
an kinh 1 th` ta khng c`n phai
, ,
,
`m
,ng minh g` thm. Gia su, c
cho na
chu
o mt
im B trong s d
a
. d
,`,
,
c l`
ngo`
ai d
on S1 tu
a AB > 1. Xet h`nh tr`
on S2 tm B b
an knh
uong tr`
,
,
C l`
cho bt k`y lun
1. Trong s c
ac d
ad
im A, B, C trong d
o
im d
a
,
, ,
,
,
, ,
khng th l`
c
o hai d
ach nhau nho hon 1, hon nua d
a2d
im c
o
im

130

,,
Chuong 12. Mt
ai tp
ac
. s b`
. h`nh hoc
. kh

,
,
,a tt ca c
c
cho,
A v`
a B. Do d
ac h`nh tr`
on S1 v`
a S2 chu
ac d
o
im d
a
,
,
,
,
c l`
a khng t hon 13 d
chu
cho.
tu
a mt
on d
o
im d
a
. trong hai h`nh tr`

,
,
tha
ng c
od
ai
. 12.3. Bn trong du`o,ng tr`on ban knh n da. t 4n doan
. d`
.
,
, ,
,
,
,
`
ng song song hoa
. c vung
1. Chung minh ra
ng c
o th ke mt
. du`ong tha
,
,
,,
, ,`,

cho.
g
oc v
oi d
ng l cho truoc v`
a ca
t t nht 2 doan
ng da
uong tha
. tha
,
,
, ,
,,
`,i giai. Gia su, l1 l`
,i l. Ky hiu
ng bt k`y vung g
oc vo
Lo
ad
u`ong tha
,
, .
,
,
,`,

d
ai c
ac h`nh chiu cua doan
th
a
ng
th
u
i
ln
c
a
c
d
u
o
ng
th
a
ng l
. d`

.
,
,
,,
, , ,

`ng
ng ba
v`
a l1 l`
a ai v`
a bi tuong ung. Boi v` d
ai cua mi d
. d`
oan
. tha
( a1 + + a4n ) + (b1 + + b4n ) 4n.
1, nn ai + bi 1. Do d
o
,
, ,,
Khng mt tnh tng qu
at gia su ( a1 + + a4n ) (b1 + + b4n ).
,
,
,,
ng d
a1 + + a4n 2n. Tt ca c
cho d
Khi d
ac d
tha
o
oan
a
`u d
uo. c
.
,
,,
`m trong
ng c
chiu xung d
od
ai 2n, boi v` ch
ung d`u na
oan
. d`
. tha
,
,
,,
,
ng
d
on b
an knh n. Nu nhu c
ac h`nh chiu cua c
ac d
tha
u`ong tr`
oan
.
,
,
,
,,
ng l khng c
ng
cho ln d
d
od
o bt d
a
u`ong tha
im chung, th` se c
a
,
,
,
,c a1 + + a4n < 2n. Do d
trn l phai c
thu
o mt
dim bi. c
ac d
o
im
.
,
,
,,
ng d
cho chiu ln d
. u`ong
cua t nht hai trong s c
ac d
tha
oan
a
o
.
,
,
t t nht hai d
,i l tai
ng d
se ca
cho.
vung g
oc vo
im d
o
oan
a
. d
. tha

,
,
. 12.4. Trn doan
tha
ng c
od
ai 1 ta t mt
s doan
tha
ng sao
. d`
.
.
.
,
,
,,
,
,a hai dim d
cho khoang c
ach giu
`ng 0,1. Chung
uo. c t bt k`y khng ba
,
,
,,
,
,
minh ra
`ng tng d. d`
ai c
ac doan
ng duo. c t khng l
on hon 0,5.
. tha
,
,
,
`,i giai. Chia d
ng ra l`
ng c
. t
Lo
am 10 d
od
ai 0,1, d
oan
oan
. d`
a
. tha
. tha
,
,
ng nhu vy.
ch
ung theo mt
ct v`
a chiu ch
ung xung mt
oan
. d
. tha
.
, . .
,
,,
,
,
,
`

Boi v` khoang c
ach giua hai d
i
m
d
u
o
c
t
b
t
k`
y
khng
b
a
ng
0,1,

.
,
,
,
,
,,
ng canh
nn c
ac d
i
m
d
u
o
c
t
c
ua
c
a
c
d
o
an
th
a
nhau khng th c`
ung

.
.
.
,
,
,
khng c
chiu xung 1 d
od
ao c
o th l`
a h`nh chiu
im. Do d
o
im n`

12.1. V du.

131

,
,
,
,
,
,,
,
ng. Suy ra tng
cua c
ac d
tha
im d
uo. c t cua nhi`u hon 5 d
oan
.
,
,
,,
,n ho,n
ng d
d
ai c
ac h`nh chiu cua c
ac d
. d`
oan
uo. c t khng lo
. tha
5.0, 1 = 0, 5.

,
,
,,
ng l na
`m trong ma
. t
. 12.5. Chung minh ra`ng nu mt
du`ong tha
.
,
,
,
,
,
pha
ng cua tam gi
ac ABC v`
a khng di qua dnh n`
ao cua tam gi
ac do
,

cho.
th` n
o ca
t khng qu
a hai canh
cua tam gi
ac da
.
,
,
,
,,
`,i giai. Ky hiu
. t pha
ng do l chia ma
. t pha
ng
Lo
v`
a l`
a hai nua ma
.
,
,
,,
d
`m trong mt
. t
cua tam gi
ac ABC. Mi
a C na
nua ma
nh A, B v`
.
,
,,
ng trn. Theo nguyn ly irichl t nht mt
. t
pha
trong hai nua ma
,
,
, . ,
,
,
a hai d
ng trn, cha
ng han
ac ABC,
pha
nh cua tam gi
. nhu , chu
,
,
,
,
,`,

ng han
t d
ng
ng l khng ca
d
tha
cha
a B. Khi do
oan
uong tha
.
. nhu A v`
,
t mt
AB, ngha l`
a n
o khng ca
ac ABC.
. trong ba canh
. cua tam gi

,
,
,, , `
,ng dim trong ma
ng mt
. 12.6. Nhu
. t pha
ng duo. c son ba
. trong ba
,
,
,
,
m`u. Chung minh ra
`ng lun t`m duo. c hai dim c`
ung m`u c
ach nhau
ng ba
du
`ng 1.
,
,
, ,
,,
`ng c
`,i giai. Gia su, hai d
Lo
ach nhau 1 d
ac
im bt k`y c
`u d
uo. c t ba
,
`
`

m`u kh
ac nhau. Xet tam gi
ac d

u
ABC
c
o
c
anh
b
a
ng
1.
T
t
c
a
c
ac

.
,
,
, ,, ,
,,
`ng c
d
od
ac m`u kh
ac nhau. Gia su d
nh cua n
uo. c t ba
im A1 d
i
,
,
,,
,
,
,
,
ng vo
i A qua d
ng BC. Boi v` A1 B = A1 C = 1, nn d
xu
u`ong tha
im
,
,,
`u
,c l`
,i m`u cua B v`
A1 c
o m`u kh
ac vo
a C, tu
a n
od
ung mt
uo. c t c`
. m

,
,
,
,
`ng nu AA1 = 3,
i d
thu. c cht d
ch ra ra
vo
ac lp
im A. C
o
a
. lun
. d
,
,
,
`m
tt ca c
th` c
ac d
a A1 t c`
ung mt
ac d
im A v`
o
im na
. m`u. Do d
,`,
trn d
on tm A b
an knh 3 c
o c`
ung mt
m`u. R
o r`
ang trn
uong tr`
,
,.
,`,
,,
,
lun t`m d
d
on d
o khoang c
ach giua ch
ung
uong tr`
o
uo. c hai d
im c
,
,
,
,
,
vy
`ng 1. Ta duo. c mu thun,
ba
ung m`u
uo. c hai d
im c`
. lun t`m d
,
,
`ng 1.
c
o khoang c
ach giua ch
ung ba

132

,,
Chuong 12. Mt
ai tp
ac
. s b`
. h`nh hoc
. kh

,
,
,
. 12.7. Cho 11 dim khac nhau trong h`nh c`u th tch V. Chung
,
,
, ,
minh ra
`ng qua tm cua h`nh c`u c
o th du. ng hai ma
. t pha
ng sao cho
,
V
,
a ph`n trong
ch
ung ca
t h`nh c`u th`
anh mt
oi th tch , m`
. "ming" v
6,
,
,
,
cho.
cua n
o khng chua trong ph`n trong bt cu mt
ao da
. dim n`
,
,
`ng mt
`,i giai. Chia h`nh c`u ra hai b
. t pha
ng d
Lo
an c`u ba
ma
i qua
.
,
,
,
,
`
a trong ph`n
cho. Mt
tm v`
a hai d
ac d
b
an cu chu
im t`u c
im d
a
,
, .
,,
,
trong nhi`u nht l`
a4d
ac d
on lai.
Chia nua h`nh c`u
im t`u c
im c`
.
,
,
ma
`ng hai ma
. t pha
ng, m`
. t pha
ng d
ba
a mi
a
i qua tm h`nh c`u v`
,
,
,,
,
`
on lai.
anh 3
hai d
im trong 4 dim c`
., Nhu vy
. nua h`nh cu chia, th`
,
,

a mt
"ming" khng chu
ao bn trong. t nht th tch cua
im n`
. d
,
,
1
,n ho,n th tch cua b
mt
an h`nh c`u.
. ming lo
3
,
. 12.8. Cho khi da din
o 9 dnh A1 , A2 , . . . , A9 .Ky hiu
. l`i P1 c
.
,,
,,
n
,ng da din
P2 , P3 , .., P9 l`
a nhu
d
u
o
c
t
ao
th`
a
nh
b
o
i
c
a
c
ph
e
p
t
inh
ti

.
. .
.

,
, , ,
,
tuong ung theo c
ac vecto A1 A2 , . . . , A1 A9 . Chung minh ra
`ng t nht
,
2 trong s 9 da din
o t nht mt
. trn c
. dim chung.
,
,
,
,
`,i giai. Xet h`nh d
Lo
a anh cua P1 qua phep vi. tu. tm A1
a din
. P l`
`ng 9 d
`m trong P. Tht
,ng minh ra
v`
a h. s 2. Ta se chu
a din
`u na
. d
.
,
,
,

ng
a c
ac d
P. Ch
ung ta se chu
vy,
, A , . . . , A9 l`
nh cua d
a din
. cho A
,1 2
, .,
,
`
`
ng minh d
ng han
m trong P. chu
ch
minh, cha
ung
iu d
o
. nhu P2 na
,
,

i phep tinh
ta ch
u y to
tin A1 A2 chuyn c
ac d
im A1 , A2 , . . . , A9
.
,
,
,
,,
,
0

0
0
i c
a trung d
to
ac d
y Ai l`
im cua d
oan
im A2 , A2 , A3 , . . . , A9 , o d
.

A2 A i ).
,
,
,
`m trong P ba
`ng
Tng th tch cua c
ac d
a din
. P1 , P2 , . . . , P9, na
,
,
,
,,
9V, o d
a th tch cua d
on th tch cua d
y V l`
a din
a din
. P1 , c`
. P
,
`ng 8V. Suy ra t nht c
ba
o hai d
od
a din
im chung (nguyn ly
. c
,
irichl cho th tch.

12.1. V du.

133

,
,
. 12.9. Trong h`nh c`u du`o,ng knh 3 da. t mt
s h`nh c`u m`
a tng
.
,
,
,,
,ng h`nh c`u n`
du`ong knh cua ch
ung ba
`ng 25 (nhu
ay c
o th giao
,
,
,
,
nhau). Chung minh ra
`ng v
oi moi
. t pha
ng t`n tai
. t pha
ng
. ma
. mt
. ma
,
song song v
oi n
o v`
a ca
t t nht 9 h`nh c`u con.
,
,
,,
`,i giai. Ch
ng
Lo
ung ta xet h`nh chiu c
ac h`nh c`u ln d
u`ong tha
,
,i ma
,n chiu v`
. t pha
ng d
cho. H`nh c`u lo
vung g
oc vo
ao mt
a
. do, an
.
,
`

ng c
ng
od
ai 3, c`
on c
ac h`nh cu con chiu v`
ao c
ac d
tha
d`
oan
. tha
,
,
, .
, ,,
, `
ng n`
. t pha
ao nhu d b`
ai ra,
nho c
o tng 25. Gia su khng c
o mt
. ma
,
,
,
,
t 8

. t pha
ng d
. t pha
ng song song voi ma
cho ch ca
ngha l`
a moi
a
. , ma
,
,
,
d
ng d
mi
h`nh c`u nho. khi d
tha
ai 3 ch thuc
o
im trong d
oan
. d`
,
,.
, .
,
`

nhiu nht l`
a8d
oan
. thang h`nh chiu cua nhung h`nh cu nho. Suy
,
,
,, ,
ra tng cua ch
ung khng qu
a 24. Nhn
uo. c su. v ly.
. d

,,
,
uc L
. 12.10. Trong h`nh vung canh
1 don vi. c
o mt
.
. du`ong gp kh
, ,
,
,
,
`
`
khng tu. ca
v
oi d. d`
ai l
on hon 1000. Chung minh ra
ng tn tai
. mt
.
,
,`,
,`,
,
,

hon
duong tha
ng m song song v
oi canh
h`nh vung v`
a ca
t duong L tai
.
.
,
500 dim.
,
, ,
,
,,
t thu
`,i giai. Gia su, li l`
, i cua d
Lo
ad
ai ma
uc, ai v`
a bi
. d`
u`ong gp kh
,
,

l`
ad
ai h`nh chiu cua n
o ln c
ac canh
cua h`nh vung. Khi d
. d`
o
.
li ai + bi . Suy ra
1000 = l1 + + ln ( a1 + + an ) + (b1 + + bn )
,c l`
. c a1 + + an 500 hoa
. c l`
Tu
a hoa
a b1 + + bn 500. Nu
,
,
,
t ln mt
tng d
ai h`nh chiu cua c
ac ma
ai 1 khng nho
. d`
. d`
. canh
. d
,
,
,
ng phai c
hon 500, th` theo nguyn ly irichl cho d
ai d
tha
o
. d`
oan
.
,
,
,
,
t gp kh
c l`
uc, tu
a
d
ac ma
im chung cho hon 500 h`nh chiu cua c
, , ,
,`,
,
,
t d
se ca
d
oc ke t`u d
uc tai
uong vung g
im chung d
o
u`ong gp kh
. t
,
nht 500 d
im.

134

` tp
12.2. Bai
.

,,
Chuong 12. Mt
ai tp
ac
. s b`
. h`nh hoc
. kh

,
,
,
. t c`u v`
a c
o din
. 12.11. Cho A l`a tp
.
. ho. p nhung dim trn mt
. ma
,,
,
,
,
,
`
`
t

n hon nua din

tch lo
t
ch
m
a
t
c
u.
Ch
u
ng
minh
r
a
ng
A
ch
u
a
t
nh
.
.
,
,
,
,`,
. p d
mt
a n
o l`
a2d
im m`
aah u duong knh cua qua c`u.
. ca
,
,,
,
,,
. 12.12. Ta goi
a h`nh tao
ac d
u`ong cheo cua
. h`nh chu thp
. l`
. boi c
`ng trong h`nh tr`
,ng minh ra
h`nh vung canh
1. Chu
on b
an knh 100
.
,
,
,,
,
,
t nhau.

. t d
ch c
o th d
ac chu thp
a
uo. c mt
. s huu han
. c
. khng ca
,
,
,
,
,
. 12.13. Trn ma. t phang cho dim O. Hoi co th da. t trn ma. t phang
,
,
,
t
5 h`nh tr`
on khng phu d
at t`u O ca
im O sao cho moi
. tia xut ph
,
,,
,
,
t o d
khng t hon hai h`nh tr`
on d
o ngha l`
a
uo. c hay khng ? (ca
y c
,
c
od
im chung).
,
,
,
,
. 12.14. Nhung dim trong ma. t phang duo.,c so,n ba`ng mt
. trong
,
,
,
,
`
ng minh ra
ng lun t`m d
hai m`u. Chu
ung m`u c
ach
uo. c hai d
im c`
`
ng 1.
ng ba
nhau d
u

, ,
CHUONG

13

`
MT
.
. S THI V ICH

,
,
,,
Nhung b`
ai to
an thi v d
ac nuo
c l`
a c
ac b`
ai to
an d
ich
in h`nh cho
. c
, ,
,
`i hoi hoc
vic
tr thng minh, s
ang tao
d
od
giai. N
o
. , vn
. dung
.
.
. sinh
,
,,
,
,
, ,

m d
c v`
o.
phai na
ac kin thuc co ban vung cha
a hiu thu d
uo. c c
a
, ,
,
giai c
ac b`
ai to
an thi v d
cao tnh phn tch
ich
. phai vn
. dung
.
,
,
,

v`
a tng ho. p c
ac kin thuc to
an hoc.
ai to
an thi v d
ich
.
. Rt nhiu b`
, ,,
,,
,,
`
c
ac nuoc c
o mt
nguyn ly irichl, c
ac chuong
. phn giai su ,dung
.
,,

truoc ta d
ai. thun
a ga. p mt
oj c mun tham
. , s b`
. tin
. cho ban
. d
,
,
khao d
ay ti chep lai
a thng k ra d
ac d
`y d
u v` chu d
` n`
y c
`
. v`
,
,
,,
ach [4] v`
a mt
s tuyn
thi v d
ac nuoc chu yu trong cun s
ich
. c
.
,
,
,
,
tp
ac. Nhung v` t`
ai liu
tham khao c
o han,
ph`n suu t`m cua ti
. kh
.
.
,
,
c cha
n chu,a d
cha
ac ban
a cho y kin.
`y d
u mong c
oc
. d
. b sung v`
,
,
,
,
b`
c v`
Mi
ai trong chuong n`
ay k`em theo tn nuo
a n
am k`y thi v d
ich
.
,,

nuoc d
o
.

,
`, 52 s nguyn bt k`y lun
. 13.1. (Anh, 1966) Chung minh ra`ng tu
,
,
,
,,
c
o th chon
a tng hoa
. c hiu
ung chia ht cho
. duo. c ra hai s m`
. cua ch
100.
,
,
,
,,
`,i giai. Tt ca c
Lo
ac s du trong phep chia cho 100, d
anh
uo. c chia th`
,
,
t`ung nh
om nhu sau: {0}, {1; 99}, {2; 98}, . . . , {49, 51}, {50}. V` c
o
,
,
tt ca 51 nh
om, m`
a lai
o 52 s, nn theo nguyn ly irichl giua
. c

136

,,
Chuong 13. Mt
` thi v d
ich
.
. s d

,
,
,
ch
ung phai c
o hai s m`
a c
ac s du trong phep chia cho 100 roi v`
ao
, ,
`

ng
mt
om. Hai s n`
ay l`
a hai s cn t`m v` nu s du cua ch
ung ba
. nh
,
, ,

nhau th` hiu


ch
ung chia ht cho 100, c`
on nu s du cua ch
ung
. cua
,
,

kh
ac nhau th` tng cua ch
ung chia ht cho 100.

,
,
,
`, tp
. 13.2. (Anh, 1970) Chung minh ra`ng tu
. ho. p tu`y y g`m n s tu.
,,
,
,

nhin lun t
ach ra duo. c mt
tp
ho. p con (kh
ac rng)
chua c
ac s m`
a
.
.
,
,

tng cua ch
ung chia ht cho n.
,
, , ,
,
`,i giai. Gia su, vo
,a c
i mt
chu
Lo
tp ho. p n`
ao d
ac s a1 , a2 , . . . , an m`
a
o
, . .
,
,
ng d
khng c
khng thoa m
an kha
ai to
an. Khi d
o s n`
ao
inh
o
. cua b`
trong c
ac s
S1 = a 1 , S2 = a 1 + a 2 , . . . , S n = a 1 + a 2 + + a n
,
chia ht cho n. V` s c
ac s du kh
ac khng trong phep chia cho
,,
n l`
a n 1, nn theo nguyn ly irichl t`m d
a Sj
uo. c hai s Si v`
,
(1 i < j n) co c`
ung s du. Suy ra hiu
a
. S,j Si = ai1 + +
, j
,,
,

i gia su n
ng
oi trn v`
a kha
ay mu thun vo
chia ht cho n, d
i`u n`
,
,,
,
ng minh.
d
ai to
an d
inh
uo. c chu
. cua b`
, ,,
, ,
. 13.3. (Anh, 1976) Gia su trong tp
u han
. ho. p hu
. X chon
. ra 50 tp
.
,,
,
,
,
,
tp
`n

ho. p con A1 , . . . , A50 sao cho mi


h
o
p
con
ch
u
a
ho
n
m
t
n
u
a
ph
.
.,
.
,, ,
,
,
,
,
tu cua tp
`ng c
o th t`m duo. c tp
. X. Chung minh ra
. ho. p con B X
,,
,,
,
,
chua khng nhi`u hon 5 ph`n tu v`
a c
o t nht mt
. ph`n tu chung cho
,
c
ac tp
. ho. p A1 , A2 , . . . A50 .

,
, ,
,, ,
,
tp
`ng n. Mi
`,i giai. Gia su, s c
Lo
ac ph`n tu cua tp
. X ba
. ho. p con
,
n
,,
,
,a khng t ho,n
d
ph`n tu, c
o ngha l`
a
uo. c chon
. A1 , . . . , A50 chu
2
,
,
,, ,
n
,,
tng s c
ac ph`n tu cua tt ca c
ac tp
ay vuo. t qu
a 50. = 25.n.
. n`
2
,, ,
`
Theo nguyn ly irichl t`n tai
m
t
ph
n
t
u
c
ua
X
thu
c
.
.
. khng t

137
,,
,
,
,ng minh vo
,i gi
chon.
hon 26 tp
a tri. bt k`y
a
. con d
. Tuong tu. ta chu
,
,
,
k < 50 giua c
ac tp
o th chon
. Ai1 , Ai2 , . . . , Aik c
. ra khng t hon
,,
,, ,
k
,
,a c`
+ 1 tp
ho. p chu
ung mt
ph`n tu. Ta ly ph`n tu cua mt
.
.
.
2
,
,
,
,
tp
a n
o thuc
ay se l`
a mt
. ho. p X m`
. khng t hon 26 tp
. (ph`n tu n`
.
,, ,
,,
,
,
`

trong 5 ph`n tu cua tp


h
o
p
B).
Ta
lo
ai
ra
26
t
p
m`
a
ch
u
a
ph
n
t
u
.
.
.
.
,,
,,
,
xet. Khi do
t`m d
d
on lai.
a
uo. c 1 ph`n tu thuc
. t nht 13 t`u 24 tp
. c`
.
,
,
,
`n

Ta lai
lo
ai
13
t
p
n`
a
y
ra,
khi
d
o
gi
u
a
11
t
p
c`
o
n
l
ai
t`
m
d
u
o
c
1
ph

.
.
.
.
.
.
,,
,
,
,
,
i 5 tp
tu thuc
ac tp
on
. khng t hon 6 trong s c
. ho. p. i vo
. ho. p c`
,,
,,
,
lai
ung. V`
a cui
uo. c 1 ph`n tu thuc
. t`m d
. khng t hon 3 trong s ch
,,
,
c`
ung t`n tai
tp
ung. Nhu vy
. mt
. ph`n tu thuc
. hai
. cui c`
. ta t`m
,
,, ,
,,
,
,
d
o th t hon v` mt
ai
uo. c khng nhi`u hon 5 ph`n tu cua tp
. X (c
. v`
,,
ph`n tu se tr`
ung nhau), ch
ung se tao
anh tp
ai ra mt
. th`
. B. Ngo`
. tp
.
,,
,
,
`
a t nht mt

bt k`y t`u A1 , . . . , A50 chu


trong
c
a
c
ph
n
t
u
d
o
.

.
,
. 13.4. (Ba Lan, 1979) Khi chia cac s tu. nhin a1 , a2 , . . . , an cho s
m
,
,,
,
,
, nhn
tu. nhin m n`
ao do
ac s du kh
ac nhau, d`ng th`oi n > .
. duo. c c
2
,
,

Chung minh ra
`ng v
oi moi
s
k

Z
t
n
t
ai
c
a
c
s
i,
j

{
1,
2,
.
.
.
,
n
}
.
.
(khng nht thit kh
ac nhau) sao cho s ai + a j k chia ht cho m.
,
`,i giai. Xet 2n s a1 , a2 , . . . , an , k a1 , k a2 , . . . , k an . V` 2n > m,
Lo
,
c
nn c
o 2 trong c
ac s d
o c`
ung s du trong phep chia cho m. Theo
o
,
d
ai to
an, c
ac s a1 , a2 , . . . , an , c
o s du kh
ac nhau trong
i`u kin
. b`
,
phep chia cho m, nn c
ac s k a1 , k a2 , . . . , k an c
ung c
o s du
,
,
, `
. p s c
ng nhau ch c
ca
kh
ac nhau. Do do
o s du ba
o th l`
a hai s c
o
,
,
i i, j n`
. Khi d
hiu
dang
ai v`
a k a j vo
ao d
ung ai + a j k
o
o
.
. cua ch
chia ht cho m.

,
,
. 13.5. (Ba Lan, 1977) Chung minh ra`ng v
oi moi
gi
a tri. a, b R v`
a
,
, .
,
e > 0, t`n tai
ac s k, m Z v`
a n N thoa m
an c
ac bt da
ng thuc
. c

,,
Chuong 13. Mt
` thi v d
ich
.
. s d

138

|na k| < e v`a |nb m| < e.


,
1
ca
`,i giai. Cho s nguyn N > v`
,i mi
. p x, y [0; 1] ta thay
Lo
a vo
e
,
`ng ca
,i hai
. p u, v thoa m
nu vo
th ba
an u = [ Nx ], v = [ Ny], khi d
o
, , ,
,i mt
. p duy nht (u, v) th`
. p ( x1 , y1 ), ( x2 , y2 ) tuong u
ng vo
ca
. ca


1

1
| x1 x2 | = (u + { Nx1 }) (v + { Nx2 }) =
N
N
1
1
= |{ Nx1 } { Nx2 }| <
<e
N
N
,,
,
v`
a tuong tu. ta c
o |y1 y2 | < e. V` u, v {0, . . . , N 1}, nn c
o tt
, 2
,
2
. p (u, v) kh
. p gi
ca N ca
ac nhau. Xet tp
a tri.
. ho. p N + 1 ca
,i l = 0, 1, . . . , N 2
x = {la}, y = {lb} vo
, ,, ,
i l = i v`
. p (gia su vo
Theo nguyn ly irichl c
o t nht hai ca
a
,
,
,
,
,
ng c`
. p (u, v). Do d
,
l = j, i > j) t`u tp
ay tuong u
ung mt
o
. ho. p n`
. ca
,
i ky hiu
vo
.
n = i j, k = [ia] [ ja], m = [ib] [ jb]
,
,,
,c c`n chu
,ng minh :
ng thu
ta nhn
ac bt d
a
. duo. c c

|na k| = |(ia [ia]) ( ja [ ja])| = |{ia} { ja}| < e,


|nb m| = |(ib [ib]) ( jb [ jb])| = |{ib} { jb}| < e.

,
. 13.6. (B, 1977) Trong h`nh tr`on co ban knh n N co 4n doan
,
,.
,
,
,
`
`
`

tha
ng du c
o d. d`
ai ba
ng 1. Chung minh ra
ng nu c
o mt
ng
. du`ong tha
,
,,
,,
,`,
cho truoc th` t`m duo. c mt
duong tha
ng kh
ac, hoa
. c song song, hoa
. c
, .
,
, ,`,

vung g
oc v
oi d
ng n`
ay v`
a ca
t t nht hai doan
ng n
oi trn.
uong tha
. tha
,
,
,
,
mt
`ng tng d
`,i giai. y ra
Lo
ai hai h`nh chiu cua mi
. d`
oan
. d
.
,
,
,
,`,
,`,
,
0

ng ln d

tha
u
o
ng
th
a
ng
l
v`
a
d
u
o
ng
th
a
ng
l
,
vung
g
o
c
v
o
i
n
o
,
khng

139
,
,
,i doan
be hon 1. Tht
od
ai l`
a 1 song song vo
. d`
. vy,
. nu vecto a c
.
,
,
,
,

ng n`
, c`
tha
ao d
on c
ac vecto x v`
a y l`
a c
ac h`nh chiu cua vecto a ln
o
,
,`,
,
0
ng l v`
d
a l th` a = x + y, suy ra | x | + |y| | a| = 1. Nhung
uong tha
,
,
,
,
`ng | x | v`
ng ba
tng cua
d
ai c
ac h`nh chiu cua d
a |y| do do
. d`
oan
. tha
,
,
,
d
ch
ung c
ung khng be hon 1. Dn
ai h`nh chiu cua tt
n tng d
. d`
,
,
,
,
,,
,
,i vy
ng khng be hon 4n. Bo
ng l
ca c
ac d
t`u hai d
oan
u`ong tha
. , tha
.
,
,
,
,,
,,
ng, tng d
v`
a l 0 c
o th chon
d
d
ai cua h`nh chiu
uo. c mt
u`ong tha
. d`
.
.
,
,
ng trn n
c
ac doan
tha
o khng be hon 2n (nguyn ly irichl). V` tt
.
,
,
,,
,
ng d
p xp trong h`nh tr`
ca c
ac d
on b
an knh n, nn ho. p
oan
uo. c sa
. tha
,
,
,,
,
ng bt k`y c
c
ac h`nh chiu cua ch
ung trn d
od
ai be hon
u`ong tha
. d`
,
,
,,
,,
,,
ng d
2n. Suy ra trn d
t`m d
ao
u`ong tha
uo. c chon
uo. c 1 d
im thuc
. v`
, .
,

ng (Nguyn ly irichl cho doan


h`nh chiu cua t nht hai d
a
oan
. th
,
,.
,
,
,`,
, ,`,

ng d
ng
ng). uong tha
ay vung g
oc voi d
tha
i qua d
im n`
uong tha
,
,
,,
,`,

d
ay. V` duong thang n`
ay
uo. c chon,
oan
. se cat t nht hai d
. thang n`
,
,
,
, ,`,

hoa. c vung g
oc voi, hoa. c song song voi d
o thoa
uong thang l, th` n
m
an di`u kin
ai to
an.
. b`
,
. 13.7. (Bungari (d` thi chon
a
. di
. tuy,n),1973) Cho a1 , a2 , . . . , an l`
,
,

nhung s nguyn kh
ac nhau trong khoang [100,200], m`
a ch
ung thoa
,
,a nhu
,ng s n`
m
an a1 + a2 + + an 11100. Chung minh ra
`ng giu
ay
,,
,

c
o t nht mt
a vit n
o o dang
o t nht hai chu s
. s, m`
. thp
. phn c

ging nhau.
,
,
`,i giai. Ch
Lo
ung ta lp
ach c
ac s trong khoang [100,200], m`
a
. danh s
,,
,
ch
ung vit o h. thp
o hai chu s tr`
ung nhau.: 100,
. phn t nht c
101, 110, 111, 112, 113, 114, 115, 116, 117, 118, 119, 121, 122,
131, 133, 141, 144, 151, 155, 161, 166, 171, 177, 181, 188, 191,
,
,
,
,
,
. t kh
ac tng tt ca
199, 200. Tng cua tt ca c
ac s trn l`
a 4050. Ma
,
,
c
ac s nguyn trong khoang [100,200] l`
a 15150. Nu trong nhung
cho a1 , a2 , . . . , an khng c
s d
o s n`
ao trong danh s
ach trn, th`
a

140

,,
Chuong 13. Mt
` thi v d
ich
.
. s d

a1 + a2 + + an < 15150 4050 = 11100, d


ay v ly. Ngha
i`u n`
,, ,
,,

l`
a trong c
ac s a1 , a2 , . . . , an c
o t nht mt
o t
. s vit o co s mu`oi c
,

nht hai chu s tr`


ung nhau.

,
. 13.8. (Bungari,1973) Trong mt
o 20000 cun s
ach,
. thu vin
. c
,,
,
gi
ng gi
ch
ung duo. c xp v`
ao nhu
a s
ach sao cho mi
a s
ach c
o t nht
,
,
mt
. c nhi`u nht 199 cun s
ach. Chung minh ra
`ng t nht
. quyn hoa
,
,
c
o hai gi
a s
ach c
o c`
ung s luo. ng s
ach.
,
, ,
,
`,i giai. Gia su, ngu,o.,c lai.
Lo
o n gi
a s
ach v`
a ch
ung
. Nu trong thu vin
. c
,,
,

`
nh s tu 1 d
d
o r`
ang l`
a n 199. Ch
ung ta ky hiu
uo. c da
n n, th` r
. ai
,,
,

. t ln gi
l`
a s cun s
ach d
a s
ach thu i, ch
ung ta se c
o 1 ai
uo. c d
a
,,
,,
,
`

tu d
199. khi d
ung ta nhn
o
iu kin
uo. c
. ai 6= a j voi i 6= j, ch
. d
20000 = a1 + a2 + + an 1 + 2 + + 199 = 19900 < 20000
,a c`
d
ay v ly. Suy ra c
o t nht hai gi
a s
ach, trn ch
ung chu
ung
i`u n`

s s
ach.
,
,
. 13.9. (Bungari, 1983) T`m h`nh vung co kch thuo,c be nht, d
,
c
trong h`nh vung do
o th sa
p xp 5 h`nh tr`
on b
an knh ba
`ng 1, sao
,
cho khng c
o 3 h`nh tr`
on n`
ao trong ch
ung c
o dim trong chung.

,
, ,
A
B
`,i giai. Gia su, h`nh vung ABCD
Lo
A1
B
1
,a 5 h`nh
c
o tm O v`
a canh
a, chu
.
O
t nhau v`
tr`
on khng ca
a d`u c
o
D1
C1
`ng 1, khi d
c
b
an knh ba
ac tm
o
,
C
D
`m trong h`nh vung
cua ch
ung na
`ng
A1 B1 C1 D1 c
o tm O v`
a canh
ba
H`nh 13.1:
.
,,
a 2 (o dy A1 B1 //AB ).
,
,
,
,
,,
,
ng ni t`u c
C
ac d
ac trung d
ac canh
u`ong tha
im cua c
i din
. d
. cua
th`
h`nh vung A1 B1 C1 D1 chia h`nh vung d
anh bn h`nh vung
o

141
, ,,
nho, o mt
ung t nht c
ung c
o hai trong sc
ac tm (Nguyn
. trong ch
,
,
. t khng
khoang c
ly irichl). Khi d
ach giua hai tm n`
ay mt
o
. ma
,
,
,
,
,
n hon d
. t kh
lo
ac khng be hon 2. Do
u`ong cheo h`nh vung be, ma
vy
o
. c
A1 B1
a 2
2 OA1 =
2=
2

2
2
suy ra a 2 2 + 2.

,
Cui c`
ung, nu a = 2 2 + 2 v`
a tm cua c
ac h`nh tr`
on l`
a c
ac
,
,
,
,,

`
d
ac d
nu trn d
im O, A1 , B1 , C1 , D1 th` tt ca c
iu kin
uo. c thoa
.

,
,
m
an. Nhu vy
cua h`nh vung c`n t`m l`
a 2 2 + 2.
. canh
.
,
,
,,
. 13.10. (Nam Tu, 1977) Cho truoc 20 s tu. nhin a1 < a2 < . . . <
,,
,
,a c
a20 khng vuo. t qu
a 70. Chung minh ra
`ng giu
ac hiu
. a j ak ( j > k )
,,
`

lun t`m duo. c t nht 4 hiu


ng nhau.
. ba
,
,
, ,
,
,
,
`,i giai. Gia su, kha
ng d
giua 19 s tu.
Lo
ai to
an l`
a sai. Khi do
inh
. cua b`
`ng nhau.
nhin a20 a19 , a19 a18 , . . . , a2 a1 khng c
o 4 s n`
ao ba
,
s 1, 2, 3, 4, 5, 6 c
. t khng qu
giua ch
Do d
ung mi
o ma
a 3 l`n.
o
, ,
,
n hon 6 (nu khng th` s c
phai lo
ac
Suy ra c
o mt
o
. trong 19 s d
,
,
,
,

s khng lon hon 6 se nhiu hon 18), c


o 3 trong 18 s c`
on lai
phai
.
, ,
,
,
,n ho,n 5, 3 trong 15 s c`
n ho,n 4,. . . Do d
tng cua
lo
on lai
o
. phai lo
ch
ung ( a20 a19 ) + ( a19 a18 ) + + ( a2 a1 ) 7 + 3(6 + 5 + 4 +
,

`ng a20 a1 70 1 = 69. Mu thun


3 + 2 + 1) = 70 khng th ba
,
,
,,
,
ng minh kha
ng d
chu
nhn
ai to
an.
uo. c d
a
inh
. cua b`
. d
,
,,
,
. 13.11. (Nam Tu, 1981) Tp
ac s {1, 2, . . . , 100} d
uo. c chia
. ho. p c
,,
,
,
,
l`
am 7 tp
`ng t nht o mt
ac tp
. ho. p con. Chung minh ra
. trong c
. ho. p
,,
con y t`m duo. c hoa
. c 4 s a, b, c, d sao cho a + b = c + d, hoa
. c ba s
e, f , g sao cho e + f = 2g.
,
,
,
`ng c
`,i giai. y ra
,a khng
Lo
o t nht mt
. trong 7 tp
. ho.,p con chu
,,
,,
,
,
,
t hon 15 s (trong tru`ong ho. p nguo. c lai,
ac tp
. tt ca c
. ho. p con

142

,,
Chuong 13. Mt
` thi v d
ich
.
. s d

,
,
ca
,a khng nhi`u ho,n 7.14 = 98 s). Mi
. p s a > b cua tp
chu
. ho. p
,,
, , ,
,,
,i hiu
. t tuong u
ta nhn
con n`
ay d
ng vo
uo. c d
a
o
uo. c
. a b. Khi d
. d
,
,
2 = 15 14 = 105 hiu, trong s do
khng t hon C15
o c
ac hiu
phai c
.
.
2
,
,
`ng nhau (v` c
ba
ac hiu
nhn d
a 99 gi
a tri. kh
ac nhau
uo. c khng qu
,
, ,, . , .
,c
i hai ca
. p s a > b, c > d ta c
ng thu
1, 2,. . . ,99). Gia su vo
o d
a
a + d = b + c. Nu a = d
a b = c d( a 6= c, b 6= d). Khi d
o
,
, ,
,
c kh
. c b = c, d
ng thu
. c
(hoa
ac khng th xy ra), th` c + b = 2a (hoa
a
a + d = 2b).
,
,
,
. 13.12. (Nam tu,1977) Chung minh ra`ng din
nh vung
. tch cua h`
,
,,
,
,
bt k`y na
`m trong tam gi
ac, khng l
on hon nua din
ac
. tch cua tam gi
.
do
,
`,i giai. (H`nh 13.2) Ta chu
,ng
Lo
A
,
,
,
ng d
minh kha
tng qu
at hon :
inh
.
,
M
din
anh bt
. tch cua h`nh b`nh h`
L
,
`m o, trong tam gi
M1
F
k`y KLMN na
ac
L1
E
,,
N
,
,
n hon nua din
ABC khng lo
. tch
K
,
C
B
.
cua tam gi
ac d
o
N1
K1
,
,
,,
mt
`ng mi
ng
y ra
u`ong tha
. d
,
H`nh 13.2:
t hai canh
KL v`
a MN ca
cua tam
.
, ,, ,
,
gi
ac ABC (c
o th o dnh tam gi
ac), ngha l`
a c
o 2 trong 4 giao d
im
,
,
,,
`m trn 1 canh
ng han,
ng
na
theo nguyn ly irichl. Cha
u`ong tha
.
. d
t BC l`n lu,o.,t o,, K1 v`
KL v`
a MN ca
a N1 . Trn canh
AC v`
a BC
. , AB,
,
,
,i KL
chon
ac d
a F sao cho L1 , M1 l`
a giao d
im D, E v`
im cua DE vo
. c
,
v`
a MN thoa m
an:
,
K1 L1 = KL; L1 M1 //K1 N1 d
`ng th`oi BF//BD
,,
`ng
h`nh b`nh h`
Khi d
anh KLMN v`
a K1 L1 M1 N1 c
od
o
u`ong cao ba
nhau d
on c
ac h`nh b`nh h`
anh BDEF v`
a K1 L1 M1 N1
n d
as y chung, c`

143
,,
,
,
y DE khng be hon d
y L1 M1 v`
a L1
c
od
a hai d
a
a
u`ong cao ha. t`u D v`
,
,,
`

ng nhau. Boi vy
xung BF ba
o: SKLMN = SK1 L1 M1 N1 SBDEF . Gia
. c
,,
EC = (1 x ) AC, c
su AE = x.AC, khi d
ac tam gi
ac ABC, ADE
o
,
`
i nhau nn ta c
v`
a EFC d
o:
ng dang
. vo
SBDEF = S ABC S ADE SEFC

= S ABC x2 S ABC (1 x )2 S ABC


1
= 2x (1 x )S ABC S ABC .
2
V` x (1 x )

1 ,
1
i x ly gi
vo
a tri. bt k`y, nn SKLMN S ABC .
4
2

,
,
,
. 13.13. (Nam tu, 1972) i v
oi mi
gi
a tri. n N, h
ay t`m s k l
on
,
,,
,
nht (k N ) thoa m
an tnh cht sau: Trong tp
ho. p g`m n ph`n tu
.
,
,
,
c
o th chon
ac nhau, sao cho hai tp
. ra k tp
. ho. p con kh
. ho. p con bt

k`y d`u c
o tp
ac rng.
. giao kh
,
,
,
,,
,
`,i giai. C d
Lo
a ch
inh
. ph`n tu ai cua tp
. ho. p X = { a1 , a2 , . . . , an } v`
,
,
,
,
`ng s
,a ph`n tu, a1 .S c
xet c
ac tp
chu
ac tp
. ho. p con
. ho. p nhu vy
. ba
,
,
,
`ng 2n1 . Suy ra
c
ac tp
a ba
. ho. p con cua tp
. , ho. p { a2 , . . . , an }, ngha l`
,
,
,
,,
,
. t kh
chon
d
k 2n1 . Ma
ac gia su d
con cua X. Ta
a
uo. c hon 2n1 tp
.
.
,
,
,,
,,
. p d
chia tt ca c
ac tp
con cua X th`
anh 2n1 ca
boi 1 tp
uo. c tao
.
.
. con
,
,
cua X v`
a ph`n b`
u cua n
o. Theo nguyn ly irichl c
o t nht 2 tp
.

chon
con d
t
ao
th`
a
nh
m
t
c
a
p,
suy
ra
ch
u
ng
khng
giao
nhau.
V
y
a
. .
. .
.
k = 2n 1 .

,
1
. 13.14. (My, 1983) Trn truc
o d. d`
ai (n
. s ly mt
. khoang c
n
,
n+1
,
,
,
`
`
N ). Chung minh ra
ng khoang n`
ay c
o chua nhiu hon
phn s
2
,
p
p, q Z, 1 q n.
ti gian dang
, trong d
o
.
q

,,
Chuong 13. Mt
` thi v d
ich
.
. s d

144

,
,
,
1
`ng trong mt
`,i giai. Gia thit ra
,a
c
Lo
ao d
od
ai , chu
o
. d`
. khoang n`
n
, p
,
, n+1

phn s ti gian , voi q {1; 2; . . . ; n}. Ta se


nhiu hon
2
q
,
,
,

`ng giua c
ng minh ra
ac mu s cua c
ac phn s n`
ay lun t`m
chu
,,

d
a mu s n`
ay chia ht cho mu s kia. Tht
uo. c hai mu s m`
. vy
.
,
,
,
,
,
r
+ . S

i dang

ta biu din c
ac mu s duo
2
.s
v
o
i
s
l`
a
s
l
e,
r

Z
.


,
,
`ng n + 1 ( ngha
c
ac s le kh
ac nhau giua c
ac s 1, 2, 3, . . . , n ba
2
,,
,
s d
s
l`
a t hon s c
ac mu
ang xet), suy ra lun t`m d
uo. c hai mu

c
q = 2r .s v`
a q1 = 2r1 s1 m`
a s = s1 v`
a r = r1 . Khi d
o mt
o
. mu
,
,
s kia, hay q1 = kq. Nhu vy
s chia ht cho mu
ac phn s
. giua c
m
l
,,
,i kq n. Khi d

chon
ac nhau dang
v`
a , vo
uo. c hai s kh
o
. d
.
q
kq

m
l < 1,
q
kq
n
,
,
1
`m trong khoang c
km l = 0, v` trong
v` ca hai na
od
ai . Do d
. d`
o
n
,`,
,,
,
truong ho. p nguo. c lai
. th`

m
l = |km l | 1 1 ,
q
kq
kq
kq
n
km = l v`
v`
a do d
a
o
nhau.

l
km
,,
=
, ngha l`
a hai phn s d
ung
uo. c chon
. tr`
kq
kq

,
. 13.15. (Vit
`ng t`n tai
ac s c
o
. nam, 1976) Chung, minh ra
. ,v s c
n

c
o
a trong c
ac biu din thp
dang
o
. phn cua mi s d
. 5 (n N ), m`
,
,
,

s 0 dung lin tip.


khng t hon 1976 chu
,
`ng vo
`,i giai. Ta chu
,ng minh ra
,i moi
Lo
ac s
. k N t`n tai
. v s c
,
m
k
m N thoa m
an d
i`u kin
. 5 = 1 (mod 2 ).

145
k
,
,,
ac s 50 , 51 , . . . , 52 lun t`m d
Tht
a
uo. c hai s 5 p v`
. vy,
. giua c
,
,
q
k

hiu
5 ( p > q) c
o c`
ung s du trong phep chia cho 2 . Khi d
o
. cua
p
q
q
p

q
k
p

ch
ung 5 5 = 5 (5
1) chia ht cho 2 nghia l`a s 5 q 1
,
r ( pq) 1 (r N ) d`u chia ht cho 2k .
v`
a tt ca c
ac s c
o dang

. 5
,
,
gi
i mi
Nhu vy
a tri. m = r ( p q), r N, ta c
o 5m 1
. vo
,
,
ung
(mod 2k ), t`u do 5m+k 5k (mod 10k ) ngha l`a k chu s tn
. , c`
, m+k
,
,
,
,
,
,
k
i k chu s tn
cua 5
tr`
ung vo
ung cua 5 . Gia su s k thoa m
an
. c`
,
,
,
,
k
1976
a khng nhi`u hon k 1976 chu s. Do d
giua k
2 > 10
chu
o
,
,
,
,
m
+
k
chu s tn
ung cua s 5
ch c
o k 1976 chu s kh
ac khng, c`
on
. c`
,
`ng 0.
1976 chu s c`
on lai
. (lin tip nhau) ba
,
,,
ng c
o n2 + 1( n N )
. 13.16. (Tip
kha
c, 1979) Trn mt
du`ong tha
.
.
,
,
,
,a ch
doan
ng. Chung minh ra
`ng, hoa
. c l`
a giu
ung c
o th chon
. tha
., n + 1
,,

doan
t nhau, hoa
. c l`
a t`m duo. c n + 1 doan
ng n`
ao
. di,mt
. khng ca
. tha
c
do
o dim chung.

,
,
,,
,,
`ng mt
`,i giai. Ta d
ng khi n
Lo
ng bn tr
ai cho d
oi ra
inh
u`ong tha
. huo
,
,.
,
,
`

d
ai d
ac, nu d
ut bn tr
ai cua
oan
oan
u m
. na,m o bn tr
. thang kh
,
,,
,
`

d
thang thu nht nam o bn tr
ai du m
ut bn tr
ai cua d
oan
oan
.
,.
,
,,
,,
d
`

, hai. Mi
ng thu

tha
o
an
th
a
ng
d
u
o
c
d
a
nh
s
b
a
ng
m
t
s
tu
o
ng
.
.
.
,
,, , ,
`ng c
,ng t`u, c
, nht d
c thu
ng
u
ac s 1, 2, . . . , n ba
ach sau: o buo
oan
. tha
,,
, , ,
bu,o
,c tip theo
mi
o tn
ung bn tr
ai cho tuong u
ng s 1. Sau d
o
. c`
,
,
,
ng chua d
nh s, d
ta lai
ung
oan
a
oan tn c`
. chon
. trong s nhung d
. tha
, . ,.
,
,
,
,
ng, kh
i c
. t cho n
bn tr
ai v`
ad
o s tuong u
ac vo
ac s cua nhung d
a
oan
.
,
,
,
,
,
,

i n

ng giao vo

tha
o (d
a
d
u
o
c
d
a
nh
s
).
N
u
d

n
bu
o
c
n`
a
o
d

y,
ta
ch
on
.

.
,
,,
,
, , , ,
,

d
u
o
c
d
o
an
th
a
ng,
nhu
ng
d

i
v
o
i
n
o
khng
ch
on
d
u
o
c
s
th
u
t
u
,
th`

.
. .
.
, .
,
`m bn tr
i n d
ng na
d
ay c
o ngha l`
a n
o giao vo
ai n
o v`
a c
o
i`u n`
oan
. tha
,
,
,
,
,
nhung s kh
ac nhau. Trong tru`ong ho. p n`
ay d
ut tr
ai cua d
`u m
oan
.
,
,
,, , ,
,,

ng d

u
o
c
ch
on
thu
c
v`
a
o
n
+
1
d
o
an
th
a
ng.
N
u
o
bu
tha
o
c

y,
d
o
an
.
.
.
.
.

146

,,
Chuong 13. Mt
` thi v d
ich
.
. s d

,
,,
ng cui c`
nh s th` theo nguyn ly irichl t nht
ung d
tha
uo. c d
a
,
,
,
,
ng vo
i nhi`u ho,n n d
ng, m`
ng
mt
a c
ac doan
oan
. trong n s u
. tha
. tha
, tu., thch ho.,p, khng giao nhau.
,i c
vo
d
ac s thu
o
,
. 13.17. (Rumani, 1978) Cac h`am s f , g, h : N N thoa man ba
di`u kin
. sau:
,
,
a) H`
am h(n) khng nhn
a tri. n`
ao tai
. gi
. nhi`u hon mt
. dim n N.
,
b) Tp
a tri. h`
am s g(n) l`
a N.
. ho. p gi

c) f (n) g(n) h(n) + 1, n N.


,
,
ng.
Chung minh ra
`ng d`ng nht thuc f (n) 1, n N, l`
a du
,
`,i giai. Ta chu
,ng minh d
,c g(n) h(n) (n N ). T`u,
Lo
`ng nht thu
dn f (n) g(n) h(n) + 1 1, n N.
v`
d
ad
o
i`u kin
. b) se dn
,
i bt k`y n N c
Vo
o
h(n) = g(n) + 1 f (n) g(n)
,
, ,,
`ng, d
,c
,i gi
ng thu
n N d
v` f (n) 1. Gia su ra
a tri. n`
ao d
i vo
o
a
ng, khi do
h(n) < g(n) = k. Theo d
g(n) = h(n) khng d
u
i`u kin
.
,

b) ta t`m c
ac s n1 , n2 , . . . , nk1 N, d
sao cho g(ni ) = i khi i =
,,

1, . . . , k 1. Boi vy
. mi s trong k s h(n1 ), h(n2 ), . . . , h(nk1 ), h(n)
,
theo nguyn ly irichl
thuc
ao t. p ho. p {1, 2, . . . , k 1}, do d
o
. v`
,
`

nhiu hon mt
h`
am s h(n) nhn
a tri. n`
ao d
ay tr
ai
o
i`u n`
. ,gi
. l`n, d
,
,
,
,
`
ng minh.
i diu kin
ng d
d
vo
inh
a
uo. c chu
. d
. a).Kha
,
,
,
,u Uo,c), 1979) Chu,ng minh ra
. 13.18. (My (Nu
`ng c
ac dnh cua n-gi
ac
,,
`

du c
o din
ac cho truoc,
. tch be, nht (n > 3), ni
. tip trong mt
. n-gi
,,
,
tr`
ung v
oi trung dim c
ac canh
cua n-gi
ac cho truoc.
.

,
, ,
`,i giai. (H`nh 13.3) Gia su, n-gi
Lo
ac d
o din
`u B1 , . . . , Bn c
. tch SB
ni
ac d
`u A1 , . . . , An din
. tip trong n-gi
. tch S A .

147
mt
nu ch
Khi d
ung khng tr`
ung nhau th` trn mi
A i A i +1
o
. canh
.
,
,
,
,
i i = 1, 2, . . . , n ( An+1 = A1 ) c
vo
o mt
ac d
nh Bi duo. c x
inh
.
. d
,`,
,,
,

trn canh
y. Tht
ly
.
. vy,
. trong truo, ng ho. p nguo. c lai
. theo nguyn
,
,

a hai d
ng han
irichl t nht c
o mt
cha
im B1
. canh,
.,
. A1 A2 , chu
,
,
,,
,,
(d
d
v`
a B2 d
ac dinh,
gia su A1 B2 > A1 B1 ), khi d
o
x
o
im Bn (o
.
,
, , ,
`m trn canh
A1 A2 An ) c
o th na
A2 A3 v`
a A1 An tuong u
ng (v` n >
.
,
,`,
3, d
a A2 An l`
ad
a khng l`
a canh
cua n-gi
ac
oan
uong cheo, m`
. A1 A3 v`
.
,
`

ng
A1 , . . . , An ), ngha l`
a B1 = A1 v`
a B2 = A2 . Ta chung minh ra
A1 B1 = A2 B2 = . . . = An Bn
`ng nhau, v`
Tht
ac B1 A2 B2 v`
a tam gi
ac B2 A3 B3 ba
. vy
. tam gi
0n 2
\
\
B\
1 A2 B2 = B2 A3 B3 = B1 B2 B3 = 180
n
0
\
\
A\
2 B1 B2 = 180 B1 A2 B2 A2 B2 B1

\
\
= 1800 B\
1 B2 B3 A2 B2 B1 = A2 B2 B3
,,
,ng minh tu,o,ng
v`
a B1 B2 = B2 B3 . Boi vy
A2 B2 = A3 B3 . Ta c
ung chu
.
,
,
,,
,c c`
ng thu
tu. c
ac d
on lai.
a
. ai
. luo. ng
SB = S A SB1 A2 B2 SB2 A3 B3 . . . SBn A1 B1 = S A nSB1 A2 B2
,
,
,n nht. Gia
nhn
a tri. nho nht, khi din
ac B1 A2 B2 lo
. gi
. tch tam gi
,,
,,
d
su A1 A2 = a, A1 B1 = x, khi d
o
ai
. luo. ng
1
1
\
B1 A2 .A2 B2 sin B\
1 A2 B2 = ( a x ) x. sin B1 A2 B2
2
2
1 a2
a
= ( ( x )2 ) sin B\
1 AB2
2 4
2
a
,n nht khi x = , ngha l`
nhn
a tri. lo
a A1 B1 = B1 A2 .
. gi
2
,
,
2
. 13.19. (CHLB uc, 1978) Mt
. b. g`m n con tem choi (n>2) mang
,
loai
c
ac nh
an hiu
an hiu
o n con tem. Hoi
. "1","2","3",. . . ,"n". Mi
. nh
. c
SB1 A2 B2 =

148

,,
Chuong 13. Mt
` thi v d
ich
.
. s d

,
,
,
c
o th xp tt ca c
ac con tem y th`
anh mt
ay thoa m
an c
ac di`u kin
. d
.
,
,
a mt
sau khng : v
oi moi
an
. x {1, 2, . . . , n} giu
. con tem mang nh
,

` a du
ng x
hiu
a con tem mang nh
an hiu
o vu
. "x" v`
. "x" tip theo lun c
con tem mang nh
an hiu
ac "x" ?
. kh
,
d
`m giu,a
`,i giai. Trong mi
Lo
oan
. na
B2 A3
A2
,i nh
hai con tem lin tip vo
an "n"
B3
B1
,
d
o n con tem. C
o n-1 d
`u c
oan
. nhu
A4
A1
,, ,
s tem o giua con tem
vy
o
. do d
B4
d
an hiu
a con
`u tin mang nh
. "n" v`
A5
,i c`
tem cui c`
ung vo
ung nh
an hiu
.
,
,
i hai con tem v`ua n
cng
d
vo
oi l`
a
o
.
2
(n 1)n + n, ngha l`a n con tem.
H`nh 13.3:
Vy
a con tem
`u tin v`
. con tem d
,
cui c`
ung trong d
ay d
ung mang nh
an hiu
`u phai c`
. "n".
,
,
i nh
Trong s n con tem vo
an "n-1" phai c
o (theo nguyn ly
`m trong t nht mt
irichl) t nht hai con tem c`
ung na
oan
. d
.
,
,, ,
,

trong s n-1 d
o
an
k
trn.
Nhu
ng
khi
d
o
o
gi
u
a
hai
con
tem
n`
a
y
.

,
,
vo
i tnh cht
mu thun
ch c
o nhi`u nht l`
a n-2 con tem; d
i`u d
o
,
cua d
ay.

,
,
. 13.20. (CHLB Nga 1972) Chn du`o,ng thang c`
ung c
o tnh cht l`
a
,
,
,
,
,

mi du`ong tha
ng chia h`nh vung th`
anh hai tu gi
ac c
o ty s din
. tch
,
2
,
,
,
c`
ba
`ng . Chung minh ra
`ng c
o t nht ba du`ong tha
ng trong s do
ung
3
,
di qua mt
. dim.
,
,
,
,,
t c
`,i giai. (H`nh 13.4) C
ng d
cho khng th ca
Lo
ac d
ac
u`ong tha
a
,
,
,,
canh
k` nhau cua h`nh vung ABCD, boi v` nu th khng th tao
.
.
, ,,
,,
,
,`,

ra d
u
o
c
hai
t
u
gi
a
c,
m`
a
l`
a
tam
gi
a
c
v`
a
ng
u
gi
a
c.
Gi
a
s
u
m
t
d
u
o
ng
.
.

149
,
,
t c
ng ca
ac canh
BC v`
a AD tai
ac dim M v`
a N. C
ac h`nh thang
tha
.
. c
,
,`,
`
ng nhau do d
ty s din
ABMN v`
a CDN M c
o c
ac d
uong cao ba
o
.
,
,
,`,
,
`

c l`
ng ty s c
tch cua ch
ung ba
ac d
a MN chia
uong trung b`nh, tu
,
,
,
,
2
ng ni trung d
d
ac canh
AB v`
a CD theo ty s .
oan
im cua c
. tha
.
3,
,
,
,
,,
Tng s c
ac dim chia c
ac d
u`ong trung b`nh cua h`nh vung theo ty
,
,
,,
2
,,
ng d
cho l`
s l`
a 4 (H`nh ve). Boi s d
a 9 v`
ad
u`ong tha
a
`u phai d
i
3
,
,
qua mt
s 4 d
oi trn, nn c
o mt
im n
. trong
. dim thuc
. t nht 3
,
,`,
ng.
d
uong tha
M

A
N

H`nh 13.4:

150

,,
Chuong 13. Mt
` thi v d
ich
.
. s d

, ,
CHUONG

14

,
,
`
BAI TP
. TU. GIAI

,
,,
`ng 15 chu
,a 11.000 d
. 14.1. Mt
o canh
ba
im.
. h`nh lp
. phuong c
.
`ng c
`ng 1 chu
,ng minh ra
,a t nht 6
Chu
o mt
an knh ba
., h`nh c`u b
,
,
cho.
d
im trong s nhung dim d
a
,
,
,

. 14.2. Cho F = { a1 , a2 , . . . , al } l`a tp


. ho. p huu han
. nhung s
,, ,
,,
nguyn duong v`
a x1 , x2 , . . . , xn , . . . . . . l`
a d
ay v han,
. moi
. ph`n tu cua
,
`
i mt
m trong F, ngha l`
trong a1 , a2 , . . . , al
n
o na
a tr`
ung vo
ao d
o
. s n`
,
,
,
,
`
ng minh ra
i moi
ng vo
. Chu
duong bt k`y t`n tai
mt
. s nguyn
. s
,
,
,.
,,
,
ph`n tu lin tip cua d
ay, m`
a tch cua ch
ung l`
a l
uy th`ua cua mt
. s
.
nguyn n`
ao d
o
,
,
. 14.3. Cho 5 dim P1 , P2 , P3 , P4 , P5 trong ph`n trong cua mt
. h`nh
,
,
,
,
vung c
o canh
1 don vi.
a khoang c
ach giua hai d
im
. Ky hiu
.
. dij l`
,
`ng t nht c
,ng minh ra
Pi v`
a Pj . Chu
o mt
trong s c
ac khoang c
ach
.

,
, ,
2
,
.
giua c
ac d
im nho hon
2
, ,,
,,
,
. 14.4. Trong mt
c
o canh
50 d
on vi. , ngu`oi ta ke d
u`ong
. h`nh vung
.
,
,
gp kh
uc sao cho moi
h`nh vung c
o khoang c
ach
. dim trn canh
.
,
,
, ,
,
,
,
`ng d
ng minh ra
nho hon 1 d
uc. Chu
ai cua cua
n d
u`ong gp kh
. d`
,,
,n ho,n 1248.
d
uc lo
u`ong gp kh
,i din
. 14.5. Cho A l`a mt
ac l`i vo
a chu vi P, c`
on r
a gi
. d
. tch S v`

,,
, ,
Chuong 14. B`
ai tp
. tu. giai
,
,
,
,,
,
. t
l`
a s thu. c duong. Ky hiu
a tp
ac d
im trong ma
. ,M l`
., ho. p tt ca c
,
,
,

`
i mi d
ng, sao cho vo
pha
im R cua M tn tai
im Q thuc
. mt
. d
. A
,
, ,
,
`
. c ba
ng r. H
m`
a khoang c
ach giua R v`
a Q nho hon hoa
ay t`m din
.
,
tch cua h`nh M.
152

`ng 70, ngu,`o,i ta nem ba h`nh


. 14.6. Trong h`nh vung co canh
ba
.
,
,i kch thu,o
,c 20x10, 25x15 v`
chu nht
a 30x30 v`
a ba h`nh tr`
on b
an
. vo
,
,
,
`
ng minh ra
ng trong h`nh vung c
knh 5. Chu
o th dich
., chuyn mt
.
,
h`nh tr`
on b
an knh 5 sao cho n
o khng c
o nhung d
im trong chung
,
i 5 h`nh d
cho.
vo
a
,
,,
uc A1 A2 . . . An v`
a
. 14.7. Trong ma. t phang cho mt
u`ong gp kh
. d
,
,`,
h`nh tr`
on b
an knh r, tm h`nh tr`
on chuyn d
trn d
ng
uong gp
.
,
,`,
kh
uc n`
ay. Cho L l`
ad
ai cua d
uc, c`
on F l`
a h`nh sinh
. d`
uong gp kh
,
,
,, ,
,`,
,ng
ra boi su. chuyn d
cua h`nh tr`
on trn d
uc. Chu
ng
uong gp kh
.
,
,c S( F ) 2Lr + r2 .
ng thu
minh bt d
a
,
,
,
,
,
`m
ng na
. 14.8. Tp
a ho. p mt
oan
. ho. p ,M l`
. s huu han
., nhung d
. tha
,
`ng
trong (0,1). Khoang c
ach giua mt
im trong M khng ba
. s d
,
,
,,
,
,
`ng tng d
ng minh ra

,od
ai cua
y 0 1 . Chu
". d`
oan
.
# nhung d
1
,
[
]
+
1
,,
,
ng m`
nh
tha
a ch
ung tao
a
v`
a su. d
a
. nn M, khng vuo. t qu
2
gi
a n`
ay l`
a chnh x
ac.
,
,
. 14.9. Trn du`o,ng tr`on ban knh 1 cho n dim P1 , P2 , . . . , Pn v`a tp
.
,
,,
,
,

ho. p A tao
b
o
i
m
t
s
h
u
u
h
an
cung
khng
c
a
t
nhau,
m`
a
t
ng
d

d`

.
.
.
. ai
,
,
`
ng minh ra
ng
cua ch
ung l`
a l ( A). Chu
,
2k
, (k = 1, 2, . . . , n 1), th` c
o th quay tp
a) Nu l ( A) >
.
n
,
,
,
,`,
,
,
a t nht k + 1 d
ho. p A trn d
on sao cho anh cua n
o chu
uong tr`
im
P1 , P2 , . . . , Pn .
trong dy

153
,
2k
b) Nu l ( A) <
, (k = 1, 2, . . . , n 1), th` c
o th quay tp
.
n
,
,
,`,
,
,
a nhi`u nht k 1
ho. p A trn d
on sao cho anh cua n
o chu
uong tr`
,
P1 , P2 , . . . , Pn .
d
im trong dy
,
,
,
,i din
. t c`u vo
. 14.10. Cho tp
im trn mt
. ho. p A, nhung d
. ma
. tch
,,
,
,
,
,
`
`
ng minh ra
a t nht
n hon nua din
t cu. Chu
ng A chu
lo
a
. tch cua m
, .
,
,
hai d
`u mt
u`ong knh cua h`nh c`u.
. d
,
,
. 14.11. Ky hiu
a nhung tp
a ch
ung c
o
. N1 ,, N2 , . ,. . , Nn l`
. ho. p, m`
,,
,,
,
,
,
,
,
i nguyn d
d
ac d
boi n
uo. c sau khi nhung d
nh cua mt
uo. c t
ng
. luo
.
,
. t v`
phep tinh
a mt
a k l`
a s tu. nhin sao cho 1 k
. tin, A l`
. b` ma
,
`
ng minh ra
ng
n 1. Chu
,
,
,
k
,
a) Nu S( A) > th` c
o th tinh
tin tp
ho. p A sao cho anh cua
.
.
r
,
,
,
,
,a t nht k + 1 d
n
o chu
a nhung d
im trong l`
nh cua N1 , N2 , . . . , Nn .
,
k
,
b) Nu S( A) <
th` c
o th tinh
tin tp
.
. ho. p A sao cho
r
,
,
,
,
,
,
,a nhi`u nht k 1 d
anh cua n
o chu
a nhung d
im trong l`
nh cua
N1 , N2 , . . . , Nn .
,
,,
,
. 14.12. Trong h`nh vung vo,i canh
1 don vi. du. ng d
uc
u`ong gp kh
.
,
,`,
,`,
,

sao cho moi


h`nh vung cat d
uong thang song song voi canh
uong
. d
.
,
,
,
,,
`

gp kh
uc khng qu
a mt
ai cua d
im. Chung minh rang d
. d`
u`ong
. d
, ,
gp kh
uc nho hon 2.
,
,
,
,
. t pha
ng cho 6 d
ng ni
. 14.13. Trong mt
ma
tha
im. Nhung d
oan
.
.
,
,
,,
,
`ng vo
,ng minh ra
,i
. p d
. c xanh. Chu
c
ac ca
im d
uo. c son m`u d
o hoa
,
c
ach tao
ac c
o c
ac canh
. h`nh nhu, vy
. t`n tai
. t nht hai h`nh tam gi
.
`
`
c`
ung mu. (c
o th hai tam gi
ac kh
ac mu nhau).

154

,,
, ,
Chuong 14. B`
ai tp
. tu. giai

, ,
CHUONG

15

,
,
,
`
` GO

LOI GIAI VA
.IY

,
,,
,
,
` go. i y
15.1. L`oi giai va
chuong 1

,
,
. 1.11. L`o,i giai: Nu tt ca cac ng quan d`u quen nhau th` vic
.
,
,`,
, `

xp b`
an bn nguoi nhu d
o g` kh
o kh
an. Gia c
o ng A
ra khng c
,
,
`
v`
a ng B khng quen nhau. Tu 2n 2 ng quan c`
on lai
ung nhu A
. c
,,
v`
a B c
o t nht n ngu`oi quen. V` n + n = 2n = (2n 2) + 2, th` t`n
,
,,
. t
ngu`oi xp d
tai
a D, m`
a ho. quen A c
ung nhu B. Khi d
o
a
. hai, ng C v`
,

c
o th xp A d
a giua ho. l`
aCd
i din
i din
. B v`
. D.
d
. 1.12. Go.,i y: Chia mt
h`nh vung th`
anh 48 d
oan
oan
. canh
.
. mi
.
,
,,
,
`

20m, khoang c
ach giua 2 d
a 0,6m, hai d
ai
oan
oan
u d`
. l`
. o hai d
,
5,9m. (Vy
=1000m=1km). Canh
thu
. 48.20m+47.0,6m+2.5,9m
.
,
hai chia ra l`
am 95 d
ach hai doan
a 0,52m, hai d
oan,
oan
. khoa ng c
. l`
.
`
d

u
d`
a
i
0,56m
(
v
y
95.10m+94.0,52m+20,56m=
1000m=1km).

.
,
,
,
2
Nhu vy
c
o
48.95=4560
manh c
o din
a s cy ch c
o
.
. tch 200m m`
,
4500, nn c`
on t nht 60 nhu vy
o cy n`
ao.
. khng c
,
nh du t`u
. 1.13. Go.,i y: Ta nhom cac ngan co c`
ung s s
ach v`
ad
a
,
,a s s
0,1,. . . ,9 (c
ac ng
an chu
ach t hon 10) v`
a mt
an c
o 10 cun.
. ng
, ,,
,
a c`
Gia su khng c
o ba ng
an chu
ung s s
ach, th` trong c
ac nh
om ta
,
nh du c
d
o t hon ba ng
an c
o c`
ung s s
ach (nhi`u nht l`
a 2), vy
a
.

156

,
,,
,
`,i giai v`
Chuong 15. Lo
a go. i y

,i 10 nh
,i mt
th` vo
om trn 10.2=20 ng
an s
ach cng
an 10 cun
. vo
. ng
,
d
nua khng cho ta s ng
an s
ach l`
a 25, dn
n v ly.
,
,
,
. 1.14. Go.,i y: Ta ly s cua t chia cho 10 th` duo.,c s du 0, 1, 2, 3,
,
4, 5, 6, 7, 8, 9. V` c
o 11 s m`
a 10 s du th` theo nguyn ly irichl
,
,
,
c
o hai s c
o c`
ung s du. Nhu vy
. hiu
. cua hai s chia ht cho 10
,
ngha l`
a n
o c
o c`
ung mt
ung.
. chu s tn
. c`
,
,
. 1.15. L`o,i giai: Goi
ac tram
a A, B, C, D, c
ac h`nh
. c
. chuyn tip l`
,
,`,
,
, ,`,

tr`
on voi d
a c
ac d
ac tram
l`
a v`
ung bao phu
uong knh l`
uong ni giua c
.
,
,
cua mt
trung tm ph
at s
ong. Ch
ung ta c
o 4 h`nh tr`
on nhu vy
a
. , v`
, . ,
,
,
`ng mt d
`m trong tu
ng minh ra
gi
phai chu
ac cua 4
im M bt k`y na
, ,, .
,
,
,
tram
on. Tht
`u d
uo. c phu boi t nht mt
im M
. d
. h`nh tr`
. vy,
. ,nu d
,,
`m trong h`nh tr`
. c
oc AMB phai l`
a t`
u hoa
na
on d
u`ong knh AB th` g
,
0
`ng 90 . Ta ni M vo
i c
,i
ba
ac tram
oc d
i vo
. A, B, C, D, tao
. ra bn g
,
,
,
, gi
c
ac canh
tu
ac. Tng cua bn g
oc n`
ay l`
a 3600 . Nhu vy
o
.
. t nht c
0
m l`
`ng 90 . Vy
. c c`
ung la
a ba
on
mt
oc t`
u hoa
. g
. M thuc
. mt
. h`nh tr`
,
,
,
,
gi
i mt
m`
a tai
cua tu
ac duo
oc t`
u.
. M nh`n canh
.
. g

,
,,
,
,
` go. i y
15.2. L`oi giai va
chuong 2
,,
. 2.11. Go.,i y: p dung
Phuong ph
ap b`
ai 2.2.
.
,,
. 2.12. Go.,i y: p dung
phuong ph
ap b`
ai 2.4
.

. 2.13. Go.,i y: a. t bk = a1 + a2 + + ak , k = 1, 2, . . . , 41 v`a ap


dung
b`
ai tp
.
. trn.
,
,i
. 2.14. Go.,i y: Nhu b`ai 2.8, nu M l`a s nguyn t c`
ung nhau vo
,
, ,
10, th` t`u su. chia ht cua l k = 111 . . . 11.10k cho M, suy ra s

,
,,
,
`,i giai v`
15.2. Lo
a go. i y chuong 2

157

111
. . . 11} chia ht cho M.
| {z
, s 1)
(k-l chu
,,
. 2.15. Go.,i y: D`
ung phuong ph
ap b`
ai 2.8. H
ay xet d
ay
N, NN, NNN, . . . , |NNN
{z. . . N} .
, s )
(1968 chu
,
. 2.16. L`o,i giai: Ta vit 1998 s 1997, 19971997, . . .
,

,
1997
.{z
. . 1997}
. V`
a xet c
ac s du trong phep chia mi
|

s 1997 la
. p 1998 l`n
cho 1998. R
vit
mt
o r`
ang khng mt
ao trong c
ac s d
o
a
. s d
. s n`
,
,
n, m`
vit l`
chia ht cho 1998 (v` 1998 l`
a s cha
a c
ac s d
a le), nhu
a
,
,
,n ho,n
vy,
ac s du cua phep chia d
ac khng, v` s c
ac s lo
`u kh
. c
,
,
,,
s c
ac s du (c
o 1998 s m`
a s c
ac s du l`
a 1997), nn t`m d
uo. c
,
,
,
c
hai s c
o s du nhu nhau, hiu
o dang
c`n t`m v`
a
o
. cua hai s d
.
chia ht cho 1998.

,
. 2.17. L`o,i giai: Ta xet 1997 s dang
1998, 19981998,. . . . S cui
.
,
,
c`
ung trong c
ac s n`
ay tao
anh t`u 1997 nh
om trong bn chu s 1,
. th`
. c mt
l`
9, 9, 8. Hoa
ac s n`
ay chia ht cho 1997 (vy,
a s
o
. trong c
. d
,
,,
,
,
. c t`m d
phai t`m), hoa
uoc hai s du nhu nhau trong phep chia cho
, .
4m a chia
hiu
1997. Khi d
ung c
o dang
o
. cua ch
. 19981998 . . . 1998.10 v`
,, ,

ht cho1997. V` 104m v`
a 1997 nguyn t c`
ung nhau nn nhn tu thu
nht, ngha l`
a s 19981998. . . 1998, chia ht cho 1997.
,
,
. 2.18. L`o,i giai: Trong n + 1 s m, m2 , . . . , mn+1 t`m duo.,c hai s
,
,
,
hiu
c
o s du nhu nhau trong phep chia cho n. Khi d
cua ch
ung
o
.
, ,, l
t
t
l

. c m (m 1) = a.n. V`
chia ht cho n. Gia su m m = a.n hoa
t
l

(m, n) = 1 nn (m , n) = 1, ngha l`a, m t 1 chia ht cho n. Vy


.
,
l

m 1 l`
a s phai t`m.

158

,
,,
,
`,i giai v`
Chuong 15. Lo
a go. i y

,
,,
,
,
` go. i y
15.3. L`oi giai va
chuong 3

,
,
,
. 3.11. L`o,i giai: Ta xet 104 cac luy th`ua khac nhau cua 3: 3, 32 , 33
4
, ,
luy th`u,a d

cho 104 . Mi
, . . . , 310 v`
a c
ac s du cua phep chia mi
o
,
,
s cho mt
ac khng khi chia cho 104 ; s c
ac s du kh
ac
. s du kh
,
,
,
4
4
, t`m d
khng l`
a 10 1; s c
ac s l`
a 10 . Do d
ac
o
uo. c hai luy th`ua kh
,
,
m
n
4
nhau 3 v`
a 3 c
o s du nhu nhau trong phep chia cho 10 , ngha l`
a
m
n
4
n
m

n
4
n
4
. c 3 (3
3 3 = 10 .l hoa
1) = 10 .l. V` 3 v`a 10 nguyn t
,
m

n
c`
ung nhau nn 3
1 chia ht cho 104 hoa. c 3mn 1 = 104 .k. T`u
,
,
,
,
suy ra 3mn = 104 .k + 1. Nhu vy,
d
o th.
o
. tra l`oi: c
,
. 3.12. Go.,i y: Ly lun
ai 3.10.
. nhu b`
,,
p dung
. 3.13. Go.,i y: Ch
u y u3 = 1986, a
phuong ph
ap b`
ai 3.4.
.
,
`ng vo
,ng minh ra
,i moi
. 3.14. Go.,i y: Ly lun
. nhu trong 3.4. Chu
. s
,
tu. nhin m t nht mt
ac s x1 , x2 , . . . , chia ht cho m. Sau
. trong c
. t m = 38 v`
da
a m = 43.
d
o
,,
,
,
,
,
. 3.15. Go.,i y: Tuo,ng tu. nhu 3.8 ta xet b. xp s-ph`n tu nhung s
,
, ,,
`ng t`n tai nhu,ng ch s i v`
,ng. D thy ra
du tuong u
a j sao cho
,
s

1 i < j k v`
a mi tng trn d
u chia ht cho k.

,
,,
,
,
` go. i y
15.4. L`oi giai va
chuong 4
,
1
,,
d
cho ve d
. 4.11. Go.,i y: Quanh mi
on b
an knh .
im d
a
u`ong tr`
15
,
. 4.12. Go.,i y: B`ai toan suy ra t`u b`ai 4.9.
,
, ,,
,
,
,
. 4.13. L`o,i giai: Gia su t`n tai dim X t`u cac dim da cho m`a no ni
,
,
, , ,
ng XX1
c
d
i6d
ac d
uo. c vo
im X1 , X2 , X3 , X4 , X5 , X6 . Khi d
o
oan
. tha

,
,,
,
`,i giai v`
15.4. Lo
a go. i y chuong 4

159

,,
,
,
trong hai c
ach sau d
o c
ach
v`
a XX2 d
y (thu. c ra c
uo. c du. ng theo mt
.
, ,
, ,
,
,

ba nhung l`
thu
a h. qua cua (b) khi ta d
i ch s)
,
(a) Nu X l`
ad
a A2 , th` XX1 < X1 X2 v`
a
im g`n nht d
n X1 v`
,
n
XX2 < X1 X2 , ngha l`
a trong tam gi
ac XX1 X2 c
o canh
X1 X2 l`
a lo
.
nht.
,
,
(b) Nu X1 g`n dim X nht, c`
on X g`n d
im X2 nht, th`
,n nht trong
XX1 < XX2 < X1 X2 ch
ung ta c
ung c
o canh
X1 X2 l`
a lo
.
tam gi
ac XX1 X2
,,
,
0
,ng minh ho`
Suy ra X\
an to`
an tuong tu. , ch
ung ta
1 XX2 > 60 . Chu
,
,
0
\
\
\
\
\
n hon 60 , d
c
ung c
o X2 XX3 , X3 XX4 , X4 XX5 , X5 XX6 , X6 XX1 lo
i`u
, ,,
,
,
,
`ng 3600 . Nhu vy,
n`
ay khng th duo. c v` tng c
ac g
oc n`
ay phai ba
.
,
,
,
,
,
,

i 5 d
mi d
im ch ni d
uo. c nhiu nht vo
im thi.
,
`ng nhau (c
. 4.14. L`o,i giai: Ta chia h`nh tr`on th`anh 6 h`nh quat
o
. ba
,

tai
d
on). Khi d
o qu
a
nh tai
o
. , tm h`nh tr`
. mi mt
. h`nh ,quat,
. khng c
,
,,
,
,
mt
ao (boi v` khoang c
ach giua hai d
im roi v`
im bt k`y trong mt
. d
, .
,
,

h`nh quat
khng
l
o
n
ho
n
1).
N
u
t
ai
m
i
h`
nh
qu
at
c
o
m
t
d
i
m th`

.,
.
.
.
,
,,
,
, ,
ta c
o th t`m d
a g
oc giua c
ac b
an knh vecto cua
uo. c hai d
im m`
,
,
,n ho,n 600 v`
khoang c
ch
ung khng lo
a do d
ach giua ch
ung khng
o
,
,
,
,
n hon 1. Do vy,
lo
o th chon
a5d
im.
. c
. khng qu
,
h`nh vung ba
`ng mt
,n ho,n gio
,i
. 4.15. L`o,i giai: Ta thay mi
. h`nh lo
,
,
,,
1
,,
,,
han
ach bin cua h`nh vung mt
u`ong c
u`ong
. boi mt
. d
. khoang 2 (d
,
1
,
ng d
n`
ay g`m bn d
a bn cung tr`
on c
o b
an knh ).
on vi. v`
oan
. tha
2

,
,,
mt
`

Mi
h`
nh
nhu
th
c
o
di
n
t
ch
b
a
ng
3
+
,
c`
o
n
120
h`
nh
d
a
d
uo. c

.
.
4
,

"vi`n ra" se phu mt


a 120.(3 + ) = 360 + 30.
. din
. tch khng qu
4
,
,
,
`

Ta bao vy bin cua h`nh chu nht


d
a
cho
b
a
ng
m
t
o chi`u
.
. dai c

160

,
,,
,
`,i giai v`
Chuong 15. Lo
a go. i y

,
,
,
1
`ng 44. Nhu, vy,
rng
. Din
tch cua dai ba
din
tch tng cng
.
.
.
.
.
2,
,
,
,
,
`ng 360 + 30 + 44 =
cua dai v`
a tt ca c
ac h`nh d
uo. c vi`n ra ba
,
,
,
c l`
404 + 30 < 404 + 94, 5 < 500, tu
a be hon din
. tch cua ,h`nh
,
,
, trong h`nh chu nht
chu nht
od
20.25 = 500). Do d
o
im O
. (S =
. c
, ,, ,
,,
d
khng bi. phu boi dai v`
a c
ac h`nh vung d
a
a
uo. c vi`n ra. Ngha l`
,
,
,
d
ach bien cua h`nh chu nht
a c
ach moi
im O c
. v`
. h`nh vung mt
.
,
1
1
,
,
n hon . H`nh tr`
khoang lo
on b
an knh c
o tm tai
a h`nh tr`
on
. O l`
2
2
c`n t`m.

,
,,
,
,
` go. i y
15.5. L`oi giai va
chuong 5

,
,
,i
. 5.11. L`o,i giai: Xet mt
o giao vo
. tp
. A bt k`y t`u 1978 tp.
. V` n
,,
,
t hon 50
1977 tp
on lai,
. c`
. v` vy
. t`n tai
. ph`n tu a A, thuc
. khng
,, ,
,
,
mt
tp
ay (tht
t`u 40 ph`n tu cua tp
. ho. p n`
. vy
. nu mi
.
. A thuc
.
,
,
,
`

`
khng nhiu hon 49 tp,
o khng nhiu hon 40.49 = 1960
. th` tt ca c
,,
`
ng). Vy
tp
ac A, d
ay khng d
ac tp
iu n`
u
. kh
. ph`n tu a thuc
. c
.
,
,
,

ho. p A, A1 , . . . , A50 . Ta se chung minh n


o se thuc
. tp
. bt k`y B t`u
,
1978 tp.
o hai tp
ao t`u c
ac tp
. Tht
. vy
. khng c
. n`
. A, A1 , . . . , ,A50
,,
,

ac voi a (v` hai tp


o
l`
a c
o phn tu chung kh
. bt k`y giao nhau ch c
, ,,
,,
,
`

mt
ph
n
t
u
chung).
Gi
a
s
u
a

B.
Khi
d
o
t
p
B
c
o
v
o
i
m
i
t
p
.
.
.
,,
,
`

A, A1 , . . . , A50 mt
ph
n
t
u
chung
kh
a
c
v
o
i
a,
suy
ra
t
p
B
c
o
khng
.
.
, ,,
,,
,
,
`
`n tu, a thuc
t hon 51 ph`n tu, d
i
u
n`
a
y
khng
th
d
u
o
c.
Suy
ra
ph

.
.
,
,
tt ca c
ac tp
h
o
p.
.
.
,
,
. 5.12. L`o,i giai: Mt
o c
ac ph`n du {0, 1, 2, . . . , 8}.
. s chia cho 9 th` c
,,
Theo nguyn ly Dirichl mo rng,
trong 55 s chon
o
.
. ra th` t nht c
,
,
,
mt
om 7 s khi chia cho 9 cho c`
ung ph`n du (nu nguo. c lai
. nh
. th`
,
,,
,
c
ac nh
om ch c
o 6 ph`n tu vy
a ta ly ra nhung 55 s).
. 6.9=54 m`
l`
Ch
ung ta ky hiu
ac s d
a a1 , a2 , a3 , . . . , a7 . V` ai+1 ai (mod 9),
o
. c

,
,,
,
`,i giai v`
15.6. Lo
a go. i y chuong 6

161

,
`ng ai+1
,ng minh ra
nn ai+1 ai {9, 18, . . .}. Ch
ung ta phai chu
, ,,
,
,
,
,
i mt
i moi
. Gia su nguo. c lai,
a i = 9 vo
ao d
o
. i n`
. vo
. i, ai+1 ,ai 18,
,,
`
`
d
ay ngha l`
a a7 a1 6.8 = 108. iu n`
ay khng th d
iu n`
uo. c v`
,, ,
,
,
a7 a1 < 100. Nhu vy
giua hai ph`n tu cua a1 , a2 , a3 , . . . , a7 c
o hai
.
,
,
`

ng 9.
s m`
a hiu
ung phai ba
. cua ch
,ng minh nhu, b`
. 5.13. Go.,i y: Cach chu
ai 5.10.
,
,
,
,, `
. 5.14. L`o,i giai: (Ban
y h`nh ) Ch
ung ta xy du. ng luo
i gm
. tu. ve l
,
, ,,
3
,
h`nh luc
nhung h`nh luc
ac c
o canh
. Mi
gi
ac c
o th phu boi
. gi
.
.
2
,
,,
,,
,,
,i b
on vo
an knh 3. C
o th tnh to
an d
d
uo. c s luo. ng luc
u`ong tr`
.
,
,i tam gi
cho l`
gi
ac m`
a ch
ung c
od
ac d
a 1+2+
im chung vo
`u d
a
3 + + 10 = 55. V` 111 = 55 2 + 1, nn c
o mt
ac trong
. luc
. gi
,
,
,,
,
,
a t nht 3 d
chon.
luoi trn chu
im trong s 111 d
im d
a
. Nhu vy
.
,,

d
on bao luc
gi
a
c
n`
a
y
c
o
t
nh
ch
t
d
a
nu.
u`ong tr`

.
,
,,
, , ,
. 5.15. L`o,i giai: Ky hiu
a s luo. ng ngu`oi lo
n nht
. A1 , A2 , . . . , Ak l`
,
,
,
,
hai ngu`oi n`
m`
a bt cu
ao c
ung khng quen nhau gi
an tip. T`u di`u
ngu,`o,i c`
kin
ai ra k 7. Mi
on lai
om quen gi
an tip t
. b`
. trong nh
,
,
,
,
,
,
,
nht mt
ung
. ngu`oi trong A1 , A2 , . . . , Ak (tru`ong ho. p nguo. c lai
. ch
,`,
,`,
,
`
ta c
o nhiu hon k nguoi, m`
a hai nguoi khng quen nhau gi
an tip).
,
,
Mt
o t nht 20 ngu`oi quen gi
an tip, v`
. trong s A1 , A2 , . . . , Ak c
,
,
,
,
,,
,
,
ng nhu vy,
nu khng d
ung ta se nhn
u
uo. c tng s luo. ng ngu`oi
. ch
. d
,
,
`ng tt ca
ng dinh
nhi`u nht l`
a 7.19 = 133 < 134. C`
on lai
ra
.
. kha
,
,,
,,
nhung ngu`oi quen gi
an tip qua c`
ung mt
a quen gi
an tip.
. ngu`oi l`

,
,,
,
,
` go. i y chuong 6
15.6. L`oi giai va

,
,
. 6.11. Go.,i y: y l`a tru`o,ng ho.,p ring cua b`ai 6.1 v`a b`ai 6.2.

162

,
,,
,
`,i giai v`
Chuong 15. Lo
a go. i y

1
. 6.12. Go.,i y: S da cho l`a x1 , x2 , x3 , x4 v`a yi = 1 + , i = 1, 2, 3, 4.
xi
,
,
`ng hai s n`
,ng minh ra
Ch c`n chu
ao d
an
y trong y1 , y2 , y3 , y4 thoa m

yi y j
,
,
,
,
2 3. Ph`n c`on lai
0
. tuong tu. nhu 2.6.
1 + 2yi y j
,
,, `
,ng minh d
. 6.13. L`o,i giai: Vo,i dinh
ng nu mt
uo. c ra
. ly Fecma chu
.
,
,,
2
2

s nguyn t p c
o dang
4k + 3 chia ht cho tng a + b , o d
y
.
,
,

`
a, b l`
a nhung s nguyn, th` p chia ht cho tung s a v`
a b. H
ay
,
,
2
2
2
2
2
2

d`
ung c
ac dang thuc sau 2 = 1 + 1 , k ( a + b ) = (ka) + (kb)2 ,
( a21 + b12 )( a22 + b22 ) = ( a1 a2 + b1 b2 )2 + ( a1 b2 a2 b1 )2 .
,
,
s vo
,ng mi
,i c
. 6.14. L`o,i giai: Cho tu,o,ng u
an bc
. hai cua chnh
,
,
s tu,o,ng u
,ng vo
,i c
n
o. Nhu vy
khi phn tch ra mi
an bc
hai cua
.
.
,
,
,
, ,
s le. M`
a s tt ca c
ac s le nho hon 2k l`
a k. Vy
nguyn ly
. theo
,
,
ty s
irichl c
o hai s trong k + 1 s c
o c`
ung ph`n c
an s le, do do
,
, ,
cua n
o se l`
a l
uy th`ua cua 2.
,
,
. 6.15. L`o,i giai: Nu giua cac s da cho co n s m`a khi chia cho n
,
,
,
,
ch
ung cho nhung ph`n du kh
ac nhau. Tng cua ch
ung se chia ht
,
,
,
,
,
,
p dung
cho n, v` n l`
a s le. Trong tru`ong ho. p nguo. c lai,
nguyn ly
. a
.
irichl.

,
,,
,
,
` go. i y chuong 7
15.7. L`oi giai va

,
, , ,
`ng moi
. 7.11. L`o,i giai: Ch
ung ta thy ra
n
. s nguyn t thu. c su. lo
,
,
,
n hon
. c 6n + 5. V` ba s nguyn t lo
hon 3 d
o dang
6n + 1 hoa
`u c
.
,
3 lp
anh mt
nn theo nguyn ly irichl phai c
o t
. th`
. cp s cng,
.
,
c l`
chia ht cho 6. Goi
nht hai s c`
ung dang,
tu
a hiu
a
o
.
. hai, s d
. d l`
,
. c l`
. c l`
cng sai cua cp s cng,
th` hiu
a d, hoa
a
.
. cua hai s y hoa

,
,,
,
`,i giai v`
15.7. Lo
a go. i y chuong 7

163

,
,
. c l`
. c 2d|6. Ch
a d|6 hoa
u y cng sai d l`
a hiu
2d. Nhu th hoa
. cua hai
n. Nhu, th v` d|3 v`
,n ho,n 3, nn n
s nguyn t lo
o l`
a s cha
a d|2 suy
ra d|6.
,
10m
. 7.12. L`o,i giai: Ta lun co lim
= . Noi cach khac t`n tai
.
m+ 1998
m
10
,,
s nguyn duong m0 sao cho m m0 th`
> 91998 . Xet mt
.
1998
,
,,
,
,

s nguyn duong n bt k`y v`


a n c
o k0 chu s n = a1 a2 . . . ak0 . Tru`ong
,,
,
ho. p n`
ay ta c
ung chon
uo. c
. d
10 N > (9k0 )1998 v`
a N > m0

(15.1)
,
,
(cu. th c
o th ly N = max{[1998lg(9k0 )] + 1, m0 + 1})
,ng minh
- Ch
ung ta se chu
,i moi
ui (n) < 10 N vo
(15.2)
. i = 1, 2, . . .
`ng qui nap.
,ng minh ba
Tht
. vy,
. ta se chu
.
,
i i = 1 ta c
1) Vo
o u1 (n) = (k1 + + k0 )1998 (9k0 )1998 < 10 N
ng khi i = 1.
(do (15.1)). Vy
u
. (15.2) d
, ,,
,i i = k, ngha l`
ng vo
2. Gia su (15.2) d
a uk (n) < 10 N . Ta c
o
u
,
N

uk+1 (n) = f (uk (n)). Theo gia thit qui nap


o
. th` uk (n) < 10 . do d
,
,

c l`
uk (n) c
o khng qu
a N chu s, tu
a uk (n) c
o dang
.
,
i p N
uk (n) = a1 a2 . . . a p vo
Theo d
inh
. ngha th`
uk+1 (n) = ( a1 + a2 + + a p )1998 (9N )1998 .
,
Do N > m0 vy
. t`u (1) suy ra
10 N > (9N )1998 .
,
T`u (15.3) v`
a (15.4) suy ra
uk+1 (n) < 10 N .

(15.3)

(15.4)

,
,,
,
`,i giai v`
Chuong 15. Lo
a go. i y

164

,i i = k + 1. Theo nguyn ly qui nap


ng vo
Vy
u
. (15.2) d
. th` (15.2)
,
i moi
ng vo
d
u
. i = 1, 2, . . ..
,,
. n
- D
ay v han
ac s nguyn duong {ui (n)}, i = 1, 2, . . . bi. cha
. c
,
,
,,
N
boi 10 nn theo nguyn ly irichl phai t`n tai
. hai ch s p < q
sao cho
u p (n) = uq (n) = u p+k (n) = uq+k (n),

,i chu k`y
N
oi c
ach kh
ac d
ay ui (n), i = p, p + 1, .. l`
a d
ay tu`n ho`
an vo
u p ( n ), u p +1 ( n ), . . . , u p + q +1 ( n ).
,
. 7.13. L`o,i giai: Cho a l`a mt
a ph`n
. s tu`y y, th` { a},= a [ a] goi
. l`
, ,
,,
,
le cua s a, o d
a ph`n nguyn cua s a. Xy du. ng d
ay
y ky hiu
. [ a] l`
,
,
i nhu sau
mo
v1 = { u1 }
v2 = { u1 + u2 }
....
v n = { u1 + u2 + + u n }
,i moi
R
o r`
ang vo
o 0 vk < 1.
. k = 1, 2, . . . , n ta c
,
,
Chia [0, 1) ra l`
am n + 1 tp
. ho. p nhu sau
1
1
2
n
0 = [0,
); 1 = [
,
); . . . ; n = [
, 1)
n+1
n+1 n+1
n+1
,
,
,
Ch c
o c
ac kha n
ang sau xy ra
. c t`n tai
1. Hoa
a v k 0 n +1 :
. k m`
,c l`
- Nu vk 0 , tu
a 0 { u1 + u2 + + u k } <
1
n+1
1
[S] S < [S] +
.
n+1

1
. t
. a
n+1

Sk = u1 + u2 + + u k : 0 { S } <

(15.5)

,
,,
,
`,i giai v`
15.7. Lo
a go. i y chuong 7

165
,
,
Vy
a s nguyn g`n S nht, nn t`u (15.5) suy ra d
i`u phai
. [S] l`
,ng minh.
chu
n
,c l`
-Nu vk n tu
a
{u1 + u2 + + uk } < 1 hay l`a
n+1
n
n
{S} < 1
S [S] < 1 hay l`a
n+1
n+1
n
+ [S] S < [S] + 1.
(15.6)
n+1
,
T`u (15.6) suy ra [S] + 1 l`
a s nguyn g`n S nht v`
a kh
ac n
o mt
.
n
1
,,
luo. ng [S] + 1 ([S] +
)=
.
n+1
n+1
. c l`
2. Hoa
a c
ac s v1 , v2 , . . . , vn 6 0 n , vy
. v1 , v2 , . . . , v n
n 1
i=1 i . Theo nguyn ly irichl t`n tai
ung mt
. vk , vl , k > l, thuc
. c`
.
,

tp
h
o
p

n`
a
o
d
o
,
1

1.

j
.
.
c
i`u d
o ngha l`
a
o
j
j+1
{ u1 + u2 + + u l } <
n+1
n+1
j
j+1
{ u1 + u2 + + u k } <
n+1
n+1
. t Sk = u1 + u2 + + uk v`
a
a Sl = u1 + u2 + + ul ta c
o
j+1
j
j+1
j
Sk [ Sk ] <
[ Sk ] +
Sk < [ Sk ] +
n+1
n+1
n+1
n+1
j
j+1
j
j+1
Sl [ Sl ] <
[ Sl ] +
Sl < [ Sl ] +
n+1
n+1
n+1
n+1
1
1
[ Sk ] [ Sl ]
Sk Sl [ Sk ] [ Sl ] +
n+1
n+1
1
1
[ Sk ] [ Sl ]
u l +1 + u l +2 + + u l [ S k ] [ S l ] +
n
+
1
n
+
1
,
Nhu vy
ay con ul +1 + ul +2 + + ul c
o s nguyn g`n nht l`
a
. d
1
,
[Sk ] [Sl ] voi d. lch
a
.
. khng qu
n+1
,
,,
,
T
om lai,
o t`n tai
ay con thoa
. trong moi
. tru`ong ho. p ta d`u c
. d
m
an yu c`u d
` ra.

166

,
,,
,
`,i giai v`
Chuong 15. Lo
a go. i y

,
s p1 = 1a = a, p2 = 2a, p3 = 3a,. . . ,
. 7.14. L`o,i giai: Xet dy
, , , ,
pm1 = (m 1) a. Ky hiu
q1 , q2 , . . . , qm1 l`
a c
ac s du tuong u
ng
.
,
`
cua dy trn chia cho m. V` theo d
ai to
an a v`
a m l`
a nguyn
iu kin
. b`
,
,

`
t c`
ung nhau, th` tt ca c
ac s du trn d
ac khng. Nu mt
u kh
.
,
,
,
,
,
`

ng 1 th` b`
d
trong c
ac s du ba
ai to
an d
ac s
a
uo. c giai. Nu tt ca c
, `
,
s trong m 1 s du q1 , q2 , . . . , qm 1
mi
du du kh
ac 1. Khi d
o
`ng mt
ba
ly irichl
. trong m 2 s 2, 3, . . . , m 1. Theo nguyn
,
,
`

ng nhau, cha
ng han
c
o t nht hai s trong q1 , q2 , . . . , qm1 ba
. nhu
,
suy ra s ( j i ) a = p j pi chia ht cho m,
qi = q j , i < j. T`u d
o
, ,
,
,
d
ay khng th xy ra. Su. v ly n`
ay do gia thit khng mt
i`u n`
. s
,
`ng 1, do d
suy ra t nht mt
du trong q1 , q2 , . . . , qm1 ba
ac
o
. trong c
,
,
`

ng 1.
s du phai ba

,
,,
,
,
` go. i y
15.8. L`oi giai va
chuong 8

,
,
,ng minh a + b = c. T`u,
ung ta se chu
. 8.11. L`o,i giai: Truo,c tin ch
,
[ an]
1
,c 0 an [ an] < 1 suy ra |
ng thu
bt d
a| < vo,i moi
a
. n1
n
n
,
[ an]
,c tu,o,ng tu.,
ng thu
= a. Ch
ung ta c
ung c
o bt d
sao cho limn
a
n
,
,
[cn]
[ an] [bn]
,c
,i han
ng thu
+
=
cho b v`
a c. Ch
ung ta ly gio
a
. cua d
n
n
n
,,
v`
a nhn
uo. c a + b = c.
. d
,,
. t a = [ a] + , b = [b] + o d

a
y 0 < 1, 0 < 1. Khi d
o
[ an] + [bn] = [([ a] + )n] + [([b] + )n] = [ a]n + [b]n + [n] + [ n],
[( a + b)n] = [([ a] + [b] + + )n] = [ a]n + [b]n + [( + )n].
,
,
,
`ng nu a
,ng minh ra
. t lai
Nhu vy
ai to
an c
o th d
a
. b`
. nhu sau: Chu
,
,
,
,
,c
ng thu
v`
a b l`
a nhung s trong khoang [0, 1) m`
a ch
ung thoa m
an d
a

[ an] + [bn] = [( a + b)n]


,
`ng 0.
,i moi
ba
vo
o
. s tu. nhin n, th` t nht mt
. trong hai s d

(15.7)

,
,,
,
`,i giai v`
15.8. Lo
a go. i y chuong 8
167
, ,,
,
,ng minh t`n tai
a b 6= 0. Ch
ung ta se chu
Gia su a 6= 0 v`
. s, tu.
,
,c (15.7) khng d
ng thu
ng. Khng mt tnh tng
nhin n sao cho d
a
u
, ,
`ng a + b < 1, v` nu ngu,o.,c lai
qu
at ch
ung ta c
o th gia thit ra
. th`
,
i n = 1.
ng thm
(15.7) se khng d
u
. ch vo
,
, ,
,`,
,
c
a b l`
a nhung s huu ty. Khi d
o
a) Ch
ung ta xet truong ho. p a v`
o
, ,
,
A
B
,,
,
,
i dang
th biu din ch
ung duo
a N l`
a nhung
y A, B v`
. a = N, N, o d
,
,
s nguyn v`
a thoa m
an 0 < A < N, 0 < B < N. D d`
ang kim tra
,
`ng (15.7) khng thoa m
,i n = N 1. Tht
ra
an vo
. vy,
.

[ a( N 1)] = [ aN a] = [ A a] = A + [ a] = A 1,
[b( N 1)] = [bN b] = [ B b] = B + [b] = B 1,
[( a + b)( N 1)] = [ A + B ( a + b)] = A + B [ a + b]
= A+B1
,
Nhu vy
. [ a( N 1)] + [b( N 1)] 6= [( a + b)( N 1)].
,
,,
,
trong c
ac s a v`
a b l`
a s v ty.
b) Ch
ung ta xet tru`ong ho. p mt
.
,
,
`ng c
Ch
ung ta se ch ra ra
o mt
a
. s tu. nhin n, m`

{ an} + {bn} 1.
(15.8)
,
,
,c (15.8) se suy ra khng th c
ng thu
d
Khi do
o (15.7).
a
, ,,
,
,
,
a mt
b`
ai 8.1 tp
Gia su a l`
. s v ty, theo
. ho. p nhung s { an}, n =
,
,
,n
1, 2, . . . l`
a tr`
u mt
a s tu. nhin lo
. trong khoang (0, 1). Ky hiu
. k l`
,
t`n tai
nht m`
a a + kb < 1. Khi d
a
o
. s tu. nhin n m`
a + kb < { an} < 1.
,
,
Nu {bn} b, th` t`u d
inh
. ngha cua k suy ra

(15.9)

{ an} + {bn} > ( a + kb) + b = a + (k + 1)b 1


,
,
,c (15.8) d
ng thu
ng.
hay n
oi c
ach kh
ac t`n tai
ad
a
u
. s tu. nhin n m`

,
,,
,
`,i giai v`
Chuong 15. Lo
a go. i y
,
,
,i vic
ng
Nu {bn} < b, th` vo
ao c
ac v cua bt d
a
. thm 1 b v`
,,
,c 0 {bn} < b, ch
thu
ung ta nhn
uo. c
. d

168

1 b b(n 1) [bn] + 1 < 1


c
i`u d
o ngha l`
a [b(n 1)] = [bn] 1 v`
a {b(n 1)} 1 b.
o
,,
, `,
Ngo`
ai ra, tuong tu. tu (15.9) suy ra
kb < a(n 1) [ an] < 1 a < 1
,

v` th [ a(n 1)] = [ an] v`


a { a(n 1)} > kb. Nhung khi d
o

{ a(n 1)} + {b(n 1)} > kb + (1 b) = 1 + (k 1)b 1,


,,
,
,
ng cho s tu. nhin n 1.
v` k 1. Trong tru`ong ho. p n`
ay (15.8) d
u
,
,
d
Nhu vy
ac s a v`
a b l`
a s v ty.
n v ly khi mt
. dn
. trong c
,
,
,
`ng {n} ch c
. 8.12. Go.,i y: Ch
u y ra
o th nhn
a tri.
.
. huu han
. gi
,,
,
,
. 8.13. Go.,i y: Ly lun
. tuong tu. nhu 8.4.
2 aa
. 8.14. Go.,i y: Xet cac s co dang
p dung
8.4.
. n = 2k v`
.

,
,,
,
,
` go. i y chuong 9
15.9. L`oi giai va
. 9.11. Go.,i y: Xet cac s kx [kx ](k = 0, 1, 2, . . . , n) v`a s 1, chia
,,
p dung
d
on lai
phuong ph
ap d
oan
oan
. [0,1] ra n + 1 ph`n, ph`n c`
. a
.
.
n`
ay.
,
c
. 9.12. Go.,i y: Vo,i mi
ach chon
. nhung s q1 , q2 , . . . qn t`n tai
. s
,
,
i n
nguyn p, vo
o 0 q1 x1 + q2 x2 + + qm xm p < 1. By gi`oi
m oan con ba
`ng nhau.
chia d
oan
.
. [0,1] ra (n + 1) d

,
,,
,
`,i giai v`
15.10. Lo
a go. i y chuong 10

169


p
,
. 9.13. Go.,i y: Hay xet hai tru`o,ng ho.,p 2 > 3 2 v`a
q



p
2 3 2

q
,
,
. 9.14. Go.,i y: Tuo,ng tu. b`ai trn.
,
,
. 9.15. Go.,i y: Gia thit vo,i moi
a q thoa m
an
. p v`
2
2
|mp + npq + sq | 1.

,
,,
,
,
` go. i y chuong 10
15.10. L`oi giai va

,
, ,
,ng
. 10.11. L`o,i giai: y l`a su. tng quat hoa b`ai 10.3. Cach chu
,,
,
,, ,
minh ho`
an to`
an tuong tu. . Ch
ung ta c
ung tao
i voi c
ac h`nh
. ra luo

ly mt
vung c
o din
vung l`
am gc r`i tinh
o
.
. tch 1. Sau d
., h`nh
,
,

tin c
ac h`nh vung c
o chua c
ac manh cua A v h`nh vung gc.
,
,
,
,
,n ho,n n.
Nhu vy
. tin cua A se lo
. tng din
. tch nhung ph`n tinh
,,
Theo nguyn ly irichl mo rng
cho din
o t nht
. tch suy ra c
,.
,
,
i h`nh vung gc
tinh
n + 1 trong s nhung ph`n cua A da
. tin to
,
,
,
,
,
ac h`nh vung nho ban d
c
od
im chung ( x0 , y0 ). Nhung d
im t`u c
`u
,
tinh
ay c
o toa. d
a ( xi , yi ) (i = 1, 2, . . . , n + 1), m`
a
n d
im n`
. l`
. tin d
,

xi x j v`
a yi y j (i, j = 1, 2, . . . , n + 1) l`
a nhung s nguyn.
,
,a b` ma
. t A
. 10.12. L`o,i giai: Ch
ung ta xet h`nh vung V, m`
a n
o chu
,
i d
. t B = V \ A. Ch
v`
a c
o canh
vo
ai n nguyn. a
ung ta thy ngay
. d`
.
,
2
2
S( B) > n k. By gi`o a
p dung
S(V ) = n v`
a S( A) < k, do d
b`
ai
o
.
,
,
2
a t nht n k + 1
10.11. V` th B c
o th tinh
o chu
. tin sao cho n
,
,
,
,
i toa. d
d
A1 , V1 , B1 l`
a c
ac anh cua phep tinh
im vo
. nguyn. Ky hiu
.
.
,
, , ,
tin trn cua A, V, B tuong u
ng. R
o r`
ang B1 = V1 \ A1 . Ch
ung ta se
,
`ng A1 chu
,ng minh ra
,a nhi`u nht k 1 d
,i toa. d
chu
im vo
. nguyn.

,
,,
,
`,i giai v`
Chuong 15. Lo
a go. i y
, ,,
,
,,
,i toa. d
Tht
ung ta c
ok d
im vo
. nguyn
. vy,
. Gia su nguo. c lai
. ch
,
,
,
2

a t nht n k + 1 dim
trong A1 . V` theo c
ach du. ng trn B1 chu
,i toa. d

trong vo
a B1 V1 , A1 V1 , A1 B1 = , Khi d
. nguyn v`
o
,
,
,
2

`
a t nht n + 1 d
i toa. d
h`nh vung V1 chu
ay
im vo
. nguyn, d
iu n`
, ,

khng th xy ra, dn d
n v ly.
170

,
,
, ,
. 10.13. L`o,i giai: C
ung nhu b`
ai 10.5 t
ac dung
ln A phep vi. tu. vo
i
.
1
,
,
,
,
0
i A .
tm tai
a h. s , ch
ung ta nhn
. v`
uo. c tp
. gc ta. d
. d
. ho. p mo
2
,
,
V` S( A) > 4n nn S( A0 ) > n. Theo nguyn ly irichl mo rng
.
,
,
0 chu

cho din
t
ch
A
a
t
nh
t
n
+
1
d
i
m
kh
a
c
nhau
(
x
,
y
)
(
i
=

i i
.
,
1, 2, . . . , n + 1) m`
a xi x j v`
a yi y j l`
a nhung s nguyn.
,
,
,
,a tt ca c
Ch
ung ta xet d
ac l`i nho nht chu
ac d
a gi
im trn. D
,
,
,
d
`ng mi
d`
ang thy ra
ac l`
a mt
trong c
ac d
nh cua d
a gi
im ( xi , yi ).
.
,
,
,
Nhung d
ac khng thuc
ao m`
a hai d`u l`
a
nh cua d
a gi
oan
. trong d
. n`
,
,
,
,
`

`
m trong d
nhung d
ac. V` th trong nhung d
im na
a gi
im ( xi , yi ) tn
,
,
,
`

m trong d
ng ni hai d
tai
a khng na
tha
im m`
oan
im trong
. mt
. d
.
,
, ,,
,
,
chon.
l`
tp
Gia su d
a ( x1 , y1 ). C
ung theo b`
ai
im d
a
im d
o
. ho. p d
.
,
,
,

i toa. d
10.5 cho ta n d
ac nhau v`
a kh
ac d
im kh
im gc toa. d
,
.
. vo
`
`
m trong A. V`
nguyn ( xi x1 , yi y1 ), i = 2, 3, . . . , n + 1, d
u na
,
,

ng nn c
tnh d
ac d
i x u
im ( x1 xi , y1 yi ), i = 2, 3, . . . , n + 1,
c
ung thuc
. A.
,
,ng minh nhu,ng d
Ch
ung ta se chu
ac d
ach trn l`
a
im x
inh
. theo c
, ,,
,
,
,
kh
ac nhau t`ung d
ung ta c
o
i m. t. Tht
. vy,
. gia su nguo. c lai
. ch

( x i x1 , y i y1 ) = ( x1 x j , y1 y j )
,
1
,i hai ch s n`
i, j m`
vo
ao d
a i, j = 2, 3, . . . , n + 1. V` vy
o
. x1 = 2 ( x i +
1
x j ) v`
a y1 = (yi + y j ). Nu i = j th` ( x1 , y1 ) = ( x j , y j ), v ly v`
2

,
,,
,
`,i giai v`
15.11. Lo
a go. i y chuong 11

171

,
,
,
ng ni hai
1 6= j. Nu i 6= j, th` ( x1 , y1 ) l`
a trung d
tha
im cua d
oan
.
,
, ,,
d
a ( x j , y j ), d
ay khng th d
ach chon.
im ( xi , yi ) v`
i`u n`
uo. c theo c
.
,
,
,
,
`ng A chu
ng minh trn ch ra ra
a t nht 2n d
Chu
ac nhau
im kh
,
,
v`
a kh
ac gc toa. d
o toa. d
ung
,
. nguyn. Nhung d
im gc toa. d
. c
. c
,
`m trong tp
na
a c
o toa. d
ai to
an.
. nguyn, suy ra kt lun
. A v`
. cua b`
dung
. 10.14. Go.,i y: Ap
b`
ai tp
.
. 10.13.
,ng minh tu,o,ng tu., 10.9.
. 10.15. Go.,i y: Cach chu

,
,,
,
,
` go. i y chuong 11
15.11. L`oi giai va
,
,c
. 11.11. a) T`u b`ai 11.3 suy ra 6 = 9 (S12 + S23 + S13 ) + S123 , tu
mt
l`
a S12 + S23 + S13 = 3 + S123 3. Do d
ac s S12 , S23 , S13
o
. trong c
, ,
khng nho hon 1.
,,
,
, ,
,c l`
b) L`oi giai: T`u b`
ai 11.3 suy ra 5 9 M2 , tu
a M2 4. Boi v`
,
,
8
,,
,
. p, din
t`u 9 d
ac c
o th tao
tch ph`n chung cua
a gi
uo. c 9. = 36 ca
. d
.
2
, , M2
1

. p d
khng nho hon
.
mt
ac ca
o
. s c
36
9
,
,

cho. Trong mi
. 11.12. L`o,i giai: Goi
a6d
im d
a
. M1 , M2 , . . . , M6 l`
,
,
,
`ng m`u d
n nht ba
tam gi
ac Mi M j Mk ta t canh
a
o. Kt qua se l`
. , lo
,
,
,
ng Mr Ms l`
mt
ad
on c
ac canh
ac se khng d
oan
o c`
o. Ch c`n
. d
. tha
. kh
,
,
,
`ng t`n tai
ng minh ra
cho v`
chu
ac c
od
a c
ac d
a
nh l`
im d
a
. , mt
. tam gi
,
,,
,
,
n nht cua tam gi
ba canh
d
lo
ac nhu
uo. c t m`u d
o. Tht
.
. vy,
., canh
.
,
,,
,
th d
a canh
ac kh
ac v` n
od
`ng th`oi l`
uo. c t m`u
. nho nht cua tam gi
,
,
,
,
,
,i c
ng ni n
cho xut ph
o vo
ac d
d
d
at 5 d
o. T`u mi
im d
a
oan
im
. tha
,
,
,,
. c t nht c
ng n`
c`
on lai.
o 3 trong nhung d
ay d
oan
uo. c t
. , Vy
. hoa
. tha
,
,,
khng d
. c t nht c
m`u d
o3d
o, hoa
oan
uo. c t m`u d
o.
. vn

,
,,
,
`,i giai v`
Chuong 15. Lo
a go. i y
,
,
, ,
,,
ng han
Nu t`u d
at 3 d
im M1 xut ph
oan
uo. c t m`u d
o (cha
. d
.

,n
M1 M2 , M1 M3 , M1 M4 ) th` trong M2 M3 M4 t nht c
o mt
canh
(lo
.
.
,
, ,,
,,
l`
nht) d
a doan
uo. c t m`u d
o, gia su do
. M, 2 M3 . Th th` trong
,
,
,
M1 M2 M3 tt ca c
ac canh
d
`u d
uo. c t m`u do.
.
,
,
,,
Nu t`u d
at t nht 3 d
im M1 xut ph
oan
. khng duo. c t m`u
,
,
ng han
d
ac
o (cha
oan
. 3d
. M1 M2 , M1 M3 , M1 M4 ) th` ta xet 3 tam gi
tam gi
M1 M2 M3 , M1 M2 M4 , M1 M3 M4 . Trong mi
ac n`
ay t nht c
o
,
,
,,
,
,
a d
mt
d
khng chu
uo. c t m`u d
o, nhung canh
nh M1 . Vy
. canh
.
.
,
,.
,,
,
`

3 d
o
an
M
M
,
M
M
,
M
M
d
u
o
c
t
m
u
d
o,
t
u
c
l`
a
3
c
anh
c
ua
.

2 3
2 4
3 4 .
.
,
`
`
M2 M3 M4 d

u
m
u
d
o.

172

,
,
,
. 11.13. L`o,i giai: Truo,c ht ta nhn
xet c
o t nht mt
tam gi
ac d
o n
.
.
,
, ,,
,
,
,
c (B`
c, cha
ng han
sa
ai 5.4). Gia su P1 P2 P3 l`
a tam gi
ac d
d
on sa
o.
.
,
,
c khng
ring P1 ta c`
on lai
6d
a ta lai
c
o mt
tam gi
ac d
im v`
on sa
.
.
.
,
,

a canh
c
o P1 l`
ad
ai 5.4). Nu tam gi
ac n`
ay khng chu
P2 P3 th`
nh (B`
.
,
,
,,
,
,
0
0

tu P1 sao cho P1 P2 P3 c
ta d
o mt
ung
uo. c kt qua, th th` c
im thu
. d
,
,,
, , `
,, 0
0
l`
a tam gi
ac d
a P3 ta d
a P3 sao cho
o. Tuong tu. d
rin P2 v`
uo. c P2 v`
,
0
0
P1 P2 P3 v`
a P1 P2 P3 l`
a tam gi
ac d
o.
,
,
,,
Truoc ht ta xet kha n
angl`
a3d
a P30 khng phn bit.
im P10 , P20 v`
.
,`,
,
,

Trong truong ho. p n`


ay mt
trong
ch
u
ng
t
ao
th`
a
nh
v
o
i
P
P
P
m
t
1 2 3
.,
.
,.
,
,
din.
h`nh tu
on lai
3 dim. Nu c
ac canh
im
. C`
. , giua mt
. trong ba d
, . ,
,
,
,
din
v`
c`
on lai
d
a2d
ad
ac
o
nh cua h`nh tu
o ta se d
uo. c hai tam gi
. l`
, ,.
,
,
,
i 2 canh
i v`
d
r`oi nhau, mt
ac c
o mt
a mt
o vo
nh mo
.
. tam gi
. d
. canh
.
,
,
,
,

din

cua tu
v`
a
tam
gi
a
c
kia
trn
t
u
di
n.
N
u
khng,
m
i
d
i
m
c
o t

. ,
.
,
,
,
,
,
din.
din
nht canh
tu
o t nht mt
o thuc
nh cua tu
. , d
. Nhu th c
. d
.
, .
,
`
`

i ca 3 d

ni vo
nh
c`
o
n
l
ai
b
a
ng
c
anh
m
u
d
en.
N
u
c
a
p
n`
a
o
trong
3

.
.
. ,
`ng canh
ni ba
o mt
tam gi
ac d
a mt
en th` ta c
en v`
o. Nu khng
. d
. d
,
,. ,
,,
ta d
ac d
od
uo. c hai tam gi
o c
nh r`oi nhau.

,
,,
,
`,i giai v`
15.12. Lo
a go. i y chuong 12
173
, , ,,
,
`ng ba d
Vy
ta c
o th gia su ra
a P30 l`
a phn bit.
im P10 , P20 v`
.
. Nu,
,
,
,
,
,
,
`ng canh
2d
a P20 , d
d
ac d
im, P10 v`
uo. c ni ba
o th` ta d
uo. c hai tam gi
o
.
,
0
0
0
0
0

tu.,
P1 P2 P3 v`
a P1 P2 P3 . Nu khng th` P1 P2 P3 v`
a P1 P2 P3 cho theo thu
,
mt
ac d
a mt
ac d
o v`
en.
. tam gi
. tam gi
,
,
,
,
, hai cua
. 11.14. L`o,i giai: Ly 1998 da duo.,c t ging nhu da thu
,
, nht sao cho ch
. t ch`ng tt ca ch
ch
ung ta v`
a da
ung ln d
ung
a thu
,
,
,
h`nh quat
trn mi
c
o tt ca c
ac vi. tr c
o th (nhu khi quay). Khi d
o
.
,
,
,
,
,
,
2

nht c

cua d
o 200 h`nh quat,
d
u
o
c
t,
t
u
c
l`
a
c
o
t
t
c
a
200
a thu

.
.
,
,
,
,,
,
,
. p h`nh quat
ca
ung nhau. Gia su c
o n vi. tr cua d
a thu
. duo. c t tr`
,,
,
. p h`nh quat

ung nhau. Khi d


hai c
o khng t hon 21 ca
uo. c t tr`
o
. d
, ,
,
,

s c
ac h`nh quat
ung nhau khng nho hon 21n. Do d
uo. c t tr`
o
. d
,,
,
2
c l`
21n 200 , tu
a n 1904, 8. Boi v` n l`
a s nguyn, nn n 1904.
. p h`nh
Suy ra c
o t nht 1998 1904 = 94 vi. tr c
o khng qu
a 20 ca
,,
quat
ung nhau.
uo. c t tr`
. d
,
,ng minh ho`
. 11.15. Go.,i y: Chu
an to`
an nhu b`
ai 11.2.

,
,,
,
,
` go. i y chuong 12
15.12. L`oi giai va

,
,
,
,ng qua
. 12.11. L`o,i giai: Ly A l`a nhung dim trn ma. t c`u di xu
,
,
,
,
,
,
,
,
,
tm cua qua c`u t`u nhung d
ho. p A. Theo gia thit tng
im cua tp
.
,
,n ho,n din
. t c`u. Theo nguyn ly
din
a A lo
. tch cua A v`
., tch ma
,
v`
irichl cho din
tch ch
ung c
od
d
a
im chung. Vy
im chung d
o
.
.
,
,
,
,
,`,

. p d
d
o l`
a ca
im d
i xung cua n
im d
u thuc
uong knh
. A tao
. ra d
,
`
qua cu.
,
h`nh chu, thp
. 12.12. L`o,i giai: Vo,i mi
on c
o tm tai
. ta xet h`nh tr`
.
1
,
,
`
`
ng minh ra
ng . Ta chu
ng nu hai
tm chu thp
a b
an knh ba
. v`
2 2

,
,,
,
`,i giai v`
Chuong 15. Lo
a go. i y
,
t nhau. Khoang
t nhau th` hai chu, thp
ca
c
ung se ca
h`nh tr`
on do
.
,
,
`ng nhau v`
t nhau khng lo
,n
c
ach giua hai tm cua h`nh tr`
on ba
a ca
,
,
,
,
,
khoang c
hon hai l`n b
an knh cua ch
ung, do d
ach giua tm cua
o
1
,
, , ,
,i ch
,n ho,n . Xet h`nh chu,
c
ac chu thp
ng vo
ung khng lo
. tuong u
2
,
,
,,
,
, nht v`
nht
ac d
ac c
anh cua h`nh chu thp
thu
a tm cua
inh
. boi c
. x
.
,
,
,
, hai. Se c
, hai d
chu thp
o mt
anh cua chu thp
i qua h`nh
. thu
. c
. thu
,
,,
,
,
,

nht, boi v` d
t chu thp
, do d
se ca
chu nht
ai cua c
anh
o
o
. d`
. d
. thu
1
1
,
,
,
,
,
`ng , c`
n hon .
ba
on d
ai d
. d`
u`ong cheo h`nh chu nht
. khng lo
2
2
,
,
,,
. t d
R
o r`
ang trong h`nh tr`
on b
an knh 100 ch c
o th d
a
uo. c mt
.
1
,

t nhau.
s huu han
ac h`nh tr`
on b
an knh khng ca
. c
2 2
174

,
, , ,,
,
. 12.13. L`o,i giai: Co th. Gia su O l`a tm cua ng
u gi
ac d
`u ABCDE.
c
Khi d
ac h`nh tr`
on ni
ac g
oc AOC, BOD, COE, DOA,
o
. tip trong c
nu.
EOB c
o tnh cht d
a
,
,
`ng 1. Tt ca 3
. 12.14. L`o,i giai: Xet mt
tam gi
ac d
o canh
ba
`u c
.
.
,
,
, ,,
,
`ng c
phai
d
o khng th d
ac m`u kh
ac nhau, do d
nh cua n
uo. c t ba
o
,
,
,
`ng 1.
c
o hai d
ung m`u, v`
a khoang c
ach giu ch
ung ba
im c`

PHU. LUC
.

,
` ANH

TP
H
O
XA
.
.
. P VA

,,
,
,
`

A.1. Tp
h
o
p
v
a
To
an
t
u
trn tp
.
.
. ho. p.

, ,
,,
`n tu, cua tp
l`
Khi ch
ung ta coi mt
ao d
a mt
i tuo. ng a n`
o
.
. d
. ph
,, ,
,
`
ho. p A, th` ky hiu
l`
a
a

A,
c`
o
n
khi
a
khng
l`
a
ph
n
t
u
c
ua
A
th`

k
y
.
hiu
a a 6 A.
. l`
,
,
,
,
Mt
tp
ho. p A goi
l`
a tp
ho. p con cua tp
ho. p B, khi moi
.
.
.
.
. ph`n
.
,, ,
,, ,
,
,
. c
tu cua tp
a ph`n tu cua tp
a A B hoa
. ho. p A l`
. ho. p B, Ky hiu
. l`
,
,
. c l`
B A. Ngha l`
a t`u a A suy ra a B hoa
a t`u a 6 B suy ra
a 6 A.
,,
,
ao, ky hiu
a . Moi
Tp
o mt
i
. l`
. d
. ph`n tu n`
. ho. p rng khng c
,
,
,
x 6 . Tp

`m trong tp
tuo. ng x d
`u khng na
. ho. p rng
. ho. p rng
,
,
`m trong moi
i moi
d`u na
a A, vo
. tp
. ho. p, ngha l`
. A.
,,
,
Hai tp
a B tr`
ung nhau khi ch
ung c
o c`
ung c
ac ph`n tu
. ho. p A v`
,
,
,,
,,
,
nhu nhau, ky hiu
a ch khi t`u
uong , A = B khi v`
. A = B. Tuong d
,
. c A B v`
a A suy ra a B v`
a t`u a B suy ra a A hoa
a B A.
,
,
, ,
,
Cho A1 , A2 , . . . , An l`
a mt
tp
ho. p. Ho. p cua
. s huu han
. nhung
.
,
,,
,
,
,
nhung tp
a mt
ac ph`n tu m`
a n
o
. ho. p trn l`
. tp
. ho. p g`m tt ca c
,
, ,
thuc
ao mt
ac tp
c
ac tp
. v`
. trong c
. ho. p A1 , A2 , . . . , An . Ho. p cua
.
,
,
ho. p A1 , A2 , . . . , An , ky hiu
l`
a
A

.
.
.

A
.
C
u
th
,
a

n
2
1
1
.
.
,
A2 . . . An ngha l`
a t`n tai
. mt
. ch s i trong {1, 2, . . . , n} sao cho
175

176

,,
,
xa.
Chuong A. Tp
a Anh
. ho. p v`

a Ai .
,
,
,
Cho A1 , A2 , . . . , An l`
a mt
. s huu han
. nhung tp
. ho. p. Giao
,
,,
,
,
,
cua nhung tp
a mt
ac ph`n tu m`
a
. ho. p trn l`
. tp
. ho. p g`m c
,
,
,
`m trong moi
n
o na
ac tp
. tp
. ho. p A1 , A2 , . . . , An . Giao c,ua c
. ho. p
A1 , A2 , . . . , An ky hiu
a A1 A2 . . . An . Cu. th, a A1 A2
. l`
,
,
i moi
. . . An ngha l`
a vo
o a Ai .
. ch s i trong {1, 2, . . . , n} ta c
,
,
,
Hiu
tp
a B l`
a mt
. ho. p A v`
. tp
. ho. p, ky hiu
. A\ B, g`m
. cua hai
,
,
,
nhung ph`n tu a c
o tnh cht a A v`
a a 6 B.

, ,
hoc
` Bai
` toan
t ho. p
A.2. Qui nap
. toan
. va
,
,
,i nguyn ly irichl c`
Trong tp
ung vo
on mt
. ho. p s tu. nhin, c`
,.
, , ,,
,
nguyn ly qui nap
an hoc
ung hay d
Dang
uo. c su dung.
on gian
. to
. c
.
. d
,
,,
,
cua phuong ph
ap qui nap
an hoc
. to
. nhu sau:
,
,
ng di`u kha
Cho mt
ay nhu
ng dinh
. d
.
K1 , K2 , . . . , K n , . . .

(A.1)

c
o tnh cht sau:
,
ng.
a)K1 l`
a kha
ng dinh
. du
,
, ,
b) V
oi mi
s tu. nhin n,nu Kn l`
a kha
ng dinh
. dus ng th` suy ra
,
ng.
Kn+1 c
ung l`
a kha
ng dinh
. du
,
,
tt ca c
ng.
Khi do
ac kha
ng dinh
ay (A.1) d`u du
. trong d
,
,
ng, sau d
a
p dung
: d
Ch
uy
b)
i`u kin
am bao cho K1 d
u
o
. a) d
.

ng, tip tuc


p dung
ng v`
cho K2 c
ung d
ung d
a tip tuc
u
u
.,a
. b) cho K3 c
.
,
,

i moi
vo
qu
a tr`nh d
ung ta chung minh mt
o
inh
.
. n. minh hoa
. ch
. d
, ,
`

p dung
ly v t ho. p m`
a rt hay a
trong cun s
ach n`
ay:
.
,,
,,
, ,
,
,
n
ng 2 tp
Moi
oi n ph`n tu c
o du
. tp
. ho. p huu han
. ,v
. ho. p con tao
. boi
,,
c
ac ph`n tu kh
ac nhau cua n
o.

,
xa. trn tp
A.3. Anh
. ho. p

177

,
`ng Kn .
,i moi
ng d
Vo
ung ta ky hiu
trn ba
i`u kha
inh
.
. s n ch
. d
,
,
,,
,
i n = 1 kha
ng d
Nhu vy
ung ta se nhn
ay (A.1). Vo
uo. c d
inh
.
. ch
. d
,,
,
,
`
ng, v` moi
ng hai tp
l`
d
o mt
o du
a
u
o
. tp
. ho. p c
. phn tu c
. ho. p con, d
,,

tp
a chnh tp
o tnh cht
os . Vy
. rng v`
. ( mt
. phn tu ) d
. (A.1) c
a).
,
,
,ng minh b) d
ng. Ngha l`
By gi`o ch
ung ta phai chu
a cho n l`
a
u
,
,
,
,
c l`
ng d
ng tu
mt
a kha
a moi
o
inh
u
. Kn d
. s tu. nhin bt k`y v`
. tp
. ho. p c
,
,,
,
,
n
`ng Kn+1
ng minh ra
ng 2 tp
n ph`n tu c
od
u
. ho. p con. C`n phai chu
,,
,
l`
ad
ung ta xet S l`
a tp
o n + 1 ph`n tu v`
a s l`
a mt
us ng. Ch
. ho. p c
.
,
,, ,
,
,
,

ph`n tu cua n
o. Ch
ung ta chia nhung tp
h
o
p
con
c
ua
S
l`
a
m
hai
l
o
p:
.
,
, .
,
,a s. Lo
,p a1 g`m tt ca c
,p a2 g`m
Lo
ac tp
h
o
p
con
c
ua
S
khng
chu
.,
.
,
,
,
,a s. Nhu,ng tp
,p a1
tt ca c
ac tp
. ho. p, con cua S chu
. ho. p con thuc
. lo
,
,
,
,
,
l`
a tp
a n
o nhn
. , ho. p con cua mt
. tp
. ,ho. p n ph`n tu S1 , m`
. t`u S sau
,
,,
,,
,
khi bo d
i ph`n tu s. Theo gia thit qui nap
. ,s luo. ng tp
. ho. p con cua
, , ,
,
,p a2 l`
. t kh
ng l`
n
od
a 2 n . Ma
ac c
ac tp
a tuong u
ng 1-1
u
. ho. p con cua lo
,
,
,
,
,
,a s
i vo
i c
vo
ac tp
. ho. p con cua S1 . Tht
. vy,
. moi
. ,tp
. ho. p con Z chu
,
, , ,
,
,
`ng c
,i tp
cua S ta cho tuong u
ng vo
ach t`u Z
. ho. p con Z1 cua S1, ba
,
,,
,,
,
,p a2
ta bo d
a s luo. ng tp
i. ph`n tu s. Ngha l`
. ho. p con cua S trong lo
,
,
,
`ng 2n . Do d
,i s lu,o.,ng tp
tt ca tp
tr`
ung vo
ho. p con cua S1 v`
a ba
o
.
.
,
,
,
,

ho. p con cua S l`


a 2n + 2n = 2n+1 . Nhu vy
ch
u
ng
ta
d
a
ch
u
ng
minh,

.
,
`
,ng minh
,i moi
vo
n,
t
u
K
d
us
ng
suy
ra
K
d
us
ng. Ch
ung ta chu

n
n +1
.
,
,
,,
ng dinh
ng.
d
o tnh cht b). Suy ra tt ca c
ac kha
uo. c (A.1) c
`u d
u
. d

A.3. Anh
xa. trn tp
. ho. p

,, ,
,
,
,
,n ho,n s
T`u nhung kh
ai nim
s ph`n tu cua mt
tp
ho. p n`
ay lo
.
.
.
,, ,
,,
,
,
nh xa.
ph`n tu cua mt
tp
ho. p kia, ngu`oi ta xy du. ng kh
ai nim
.
.
. a
, ,
,

d
a chnh x
ac h
oa trong khi chung minh.
giai thch v`

178

,,
,
xa.
Chuong A. Tp
a Anh
. ho. p v`

,
,
`ng cho mt
nh xa. f : A B t`u tp
Ch
ung ta n
oi ra
ao
. ho. p A v`
. a
,,
, , ,
,
,
`
i mt
tp
. ho. p B, khi moi
. phn tu a thuc
. A cho tuo,ng ung vo
. ph`n
,,
,,
,i ph`n
nh xa. f vo
tu f ( a) thuc
B. Ph`n tu f ( a) goi
l`
a gi
a tri. cua a
.
.
,
, ,
,,
,
,
nh xa. s tu. nhin v`
ao s
tu a cua A. C
o th ch ra nhi`u v du. nhu a
,
,
,
,
,
n, vo
ng vo
i moi
i 2n,. . . .
cha
. n cho tuong u
,
,`,
,
nh xa:
nh xa.
Nguoi ta c`
on d
ua ra mt
. s tnh cht cua a
. Mt
. a
,
,
nh, nu t`u f ( a1 ) = f ( a2 )( a1 , a2 A) suy ra
f : A B goi
ad
on a
. l`
,
,,
,
nh khi moi
a1 = a2 . N
oi c
ach kh
ac f l`
ad
ac nhau cua
o n a
. ph`n, tu kh
,
, , ,
,i nhu,ng ph`n tu, kh
A cho tuong u
ng vo
ac nhau cua B.
,,
,i moi
nh xa. f : A B goi
nh, nu vo
Mt
a
l`
a to`
an a
ph`n tu
.
.
.
,
,
,,
cua b cua B t`n tai
o nhi`u v
. ph`n tu a thuc
. A sao cho f ( a) = b., C
,
,
nh xa. loai
du. v` a
n`
ay, nhu phep chiu ln truc
toa. d
d
. cua moi
im
.
.
.
,
. t pha
ng.
trong ma
,
,
nh xa. f : A B goi
nh, khi n
Mt
a song a
od
ad
`ng th`oi l`
o n
. a
. l`
,,
,i moi
nh v`
nh. Nu f : A B l`
nh, th` vo
a
a to`
an a
a song a
ph`n tu b
.
,
,,
cua B t`n tai
. duy nht ph`n tu a thuc
. A sao cho f ( a) = b.
. t g(b) = a khi d
nh f : A B. Nu ch

Cho song a
ung ta d
a
a
,,
nh xa. g : B A goi
nh
aa
c
o f ( a) = b, ch
ung ta nhn
uo. c a
. d
. l`
,, ,
,

1
xa. nguo. c cua f v`
a ky hiu
a f
= g. Nhu vy
a
. l`
. f ( f (b)) = b v`
,

1
i moi
f ( f ( a)) = a vo
a b thuc
. a thuc
. A v`
. B.
,
nh xa. f : A B v`
nh xa.
Cho hai a
a g : B C, c
o th tao
. ra a
,
`ng cng thu
c h( a) = f ( g( a)). Ky hiu
h : A C ba
. h = g f.

,
`
TAI LIU
. THAM KHAO

,
, ,
[1] Tuyn tp
am tap
an hoc
a Tui tre, NXBGD, 1997.
. 30 n
. ch To
. v`
[2] 255 b`
ai to
an s hoc
anh, N. N.
. chon
. loc,
. V.D. Thuy,
. T. C. Th`
,,
am,
So GD H`
a ty, 1993
.
,
,
,
[3] 10000 B`
ai to
an so cp - D
ay s v`
a gi
oi han,
. Phan Huy Khai,
NXB H`
a ni,
. 1997.
,,
[4] C
ac d` thi v dich
an c
ac nuoc, Xv. Cnhiagin, G.A.Tnian,
. to
,
If. Sarugin, NXB GD 1996.
,
,
[5] C
ac b`
ai to
an h`nh hoc
ng, V.V. Praxolov, Tp
. pha
. II, NXB Hai
ph`
ong 1997.
,
[6] Zadatri Vsesaiunuc matematitreskii olimpiada, N.B. Vasilev,
A.A. Egorov, Maskova- Nauka 1988.
[7] Matematichskii akvarium V. A. Uphnarovskii, Kishinev " shtiinsa" 1987.(Ting Nga).
[8] Problem-Solving Through Problems, Loren C. Larson, SpringVerlag, 1983.
[9] Princil na Dirichle, Ivan Prodanov, "Narodna Prosveta", Sophia
1988 (Ting Bungari).
179

180

Muc
. luc
.

,
[10] Zadachi da idvunklasna Rabota po matematika,
R.Rusev, K.Bankov, Sv. Slavchev, "Narodna Prosveta", Sophia
1986 (Ting Bungari).
[11] Sbornhik ot Zadachi za Matematicheski Olympiadi,
St. Budurov, V. Phlorov, "Narodna Prosveta", Sophia 1966
(Ting Bungari).

NI
. DUNG

`,i n
Lo
oi d
`u . . . . . . . . . . . . . . . . . . . . . . . . . . . . . . . . . . . . . . . . . . . . . . . . . . .
,,
Chuong 1. Nguyn l
y irichl v`
a v du. . . . . . . . . . . . . . . . . . . . . . .

1.1. Nguyn ly irichl . . . . . . . . . . . . . . . . . . . . . . . . . . . . . . . . . . . . .

1.2. V du.
. ..................................................

1.3. B`
ai tp.
. ...............................................
,,
Chuong 2. S hoc
. ..........................................
,
2.1. Phep chia s tu. nhin . . . . . . . . . . . . . . . . . . . . . . . . . . . . . . . . .

13

2.2. Vdu. . . . . . . . . . . . . . . . . . . . . . . . . . . . . . . . . . . . . . . . . . . . . . . . . . .

14

2.3. B`
ai tp.
. ...............................................
,,
Chuong 3. D
ay s . . . . . . . . . . . . . . . . . . . . . . . . . . . . . . . . . . . . . . . . . .

20
23

3.1. Nguyn ly irichl cho d


ay s v han
. ..................

23

3.2. V du. . . . . . . . . . . . . . . . . . . . . . . . . . . . . . . . . . . . . . . . . . . . . . . . . .

23

3.3. B`
ai tp.
. ...............................................
,,
Chuong 4. H`nh hoc
. ........................................

31
33

4.1. V du. . . . . . . . . . . . . . . . . . . . . . . . . . . . . . . . . . . . . . . . . . . . . . . . . .

33

4.2. B`
ai tp.
. ...............................................
,,
,,
Chuong 5. Mo rng
nguyn l
y irichl. . . . . . . . . . . . . . . . . . . . .
.
,,
5.1. Nguyn ly irichl mo rng
. ...........................

41
43

5.2. V du. . . . . . . . . . . . . . . . . . . . . . . . . . . . . . . . . . . . . . . . . . . . . . . . . .

44

181

11

13

43

182

NI
. DUNG

5.3. B`
ai tp.
. ...............................................
,,
ai tp
Chuong 6. B`
. s hoc
. nng cao . . . . . . . . . . . . . . . . . . . . . . . .
,
, ,
6.1. inh
. ly co ban cua s hoc
. .............................

52
55

6.2. V du. . . . . . . . . . . . . . . . . . . . . . . . . . . . . . . . . . . . . . . . . . . . . . . . . .

55

6.3. B`
ai tp.
. ...............................................
,,

Chuong 7. B`
ai tp
. dy s nng cao . . . . . . . . . . . . . . . . . . . . . . . .

64
67

7.1. V du. . . . . . . . . . . . . . . . . . . . . . . . . . . . . . . . . . . . . . . . . . . . . . . . . .

67

7.2. B`
ai tp.
. ...............................................
,,
, ,
Chuong 8. S thu. c vo
i tp
u mt
. tr`
. .........................

78
79

8.1. Tp
u mt
. tr`
. ...........................................

79

8.2. V du. . . . . . . . . . . . . . . . . . . . . . . . . . . . . . . . . . . . . . . . . . . . . . . . . .

80

8.3. B`
ai tp.
. ...............................................
,
,,
,ng u
,ng dung
kh
ac cua nguyn l
y irichle . .
Chuong 9. Nhu
.
,
,
9.1. Xp x mt
. s thu. c . . . . . . . . . . . . . . . . . . . . . . . . . . . . . . . . . . . .

87

55

89
89

9.2. B`
ai tp.
. . . . . . . . . . . . . . . . . . . . . . . . . . . . . . . . . . . . . . . . . . . . . . . . 99
,,
Chuong 10. Nguyn l
y irichl cho din
. tch . . . . . . . . . . . . . 101
,
10.1. Ph
at biu nguyn ly irichl cho din
. tch . . . . . . . . . . 101
10.2. V du. . . . . . . . . . . . . . . . . . . . . . . . . . . . . . . . . . . . . . . . . . . . . . . .

105

10.3. B`
ai tp
. .............................................
, ,
,,
Chuong 11. To
an hoc
. t ho. p . . . . . . . . . . . . . . . . . . . . . . . . . . . . . .

117
119

11.1. V du. . . . . . . . . . . . . . . . . . . . . . . . . . . . . . . . . . . . . . . . . . . . . . . .

119

11.2. B`
ai tp
. .............................................
,,
Chuong 12. Mt
ai tp
ac . . . . . . . . . . . . . . . .
. s b`
. h`nh hoc
. kh

127
129

12.1. V du. . . . . . . . . . . . . . . . . . . . . . . . . . . . . . . . . . . . . . . . . . . . . . . .

129

Muc
. luc
.

183

12.2. B`
ai tp
. .............................................
,,
Chuong 13. Mt
` thi v dich
. ........................
. s d
,
,,
,
ai tp
Chuong 14. B`
. tu. giai . . . . . . . . . . . . . . . . . . . . . . . . . . . . . . . .
,
,,
,
`,i giai v`
..............................
Chuong 15. Lo
a go. i y
,
,
,
,
,
15.1. L`oi giai v`
a go. i y chuong 1. . . . . . . . . . . . . . . . . . . . . . . . . . .
,
,,
,
,
15.2. L`oi giai v`
a go. i y chuong 2. . . . . . . . . . . . . . . . . . . . . . . . . . .
,,
, ,
,
15.3. L`oi giai v`
a go. i y chuong 3. . . . . . . . . . . . . . . . . . . . . . . . . . .
,,
, ,
,
15.4. L`oi giai v`
a go. i y chuong 4. . . . . . . . . . . . . . . . . . . . . . . . . . .
,,
, ,
,
15.5. L`oi giai v`
a go. i y chuong 5. . . . . . . . . . . . . . . . . . . . . . . . . . .
,,
, ,
,
15.6. L`oi giai v`
a go. i y chuong 6. . . . . . . . . . . . . . . . . . . . . . . . . . .
,,
, ,
,
15.7. L`oi giai v`
a go. i y chuong 7. . . . . . . . . . . . . . . . . . . . . . . . . . .
,,
, ,
,
15.8. L`oi giai v`
a go. i y chuong 8. . . . . . . . . . . . . . . . . . . . . . . . . . .
,,
, ,
,
15.9. L`oi giai v`
a go. i y chuong 9. . . . . . . . . . . . . . . . . . . . . . . . . . .
,,
, ,
,
15.10. L`oi giai v`
a go. i y chuong 10 . . . . . . . . . . . . . . . . . . . . . . . .
,,
, ,
,
15.11. L`oi giai v`
a go. i y chuong 11 . . . . . . . . . . . . . . . . . . . . . . . .
,,
, ,
,
15.12. L`oi giai v`
a go. i y chuong 12 . . . . . . . . . . . . . . . . . . . . . . . .
,
nh xa. . . . . . . . . . . . . . . . . . . . . . . . . . . . .
Phu. luc
aA
. A. Tp
. ho. p v`
,,
,
,
A.1. Tp
a To
an tu trn tp
. ho. p v`
. ho. p. . . . . . . . . . . . . . . . . . . . .
, ,
A.2. Qui nap
an hoc
a B`
ai to
an t ho. p . . . . . . . . . . . . . . . . .
. to
. v`
,
xa. trn tp
A.3. Anh
. ho. p . . . . . . . . . . . . . . . . . . . . . . . . . . . . . . . . .

134

Muc
. luc
. .................................................

135
151
155
155
156
158
158
160
161
162
166
168
169
171
173
175
175
176
177
180

,
HU
,U IN
NGUYN

, ,
,NG DUNG
P IRICHL VA
`U
PHUONG PHA
.
,
, ,
c
Ebook
1.0 cua cun s
ach nguyn gc t`u ban in, c
ac ban
. tham
,
,
,

`
khao, cho y kin sai s
ot v`
a loi khuyn t
ai ban. Moi
. lin h.
,
,
,

T
ac gia: Nguyn Huu in
in
. thoai:
. 0989061951
Email: huudien@vnu.edu.vn
Web: http://nhdien.wordpress.com

,
Chiu
ach nhim
. tr
. xut ban:
,, ,
Bin tp
a sua ban in:
. v`
,
Tr`nh b`y v`
a ch ban:
Tr`nh b`y b`a:

,
NG HAI
T A
PHU

,
,

` LINH
HUU IN, PHU
, ,
HUONG LAN

,
BAN KHOA HOC VA
` XUT
` KY
THUT
NHA
.
.
,
`
`
70 TRN HUNG AO
. - HA NI
.
6.6T7.3
290 - 4-98
KHKT 98
,
,
In 1000 ban kh 14, 5 20, 5 tai
an Vit
. Cng ty In Cng o`
. Nam
,
191 Son Ty - ng a - H`
a Ni.
. Giy phep XB s: 290-22/4/98.
,
,
In xong v`
a np
ang 1 n
am 1999.
. luu chiu th

You might also like